Canadian Boards - Neonatology

Ace your homework & exams now with Quizwiz!

With the new AS-3 preservative, how long can blood be stored and used for?

Store for at least 4 weeks Once used, 5 days (if using for mini, slow, top-up transfusions) At 5 days K+ is very high If want a big bolus, need <5d blood CPS Red blood cell transfusion in newborn infants

bleeding from the leptomeningeal arteries or bridging veins

Subarachnoid hemorrhage • Due to bleeding from the leptomeningeal arteries or bridging veins. • Majority are asymptomatic; may be detected due high RBCs on a lumbar puncture • May develop benign seizures on the 2nd day of life. • Rarely fatal • Consider AVM if neurological abnormalities - do CT or MRI. Neo Notes

Collection of blood beneath the aponeurosis the entire length of the occipital-frontalis muscle after birth

Subgaleal hemorrhage • Collection of blood beneath the aponeurosis the entire length of the occipital-frontalis muscle. • Bleeding can be very extensive: can even extend to neck. • firm fluctuant mass, increases in size after birth. • Often with vacuum-assisted delivery • Many develop a consumptive coagulopathy due to massive blood loss. • Must monitor for hypotension and hyperbilirubinemia. • Typically resolve over a 2-3 weeks Neo Notes

o transient eye deviations, nystagmus, blinking, mouthing, abnormal extremity movements (rowing, swimming, bicycling, pedaling, and stepping), fluctuations in heart rate, hypertension episodes, and apnea. More common in premature

Subtle Seizures Neo Notes

What interventions are NOT helpful in preventing hyperbilirubinemia?

Supplementing breast milk with water/ dextrose water Glycerine suppositories/enemas (to decrease enterohepatic circulation of bilirubin) L-aspartic acid Enzymatically hydrolyzed casein Whey/casein/clofibrate CPS Hyperbili

CPS recommendation: When should IV supplementation be used for hypoglycemia?

Symptomatic <2.6 <1.8 after one effective feed <2 after subsequent feeds <2.6 consistently CPS Hypoglycemia

Does erythropoeitin contain albumin?

YES CPS Red blood cell transfusion in newborn infants

Canadian Pediatric Review 2018 Perinatal-Neonatal Medicine When PPV begins, the assistant listens for increasing HR for the first 15 seconds of PPV - match the action with the observation (some answers can be used more than once): A. "HR is increasing" B. "HR is not increasing, chest is moving" C. "HR is not increasing, chest is not moving" D. "The chest is moving now" E. "HR is at least 100 at 30s" F. "HR is >60 <100 at 30s" G. "HR is <60 at 30s" H. "HR not improving, chest is moving well, PPV x 1 minute...." 1. Ventilation corrective steps (MRSOPA) 2. Continue PPV until spontaneous efforts 3. PPV continues for 15s and reassess 4. Begin 100% O2 and chest compressions 5. Intubate or insert LMA 6. Ventilate for 30 seconds and reassess

a - 3. PPV continues for 15s and reassess b - 3. PPV continues for 15s and reassess (complete 15s because may still be recruiting - try 30s if getting good chest movement) c - 1. Ventilation corrective steps (MRSOPA) d - 6. Ventilate for 30 seconds and reassess e - 2. Continue PPV until spontaneous efforts f - 5. Intubate or insert LMA (do this if HR isn't coming up despite effective bagging) g - 4. Begin 100% O2 and chest compressions h

prethreshold ROP is ROP with

a high likelihood of progressing to threshold ROP. CPS ROP

Retinopathy of prematurity is

a proliferative disorder of the developing retinal blood vessels in preterm infants. Disease of rentinal vasculature Affects mostly preterm infants Cause blindness and poor visual acuity CPS ROP

CPS Intubation - premedication is to administer

a vagolytic (intravenous atropine 20 μg/kg), a rapid-acting analgesic (IV fentanyl 3 μg/kg to 5 μg/kg; slow infusion) and a shortduration muscle relaxant (IV succinylcholine 2 mg/kg). CPS Intubation

Tx of ABO incompatibility

TREATMENT: Phototherapy may be effective in lowering serum bilirubin levels. In rare severe cases, treatment is directed at correcting dangerous degrees of anemia or hyperbilirubinemia by exchange transfusions with type O blood of the same Rh type as the infant. Some infants with ABO hemolytic disease may require transfusion of packed RBCs at several weeks of age because of slowly progressive anemia. Post-discharge monitoring of hemoglobin/Hct is essential in newborns with ABO hemolytic disease. Neo Notes

How and when should the bilirubin be screened?

TSB or TcB between 24-72h of life If treatment level treat If not treatment level plot on severe hyperbili risk nomogram Document TSB/TcB and risk zone for parents and arrange for followup TSB can be done with metabolic screening Minimize pokes TcB at dc or 72h of life (whichever comes first) CPS Hyperbili

Decreased risk factors for Development of Severe Hyperbilirubinemia in Infants > 35 weeks

TSB or TcB level in the low-risk zone Gestational age ≥41 wk Exclusive bottle feeding Black race Discharge from hospital after 72 hr Neo Notes

what happens to gut in NEC

The disease is characterized by various degrees of mucosal or transmural necrosis of the intestine Neo Notes

Who is prioritized in receiving banked human milk

The most critical demand for human donor breast milk is for the most vulnerable neonates who are either preterm or require gastrointestinal surgery as a newborn. There was a reduction in NEC in donor breast milk-fed neonates. serological screening and physician consent. Serology includes testing for hepatitis B and C as well as HIV and the human T cell leukemia virus. All milk must be properly collected, stored, pasteurized and cultured in accordance with food preparation guidelines Although there may be a risk of allergic reaction to human donor breast milk, human breast milk is species specific and, thus, the risk is not higher than the alternative - formula feeding. CPS milk banking

What are the three concerns regarding folic fortification?

Toxicities from OVERDOSE = will not happen based on fortification amounts. Safe as long as <10mg/day Masking pernicious anemia (B12 deficiency w/ neurologic complications) Studies/surveys shown that at the fortification doses, has not happened The biggest consumer of food is adult males! IF we increase folate in everything - males might get a much higher intake! CPS Folic Acid and Neural tube defects

What has shown to help parents in shared decision making?

"decision aids" - tools to provide information and help individuals clarify and share values Shown to help HCP and parents: Make better decisions Patients participate more, know more, more realistic expectations CPS Counselling and management for anticipated extremely preterm birth

Gestational age

# weeks from 1st d of last period

How much folate should we aim to add to flour in canada?

0.5-1mg daily intake CPS Folic Acid and Neural tube defects

what is the most important and common indicator of significant neurologic dysfunction in the neonatal period.

1. Neonatal Seizures Neo Notes

For newborn infants whose parents refuse an intramuscular injection of Vit K, the physician should recommend an oral dose of _____ vitamin K1 at the time of the first feeding. Use of the parenteral form of vitamin K for oral administration is all that is currently available. This should be repeated at ______ and ______.

2.0 mg two to four weeks six to eight weeks of age CPS Vit K

Canadian Pediatric Review 2018 Perinatal-Neonatal Medicine Which of the following is NOT an association or complication of LGA? 1. Birth trauma (dystocia, fractures, ICH, hip) 2. Asphyxia / HIE 3. Anemia 4. Hypoglycemia

3. Anemia

CPS recommendation: How should BG be tested?

A method that is quality controlled Accurate Reliable in range of 1-3mmol/L CPS Hypoglycemia

What infectious diseases does breast milk decrease the incidence of?

Bacterial meningitis Bacteremia Diarrhea Respiratory tract infections Otitis media UTI CPS baby friendly

What kind of light wavelength is necessary? CPS Hyperbili

Blue green CPS Hyperbili

ROP schedule

CPS ROP

Complications of surfactant administration

Complications - transient hypoxia, bradycardia and hypotension, blockage of ETT and pulmonary hemorrhage Neo Notes

Dx of HIE

Diagnosis - MRI is preferred because of its increased sensitivity and specificity early in the process and its ability to outline the topography of the lesion. Amplitude integrated EEG (aEEG) helps determine the infants at highest risk for significant brain injury. Neo Notes

What should you do in situations where the ten breastfeeding initiative rules cannot be met?

Example = nicu But breastmilk so important for them If not available, need to offer pasteurized human donor milk CPS baby friendly

What should you do for a mom who can't breastfeed?

Example: Surrogate Adoptive mother Just can't Need to still offer as many of the BFI practices as possible Do kangaroo care within an hour CPS baby friendly

Name 6 SSRIs CPS SSRI's Which are the most commonly used?

Fluoxetine Paroxetine sertraline CPS SSRI's

What metabolic diseases should not have breastfeeding?

Galactosemia PKU used to be contraindicated, but now, OKAY as long as supplement with low phenylalanine formula and monitoring phenylalanine levels CPS baby friendly

When should meconium aspiration syndrome children be given surfactant?

Intubated >50% oxygen CPS surfactant

seizures that are asynchronous twitching of several parts of the body (usually no EEG discharge)

Myoclonic Seizures: rapid nonrhythmic jerks - Multifocal Neo Notes

What are physiologic indicators that you should start iNO?

Oxygenation index 20-25 PaO2 <100mmHg despite 100% oxygen CPS iNO

Who needs donor breast milk the most?

Premature Those needing GI surgery CPS Milk banking

Prep for shared decision making meeting w/ parents

Preparation and setting - Speak with the mother's HCP, obtain all relevant information regarding maternal and fetal health - Ensure use of interpretive services by a professional translator, if required Create a comfortable environment • Make sure to talk with both parents, if feasible • Make sure the consult is not disturbed (e.g., turn pager to vibrate, close the door or curtains, let the nurse know) • Sit down, shake hands (if appropriate) and introduce yourself first, slowly and clearly • Demonstrate openness to communication with and involvement of parents • Ask about participants (e.g., use names, including the infant's name, if known, and if parents agree you can use it) Assess parental knowledge of prematurity issues, along with perspectives, concerns, expectations, needs and preferences • Ask what they know about prematurity • Make sure you understand and acknowledge their values, perspectives and concerns (e.g., cultural/social background, religious beliefs, family structure) • Adjust ways of communicating information to respect their values and preferences • Support their involvement in decision making with During consultation Ensure factors that are important to parents are discussed (e.g., Ask what is important to them) • Typically, parents want to know about likelihood of survival, risk of NDD (with related challenges and opportunities), what medical problems might be encountered, possible treatments, what a preterm baby looks like, what it is like to be in the NICU, what happens after birth, how to manage breastfeeding, and their own role in the NICU (with explanations) Discuss different choices or options • Present parents with the choices they need to make, clearly and accurately • Offer appropriate management options based on the clinical situation Share weighted and balanced information • Include both positive and negative aspects of care, pros and cons of the options, treatable and untreatable conditions • Disclose potential outcomes according to parental preferences: - Use grading words (majority, most, significant, some, a minority) and numbers when possible (6 out of 10, rather than 60%) - Use a consistent denominator when presenting different options, outcomes or event rates, to make the information easier to understand, interpret or compare (XX out of 10, 100 or 1000). For example, saying 'Out of 100 babies, 20 will die, 20 will survive with NDD, and 60 will survive with no NDD,' is better than saying 20 out of 100 for one outcome, then 1 out of 5 for another. Disclose uncertainty (i.e., the limits of statistics when applied to a particular baby) Show compassion and acknowledge parental distress • Reassure parents that they did not do anything to cause preterm birth • Confirm the uniqueness of their family and of the unborn baby • Acknowledge their baby as a being, not a GA • Provide value-neutral information (i.e., by including the positives of having an infant they can love and cherish) • Be honest Provide support and give parents realistic hope • Validate their situation as very difficult and their reactions as understandable • Tell them that every hour, day and week that the pregnancy continues (with baby and mom in stable condition) has positive effects • Make sure they know that they are not alone • Make sure they understand that you are there to provide more information and answer new questions • Invite them to write their questions down as they think of them, for next time • Meet with parents the following day, if possible, or at any time after the initial consultation CPS Counselling and management for anticipated extremely preterm birth

What should be two additional functions of the milk bank?

Promotion of breastfeeding in the community Human milk research CPS Milk banking

Recommendations by CPS a) what should we do about breastfeeding and societal thoughts? B) why?

Protect Promote Support Because it is the ideal form of nutrition for all infants/young children CPS baby friendly

What other "disabilities" are not accounted for? CPS Counselling and management for anticipated extremely preterm birth

Regulatory/behavioural difficulties ADHD Language delays Health related issues Hospital readmissions All happen more frequently to prems CPS Counselling and management for anticipated extremely preterm birth

What are risk factors for SEVERE hyperbili? (11)

Remember, not useful on own because very common. Need to use in conjunction w/ TSB. CPS Hyperbili

34. Term male, delivered by emergency C-section for fetal distress. Meconium present, suctioned below cords. Now needing 100% O2. Radial gas PaO2 is 80. Umbilical gas PaO2 is 40. What is the diagnosis? a) PPHN b) Meconium aspiration c) TGA

a) PPHN Radial gas = Preductal Umbilical gas = Postductal Difference >20 mmHg (or 5 percent oxygen sat) suggests R L shunting through ductus Primary Pulmonary Hypertension of the Newborn (PPHN): • Occurs in post-term, term or near-term infants • Characterized by severe hypoxemia o Without evidence of lung or structural heart disease o Often seen with asphyxia or meconium-stained fluid • Significant R L shunting through a PDA and through intrapulmonary channels • Diagnosis suggested by ductal differences and confirmed by Echo (ruling out structural congenital heart disease) • Treatment: supportive, most important therapy is assisted ventilation; inhaled NO (selective pulmonary artery vasodilating agent); may need ECMO A N18 p744 -a PaO2 gradient b/w a preductal (right radial artery) and a postductal (umbilical artery) site of blood sampling > 20 mmHg suggests right to left shunting through the ductus arteriosus, as does a saturation gradient > 5% b/w a preductal and postductal site PPHN ▪ Severe hypoxemia due to elevated pulmonary vascular resistance and PHT ▪ PPHN occurs in term and post-term infants. ▪ Predisposing factors include: o Birth asphyxia, meconium aspiration syndrome, early-onset sepsis, RDS, hypoglycemia, polycythemia, maternal use of nonsteroidal anti-inflammatory drugs with in utero constriction of the ductus arteriosus, maternal late trimester use of selective serotonin reuptake inhibitors, and pulmonary hypoplasia as a result of diaphragmatic hernia or Potters sequence, amniotic fluid leak, oligohydramnios, or pleural effusions. PPHN can also be idiopathic. ▪ In suspected cases must rule out cyanotic congenital heart disease particularly TAPVD. PATHOPHYSIOLOGY: ▪ Persistence of the fetal circulatory pattern of RL (pulmonary systemic) through the PDA & PFO after birth is due to excessively high pulmonary vascular resistance. ▪ After birth, PVR normally declines rapidly as a consequence of vasodilation secondary to: o gas filling the lungs, o a rise in postnatal Pao2, o a reduction in Paco2, o increased pH, o and release of vasoactive substances. ▪ Increased neonatal pulmonary vascular resistance may be: (1) Maladaptive from an acute injury (no normal vasodilation in response to increased oxygen and other changes after birth); (2) The result of increased pulmonary artery medial muscle thickness and extension of smooth muscle layers into the usually nonmuscular, more peripheral pulmonary arterioles in response to chronic fetal hypoxia; (3) Due to pulmonary hypoplasia (diaphragmatic hernia, Potter sequence); or (4) Obstructive as a result of polycythemia or TAPVR, or alveolar capillary dysplasia. ♣ Regardless of etiology, profound hypoxia from right-to-left shunting and normal or elevated Paco2 are present. ♣ PPHN should be suspected in all term infants who have cyanosis (with or without fetal distress), IUGR, meconium-stained amniotic fluid, hypoglycemia, polycythemia, diaphragmatic hernia, pleural effusions, and birth asphyxia PRESENTATION: ♣ Infants become ill in the delivery room or within the 1st 12 hr of life. ♣ PPHN related to polycythemia, idiopathic causes, hypoglycemia, or asphyxia may result in severe cyanosis with tachypnea, although, initially, signs of respiratory distress may be minimal. ♣ Infants who have PPHN associated with meconium aspiration, group B streptococcal pneumonia, diaphragmatic hernia, or pulmonary hypoplasia usually exhibit cyanosis, grunting, flaring, retractions, tachycardia, and shock. ♣ Multiorgan involvement may be present. ♣ Myocardial ischemia, papillary muscle dysfunction with mitral and tricuspid regurgitation, and biventricular dysfunction produce cardiogenic shock with decreased pulmonary blood flow, tissue perfusion, and oxygen delivery. ♣ The hypoxia is quite labile and often out of proportion to the findings on chest roentgenograms (X-rays). MANAGEMENT: ♣ Initial management includes oxygen administration and correction of acidosis, hypotension, and hypercapnia. ♣ Cardiac output is maintained with the use of inotropic agents and with judicious volume replacement. ♣ Endotracheal intubation and mechanical ventilation are almost always necessary for the newborn with PPHN. ♣ The goal of mechanical ventilation should be to maintain normal functional residual capacity (FRC) by recruiting areas of atelectasis, as well as to avoid overexpansion. ♣ In newborns with severe airspace disease who require high peak inspiratory pressures (ie, >30 cm water) or mean airway pressures (>15 cm water), consider HFV to reduce barotraumas and associated air leak syndrome. ♣ When HFV is used, the goal should still be to optimize lung expansion and FRC and to avoid overdistension. ♣ Surfactant therapy may be a consideration particularly in patients with MAS. ♣ Because of their lability and ability to fight the ventilator, newborns with PPHN require sedation usually in the form of morphine or fentanyl, often in combination with a benzodiazepine. ♣ The use of paralytic agents is controversial and reserved for the newborn that cannot be treated with sedatives alone. Muscle relaxants may promote atelectasis of dependent lung regions and ventilation-perfusion mismatch. ♣ Nitric oxide, produced endogenously in endothelial cells at the time of transition from fetal to neonatal life, diffuses to the vascular smooth muscle cell, where it increases the activity of soluble guanylate cyclase. ♣ This leads to the intraconversion of guanosine triphophate to cyclic guanosine monophosphate (cGMP), which causes smooth muscle relaxation leading to pulmonary vasodilation. ♣ Similarly, exogenous inhaled nitric oxide (iNO) diffuses from the alveolus to the smooth muscle cells with similar effects. ♣ iNO is then rapidly bound and inactivated by reduced hemoglobin in the vascular space, thus avoiding concomitant reductions in systemic blood pressure. ♣ Treatment with iNO is indicated for newborns with an oxygen index (OI) of 25 or more. ♣ In 2 large, randomized trials, NO reduced the need for ECMO support by approximately 40%. ♣ Contraindications to iNO include congenital heart disease characterized by left ventricular outflow tract obstruction (eg, interrupted aortic arch, critical aortic stenosis, hypoplastic left heart syndrome) and severe left ventricular dysfunction. ♣ The optimal starting dose is 20 ppm. ♣ Higher doses are associated with side effects including methemoglobinemia and increased levels of nitrogen dioxide, a pulmonary irritant. ♣ Most newborns require iNO for less than 5 days. ♣ The dose can be weaned to 5 ppm after 6-24 hr of therapy. ♣ The dose can then be weaned slowly and discontinued when the Fio2 is <0.6 and the iNO dose is 1 ppm. ♣ Abrupt discontinuation should be avoided as it may cause rebound pulmonary hypertension. ♣ Therapy with continuous inhaled or intravenous prostacyclin (PGI2) has improved oxygenation and outcome in infants with PPHN. ♣ PGI2 is a prostaglandin member of the family of lipid molecules known as eicosanoids produced in endothelial cells by the actions of prostacyclin synthase. ♣ It inhibits platelet activation and is also an effective vasodilator. ♣ Its vasodilatory properties come from its ability to raise cAMP and subsequent Protein Kinase levels ♣ When given centrally as a continuous infusion, or given in an inhaled formulation PGI2 leads to smooth muscle relaxation and vasodilation. ♣ However the role of PGI2 is mostly limited to PAH, when the pulmonary hypertension is secondary to a fixed vasculatory lesion (e.g. narrowed pulmonary arteries) rather than delayed transition as in PPHN. Extracorporeal Membrane Oxygenation: ♣ In 5-10% of patients with PPHN, the response to 100% oxygen, mechanical ventilation, and drugs is poor. ♣ In such patients, the OI has been used to predict mortality rates >80%. ♣ An OI > 40 that is unresponsive to nitric oxide inhalation predict a high mortality rate and are indications for ECMO: OI = (MAP × Fio2 × 100) ÷ Postductal Pao2 ♣ ECMO is prolonged cardiopulmonary bypass that is used to treat newborn infants (<1 week old) with reversible pulmonary disease that has been complicated by PPHN. ♣ Although overall survival is about 70% to 80%, it varies by diagnosis, with rates of more than 90% for MAS, 75% for sepsis, and 50% for CDH. ♣ Because ECMO requires complete heparinization to prevent clotting in the circuit, and there is therefore a major risk of systemic bleeding and ICH with LTND, baseline criteria for newborns considered for ECMO are generally as follows: o Gestation of more than 34 weeks o Weight more than 2000 g o No major intracranial hemorrhage on cranial sonograms (i.e. larger than a grade II hemorrhage) o Reversible lung disease or mechanical ventilation for 7-14 days o No pulmonary hemorrhage o No history of severe asphyxia o No evidence of lethal congenital anomalies or inoperable cardiac disease ♣ Complications of ECMO include: o thromboembolism, air embolization, bleeding, stroke, seizures, atelectasis, cholestatic jaundice, thrombocytopenia, neutropenia, hemolysis, infectious complications of blood transfusions, edema formation, and systemic hypertension. ♣ Under normal conditions, vascular smooth muscle cGMP is degraded by phophodiesterase-5 (PDE-5). ♣ As an inhibitor of PDE-5, Sildenafil (Viagra) prolongs the half-life of cGMP and by so doing could prolong the vasodilating effects of NO. N19 (Chpt on PPHN), PS p475-476, SK p584-585, EMed - Persistent Newborn Pulmonary Hypertension, WIKI - PPHN

43. 33 week premature infant is born to a mother with hypertension. Baby is SGA. What other associated findings do you expect? a) Polycythemia b) Hyperglycemia c) Hypomagnesemia d) Hypercalcemia

a) Polycythemia - Nelson's: Maternal hypertension -> symmetric IUGR if severe/acute, asymmetric if chronic, intrauterine fetal demise via fetal hypoxia and placental insufficiency - Polycythemia occurs in settings of chronic arterial oxygen desaturation; neonatal - can occur in normal environment, twin-to-twin transfusion, infant of diabetic mother, SGA, infants of hypertensive mothers.. Not b) Hyperglycemia (SGA, premature baby more likely to be hypoglycemic) c) Hypomagnesemia d) Hypercalcemia (LBW infants typically require calcium supplementation to avoid metabolic bone disease) Polycythemia • Hct 65% (centrally, peripheral heelstick Hct falsely elevated) • Increased risk: high altitudes, post-term infants, SGA, recipient of twin-twin transfusion, delayed clamping of umbilical cord, IDM or infants of hypertensive mothers, Trisomies, neonatal Graves disease, hypothyroidism, Beckwith-Wiedemann **Infants of diabetic or hypertensive mothers and those with growth restriction may have to chronic fetal hypoxia, which stimulates erythropoietin production and increases RBC production. • Clinical manifestations irritability, lethargy, tachypnea, respiratory distress, cyanosis, feeding disturbances, hyperbilirubinemia, hypoglycemia, and thrombocytopenia o Severe complications: seizures, stroke, pulmonary HTN, NEC, renal vein thrombosis, renal failure • Treatment partial exchange transfusion with NS (if Hct > 70-75% or sings of hyperviscosity) • Prognosis unclear, reported adverse outcomes include speech deficits, abnormal fine motor control, reduced IQ, school problems o Unclear whether partial exchange transfusion improves the long-term outcome o Most asymptomatic infants develop normally. Problems of IUGR and SGA: ♣ Intrauterine fetal demise ♣ Temperature instability ♣ Perinatal asphyxia ♣ Hypoglycemia ♣ Polycythemia-hyperviscosity ♣ Reduced oxygen consumption/hypothermia ♣ Dysmorphology ♣ Pulmonary hemorrhage EN p216 IUGR is present when fetal growth stops and, over time, declines to <5th%ile of growth for gestational age or when growth proceeds slowly, but absolute size remains <5th%ile. Fetal causes (reduce innate growth potential): fetal rubella infection; primordial dwarfing syndromes; chromosomal abnormalities; congenital malformation syndromes. Placental causes: congenital infection; placental tumors; chronic abrutio placentae; twin-to-twin transfusion syndrome; placental insufficiency. Maternal causes: include severe peripheral vascular disease that reduce uterine blood flow (chronic hypertension; diabetic vasculopathy; preeclampsia/eclampsia); reduced nutritional intake; alcohol; cigarette smoking; drug abuse; uterine constraint; reduced weight gain during pregnancy EN p216 ♣ Terms IUGR and SGA are not synonymous. ♣ IUGR represents a deviation from expected growth patterns secondary to an adaptation to unfavorable intrauterine conditions that result in permanent alterations in metabolism, growth and development ♣ SGA describes an infant whose BW is statistically <10th%ile or 2 standard deviations below the mean BW for gestational age EN p236

75. Neonate with rash at 36 hours of life a) erythema toxicum b) millaria c) HSV d) Pustulosis

a) erythema toxicum

12. Match the following feeding options with the presentation. You can use each one more than once. Breastfeeding ad lib, protein hydrolysate formula; formula with decreased lipids; NG feed of standard calorie formula a. Chylothorax b. Babe with Cow's milk protein allergy (CMPA) c. Babe admitted to intensive care, decreased LOC d. Babe living in Africa with a mother who has HIV

a. Chylothorax - formula with decreased lipids - pleural collection of fluid formed by escape of chyle from thoracic duct or lymphatics into thoracic cavity - occurs most often as complication of cardiothoracic surgery, or with chest injury/intrathoracic malignancy/congenital anomalies of lymphatic system; refractory chylothorax in fetus associated with missense mutation in intengrin a-alpha-9 - symptoms similar to pleural effusion (respiratory distress) - spontaneous recovery occurs in >50% of cases of neonatal chylothorax - treat initially with enteral feeding with low fat or medium-chain triglyceride, high-protein diet, or parenteral nutrition; if no resolution in 1-2 weeks, TPN - thoracentesis as needed to relieve pressure b. Babe with CMPA - protein hydrolysate formula - food protein-induced proctocolitis - continued breastfeeding if mother willing to eliminate food from her diet (all dairy, then maybe soy protein as well) - or extensively hydrolyzed formula c. Babe admitted to intensive care, decreased LOC - NG feed of standard calorie formula - need adequate nutrition to heal; no clear reason for special formula; can't BF ad lib d. Babe living in Africa with a mother who has HIV - breastfeeding ad lib - Nelson's: Although most effective way to prevent postpartum transmission would be to eliminate BF altogether, increasing evidence that early weaning may not be safe in resource constrained settings, due to high risk of malnutrition and diarrhea in formula-fed infants - WHO recommends exclusive BF for at least 6 months in resource-limited settings, unless there is affordable/sustainable replacement feeding option

1. 30 wk SGA prem, now 2 wks old in NICU, baby presents with septic picture. a. List 3 bacteria that could be responsible: b. Initial choice of antibiotic treatment.

a. List 3 bacteria that could be responsible: Coagulase-negative Staphylococcus, S aureus, E. coli b. Initial choice of antibiotic treatment. Vancomycin and Gentamycin (?Vancomycin and Gentamycin to cover for Staph - most common nosocomial infection) Nosocomial - CONS. S. aureus, E. coli a. List 3 bacteria that could be responsible: • Coagulase negative Staph • Staph Aureus • Group B Strept b. Initial choice of antibiotic treatment: Ampicillin and Gentamicin (can also consider use of Cloxacillin/Vancomycin; or Cefotaxime

1. Newborn has significant respiratory distress and CXR that is consistent with pneumonia. She is ventilated with PIP 36, PEEP 5 rate 60 bpm, FiO2 1.0, she is not saturating very well. a. What is the likely diagnosis? b. What intervention should you start now?

a. What is the likely diagnosis? PPHN b. What intervention should you start now? iNO

1. Why can't you cool a neonate? a. prematurity b. ph 6.9 c. apgars

a. prematurity

tx of ______ may lessen severity of RDS

acidosis, hypoxia, hypotension and hypothermia Neo Notes

Readiness for intestinal feeds

active bowel sounds, passage of meconium, and the absence of abdominal distention, bilious gastric aspirates, and emesis • Oral feeding requires: strong suck, coordinated swallowing, epiglottal and uvular closure of the larynx and nasal passages, and normal esophageal motility (usually by 34 weeks) • Trophic feeds: enhanced gut motility, improved growth, decreased need for parenteral nutrition, fewer episodes of sepsis, and shortened hospital stay • Gradually advancing feeds reduces NEC Neo Notes

Premature infants with NEC who require surgical intervention or who have concomitant bacteremia are at increased risk for ______

adverse growth and neurodevelopmental outcome. Neo Notes

Surfactant deficiency pathophysiology

alveolar atelectasis, hyaline membrane formation and interstitial edema decreased lung compliance Neo Notes

o Spasms, focal clonic or tonic, and generalized myoclonic seizures _____ associated with electrographic discharges (epileptic seizures)

are, as a rule, Neo notes

76. Full term baby delivered to an O+ Mom. Looks well but pale. Hgb is 70, he is hemodynamically stable. What is the most likely diagnosis? a) ABO incompatibility b) Chronic fetal maternal hemorrhage c) Rh incompatibility

b) Chronic fetal maternal hemorrhage - must be chronic with hemodynamic stability Not a) ABO incompatibility - rarely causes significant anemia (Nelson's) c) Rh incompatibility • Baby is pale, NOT jaundiced therefore blood has left the baby fetal maternal hemorrhage o Occurs in 10-15 percent of pregnancies o Presents with chronic blood loss in utero, marked pallor, less distress, low Hb with microcytic indices, and if severe heart failure

48. Which of the following is the best predictor of risk of Rh autoimmune hemolytic disease at the time of delivery? a) Bili in the cord b) Hgb in the cord c) Mom's anti-Rh titers d) Gestational age

b) Hgb in the cord vs a) bili in the cord Nelson's: - cord blood hemoglobin content varies and is usually proportional to the severity of the disease (but may be within normal range because of compensatory bone marrow and extramedullary hematopoiesis) - Rh titer should be tested in at-risk women at 12-16, 28-32, and 36 weeks of gestation; presence of elevated antibody titers at the beginning of pregnancy, rapid rise in titer, or a titer of 1:64 or greater suggests significant hemolytic disease, but exact titer correlates poorly with severity of disease - cord bilirubin usually between 3-5mg/dL, bilirubin is cleared by placenta so generally only rises significantly after birth Rh Autoimmune hemolytic disease • Isoimmune hemolytic disease from D antigen o Rh positive fetal blood (inherited from father) Rh negative mother ♣ Causes anti Rh antibody formation (sensitization) ♣ IgG crosses the placenta and causes hemolysis in fetus • Clinical manifestations vary from only laboratory evidence of mild hemolysis to hydops fetalis o Jaundice may be absent at birth (placental clearance) ♣ Severe cases bilirubin pigments stain the amniotic fluid, cord and vernix • Cord levels of bilirubin reflect severe hemolysis ♣ Generally becomes evident on the first day of life, risk of kernicterus o Laboratory Data: ♣ Direct Coombs test is usually positive ♣ Anemia present ♣ Cord blood hemoglobin content varies and is usually proportional to the severity of disease ♣ Cord bilirubin is generally elevated • Antenatal diagnosis = maternal titer of IgG antibodies to D antigen, determination of fetal Rh status (isolating fetal cells from maternal circulation); monitor severity via Doppler u/s of middle cerebral artery • Postnatal diagnosis = baby of any Rh negative woman should have blood tests for ABO blood group, Rh type, Hct and hemoglobin, and DAT. If positive DAT do baseline serum bilirubin • Treatment: o In utero transfusions o Supportive care at delivery (assessment of need for transfusion) o Exchange transfusion - indication controversial o Close monitoring (bloodwork every 4-6 hrs) o IVIG may reduce hemolysis, peak serum bilirubin levels and need for exchange (as well as decrease the duration of PT and length of stay) • Late Complications: o Monitor for development of anemia and cholestasis (inspissated bile syndrome) o Portal vein thrombosis • Prevention of Rh Sensitization: o Intramuscular injection of 300 mcg of human anti-D globulin (RhoGAM) within 72 hours of delivery of an Rh-positive infant, etctopic pregnancy, abdominal trauma, amniocentesis, CVS or abortion ♣ Eliminates potentially antigenic fetal cells from maternal circulation o Dose at 28-32 weeks and again at birth is more effective than single dose C N19 p615 N18 p768 Hemolytic Disease of the Newborn caused by Rh Incompatability ♣ Rh type directs the production of a number of blood group factors (C, c, D, d, E, e) ♣ 90% are due to D antigen ♣ 3X more common in caucasians ♣ Ab formation against D antigen may be induced in the unsensitized Rh-negative recipient mother ♣ Once sensitization has taken place, smaller doses of antigen stimulate an increase in Ab titer ♣ Concerningifhighatstartofpregnancy,rapidriseintitre,titreof greaterthan1:64 ♣ IgG crosses the placenta hemolytic manifestation ♣ Rarely occurs during a 1st pregnancy - maternal-fetal transfusion occurs near the time of delivery, too late for the mother to become sensitized and transmit antibody to her infant before delivery ♣ 55% of Rh + fathers are heterozygous (D/d) and may have Rh - offspring ♣ Maternal-fetal transfusion only occurs in 50% of pregnancies ♣ Capacity of Rh - women to produce Abs is variable ♣ Overall incidence of isoimmunization is low ~ 10% ♣ When mother and fetus have ABO incompatability, mother is partially protected against sensitization by the rapid removal of Rh + cells from her circulation by her pre-existing anti-A or anti-B which are IgM Abs and do not cross ♣ Once a mother has been sensitized her infant is likely to have hemolytic disease ♣ Severity of Rh illness worsens with successive pregnancies ♣ 1st affected infant after sensitization my represent the end of the mother's childbearing potential argues for RhoGAM after delivery of each Rh positive infant, successful strategy to reduce Rh hemolytic disease ♣ Severity of disease ranges from only mild lab evidence of hemolysis to severe anemia, cardiac decompensation, massive HSM, anasarca, circulatory collapse ♣ Hydrops fetalis = excessively abnormal fluid in 2 or more fetal compartments ♣ Non immune conditions are a frequent cause of hydrops since RhoGAM ♣ Severity of hydrops related to anemia severity and reduction in albumin due in part to hepatic dysfunction ♣ Jaundice may be absent at birth b/c of placental clearance of lipid-soluble unconjugated bilirubin ♣ Risk of bilirubin encephalopathy and kernicterus after delivery b/c of rapid rise in 1st day ♣ Infants born after intrauterine transfusion usu have very high (but extremely variable) cord levels of bilirubin, which reflects the severity of the hemolysis and effects on hepatic function ♣ DAT usually + ♣ Elevated Ab titers suggest significant hemolytic disease, although the exact titer correlates poorly with the severity of disease ♣ U/S, PUBS (percutaneous umbilical blood sampling) - progressin of disease ♣ Hydrops is present when fetal hemoglobin is < 5 g/dL ♣ Amniocentesis - bilirubin is cleared by the placenta, significant proportion enters the amniotic fluid ♣ Immediately after birth, cord blood for ABO, Rh type, Hgb, DAT ♣ Rx goals: 1. Prevent death from severe anemia; 2. Avoid neurotoxicity from hyperbilirubinemia o in utero transfusion o transfusion at delivery o immediate full or partial exchange transfusion o serum bilirubin immed and at 4-6 hr intervals o careful monitoring of the serum bilirubin until a falling trend has been demonstrated w/o phototherapy

10. 2 day old baby with lots of oral secretions difficulty feeding. Best test to confirm diagnosis? a) CXR b) Insertion orogastric tube c) Abdominal ultrasound

b) Insertion orogastric tube TEF • Esophageal atresia: most common congenital anomaly of the esophagus o >90% have an associated tracheoesophageal fistula • Fifty percent of infants are nonsyndromic without other anomalies o associated anomalies: VATER or VACTERL • Infants with EA become symptomatic immediately after birth: excessive secretions that cause drooling, choking, respiratory distress, and the inability to feed. • Reflux of gastric contents through the TEF results in aspiration pneumonia • H-Type may be asymptomatic in the newborn period: later, prolonged history of mild respiratory distress associated with feeding or recurrent episodes of pneumonia. • Diagnosis: o attempting to pass a catheter into the stomach - cannot be passed further than 10 to 15 cm. ♣ confirmed with CXR showing catheter curled in the upper esophageal pouch. ♣ A distal TEF often can be seen on a lateral chest radiograph; both views will reveal a gas-filled gastrointestinal tract. o If uncertain or suspect proximal TEF: water-soluble contrast material placed in the esophageal pouch under fluoroscopic guidance will confirm the presence of EA. The contrast material must be removed immediately to avoid regurgitation and aspiration. a. CXR - won't be abN w/o ng inserted b. Insertion orogastric tube - failure to insert best test to confirm esophageal pouch c. Abdominal ultrasound - won't help dx of TEF; may see a dilated stomach EN p479 ♣ Esophagus and trachea develop in close proximity to each other during weeks 4-6 of fetal life. ♣ Defects in the mesenchyme separating the 2 structures result in TEF, often in association with other anomalies (involving the kidneys, heart, spine, or limbs) ♣ Occurs in about 1:3000 live births. ♣ Most common forms of TEF occur with esophageal atresia (EA); the H-type TEF w/o atresia is uncommon, as is EA w/o TEF. ♣ Associated defects include the VACTERL association: vertebral anomalies (70%), anal atresia/imperforate anus (50%), cardiac anomalies (30%), TEF (70%), renal anomalies (50%), and limb anomalies (polydactyly, forearm defects, absent thumbs, syndactyly) (70%). ♣ A single artery umbilical cord is often noted. EN p478 Presentation: History of maternal polyhydramnios; excessive neonatal oral secretions; choking /cyanosis with feeding; unable to pass catheter into the stomach (except H-type); recurrent aspiration pneumonia (H-type) Diagnosis is based on inability to pas NG/OG tube into the stomach (except H-type) SK p349 XR -coiled feeding tube in the esophageal pouch, air distended stomach - pure EA airless scaphoid abdomen Management: NPO; Replogle tube set to low continuous suction; maintenance IV fluids; antibiotics if aspiration pneumonia present; prone positioning preferable; surgical ligation w/ end to end anastomosis or possibly neoesophagus Complications: anastomotic leak; refistulization; anastomotic stricture; GERD; reactive airways disease; tracheomalacia SK p351

2. Infant born at 37 wks. No risk factors and Coombs neg. Now 30 hrs and bili 277. Provided with graphs from CPS hyperbili station. a) send home with follow up b) intensive photo and recheck bili in 2-6 hour c) prepare for exchange transfusion

b) intensive photo and recheck bili in 2-6 hour Exchange transfusion for hyperbilirubinemia (HBR) in healthy full term infants without eveidence of hemolysis is almost never required - PS p453 PHYSIOLOGIC JAUNDICE (ICTERUS NEONATORUM): The level of indirect bilirubin in umbilical cord serum is 15-50 mmol/L and rises at a rate of <75 mmol/L/24 hr; thus, jaundice becomes visible on the 2nd-3rd day, usually peaking between the 2nd and 4th days at 75-100 mmol/L and decreasing to below 35 mmol/L between the 5th and 7th days of life. Jaundice associated with these changes is designated physiologic and is the result of increased bilirubin production from the breakdown of fetal red blood cells combined with transient limitation in the conjugation of bilirubin by the immature neonatal liver. A search to determine the cause of jaundice should be made if 1. it appears in the 1st 24-36 hr of life 2. serum bilirubin is rising at a rate faster than 75 mmol/L/24 hr 3. serum bilirubin is >200 mmol/L in full-term infants (especially in the absence of risk factors) or 170-200 mmol/L in preterm infants 4. jaundice persists after 10-14 days of life 5. direct bilirubin is >30 mmol/L at any time. Other factors suggesting a nonphysiologic cause of jaundice are: ♣ family history of hemolytic disease ♣ pallor ♣ hepatomegaly ♣ splenomegaly ♣ failure of phototherapy to lower bilirubin ♣ vomiting, lethargy, poor feeding or excessive weight loss ♣ apnea +/- bradycardia ♣ abnormal vital signs ♣ hypothermia ♣ light-colored stools and dark urine ♣ signs of kernicterus PATHOLOGIC HYPERBILIRUBINEMIA: occurs when timing, duration, or pattern varies significantly from physiologic jaundice or if the infant is at special risk for neurotoxicity. The greatest risk is the development of kernicterus (bilirubin encephalopathy with neuronal necrosis of basal ganglia and brainstem nuclei), which is dependent on the level of indirect bilirubin, duration of exposure to elevated levels, the cause of jaundice, and the infant's well-being. Symptoms would include: lethargy, hypotonia (with hypertonia later), high pitched cry, opisthotonus, and seizures. Late sequale include: hearing deficits, oculomotor disturbances (choreathetoid movements), developmental delay, athetoid CP, and dental dysplasia. RF for Jaundice ♣ Prematurity ♣ Sibling with severe HBR ♣ Bruising ♣ Cephalohematoma ♣ Dehydration ♣ Asian decent ♣ Exclusive breastfeeding ♣ IDM ♣ G6PD deficiency ♣ Maternal blood group O or Rhesus negative Adverse effects of Phototherapy ♣ Loose stools ♣ Increased insensible water loss ♣ Skin rashes ♣ Overheating ♣ Potential for burns ♣ Bronze baby syndrome ♣ Potential for retinal damage Contraindications to Phototherapy: Family history of light sensitive porphyria. Presence of direct HBR is only a relative contraindication (decreased effectiveness, may precipitate Bronze Baby Syndrome) SK p588-590, PS p450-453, EN p247-249

Breastfeed decreases risk of what infections

bacterial meningitis, bacteremia, diarrhea, respiratory tract infections, otitis media and urinary tract infections CPS baby friendly

8. Baby with jaundice, low platelet count, looks septic. What is the cause? (Lab values given) a) TORCH infection b) Galactosemia c) Bacterial sepsis

c) Bacterial sepsis • Presentation of TORCH: IUGR, prematurity, hepatosplenomegaly and jaundice, blueberry muffin-like rash, thrombocytopenia and purpura, and microcephaly and intracranial calcifications, Chorioretinitis, sensorineural hearing loss, may present with sepsis-like picture • Presentation of galactosemia: jaundice, lethargy, hepatic failure, e coli sepsis

55. Abstinence from methadone? Neonatal sx? a) Hyporeflexia b) Constipation c) Sneezing d) Lethargy

c) Sneezing Wakefulness Irritability Tremulousness, temperature instability, tachypnea Hhyperactivity, high-pitched cry, hyperacusis, hyperreflexia, hypertonus Diarrhea, diaphoresis, disorganized suck Rub marks, respiratory distress, rhinorrhea Apneic attacks, autonomic dysfunction Weight loss or failure to gain weight Alkalosis (respiratory) Lacrimation (Sneezing, seizures, sweating) - SK p600 PS p427 Neonatal abstinence Syndrome • Drugs of abuse are LMW & lipophilic, thus cross placenta & accumulate in fetus/amniotic fluid Opiates Cocaine Alcohol Mechanism Bind to opioid receptors Symptoms IUGR & perinatal distress, immediatly after delivery OR up to 1-2 weeks of age (most have signs at 2-3 days of life) If exposed to methadone, withdrawal symptoms may occur beyond 2 weeks of age Variable clinical course Restlessness, agitation, tremors, wakefulness, feeding problems (may persist up to 3-6 mos of age) incidence of RDS & Hyperbili risk of SIDS, strabismus, slow growth (most catch up by 1-2yr) Mechanism Prevents reuptake of NTs & affects permeability of Na ions Symptoms Vasoactive properties for fetal cerebral perfusion, placental flow, causes maternal HTN Irritability, hypertonia, tremor, high pitched cry, hyperreflexia, frantic fist sucking, feeding problems, sneezing, tachypnea, abN sleep patterns A specific cocaine withdrawal pattern has not been described **The symptoms above may represent cocaine intoxication, rather than withdrawal** Growth catches up, but not HC!! Mechanism Anxiolytic, analgesic, depressant Increases congenital anomalies & impaired intellect Symptoms General tremors, irritability, hypertonicity, twitching, tachypnea, hyperacusis, opithotonus, seizures, sbdominal distention, vomiting Facial microcephaly, microphthalmos, short palpebral fissures, flat philtrum, thin upper lip, hypoplastic maxilla Other irritabily in infancy, hyperactivity in childhood Barbituates Benzodiazepines PCP Similar to narcotic withdrawal, but appear later (1-2 weeks of life) Symptoms last 2-6 weeks Similar to narcotic withdrawal, including seizures, and appear shortly after birth Similar to narcotic withdrawal, appear within 24 hrs of birth Marijuana May have shorter gestation & lower BW, no evidence for profound dysfunction • Associated Risk factors for maternal drug abuse o Low SES, Poor antenatal care, Teeneage or single mothers, Poor education • Associated conditions o Infxn (HIV positive serology), polysubstance use, poor nutrition, anemia • Associated Obstetrical Factors o Premature delivery, PROM, chorioamnionitis, fetal distress, IUGR • Complications associated with the drug itself o HTN, abruptio placenta, arrhythmias, ischemia/infarcts, stroke, respiratory arrest, fetal demise (THESE ARE WITH COCAINE) • Signs & Symptoms - CNS irritability, altered neurobehavioral organization, abN sympathetic activation o Hyperirritability (DTR's, Hypertonia, Hyperacusis, Tremors, High pitched cry) o Seizures o Wakefulness o Poor suck & swallow o Regurgitation/emesis o Loose stool o Apnea o Tachypnea o Yawning/Hiccups o Sneezing/stuffy nose o Mottling o Fever o Failure to gain weight o Lacrimation • Diagnostic Tests Urine Meconium Hair Reflects last few days before pregnancy False Negatives dilution (low SG) high NaCl (high SG) False Positives codeine detected as morphine ranitidine, ephedrine detected as amphetamines Drugs may be present up to 3 days after delivery More sensitive than urine Reflects a longer time frame prior to delivery Needs to be processed before testing (lab burden) Grows at 1-2cm/month (mom's hair can be tested over long time period) If used from the infant, represents the last trimester Same as stool, needs to be processed before testing (lab burden) • Treatment o Goal is to reduce symptoms, restore normal sleep and feeding patterns, minimize duration of exposure and dosage of therapeutic agent o Minimal stimulation (dark, quiet area) o Swaddling & positioning o Prevent excessive crying (pacifier, feed on demand) o Drug treatment ♣ Treatment with same class of drug as that causing the withdrawal • Oral morphine: first line in opioid withdrawal; use 0.02 mg/kg q4h • Phenobarbital: second line in opioid withdrawal; first line in multidrug use • Diazepam used only as an adjunct to other medications

Poor prognostic factors in BPD

cor pulmonale, IVH, prolonged ventilation, pulmonary hypertension, O2 need at 1 yr Neo Notes

Variable deceleration

cord compression Neo Notes

Injury to the spine/spinal cord in neonatal period Apnea on ____ and poor motor recovery by ____ are poor prognostic signs.

day 1 3 mo Neo notes

In hemolytic dz of the NB The presence of _____ at the beginning of pregnancy, _____ in titer, or a titer of ____ or greater suggests significant hemolytic disease.

elevated antibody titers a rapid rise 1:64 Neo Notes

Clinical manifestations of neonatal polycythemia include

irritability, lethargy, tachypnea, respiratory distress, cyanosis, feeding disturbances, hyperbilirubinemia, hypoglycemia, and thrombocytopenia. Severe complications include seizures, stroke, pulmonary hypertension, necrotizing enterocolitis, renal vein thrombosis, and renal failure. Many affected infants are asymptomatic. Neo Notes

intrapartum antibiotic prophylaxis _____ recommended when a caesarean section is performed before onset of labour when _____

is not membranes are intact, regardless of GBS status. It should be noted that IAP does not reduce the incidence of late-onset GBS disease CPS term infants at increased risk for early onset bacterial sepsis

Intubated newborn infants with pulmonary hemorrhage which _______ should receive exogenous surfactant therapy as one aspect of clinical care

leads to clinical deterioration CPS surfactant

• Low Birth Weight:

less than 2500g Neo Notes

In neonates, Hirschsprung disease must be differentiated from ______

meconium plug syndrome, meconium ileus, and intestinal atresia. Neo Notes

CXR of RDS

o CXR - characteristic fine reticular granularity of the parenchyma + air bronchograms Neo Notes

CDH associated Chromosomal syndromes

o Chromosomal syndromes: trisomy 21, trisomy 13, trisomy 18, Fryn, Brachmann-de Lange, Pallister-Killian, and Turner. Neo notes

o Four distinct pathologic stages of classic BPD:

o Four distinct pathologic stages of classic BPD: o acute lung injury, exudative bronchiolitis, proliferative bronchiolitis, and obliterative fibroproliferative bronchiolitis. Neo Notes

Outcomes in ROP

o In >90% of at-risk infants: spontaneous arrest and regression, with little or no residual effects or visual disability. o Less than 10% of infants progress to severe disease, with significant extraretinal vasoproliferation, cicatrization, detachment of the retina, and impairment of vision. Neo Notes

• Pneumopericardium

o May be asymptomatic, but usally manifests as sudden shock with tachycardia, muffled heart sounds and decreased pulses ♣ TAMPONADE Neo Notes

Mechanical ventilation goals for BPD/RDS management

o Mechanical ventilation: ♣ Allow hypercapnia with pH >7.20 and a Pao2 of 50-70 mm Hg with an oxygen saturation of 90-95%. Neo Notes

seizures that affect the flexor muscles of the upper extremities ( may see EEG changes)

o Myoclonic Seizures: rapid nonrhythmic jerks Focal : Neo notes

How does phototherapy work

o Phototherapy: converts bilirubin to lumirubin which can be extreted by the kidneys in the unconjugated state Neo Notes

Protective factors in BPD

o Protective factors: Vitamin A supplementation , Early use of nasal CPAP and rapid extubation Neo Notes

Risk factors for ROP

o Risk factors: prematurity, Oxygenation, respiratory distress, apnea, bradycardia, heart disease, infection, hypercarbia, acidosis, anemia, and the need for transfusion Neo Notes

The foreskin can also become entrapped behind the glans

paraphimosis CPS circumcision

MECONIUM PLUGS are associated with _____

small left colon syndrome in IDM, CF, rectal aganglionosis, maternal opiate use, and magnesium sulfate therapy for preeclampsia The plug may be evacuated by glycerin suppository or rectal irrigation saline. Neo Notes

Fetal maturity may be estimated by determining the

surfactant content of amniotic fluid. Neo Notes

The onset of apnea in a previously well premature neonate after _____ or in a term infant at any time is a critical event that warrants immediate investigation

the 2nd wk of life neo notes

Perinatal brachial plexus palsy (PBPP) - If physical examination shows incomplete recovery by ______, full recovery is unlikely.

three to four weeks CPS Perinatal brachial plexus palsy

o Subtle, generalized tonic and other myoclonic seizures are _____ discharges and thus are thought to usually represent release phenomena with abnormal movements secondary to brain injury rather than true epileptic seizures.

usually not associated with Neo Notes

The predominant cause of antepartum fetal distress is _______

uteroplacental insufficiency Neo Notes

Alagille syndrome

♣ Alagille = intrahepatic bile duct paucity + butterfly vertebrae + tubulointerstitial nephropathy + peripheral pulmonic stenosis Neo Notes

Surfactant administration Improves _____ but does not

♣ Improves survival and reduces air leak but has not consistently reduced the incidence of BPD Neo Notes

Indications for RBC transfusion in newborns Anemia

'Top-up' transfusions should be used to maintain hemoglobin levels >75 g/L in convalescent preterm infants For infants in the first and second week of life, minimum hemoglobin levels of 100 g/L and 85 g/L, respectively, are recommended. Infants needing respiratory support may require transfusion at higher hemoglobin thresholds Anemia: Anemic hypoxia occurs when the level of circulating hemoglobin is inadequate to meet the oxygen demands of tissue. The circulation's ability to deliver oxygen to tissue is a function of blood flow, along with the amount and functional capabilities of hemoglobin. The level of hemoglobin at which anemic hypoxia occurs can depend on the functional ability of RBCs or of hemoglobin to accept, transport and unload oxygen at tissue oxygen tensions, functions which are, in turn, affected by RBC physiology, the nature of the hemoglobin itself and the environment in which it functions.[24] The RBCs and hemoglobin of stored blood are functionally impaired; some dysfunctions improve with time in the recipient's circulation. Therefore, the point at which any infant is functionally anemic post-transfusion cannot be defined easily by blood hemoglobin content alone. CPS Red blood cell transfusion in newborn infants

detection of the 1st audible fetal heart tones ______, and initial fetal movements _____ are also helpful.

(16-18 wk) (18-20 wk) Neo Notes

Fetal bradycardia

(<120 beats/min) fetal hypoxia, maternal anesthetic or β-adrenergic blocker, and heart block ± CHD Neo Notes

Fetal Tachycardia

(>160 beats/min) early fetal hypoxia, maternal fever, maternal hyperthyroidism, maternal β-sympathomimetic or atropine therapy, fetal anemia, infection, and some fetal arrhythmias. Neo Notes

POLYHYDRAMNIOS

(>2L or AFI >24cm, 1-5%) = 2 (L) much fluid 4 the baby who is open (spina bifida/cleft, hernia), tri-ing (tri 18/21) to be small (achondroplasia), has been klip'd (klippel-feil) and is failing (HF = hydrops etc.) Congenital anomalies: Anencephaly, hydrocephaly, TEF, duodenal atresia, spina bifida, cleft lip or palate, CCAM, diaphragmatic hernia Syndromes: Achondroplasia, Klippel-Feil, trisomy 18/21, TORCH, hydrops fetalis, multiple congenital anomalad Other: Diabetes mellitus, twin-twin transfusion (recipient), fetal anemia, fetal heart failure, polyuric renal disease, neuromuscular diseases, nonimmune hydrops, chylothorax, teratoma Idiopathic (60%) Neo notes

1. 28 wk prem now 28 days old and on oxygen for CLD. On 150 cc/kg/d EBM by gavage, also on Fe, aldactazide, vit E and fluticasone. Calculate the caloric intake, what should his caloric intake be? What 2 ways do you recommend optimizing nutrition?

- 150mL/kg/day = 100kcal/kg/day (0.67 kcal/mL) - Should be: 150kcal/kg.day with CLD - 2 ways to optimize nutrition: o add fortify o increase rate or volume of feed

17. 20 day old infant exclusively breastfed who is only gaining 5-10g a day. Mom seeing a lactation consultant. Notes very painful to breastfeed and baby keeps slipping off. On exam is only ankyloglossia. What is normal daily weight gain for baby? What are your recommendationsx2

- 20-30g per day - Recommendations: - Consider frenotomy - refer to experienced practitioner - (Nipple shield) - Encourage continuation of breastfeeding and supplement with formula as needed - Close monitoring of weight ongoing

25. An infant born at 26 weeks gestation is now 28 weeks (2 weeks old). He is doing well and is going to be transferred to the level II nursery. When should he be screened for ROP? a) If normal now, no need to do further follow-up b) Do eye exam now c) Do eye exam at 32 weeks d) Do eye exam at 38 weeks e) Do eye exam at 52 weeks

- 31 weeks CPS Statement: If 22-27 weeks, screen @ 31 weeks; if 28 -30+6, screen at 4 weeks of life CPS statement: vision screening in premature infants • Screen: less than 31 weeks, less than 1250 g • Timing: o Infants less than 27 weeks GA: screen at 31 weeks PMA o Infants 27 weeks or more GA: screen at 4 weeks of age Stages of ROP ▪ Stage I: Line of demarcation separates vascular and avascular retina ▪ Stage II: Ridging of line of demarcation as a result of scar formation ▪ Stage III: Extraretinal fibrovascular proliferation present ▪ Plus disease: In stages II and III plus disease refers to active inflammation as manifested by tortuosity of retinal vessels, which increases the risk for progression of ROP ▪ Stage IV: Subtotal retinal detachment ▪ Stage V: Complete retinal detachment PS p426 Recommendations: ▪ Current evidence suggests that screening infants with the following: o GA of 30 6/7 weeks or less (regardless of birth weight); AND o birth weights of 1250 g or less o Screening of infants with birth weights of between 1251 g and 2000 g is appropriate if the neonatologist believes the baby to be at high risk because of the severity and complexity of the neonatal clinical course. o Risk factors may include severe and unstable respiratory disease, hypotension requiring inotropes and prolonged ventilatory or oxygen therapy ▪ Infants with GA of 26 6/7 weeks or less at birth - initial screen at 31 weeks' PMA ▪ Infants with GA of 27 weeks or more at birth - initial screen at four weeks' CA. ▪ Follow-up examinations should be recommended by the examining ophthalmologist. ▪ For follow-up examinations, the AAP suggested schedule is the following: o One week or less follow-up: ♣ Stage 1 or 2 ROP in zone I ♣ Stage 3 ROP in zone II. o One- to two-week follow-up: ♣ Immature vascularization (stage 0) in zone I ♣ Stage 2 ROP in zone II ♣ Regressing ROP in zone I. o Two-week follow-up: ♣ Stage 1 ROP in zone II ♣ Regressing ROP in zone II. o Two- to three-week follow-up: ♣ Stage 1 or 2 ROP in zone III ♣ Regressing ROP in zone III. ▪ Retinal ablative therapy should be considered for high-risk prethreshold ROP: o Zone I - any stage ROP with plus disease. o Zone I - stage 3 ROP with or without plus disease. o Zone II - stage 2 or 3 ROP with plus disease. ▪ Retinal ablative therapy should be performed for threshold ROP (at least five contiguous or eight cumulative clock hours of stage 3 ROP in zone 1 or 2 in the presence of plus disease). Treatment should be performed within 72 h of examination. ▪ Cessation of ROP screening depends on eye findings and PMA. AAP indications for stopping screening examinations include the following: o Complete vascularization; o Zone III vascularization without previous zone I or II ROP; o PMA of 45 weeks and no prethreshold disease or worse ROP; o Regression of ROP. ▪ Infants who have had ROP are at risk of poor visual acuity and other visual disturbances, regardless of whether treatment was required. These infants require long-term ophthalmological follow-up.

40. Seizing neonate with maternal drug use

- ? repeat, bolus morphine vs phenobarb Neonatal Abstinence Syndrome (withdrawal) • Maternal drug use effects on fetus: o Chronic or intermittent drug exposures o Poor maternal nutrition o Acute withdrawal shortly after birth o Longterm effects on physical growth and neurodevelopment • Heroin addiction: o 50 percent incidence of LBW infants, higher rate of stillbirths (but not cong anomalies) o Withdrawal is seen in 50-75 percent of infants ♣ Begins in first 48 hours ♣ Depends on maternal daily dose and duration of addiction; as well as timing of last exposure ♣ Risk of RDS and hyperbilirubinemia may be reduced in preterm infants of heroin users ♣ Tremors and hyperirritability most prominent symptoms (limbs frequently rigid, hyperreflexic and resistant to flexion or extension) ♣ Other signs: hyperacusis, tachypnea, diarrhea, vomiting, high-pitched cry, fist sucking, poor feeding and fever o Diagnosed with history and clinical findings ♣ Meconium testing is more accurate than neonatal urine drug testing • Methadone addiction: o Severe withdrawal symtoms, incidence 20-90 percent o Clinical manifestations are similar, except higher incidence of seizures (10-20%) and later onset (2-6 weeks) • Alcohol withdrawal: o Uncommon, if mom is drinking immediately before delivery, baby may be affected as it crosses the placenta ♣ Hypoglycemia and metabolic acidosis may be present • Cocaine abuse: o Withdrawal is unusual o Pregnancy may be complicated by premature labor, abruption and fetal asphyxia ♣ IUGR and neurobehavioural deficits • Treatment o Care in a quiet environment with reduction of external stimuli and swaddling o Methadone for heroin and methadone withdrawal o Mortality is less than five percent ♣ Neonatal withdrawal signs and symptoms usually begin at 1-5 days of life with maternal heroin use and at 1 to 4 weeks with maternal methadone addiction ♣ Clinical manifestations of withdrawal include: o Sneezing, yawning, ravenous appetite, emesis, diarrhea, fever, diaphoresis, tachypnea, high pitched cry, tremors, jitteriness, poor sleep, poor feeding, and seizures. ♣ The illness tends to be more severe during methadone withdrawal. ♣ Initial treatment includes swaddling in blankets in a quiet, dark room. ♣ When hyperactivity is constant, and irritability interferes with sleeping and feeding, or when diarrhea or seizures are present, pharmacologic treatment is indicated. EN p232 ♣ SSRIs are being prescribed with increasing frequency to pregnant women with depression. ♣ Recent data suggest that within days of birth, infants experience withdrawal symptoms, including irritability, crying, hypertonia, and seizures. ♣ The drug that figures most prominently is paroxitine (Paxil), but similar symptoms have been reported with fluoxitine (Prozac), Sertraline (Zoloft), and citalopram (Celexa). PS p427 Distinguishing a seizure from nonseizure activity/tremor, Seizure is: ♣ Not elicited by sensory stimuli ♣ Not suppressed by gentle restraint/repositioning ♣ Accompanied by autonomic phenomena (vital sign changes) ♣ Associated with abnormal eye movements ♣ Often present with >1 seizure type ♣ Accompanied by an abnormal neurological exam ♣ Suggested by the presence of an abnormal EEG in a term infant or abnormal neuroimaging SK p596, PS p564 Management of above patient: 1. ABCs and oxygen 2. Cardiac and O2 sat monitoring 3. Load with Phenobarbital 20mg/kg - if seizure persists can add to total loading up to max 40mg/kg 4. If seizure continues load with 20mg/kg phenytoin 5. Bloodwork: glucose, electrolytes, calcium, phosphorous, and magnesium and consider CBC + diff, CRP, BCx and LP once seizure ceases 6. Commence treatment for drug withdrawal - oral morphine 0.02mg/kg q4H 7. May require ongoing maintenance of Phenobarb 5mg/kg/d +/- Phenytoin 6mg/kg/d SK p599, PS p563-564

1. Neonate with BPD being treated with furosemide. List 4 side effects.

- Alkalosis - Hyponatremia - Hypokalemia - Hypocalcemia - Hypercalciuria - Renal stones - Ototoxicity Furosemide side effects: • Hypokalemia, hyponatremia, hypocalcemia • Alkalosis • Ototoxicity • Nephrocalcinosis • Azotemia • Cholelithiasis Dehydration Electrolyte loss (ie. Na, K, alkolosis) Hypercalciuria renal stones and / or nephrocalcinosis Ototoxicity

7. How do you clinically determine the gestational age of a premature baby?

- Ballard score (neuromuscular and physical maturity)

Perinatal Brachial Plexus Palsy

- Birth trauma is the most common cause - Associated with shoulder dystocia, LGA, maternal diabetes and instrumental delivery - Recent evidence suggests that 20-30% of infants will have residual defects Recommendations: - Counselling of parents should include the following information: o It is not always preventable o 75% of infants recover completely within the 1st month of life o 25% experience permanent impairment and disability - If there is incomplete recovery by the end of the 1st month, referral to a multidisciplinary brachial plexus team should be made. The team should include neurologists, physiatrists, rehabilitation therapists and plastic surgeons. - Management and prognosis is based on history, electrodiagnostic procedures, diagnostic imaging, and physical examination - There are no RCTs evaluating surgical vs non-surgical management - Secondary soft tissue and bone reconstruction may improve function, but is inferior to primary intervention Neo notes

1. Newborn infant with Apgars of 1 and 6: a. What 5 complications would you anticipate in the next few days: b) What 2 tests at discharge, if normal, would likely indicate a normal neurological outcome:

- Complications of perinatal asphyxia (Nelson's) o Seizures (due to HIE/cerebral edema, hypocalcemia, hypoglycemia, or infection) o Persistent pulmonary hypertension o Respiratory distress syndrome o Hematuria o Acute tubular necrosis o Heart failure/cardiogenic shock ♣ Bottom five secondary to inadequate perfusion a. b) What 2 tests at discharge, if normal, would likely indicate a normal neurological outcome: - MRI - EEG Nelson's: "normal MRI and EEG findings are associated with good recovery, whereas severe abnormalities predict poor outcome" - Other predictors of poor outcome: o Initial gas pH <6.7 o 5-min Apgar 0-3 o High base deficit (>20-25 mmol/L) o Decerebrate posture o Lack of spontaneous activity o Severe encephalopathy (flaccid coma, apnea, absence of oculocephalic reflexes, and refractory seizures)

1. You are working in the NICU and are called to see a 4 day old female born at 28 weeks. She has vomited with her last 2 NG feeds and now is noted to be distended with discoloration of the abdominal skin. You are concerned about necrotizing enterocolitis. You place the baby npo, start antibiotic treatment, and order an abdominal XR. Name 3 XR signs of NEC.

- Dilation and thickening of bowel wall - Air fluid levels - Pneumatosis - Pneumoperitoneum/free air - Portal venous gas Name 3 XR signs of NEC. Pneumatosis intestinalis Portal venous gas Pneumoperitoneum Sentinel loops • Early: Abnormal gas pattern with dilated loops of bowel • Pneumatosis intestinalis (Stage II-III NEC) • Pneumoperitoneum (Stage IIIB NEC) • Portal venous gas • Sentinel loop: fixed loop of bowel suggestive of necrotic bowel and/or perforation even in absence of pneumatosis intestinalis Modified Bell staging criteria for necrotizing enterocolitis (NEC) in neonates Stage Classification of NEC Systemic signs Abdominal signs Radiographic signs IA Suspected Temperature instability, apnea, bradycardia, lethargy Gastric retention, abdominal distention, emesis, heme-positive stool Normal or intestinal dilation, mild ileus IB Suspected Same as above Grossly bloody stool Same as above IIA Definite, mildly ill Same as above Same as above, plus absent bowel sounds with or without abdominal tenderness Intestinal dilation, ileus, pneumatosis intestinalis IIB Definite, moderately ill Same as above, plus mild metabolic acidosis and thrombocytopenia Same as above, plus absent bowel sounds, definite tenderness, with or without abdominal cellulitis or right lower quadrant mass Same as IIA, plus ascites IIIA Advanced, severely ill, intact bowel Same as IIB, plus hypotension, bradycardia, severe apnea, combined respiratory and metabolic acidosis, DIC, and neutropenia Same as above, plus signs of peritonitis, marked tenderness, and abdominal distention Same as IIA, plus ascites IIIB Advanced, severely ill, perforated bowel Same as IIIA Same as IIIA Same as above, plus pneumoperitoneum

1. GBS unknown, indications for treatment of infant (3)

- Fever in infant - WBC <5 - Unwell

Canadian Pediatric Review 2018 Perinatal-Neonatal Medicine Antenatal screening:

- First trimester screen (FTS): (Detection 85%, False + 5%) • Maternal age • 12 week nuchal translucency + serum PAPP-A, beta-HCG - Maternal serum screen (Detection 60%, false 5%) • 1 blood test: 15-20 weeks (AFP, E, HCG) - Integrated prenatal screen (IPS) • FTS + MSS - Results: • Trisomies: high HCG, low PAPP-A, low AFP • Open NTD: high AFP (all the Afp Escapes in NTDs) • Placental insufficiency: low PAPP-A (P = Placenta)

- What does the evidence say that iNO can do? - Who should be treated with iNO?

- For PPHN decreases ecmo and death at doses of 10-80ppm (mostly preventing ECMO) For diaphragmatic hernia not decrease death - >/=35wks infant Hypoxemic respiratory failure Not responding to appropriate respiratory management CPS iNO

1. Baby with hypoglycemia, name 10 bloodwork that needs to be done BEFORE treatment to diagnose the cause of the hypoglycaemia

- Glucose, Insulin, growth hormone, cortisol, beta-hydroxybutyrate/ketones, free fatty acids, lactate, uric acid, ammonia, thyroxine/TSH, IGFBP-1 - Chart in Nelson's

7. Name 3 perinatal complications for a post-term baby born at 42 weeks.

- Hypoglycemia (?LGA) - Meconium Aspiration Syndrome - Brachial plexus injury secondary to shoulder dystocia - Perinatal depression, PPHN, Hypocalcemia, Polycythemia

1. Premature baby with apneas, temp instability, abdo distention with bowel loops palpable, not tolerating feeds a. Management/treatment x4 lines

- NPO - IVF/Parenteral Nutrition - Broad-spectrum antibiotics (Cefotaxime + Vancomycin +/- Metronidazole) - Close monitoring with x-ray/involvement of surgical team

19. A child presents at four days old with purulent exudate coming from one eye. Otherwise feeding well. She was born to a mother who had untreated Gonorrhea during the pregnancy, and you suspect ophthalmia neonatorum. What are FIVE management strategies that you should undertake to treat this child?

- Nelsons, CPS Statement, Public health - *admit to hospital and consult ID - *conjunctival culture for gonorrhea and chlamydia - Ceftriaxone IV x 1 dose (50mg/kg/dose x 1 - not to exceed 125mg) - irrigate eye with saline - high frequency until purulent discharge cleared - CSF and blood culture if child unwell In most instances, neonatal ophthalmia is a mild illness. The exception is ophthalmia due to infection with N gonorrhoeae.[2] Without preventive measures, gonococcal ophthalmia occurs in 30% to 50% of infants exposed during delivery[3]-[5] and may progress quickly to corneal ulceration, perforation of the globe and permanent visual impairment.[6] Infants at increased risk for gonococcal ophthalmia are those whose mothers are at risk for sexually transmitted infections (STIs).[7] Managing newborns exposed to N gonorrhoeae: A system should be established to ensure that all infants born to mothers found to have untreated N gonorrhoeae infection at delivery are treated. If the mother's test results are not available at discharge, a plan must be in place to ensure that she can be contacted promptly if the results are positive. The mother must also be advised to watch her infant for eye discharge in the first week of life and told whom to contact immediately if this symptom develops, or if the child is unwell in any way. When there is doubt about maternal compliance with this recommendation and the mother is considered to be at risk for gonococcal infection, administering one dose of ceftriaxone should be considered for the infant before discharge. Infants born to women with untreated N gonorrhoeae infection at the time of delivery, including those born by Caesarian section,[34] should be tested and treated immediately without waiting for test results.[28] Infants exposed to N gonorrhoeae who appear to be healthy at birth, both term and preterm, should have a conjunctival culture for N gonorrhoeae and receive a single dose of ceftriaxone (50 mg/kg to a maximum of 125 mg) intravenously or intramuscularly. The preferred diluent for intramuscular ceftriaxone is 1% lidocaine without epinephrine (0.45 mL/125 mg). This intervention is both safe and effective. Biliary stasis from ceftriaxone is not considered to be a risk with a single dose. (Ceftriaxone is contraindicated in newborns receiving intravenous calcium. A single dose of cefotaxime [100 mg/kg given intravenously or intramuscularly] is an acceptable alternative.) If the exposed infant is unwell in any way, blood and cerebrospinal fluid cultures should also be performed. Infants with established gonococcal disease require additional investigation and therapy in consultation with a specialist in paediatric infectious diseases.[2][7][28] Historically, the purpose of prophylaxis for neonatal ophthalmia was to prevent devastating neonatal eye infection due to N gonorrhoeae. Tetracycline and erythromycin ointments have been considered to be acceptable alternatives for preventing gonococcal ophthalmia.[2][10] However, N gonorrhoeae strains isolated in Canada in 2012 showed considerable resistance to these agents, with tetracycline at 30% and erythromycin at 23%.[11] Whether this resistance can be overcome by the high local antibiotic levels achieved by topical application is unknown, and there are no recent studies of the efficacy of ophthalmia prophylaxis with these agents. Infants born to women with untreated chlamydia infection at delivery have a 50% risk of acquiring chlamydia, a 30% to 50% risk of developing neonatal conjunctivitis and a 10% to 20% risk of developing chlamydia pneumonia.[21] Topical ocular prophylaxis does not prevent transmission from mother to infant, does not reliably prevent neonatal conjunctivitis and does not prevent pneumonia.[10][22]-[26] Oral erythromycin prophylaxis of infants born to untreated mothers has been used in the past but has not been recommended since the association between erythromycin and pyloric stenosis was recognized.[27] Routine prenatal screening for C trachomatis and treatment of identified infections during pregnancy is the preferred option for preventing neonatal conjunctivitis and other infections in newborns caused by this organism. Close clinical follow-up of exposed infants is recommended.[2][28] The Public Health Agency of Canada recommends testing conjunctival and nasopharyngeal secretions of symptomatic infants and treating those who show positive results.

1. What are 4 measures for pain control in a neonate who is getting an IV start

- Oral glucose - Non-nutritive sucking on pacifier - Kangaroo care - Swaddling - Topical anesthetic if time permits

1. Neonate has lots of secretions needing suctioning and some respiratory distress. What is the single MOST important investigation to diagnose this disorder:

- Pass a nasogastric tube +/- x-ray (reveals coiled tube) Nelson's: TEF/EA in neonate presents with frothing and bubbling at mouth and nose, with coughing, cyanosis and respiratory distress - early onset respiratory distress - inability to pass tube suggests EA - when feasible, managed with surgical ligation of TEF and primary end-to-end anastomosis of esophagus

1. List 3 causes of cyanotic congenital heart disease with decreased pulmonary blood flow.

- Pulmonary atresia - Tricuspid atresia - Tetralogy of Fallot

1. List 4 ways to minimize pain in a neonate from procedures (e.g. heelprick, IV, venipuncture, suction) in a level 2 NICU.

- Repeat: oral sucrose, swaddle, non-nutritive sucking, kangaroo care, topical anesthetic (use sparingly)

1. A 12-hour old infant is noted to have abnormal movements, lasting 10-20 sec, observed by a nurse. Name THREE things that would make this activity UNlikely to be epileptic in nature.

- Suppressible - Precipitated by stimulation - No other VS changes - Not detectable on continuous EEG monitoring

Signs and Symptoms of SEPSIS in the neonate can be nonspecific; or focal signs of infection including

- Temperature instability - Hypotension - Poor perfusion - Apnea - Resp Distress - Lethargy - Seizures - Feeding intolerance - Jaundice - Petechiae Neo Notes

Mechanism of cooling in HIE

- amelioration of apoptosis - decreased loss of high-energy phosphates (ATP) - reduced oxygen consumption - reduced release of nitric oxide, glutamate, free radicals and excitatory amino acid neurotransmitters, and the induction of genes that reduce neuronal death Multifactorial Decrease oxygen consumption Reduce loss of high energy phosphates Reduce release of glutamate, free radicals, nitric oxide, amino acid neurotransmitters Induce genes that reduce cell death CPS Hypothermia in HIE

1. Baby prem 33 wks with RDS. HR is 50, apneic and limp. What are your next 2 mngt steps? Show very baby XR -completely opacified lung fields (+/- bilateral pleural effusions). a. What does this baby likely have? b. Name 2 findings on XR.

- bag mask ventilate - compressions a. What does this baby likely have? RDS/HMD b. Name 2 findings on XR. Ground glass appearance, reduced lung volumes, air bronchograms a. What are your next 2 mngt steps? Start IPPV/Intubate and Chest compressions b. Show very baby XR -completely opacified lung fields (+/- bilateral pleural effusions) What does this baby likely have? Respiratory distress syndrome c. Name 2 findings on XR. • Fine reticular granularity (ground glass appearance) • Air bronchograms What are your next 2 mngt steps? Support ventilation and oxygenation -O2, Bag and mask ventilation, and prepare for intubation and administration of surfactant Support circulation -chest compressions Show very baby XR -completely opacified lung fields (+/- bilateral pleural effusions). What does this baby likely have? Respiratory distress syndrome Name 2 findings on XR. Fine reticular granularity Air bronchograms

1. Child with distress, can't pass NG through nare. List 3 other findings on physical exam you'd look for?

- choanal atresia - associated with CHARGE syndrome - Coloboma - Cardiac murmur - Ear anomalies Nelson's: unilateral or bilateral septum (bony > membranous) between nose and pharynx - other anomalies in 50-70%; 10-20% have CHARGE syndrome (coloboma, heart disease, atresia choanae, retarded growth and development, genital anomalies/hypogonadism, ear anomalies/deafness) Choanal Atresia/Stenosis associated with CHARGE syndrome: • Coloboma • Heart Defects • Atresia of choanal • Renal/genitourinary ;) • Growth retardation • Ear abnormalities, deafness Choanal atresia: • 1 in 7000; unilateral or bilateral o 50% have other congenital abnormalities, anomalies more frequent in those with bilateral • Dx established by inability to pass catheter thru each nostril 3-4cm into the nasopharynx Coloboma Genitourinary anomalies Ear anomalies Heart murmur Choanal atresia Ass'd with CHARGE Coloboma Heart disease Atresia choanae Retarded growth or development Genital anomalies Ear anomalies or deafness N p 1743 Choanal Atresia -1 in 7000 -unilateral or bilateral -50% have other congenital anomalies, anomalies more frequent in those with bilateral atresia -dx established by inability to pass a firm catheter through each nostril 3-4 cm into the nasopharynx -newborns have a variable ability to breathe through their mouths, so nasal obstruction does not produce the same symptoms in every infant -when the obstruction is unilateral, the infant may be asymptomatic for a prolonged period, often until the 1st respiratory infection, when unilateral nasal discharge or nasal obstruction may suggest the dx -infants with bilateral choanal atresia who have difficulty with mouth breathing make vigorous attempts to inspire, often suck in their lips, and develop cyanosis

1. Transitioning kid - 5 points Fetus to baby describe what happens:

- closure of Foramen ovale - closure of ductus venosus - closure of ductus arteriosus - decrease of pulmonary venous pressures - increase of systemic venous pressures Nelson's: - baby cries, rapid drop in PaO2, decreases PVR - SVR increases with low resistance circuit of placenta gone - Increased SVR and decreased PVR causes PDA to shunt L -> R then PDA gradually closes based on high arterial PaO2 - High LA pressures from volume returning to lungs cause foramen ovale to close - Removal of placenta also causes ductus venosus to close

1. Three ways that maternal SLE can affect neonate.

- congenital heart block (most feared, usually permanent) - characteristic annular or macular rash typically affecting the face, trunk, scalp - cytopenias - hepatitis Nelson's: neonatal lupus results from passive transfer of maternal IgG autoantibodies to fetus - noncardiac manifestations of neonatal lupus are usually reversible, but congenital heart block is permanent

TERATOGENS Category X

- contraindicated in pregnancy Neo Notes

TERATOGENS Category C

- definite risk shown in animal studies but no human studies or no data for animals or humans Yet to CCCC what it does in humans Neo Notes

4 risk factors for increased chance of kernicterus in a baby with hyperbilirubinemia

- dehydration, respiratory distress, prematurity, acidosis • Prematurity • Asphyxia • IVH • Hemolysis CPS: Kernicterus = pathologic finding of deep-yellow staining of neurons and neuronal necrosis of basal ganglia and brainstem nuclei - does not occur with TSB below, 340, rare if not above 425 - variable susceptibility; increased risk with dehydration, hyperosmolarity, respiratory distress, hydrops, prematurity, acidosis, hypoalbuminemia, hypoxia, seizures Kernicterus: • Dependent on the level of indirect bilirubin, duration of exposure to elevated levels, cause of jaundice and the infant's wellbeing • Occurs at lower bilirubin levels in: o Preterm infants o Presence of asphyxia, IVH, Hemolysis or drugs that displace bilirubin from albumin • CPS Statement: o All of the reasons for the variable susceptibility of infants are not known; however, dehydration,hyperosmolarity, respiratory distress, hydrops, prematurity, acidosis, hypoalbuminemia, hypoxia and seizures are said to increase the risk of acute encephalopathy in the presence of severe hyperbilirubinemia Risk Factors for Development of Severe Hyperbilirubinemia in Infants > 35 weeks: Major risk factors Predischarge TSB or TcB level in the high-risk zone Jaundice observed in the first 24 hr Blood group incompatibility with positive direct antiglobulin test, other known hemolytic disease (G6PD deficiency) Gestational age 35-36 wk Previous sibling received phototherapy Cephalohematoma or significant bruising Exclusive breastfeeding, particularly if nursing is not going well and weight loss is excessive East Asian race Minor risk factors Predischarge TSB or TcB level in the high intermediate-risk zone Gestational age 37-38 wk Jaundice observed before discharge Previous sibling with jaundice Macrosomic infant of a diabetic mother Maternal age ≥25 yr Male gender Decreased risk TSB or TcB level in the low-risk zone Gestational age ≥41 wk Exclusive bottle feeding Black race Discharge from hospital after 72 hr 4 risk factors for increased chance of kernicterus in a baby with hyperbilirubinemia Prematurity Sepsis Hypoalbuminemia Hemolysis Drugs that displace bilirubin from albumin IVH Asphyxia ABO incompatability G6PD deficiency Early discharge from hospital Visible jaundice @ less than 24 hours of age Breastfeeding Visible bruising, eg. cephalohematoma Sibling with a history of severe hyperbilirubinemia CPS Statement FN07-02 Hyperbilirubinemia Guidelines N p759

• Intrauterine growth retardation (IUGR) o Symmetric

- earlier onset, associated with diseases that seriously affect fetal cell number (chromosomal, genetic, malformation, teratogenic, infectious or severe maternal hypertensive etiologies) Neo Notes

• Intrauterine growth retardation (IUGR) o Asymmetric

- late onset, associated with poor maternal nutrition or with late onset or exacerbation of maternal vascular diseases Neo Notes

16. List 6 pieces of advice for the prevention of SIDS in the neonate.

- no in utero smoking - supine sleeping position - no co-sleeping - room sharing for 6-12 months - breastfeed if possible - pacifier use after breastfeeding established

TERATOGENS Category B

- no/some risk shown in animal studies but no human studies might B ok But don't know for sure, CCCC category C drugs that C effects in animals Neo Notes

18. You are taking care of a baby in the NICU who was born at 32 weeks gestation. She is doing well and is now approximately a week old chronologically. The parents ask you to give an estimate of when they can take her home. What are FOUR developmental criteria that must be met before discharge from hospital can be considered?

- normal body temperature - apnea free period of 5-7 days - maintain FiO2 - sustained weight gain - ... CPS statement

CXR in meconium aspiration

- patchy infiltrates, coarse streaking of both lung fields, increased anteroposterior diameter, and flattening of the diaphragm

Before discharge home, preterm infants must be completely evaluated, including:

- provincial newborn screening; - assessment for respiratory syncytial viarus (RSV) prophylaxis and administration, if indicated; - cranial imaging at near-term, if indicated by gestational age; - retinopathy of prematurity (ROP) screening, if indicated by gestational age or birthweight; - hearing screening; - immunizations according to chronological age and provincial/territorial schedule; and - predischarge physical examination, including measurement of weight, length and head circumference. The discharge team must determine each family's caregiving and psychosocial readiness for their infant's discharge, including assessment of the home environment. The family should receive predischarge education that includes safe sleep practices and SIDS prevention. Infant cardiopulmonary resuscitation training is highly desirable.Parents should be able to: - independently and confidently care for their infant; - provide medications, nutritional supplements and any special medical care; - recognize signs and symptoms of illness and respond appropriately, especially in emergency situations; and - understand the importance of infection control measuresand a smoke-free environment. The infant's health care team must ensure that an appropriate follow-up plan is in place before discharge, and that all aspects of the plan are communicated to and understood by the parents. Follow-up may include: • identification of and communication with the identified primary care physician, and providing a written or electronic summary of each infant's birth history and care; • follow-up by a qualified health care professional within 72 h; - medical and surgical follow-up appointments as required, including ROP screening; • neonatal neurodevelopmental follow-up, if indicated; • follow-up of hearing and newborn screening results; • RSV prophylaxis, if required; • community resources and supports; and • a neonatologist's or paediatrician's advice and support to the primary care physician, as needed. CPS facilitating discharge of preterms infants

TERATOGENS Category D

- some risk but with a benefit that may exceed that risk for the treated life-threatening condition DDDon't delay if you really need it Neo Notes

4. 4 months old ex 28 weeks with stridor (positional). History of PDA ligation. Aside from laryngomalacia name 3 things on DDX

- subglottic stenosis - vocal cord paresis - viral infection (croup) - vascular ring - vascular sling - hemangioma

Mechanism of brain damage in HIE

0-6h: Energy failure stage 1 = dec'd perfusion = dec'd ATP = Na/K pump = cells depol = release of excitatory aa's = Ca into cell = cell death 6-12 h: Normalization of oxidative metabolism w/ reperfusion = INTERVENE w/ COOLING 12-36h 7-14d: Energy failure stage 2: Apoptosis/Mito failure/Cytoxic edema/aa's/free radicals/cell death ********************** Poor perfusion and oxygenation to the brain Fall in: ATP Na/K pump Cell depolarization Lactic acidosis Release of excitatory amino acids Calcium entry into the cell Cell necrosis Break: 6-12h Normalization of oxidative metabolism after resuscitation and reperfusion Energy failure stage 2: 12-36h 7-14d Apoptosis Mitochondrial failure Cytotoxic edema Incr AA + release of free radicals Cell death At the cellular level, hypoxia-ischemia results in two phases of energy failure. The primary phase follows the reduction in blood flow and oxygen supply with a fall in ATP, failure of the Na+/K+ pump, depolarization of cells, lactic acidosis, release of excitatory amino acids, calcium entry into the cell and, if severe, cell necrosis. Following resuscitation and reperfusion, there is a latent period with normalization of oxidative metabolism lasting 6 h to 12 h, which is the therapeutic window for neuroprotective interventions. The secondary phase of energy failure develops at 12 h to 36 h, and may last seven to 14 days with initiation of apoptosis, mitochondrial failure, cytotoxic edema, accumulation of excitatory amino acids and release of free radicals terminating in cell death. This secondary phase is associated with worsening of HIE and correlates with poor outcomes CPS Hypothermia in HIE

Glucagon by intravenous bolus (_____mg/kg) or infusion (_____) has been observed to raise blood glucose and prevent recurrent episodes of hypoglycemia in both term and preterm infants. Alternative therapies include ______, but data are limited in their use for the initial management of hypoglycemia.

0.1 mg/kg to 0.3 mg/kg 10 μg/kg/h to 20 μg/kg/h hydrocortisone, diazoxide and octreotide CPS Hypoglycemia

CPS recommendation: what is the rate of complications in circumcision? What research is necessary?

0.2-2% Most are minor but some are major occasionally Need research for: Better epidemiological data on incidence of surgical complications Later complications Problems associated with no circumcision CPS circumcision

Vitamin K1 should be given as a single intramuscular dose of _____ (birthweight _____or less) or ______ (birthweight greater than _____) to all newborns within the first ______ following initial stabilization of the baby and an appropriate opportunity for maternal (family)-baby interaction.

0.5 mg 1500 g 1.0 mg 1500 g 6 h after birth IM vitamin K1 <1500g = 0.5mg >1500g = 1mg Within 6hrs After stabilization and family-baby interaction CPS Vit K

Ten steps to successful breatfeeding:

1 Have a written breast-feeding policy that is routinely communicated to all health care staff. 2 Train all health care staff in the skills necessary to implement this policy. 3 Inform all pregnant women about the benefits and management of breast-feeding. 4 Help mothers initiate breast-feeding within a half hour of birth. 5 Show mothers how to breast-feed and how to maintain lactation even if they should be separated from their infants. 6 Give newborn infants no food or drink other than breast milk unless medically indicated. 7 Practice rooming-in (allow mothers and infants to remain together) 24 hr a day. 8 Encourage breast-feeding on demand. 9 Give no artificial teats or pacifiers (also called dummies or soothers) to breast-feeding infants. 10 Foster the establishment of breast-feeding support groups and refer mothers to them on discharge from the hospital or clinic. Neo Notes

What is the risk of early onset GBS disease in an infant whose mother is GBS positive but does not receive IAP?

1% Remember 2/1000 live births is GBS disease without IPA CPS term infants at increased risk for early onset bacterial sepsis

What is the incidence of neural tube defect in Canada? Where are the highest rates?

1-4/1000 Atlantic canada - east to west! Is less! CPS Folic Acid and Neural tube defects

Reducing pain from other major procedures Retinal examination and surgery for retinopathy of prematurity

1. Although there are insufficient data to make a specific recommendation, retinal examinations are painful, and pain relief measures should be utilized. A reasonable approach would be to administer local anesthetic eye drops and oral sucrose. 2. Retinal surgery should be considered major surgery, and effective pain relief, based on opiates, should be provided. CPS Prevention and management of pain in the neonate

Canadian Pediatric Review 2018 Perinatal-Neonatal Medicine Baby C is now term corrected. She was treated with caffeine initially then it was discontinued at 34 weeks. She now continues to have apneas and bradycardias up to 5 times per day. What is the most likely cause of the apneas? 1. Apnea of prematurity 2. Gastroesophageal reflux disease 3. An undiagnosed inborn error of metabolism 4. Intraventricular hemorrhage 5. Kangaroo care / skin-to-skin contact exposure

1. Apnea of prematurity Apnea • Definition: cessation of breathing 20 seconds or 10-20s with bradycardia (<80) • Types: central, obstructive, mixed • DDX: Never normal in a term baby - Maternal drug, sepsis/meningitis, ICH, seizure, GERD, hypoxia/acidosis, metabolic (hypoglycemia, hypocalcaemia), polycythemia, PDA, central hypoventilation syndrome • Treatment: - Cardio resp monitoring, (respiratory support), treat cause • Apnea of prematurity: - Caffeine (decreases time on respiratory support) - Up to 20% of VLBWs still have apnea at term corrected, up to 44 weeks - "8 days free of apnea for discharge" late preterm Prematurity • LBW < 2500g VLBW < 1500g ELBW <1000g - 22 weeks 500g (+100g per week) to 26 weeks (900g) • Mortality by GA - 40-50% 24 weeks, 25% 25 weeks, <1% > 30 weeks • Complications - Apnea of prematurity - Respiratory distress syndrome - Chronic lung disease - PDA - IVH - Anemia requiring transfusions - Sepsis, NEC - ROP - Neurodevelopment: • CP, cognitive, hearing, blindness, learning disability, behaviour • Late-preterm: 34-36 weeks GA - respiratory distress, temperature instability, hypoglycemia, kernicterus, apnea, seizures, and feeding problems, as well as higher rates of rehospitalisation

1. Baby with constipation. Barium shows rectal ampulla narrow with dilated proximal bowel. a. 2 investigations to confirm the diagnosis? b. What would you expect as the result of each investigation?

1. Baby with constipation. Barium shows rectal ampulla narrow with dilated proximal bowel. a. 2 investigations to confirm the diagnosis? - Rectal suction biopsy - Anorectal manometry b. What would you expect as the result of each investigation? - Rectal biopsy - hypertrophied nerve bundles that stain positively for acetylcholinesterase with absence of ganglion cells - Anorectal manometry - failure of internal anal sphincter to relax in presence of dilated balloon in rectum (causing distension) What would you expect as the result of each investigation? Rectal manometry -paradoxical rise in pressure with rectal distension Rectal biopsy -aganglionic bowel Nelson's: Hirschsprung Disease/Congenital Aganglionic Megacolon - Absence of ganglion cells in submucosal and myenteric plexus; extends proximally from anus for variable distance - Histologically see absence of Meissner and Auerbach plexus, and hypertrophied nerve bundles with high concentrations of acetylcholinesterase between muscle layers and in submucosa - CP: o Failure to pass meconium, distended abdomen, bilious emesis/aspirates with feed intolerance - Diagnosis: o Gold standard = rectal suction biopsy ♣ Large number of hypertrophied nerve bundles that stain positively for acetylcholinesterase with absence of ganglion cells o Anorectal manometry - measures pressure of internal anal sphincter while balloon is distended in rectum ♣ Normal: rectal distension initiates relaxation of sphincter ♣ Hirschsprung: sphincter fails to relax o Unprepared contrast enema - especially useful after 1 month of life once proximal ganglionic segment is dilated (normal in 10% of newborns) ♣ Findings based on presence of abrupt narrow transition zone between normal dilated proximal colon, and smaller-caliber obstructed distal aganglionic segment - Treatment: o Operative intervention Diagnosis = Hirshsprung's a. 2 investigations to confirm the diagnosis? Rectal suction biopsy Rectal contrast enema Anorectal manometry b. What would you expect as the result of each investigation? AXR - signs of distal intestinal obstruction Rectal biopsy - no ganglion cells present, increased acetylcholinesterase staining Contrast rectal enema - transition zone identified; delay evacuation (> 24 hr) Anorectal manometry - paradoxical rise in pressure with rectal distension (or no sphincter relaxation at all)

Canadian Pediatric Review 2018 Perinatal-Neonatal Medicine 39 week GA infant born after c-section is grunting at 10 minutes of age. You examine and notice a RR of 70 and minimal indrawing. O2 saturations are 98% in room air. A chest x-ray is performed. What do you do next? 1. Continue to observe, no further investigation at this time 2. Give a trial of CPAP, PEEP 5 cm H2O 3. Give a dose of sedation to decrease work of breathing and prevent flipping into PPHN 4. Full septic work-up and start antibiotics

1. Continue to observe, no further investigation at this time TTN

Canadian Pediatric Review 2018 Perinatal-Neonatal Medicine In contrast to 'classic' bronchopulmonary dysplasia, the 'new' BPD is characterized by different histopathology. The most striking abnormality is: 1. Decrease in alveolar septation 2. Diffuse leukocytic infiltration 3. Hypertrophy of airway smooth muscle 4. Lung parenchymal fibrosis

1. Decrease in alveolar septation old BPD was 27-28 weeks that had bad lungs because of the ventilator vs new BPD which is inadequate development Bronchopulmonary dysplasia • Definition: - Oxygen dependence beyond 28 days or at 36 weeks postgestational age - Incidence 25% in <1500g • Postnatal Corticosteroids: - decrease ventilation time but not overall mortality - risk of neurodevelopment sequelae • Recommendations: - not recommended in 1st week of life for BPD prevention, routine use inhaled steroids also not recommended - could consider later use: for ventilator-dependent, severe CLD, low-dose with tapering short course (7-10 days)

Reducing pain from other major procedures Intercostal drains

1. General nonpharmacological measures; 2. Slow infiltration of the skin site with a local anesthetic before incision unless there is life threatening instability. If there is inadequate time to infiltrate before the insertion of the chest tube, local skin infiltration after achieving stability may reduce later pain responses and later analgesic requirements. 3. Systemic analgesia with a rapidly acting opiate, such as fentanyl. CPS Prevention and management of pain in the neonate

Reducing pain from other major procedures Chest drain removal

1. General nonpharmacological measures; and 2. A short-acting, rapid-onset systemic analgesic. CPS Prevention and management of pain in the neonate

Early-onset neonatal bacterial sepsis (EOS) - Well late preterm infants 35 to 36 weeks' GA

1. If infants are stable enough to remain with their mother in a mother and baby unit, they can be managed similar to infants ≥37 weeks' GA, but should be observed in hospital for at least 48 h. CPS term infants at increased risk for early onset bacterial sepsis

Early-onset neonatal bacterial sepsis (EOS) - Newly born unwell term infants (≥37 weeks' GA) should receive (w/u and/or tx)

1. Infants with clinical signs of sepsis (respiratory distress, temperature instability, tachycardia, seizures, hypotonia, lethargy, poor peripheral perfusion, hypotension, acidosis) require prompt investigation, including: CBC blood culture lumbar puncture and initiation of empirical intravenous antibiotic therapy. Ampicillin and an aminoglycoside provide coverage for the most common pathogens associated with early onset sepsis (EOS). Infants who have respiratory signs should also have a chest x-ray. (Strong recommendation, moderate quality evidence.) 2. Infants with early respiratory signs only and without risk factors for sepsis may be observed for up to 6 h before initiating investigations for sepsis and antibiotic therapy. (Strong recommendation, low quality evidence.) CPS term infants at increased risk for early onset bacterial sepsis

Canadian Pediatric Review 2018 Perinatal-Neonatal Medicine Which of the following is the most likely association found in the neonate? 1. Maternal Graves disease - hyperthyroidoism 2. Maternal diabetes mellitus - hyperglycemia 3. Maternal hyperparathyroidism - hyponatremia 4. Maternal SLE - supraventricular tachycardia 5. Maternal vitamin D deficiency - hypercalcemia

1. Maternal Graves disease - hyperthyroidoism 2. Maternal diabetes mellitus - hyperglycemia (hypo) 3. Maternal hyperparathyroidism - hyponatremia (hypocalcemia) 4. Maternal SLE - supraventricular tachycardia (heart block) 5. Maternal vitamin D deficiency - hypercalcemia (hypocalcemia)

1. Mom with no prenatal care gives birth to kid with anencephaly. She asks if the kid's organs (heart, kidneys) could be donated. What answer do you give her (1 point)? Why (1 point)?

1. Mom with no prenatal care gives birth to kid with anencephaly. She asks if the kid's organs (heart, kidneys) could be donated. What answer do you give her (1 point)? Why (1 point)? - Not possible, because cannot meet 'dead donor' rule (of brain death) because usually have adequate brainstem function to maintain respiration and heart rate after birth, and by the time brain death has occurred, the organs will have had ischemic damage (cardiovascular and respiratory functions deteriorate gradually before terminal event) -Anencephaly = CNS abnormality characterized by congenital absence of forebrain, skull and scalp; usually die within days or weeks without life-support - 2005 CPS statement (no longer an active one!): - organ donation from anencephalic infants should not be undertaken due to serious difficulties surrounding establishment of brain death in these infants, and lack of evidence to date supporting successful organ transplantation - there should be no modification of standard infant brain death criteria to include infants with anencephaly

Canadian Pediatric Review 2018 Perinatal-Neonatal Medicine During your postnatal ward round at a busy Canadianlevel II centre, in which of these scenarios are you least likely to recommend breastfeeding? 1. Mother is HIV positive on antiretroviral medications 2. Mother has a single oral lesion consistent with HSV 3. Mother has a history of Hepatitis B acquired in childhood 4. Mother is in a methadone program and drug screens are consistently negative for other substances

1. Mother is HIV positive on antiretroviral medications

1. Name 4 perinatal risk factors for the development of sensorineural hearing loss.

1. Name 4 perinatal risk factors for the development of sensorineural hearing loss. i. FMHx of permanent hearing loss ii. Congenital infections iii. Craniofacial abnormalities iv. NICU stay >2 days OR NICU stay of any duration + ototoxic drugs, ECMO, assisted ventilation, or hyperbilirubinemia requiring exchange transfusion -Congenital infections - CMV (most common infectious cause of congenital SNHL), toxoplasmosis, syphilis, rubella - Neonatal GBS sepsis/meningitis (pathogens S. pneumo, H. flu) - Ototoxic drugs in pregnancy (aminoglycosides, loop diuretics, chemotherapeutic agents) - Exposure to certain chemicals (quinine, lead, arsenic) - can occur both pre and postnatally Nelson's: - damage to or maldevelopment of structures in the inner ear can cause sensorineural hearing loss o causes: hair cell destruction from noise, disease, or ototoxic agents; cochlear malformation; perilymphatic fistula of round or oval window membrane; lesions of acoustic division of 8th nerve - conductive hearing loss is caused by dysfunction in transmission of sound through external or middle ear, or by abnormal transduction of sound energy into neural activity in inner ear and 8th nerve (most common form of hearing loss in children) CPS: 1) FMHx of permanent hearing loss 2) Congenital infections 3) Craniofacial abnormalities 4) NICU stay >2 days OR NICU stay of any duration + ototoxic drugs, ECMO, assisted ventilation, or hyperbilirubinemia requiring exchange transfusion

1. Name four contraindications to breastfeeding (not including drugs).

1. Name four contraindications to breastfeeding (not including drugs). - HIV infection - Human T-cell leukemia - CMV (in preterm infants) - Active TB - Hepatitis B virus (until infant receives hepatitis B immune globulin and vaccine) (straight from Nelson's) - CPS Statement on Breastfeeding...

1. Neonate with lung disease on ventilator (ACVG). Settings given, RR 40. Gas given (something like pH 7.2, CO2 58, bicarb I don't remember) Hct 0.48. a. What vent change would you make? b. Same baby does well for 48hrs then becomes mottled with poor BP and new gas: pH 7.1, CO2 40, base deficit really high, Hct 0.3. What has happened?

1. Neonate with lung disease on ventilator (ACVG). Settings given, RR 40. Gas given (something like pH 7.2, CO2 58, bicarb I don't remember) Hct 0.48. a. What vent change would you make? - increase RR (must blow off CO2) b. Same baby does well for 48hrs then becomes mottled with poor BP and new gas: pH 7.1, CO2 40, base deficit really high, Hct 0.3. What has happened? - sepsis - metabolic acidosis (high base deficit), hypotension/mottled - Nelson's: nonspecific signs and symptoms - temperature instability, hypotension, poor perfusion with pallor and mottled skin, metabolic acidosis, tachycardia or bradycardia, apnea, respiratory distress, grunting, cyanosis, irritability....

1. Newborn weighs 1.8 kg. Jittery and found to have glucose 1.3. Repeat glucose is 0.8 a. What is this baby's glucose requirement in mg/kg/min? (1) b. Write your IV order.

1. Newborn weighs 1.8 kg. Jittery and found to have glucose 1.3. Repeat glucose is 0.8 a. What is this baby's glucose requirement in mg/kg/min? (1) 5.5mg/kg/min b. Write your IV order. D10% 2mL/kg mini bolus, then D10% 80mL/kg/day CPS Statement: - initial glucose infusion regime is 80mL/kg/day of 10% dextrose, providing 5.5mg/kg/min of glucose. - Infants with very low glucose levels, particularly <1.8, should be managed with expedience - A single minibolus of 2mL/kg of D10% at start of infusion more rapidly achieves steady state levels, but benefit of practice in asymptomatic babies is uncertain (no comment under symptomatic but presume benefit)

Canadian Pediatric Review 2018 Perinatal-Neonatal Medicine After performing chest compressions for 45 seconds, while providing PPV with an LMA in position with good chest rise in 100% - the HR is 20 beats per minute. Which is the most appropriate next step? 1. Obtain emergency umbilical venous access 2. Insert intraosseous catheter 3. Give 1 dose of epinephrine 1:10 000 concentration, via the LMA 4. Intubate the baby, continue CPR and reassess in 120 seconds 5. Consider discontinuing resuscitation

1. Obtain emergency umbilical venous access 2. Insert intraosseous catheter

Early-onset neonatal bacterial sepsis (EOS) - Newly born well-appearing infants (≥37 weeks' GA) should receive (w/u and/or tx)

1. White blood cell (WBC) indices (total WBC count, absolute neutrophil count [ANC], ratio of immature to total neutrophils) or a single C-reactive protein should not be used routinely as screening or diagnostic tests for EOS, nor to routinely exclude EOS 2. For GBS-positive mothers with adequate intrapartum antibiotic prophylaxis (IAP), no additional risk factors OR mothers who are GBS-negative or GBS-unknown status, with one other risk factor and adequate IAP: Infants do not require investigation or treatment for sepsis. They may be discharged home after 24 h if they remain well, meet other discharge criteria and if parents understand signs of sepsis and when to seek medical care. (Strong recommendation, high quality evidence for GBS positive mother, adequate IAP; low quality evidence for GBS-negative or -unknown mothers.) 3. For GBS-positive mothers with inadequate IAP and no additional risk factors OR mothers who are GBS-negative or GBS-unknown status, with one other risk factor and inadequate IAP: Infants should be examined at birth, observed closely in hospital with vital signs every 3 h to 4 h, and reassessed before discharge home. They may be discharged home after 24 h if they remain well and meet other discharge criteria, providing there is ready access to health care and the parents understand and are able to seek medical care if the infant develops signs of sepsis. Routine investigation or treatment is not required. (Strong recommendation, low quality evidence.) 4. Multiple risk factors for sepsis and/or chorioamnionitis: Infants should be investigated and treated using an individualized approach that includes consideration of the severity of risk factors and maternal antibiotic therapy. At minimum, infants should have close observation in hospital for at least 24 h with vital signs every 3 h to 4 h and reassessment before discharge. A CBC done after 4 h of age may be helpful; WBC <5 x 109/L and ANC <1.5 x 109/L have the highest positive predictive value. Some infants may warrant investigation and antibiotic therapy. CPS term infants at increased risk for early onset bacterial sepsis

At-risk babies who have a blood glucose of less than _____ at ______ despite one feed (breastfeed or approximately _____ of formula or glucose water), or less than _____ after subsequent feeding, should receive an ______.

1.8 mmol/L 2 h of age 5 mL/kg to 10 mL/kg 2.0 mmol/L intravenous dextrose infusion CPS Hypoglycemia

Infants with very low glucose levels, particularly those with levels less than _____, should be managed with some expedience, confirming response to intervention in a timely fashion (a response to intravenous interventions should occur within ____). A single minibolus of ____ of _____ at the start of an infusion more rapidly achieves steady state levels, but the benefit of this practice in asymptomatic babies is uncertain. Due to the short duration of action of glucose, _____ are not recommended.

1.8 mmol/L 30 min 2 mL/kg 10% dextrose repeated miniboluses without an increase in the infusion rate CPS Hypoglycemia

At-risk infants with glucose levels less than _____ (assuming one effective feed), or repeatedly less than _____, require intervention Symptomatic infants should be treated immediately for blood glucose levels less than _____; there should be concurrent ______.

1.8 mmol/L on one occasion 2.6 mmol/L 2.6 mmol/L investigation and management of the underlying cause CPS Hypoglycemia

CPS recommendation: when can use enteral supplementation for hypoglycemia? When do you recheck sugar?

1.8-2.5 mmol/L Asymptomatic Recheck in 60mins CPS Hypoglycemia

Apnea of prematurity is defined as cessation of breathing for ≥20 s or _____ if accompanied by ______ or oxygen saturation (SaO2) ____% in infants ____ weeks' PMA

10 s to 20 s bradycardia (heart rate <80 beats/min) <80% <37 wks Although most preterm infants are free of apneic and bradycardic spells by 36 weeks' PMA, very preterm infants show more variability in resolution, and apnea may persist up to 44 weeks' PMA CPS facilitating discharge of preterms infants

there are differences between capillary and venous whole blood and plasma glucose levels in the range of _____ variation, whole blood being ____ than plasma

10% lower CPS Hypoglycemia

Meconium found in ____percent of births

10-15% Neo Notes

AGA

10-90th percentile for a given GA

The half-life of caffeine is prolonged in neonates (approximately _____) and infants may be at risk for recurrence of apnea for several days after it is discontinued.

100 h CPS facilitating discharge of preterms infants

Integrated Test for Trisomy 21

11-14 weeks: hCG and PAPP-A + nuchal translucency 15-20 weeks: AFP, uE3 and inhibin A Neo Notes

In the treatment of hypoglycemia, Intravenous dextrose can be weaned when levels have been stable for _____.

12 h CPS Hypoglycemia

hypoglycemia usually occurs in LGA infants and IDMs within ______, and screening beyond this period is not required if blood glucose is maintained at ______ or higher

12 h of birth 2.6 mmol/L Is the baby at risk? SGA, LGA, IDM or preterm = CHECK GLUCOSE AT 2 HRS AND EVERY 3-6 HOURS (BEFORE FEEDS) AS LONG AS INFANT REMAINS WELL, UNTIL FEEDS ARE ESTABLISHED AND GLUCOSE ≥2.6 mmol/l <1.8 mmol/l at 2 hours of age or <2.0 mmol/l at subsequent checks CONSIDER IV TREATMENT 1.8 - 2.0 mmol/l at 2 hours of age or 2.0 - 2.5 mmol/l at subsequent checks = Refeed and recheck glucose in 1 hour >2.0 mmol/l at 2 hours of age or ≥2.6 mmol/l at subsequent checks NOTE: REPEATED GLUCOSE LEVELS < 2.6 mmol/l IN AN AT RISK BABY REQUIRE FURTHER INTERVENTION WHEN TO STOP TESTING: IDM AND LGA DO NOT REQUIRE RETESTING IF SUGAR ≥2.6 mmol/l AFTER 12 HOURS OF AGE AS THEY USUALLY PRESENT BY THIS TIME. PRETERM AND SGA INFANTS DO NOT REQUIRE RETESTING AFTER 36 HOURS PROVIDED STABLE LEVELS AND INTAKE ARE ACHIEVED Is the baby unwell? ROUTINE CARE: INITIAL FEED Is the baby at risk? SGA, LGA, IDM or preterm BABY BECOMES UNWELL CHECK GLUCOSE NOW: INVESTIGATE FOR CAUSE AND TREAT ANY UNDERLYING CONDITION <2.6 mmol/l in an unwell baby CONSIDER IV TREATMENT CPS Hypoglycemia

How/how much surfactant to give

120 mg of liquid phospholipids/kg body weight for the first dose, larger initial doses do not lead to further improvements in outcomes via ET tube There is no evidence to support the practice of placing the infant in multiple different positions during the administration of surfactant. CPS surfactant

When can you start to wean dextrose infusions?

12h of being stable CPS Hypoglycemia

What is the general risk of severe disability in 23-25wks? CPS Counselling and management for anticipated extremely preterm birth

18-60% CPS Counselling and management for anticipated extremely preterm birth

When was the first human milk bank opened in the world, in US? Why did so many of them close in 1980? Where is the only milk bank in canada? How many milk banks are there in north america?

1909- vienna austria 1919- boston 1980's- closed bc of HIV Only one in canada = BC 11 in north america CPS Milk banking

The onset of NEC usually occurs in the ____ but can be as late as _____ in VLBW infants

1st 2 wk of life 3 mo of age Neo Notes

it is recommended that screening be initiated in at-risk babies at ______ and should be continued until the period of risk is considered over

2 h of age (after an initial feed) 'Symptomatic infants' should have a blood glucose assessment without delay as part of the workup for diagnostic and therapeutic purposes. CPS Hypoglycemia

How frequently should asymptomatic, at-risk infants be screened for hypoglycemia? When should it be discontinued

2 h of age and every 3 h to 6 h after this, in keeping with breastfeeding practices. Testing may be discontinued after 12 h in LGA infants and IDMs if blood glucose levels remain at 2.6 mmol/L or higher, and after 36 h in SGA and preterm infants if feeding has been established and blood glucose levels remain at 2.6 mmol/L or higher. Symptomatic and unwell babies require immediate glucose testing CPS Hypoglycemia

umbilical vein catheter may be used up to _____

2 weeks Neo Notes

What is the recurrence rate for neural tube defects if a woman has had a previously affected pregnancy?

2% (20/1000) to 4-5% (40-50/1000) Ie. 5-10x greater risk CPS Folic Acid and Neural tube defects

Current evidence shows that the infants who benefit from hypothermia are ______ infants ___weeks' gestation with HIE who are ____ of age and who meet both treatment criteria A and B: What are criteria A and B

2, 3,4,5,6,7, 10, 16 10, 10, 7-16 = 10 min apgar <5, 10 mins resusc, <7pH/16 base deficit + ENCEPH 1 = 1h gas 2 = Sarnat II, 0.5 degrees q2h rewarming 3 = 36 wks, Sarnat III 5 = Apgar <5 at 10 mins, 0.5 degrees q2h for rewarming 6 = 6 hours old 7 = pH <7, 72h cooling (48-72) 10 = 10 min apgar, 10 mins of resuscitation 16 = base deficit >16 mmol/L in cord or arterial blood gases measured within 1 h of birth. Criteria A = 2 of two 10's (2x5 =10) - <5 apgar @ 10 OR - 10 mins vent/resusc OR - 7-16 (<7ph or >16 BD) +++++ II/III Sarnat (Mod/severe enceph) Indications: (>36 weeks) BOTH 1&2 1. Any 2 of: - APGAR < 5 at 10 min, - ventilation 10min, - acidosis pH <7 BD >16 (on cord or at 1h) 2. Signs of moderate to severe encephalopathy SARNAT staging in HIE A - Autonomic/ N - Neuromuscular S - Seizures L - Level of consciousness E - EEG R - Reflexes Stage 1 = hyperalert/hyperalert eyes (dilated)/hyperaltert system (sympathetic) Alertness: Hyperalert Spontaneous Activity: NL Muscle tone: NL Posture: Mild distal flexion Myoclonus: Present Stretch reflexes: Overactive Suck: Weak or absent moro: Strong Oculovestibular: Normal Tonic neck: slight Autonomic: Generalized sympathetic (inc'd HR/RR) Seizures: None EEG: Normal (awake) Pupils: Mydriasis Duration: 1-3 days Stage 2 = hypotonic (hypotonic autonomic too), myoclonic + seizures, miosis also Alertness: Lethargic or obtunded Spontaneous Activity: Decreased Muscle tone: Mild Hypotonia Posture: Strong distal flexion Myoclonus: Present Stretch reflexes: Overactive Suck: Weak or absent moro: Weak Oculovestibular: Overactive Tonic neck: strong Autonomic: Generalized parasympathetic (brady) Seizures: Common; focal or multifocal EEG: Early: low-voltage continuous delta and theta. Later: periodic pattern (awake) Seizures: focal 1-to 1-Hz spike-and-wave Pupils: Miosis Duration: 2-14 Stage 3 = flat/decerebrate + seizure Alertness: Stuporous Spontaneous Activity: Absent Muscle tone: Flaccid Posture: Intermittent decerebration Myoclonus: Absent Stretch reflexes: Decreased or absent Suck: absent moro: absent Oculovestibular: Weak or absent Tonic neck: absent Autonomic: Both systems depressed Seizures: Common; focal or multifocal EEG: Early: Early: periodic pattern with Isopotential phases. Later: totally isopotential Seizures: Uncommon (decerebration) Pupils: Variable; unequal; Duration: Hours to weeks SARNAT STAGING Stage 1: 100% normal Stage2: 70% normal →→→25% neuro sequelae →→→5% mortality Stage 3: 0% normal, 20% neuro sequelae, 80% mortality Any two of the following: • Apgar score <5 at 10 min of age. • Continued need for ventilation and resuscitation at 10 min of age. • Metabolic acidosis with pH <7 or base deficit >16 mmol/L in cord or arterial blood gases measured within 1 h of birth term and late preterm >/=36wks </=6hours old Criteria a and b satisfied A: 2/3 Apgar <5 @ 10mins A/V gas <1h old: pH<7 BE >16 Ventilation/ resuscitation @10mins B: Sarnat stage 2/3 (moderate/severe) Evidence of seizure Or - >/= 1 sign in 3/6 categories +/- aEEG (amplitude-integrated EEG) 5.5h x 20mins for ?sz Criteria A Any two of the following: • Apgar score <5 at 10 min of age. • Continued need for ventilation and resuscitation at 10 min of age. • Metabolic acidosis with pH <7 or base deficit >16 mmol/L in cord or arterial blood gases measured within 1 h of birth. AND Criteria B • Moderate (Sarnat stage II) or severe (Sarnat stage III) encephalopathy demonstrated by the presence of seizures or at least one sign in at least three of the six categories shown in Table 1 All infants who are depressed at birth should be assessed to determine whether they fulfill criteria A. Infants who fulfill criteria A should then undergo a careful neurological examination to determine whether they fulfill criteria B. Infants who meet both criteria should be offered hypothermia. If possible, it is helpful to assess infants with an amplitude-integrated electroencephalogram for at least 20 min before 5.5 h of age to document abnormal tracings or seizures CPS Hypothermia in HIE

Canadian Pediatric Review 2018 Perinatal-Neonatal Medicine Which of the following statements is false? 1. Antenatal steroids are indicated for women presenting with risk of preterm delivery at < 34 weeks gestation. 2. Antenatal steroids are used for neuroprotection and decrease the risk of cerebral palsy. 3. Magnesium sulphate is indicated for women presenting with risk of preterm delivery at < 32 weeks gestation. 4. Magnesium sulphate is used for neuroprotection and decreases the risk of cerebral palsy.

2. Antenatal steroids are used for neuroprotection and decrease the risk of cerebral palsy. mag = neuro protection Fetal (maternal) interventions • Prevention of birth defects - Folic acid (peri-conception + pregnancy) - Glucose control in diabetes (same) • In prematurity - Antenatal steroids <34 weeks • lung development, IVH, NEC, mortality - Antenatal magnesium sulphate <32 weeks • neuroprotection (cerebral palsy) • Prevention / treatment of anemia / jaundice - Anti-D globulin (Rhogam) at 28 weeks - Fetal transfusions (for hydrops) • Treatment of fetal disease - Arrhythmia - Fetal surgery (CCAM hydrothorax shunt, severe CDH, Obstructive uropathy, Twin twin transfusion)

Canadian Pediatric Review 2018 Perinatal-Neonatal Medicine A 3 year old child was referred to you with both expressive and receptive speech delay. On history, she was born to a healthy mother G2P1 at 38 weeks GA by SVD, with APGARS 9, 9. Birth weight was 2130g, HC 30cm. Hearing screen prior to discharge was normal. Which of the following is the most likely reason for her speech delay? 1. Intraventricular hemorrhage related to her low birth weight 2. Deafness related to congenital CMV 3. Periventricular leukomalacia secondary to intrauterine growth restriction 4. Developmental delay secondary to fetal alcohol syndrome

2. Deafness related to congenital CMV

Canadian Pediatric Review 2018 Perinatal-Neonatal Medicine Baby M is a 37+2 week infant, born to an A positive mother, following a normal pregnancy. At 24 hours total bilirubin is 135. She is feeding well, and mother is ready for discharge. Which of the following are most appropriate? 1. Routine care, discharge home to follow up with family physician within 48 hours 2. Discharge home to return for a bilirubin check in the postnatal clinic in 24 hours 3. Keep in hospital and recheck bilirubin in 24 hours 4. Start phototherapy and recheck bilirubin in 24 hours

2. Discharge home to return for a bilirubin check in the postnatal clinic in 24 hours Causes of Neonatal Jaundice • Unconjugated - Dehydration - 'Breastmilk' - Infection - Polycythemia - Hemolysis - Endocrinopathies - Extravascular blood - Genetic disorders - Increased enterohepatic circulation (GI obstruction, delayed meconium) • Conjugated - Extrahepatic obstructive • Biliary atresia, choledochal cysts - Bacterial infection - TORCH - Neonatal hepatitis • viral, bacterial, parasitic, idiopathic - Metabolic: • alpha-1 antitrypsin, IEM, endocrinopathies, CF, Iron storage disease, bile acid synthesis defects - Cholecystasis syndromes • Dubin Johnson, Byler - Toxic: hyperalimentation • ↑production - ↑breakdown of fetal erythrocytes - High RBC mass • ↓hepatic excretion: - low [binding protein] - low glucuronyl transferase fn

Canadian Pediatric Review 2018 Perinatal-Neonatal Medicine In a healthy baby the transition from fetal to neonatal circulation involves: 1. Closure of the ductus venosus, becoming the ligamentum of teres, the remnant of the umbilical artery 2. Functional closure of the foramen ovale in response to increased left atrial pressure 3. Blood flow in the ductus arteriosis continuing to shunt right to left until it closes 4. The ductus arteriosus closing in response to decreasing Pa02

2. Functional closure of the foramen ovale in response to increased left atrial pressure 1. is actually remnant of the umbilical vein 3. starts shunting L to R before it closes 4. closes due to increasing (not decreasing) PaO2 Shunts: 1. Ductus Venosus UV -> IVC 2. Foramen Ovale RA -> LA 3. Ductus Arteriosus PA -> Ao

Canadian Pediatric Review 2018 Perinatal-Neonatal Medicine Which of the following constellations of features BEST describes fetal alcohol syndrome? 1. Elfin facies, irritability, and supravalvular aortic stenosis 2. Growth deficiency, microcephaly, developmental delay, short palpebral fissures 3. IUGR, triangular-shaped face, clinodactyly 4. Short stature, webbed neck, pulmonic stenosis 5. Weakness, club feet, immobile face, inadequate respirations

2. Growth deficiency, microcephaly, developmental delay, short palpebral fissures 1. williams - Will ferrel = Elf - he's SUPRAA(ortic) STenning (stenosis) 3. russel silver 4. turners = pul(monic) the neck skin in because it's sticking out (pulmonic stenosis) 5. congenital myotonia picture

Canadian Pediatric Review 2018 Perinatal-Neonatal Medicine Which of the following is the correct association? 1. Oesophageal atresia - oligohydramnios 2. IUGR - oligohydramnios 3. Posterior urethral valves - polyhydramnios 4. Renal agenesis - polyhydramnios

2. IUGR - oligohydramnios esoph atresia = poly bc can't swallow PUV/Renal agenesis = low UO = oligo

Canadian Pediatric Review 2018 Perinatal-Neonatal Medicine Which of the following is true? 1. Cord clamping should be delayed for at least 30- 60 seconds in term and preterm newborns who require resuscitation at birth. 2. Initial assessment of HR should be made using a stethoscope auscultating the left side of the chest. 3. PPV in newborns >35 weeks GA should begin with 21-30% FiO2. 4. When PPV begins, the assistant listens for increasing HR, if he/she announces 'heart rate is increasing', transition to CPAP

2. Initial assessment of HR should be made using a stethoscope auscultating the left side of the chest. 1. delayed cord clamping is recommended in babies not needing resuscitation 3. <35 wks 4. Transition to CPAP requires more than just this

You are stabilizing a 41 week baby in whom you have already suctioned copious meconium. Now at 30 minutes, she is making spontaneous respiratory efforts with significant work of breathing on mask CPAP PEEP 7 in 100% FiO2. Saturations are 88% and she has peripheral IV access. Cap gas is: 7.16/60/36/17. Your next step is to: 1. Confirm the diagnosis of meconium aspiration syndrome with a CXR 2. Intubate using sedation and muscle relaxation 3. Increase PEEP to 8 and reassess 4. Insert a UVC and UAC for better access and PaO2 monitoring

2. Intubate using sedation and muscle relaxation

Canadian Pediatric Review 2018 Perinatal-Neonatal Medicine Which of the following is true? 1. Erb's palsy involves C6, C7, & C8 2. Klumpke's palsy involves C7, C8, & T1 3. Facial nerve palsy -> persistently closed eye 4. In full nerve injury, neuroplasty is advised at the end of the first year of life

2. Klumpke's palsy involves C7, C8, & T1 E -> K (alphabetical)

Canadian Pediatric Review 2018 Perinatal-Neonatal Medicine Which of the following is false? 1. IVH is the most frequent type of intracranial hemorrhage in the newborn 2. Most IVH occur with symptoms of bradycardia, desaturation, and acidosis 3. Post-hemorrhagic hydrocephalus occurs due to adhesive arachnoiditis 4. Prematurity is the biggest risk factor for IVH

2. Most IVH occur with symptoms of bradycardia, desaturation, and acidosis note PVL = hypoxia so do HUS late to look for PVL due to hypoxia IVH • Germinal matrix hemorrhage, - 33% in <1500g; 60% in infants <1000g - Screening: <1500g or < 31+ 6 weeks • Associated with: - prematurity, RDS, PPV, hypocarbia, metabolic acidosis, coagulation disorders, unstable cerebral circulation • Clinical features: - 60% asymptomatic - Changes in tone, nystagmus, - Deterioration (apnea, bradycardia, acidosis, seizure, hyperglycemia, hyperkalemia) - Massive collapse (seizure, coma, shock, anemia) IVH • Diagnosis: ultrasound; Grade I-IV / Staging - I = Subependymal - II = no ventricle dilatation - III = distended ventricles, >50% filled with blood - IV = parenchymal involvement • Complications - Post hemorrhagic ventricular dilatation - Porencephaly

Canadian Pediatric Review 2018 Perinatal-Neonatal Medicine You attend the resuscitation of a 34 week infant. After stimulation and drying, the HR is 120 and the infant has secondary apnea. Which of the following would provide the most appropriate next step in supporting ventilation? 1. Provide PPV using a self-inflating bag, not attaching oxygen, for resuscitation in room air 2. Provide PEEP using a self-inflating bag at 5 cm of H20, using an oxygen blender, set to 21% 3. Using a t-piece resuscitator, provide supplemental oxygen, 30% FiO2 4. Using a flow inflating bag provide CPAP of 7 cm H20, using an oxygen blender, set to 100%

2. Provide PEEP using a self-inflating bag at 5 cm of H20, using an oxygen blender, set to 21%

Canadian Pediatric Review 2018 Perinatal-Neonatal Medicine You are paged by a family physician in the community about a baby with a bilirubin level of 350. Baby was born at 36+6 weeks and is now 96 hours old. Birth weight was 3100g and current weight is 3000g. Mother is a healthy woman of Scottish descent, GBS positive in previous pregnancy, and blood type O. Baby is exclusively breastfeeding. Which of the following is the correct next step? 1. Perform a full septic workup including LP (this is significant jaundice with risk factors for sepsis) 2. Send a CBC and coombs test now 3. Start phototherapy and recheck bilirubin in 12 hours with a CBC and coombs test 4. Baby is older now, the risk of kernicterus is much lower than in the first 24 hours, if she is feeding well, with good hydration, can repeat in 24 hours

2. Send a CBC and coombs test now Red flags Onset before 24 hours Hemolysis is a predictor of severity Pallor, Unwell Hepatosplenomegaly Pale stools, dark urine Conjugated hyperbilirubinemia Investigations Total and conjugated bilirubin CBC, blood group, Coombs test +/- G6PD, blood film Liver function, Metabolic / Endocrine workup Severe Hyperbilirubinemia • Acute bilirubin encephalopathy: - Lethargy, decreased tone & suck - Increased tone, opisthoclonus, retrocollis - High pitched cry, seizure, coma • Chronic bilirubin encephalopathy - Athetoid CP, seizures, DD/MR, hearing, oculomotor, dental dysplasia • Severe: TSB > 340 umol/L in 1st 28d • Critical: TSB > 425 umol/L in 1st 28d Clinical signs of acute bilirubin encephalopathy → immediate EXCHANGE transfusion

Canadian Pediatric Review 2018 Perinatal-Neonatal Medicine A 4.3 kg term newborn IDM was breast feeding on hypoglycemia protocol with Mom but at 4 hours is sleepy with a blood glucose of 1.9. What is the most appropriate next step? 1. Supplement breast feeding with formula and recheck blood glucose in 2 hours 2. Start IV D10W at 4-6 mg/kg/min and recheck in 30 minutes 3. Give a IV D10W 2ml/kg bolus then start D12.5W at 80ml/kg/day and recheck in 30 minutes 4. Send a critical sample then supplement breast feeding with formula and recheck blood glucose in 1 hour

2. Start IV D10W at 4-6 mg/kg/min and recheck in 30 minutes Hypoglycemia • Level < 2.6 mmol/L: adverse neuro outcome • Test and treat all symptomatic babies • Screening recommended in: - IDMs, preterm infants (<37 weeks) - SGA infants (<10th%), LGA (>90th%) • Frequency: - 1st @ 2hours of age (after feeding) - q3hours until 2 ≥ 2.6 mmol/L or - Testing may be discontinued after 12 h (LGA & IDMs) and 36h (SGA, preterm) • Give enteral supplementation: - asymptomatic infants with blood glucose levels of 1.8 - 2.5 mmol/L recheck in 60 minutes • Symptomatic or persistent hypoglycemia - Bolus 2cc/kg D10W IV and - GIR 4-6 mg/kg/min up to 10-12mg/kg/min then - Refractory: glucagon, steroids, diazoxide - Endocrine consult - Send critical labs (before correction) • Glucose, insulin, ketones/ketoacids, thyroid, cortisol, metabolic workup Hypoglycemia • Decreased substrate availability - SGA, preterm, discordant twin • Increased utilization - Hyperinsulinism (IDM, Beckwith-Wiedemann, SGA), - Polycythemia • Inability to use glucose - Metabolic: glycogen storage disease type I, galactosemia, fructose intolerance, IEM aa • Other - Birth asphyxia, CAH, hypopituitarism

Canadian Pediatric Review 2018 Perinatal-Neonatal Medicine Which of the following are true? 1. Caput succadaneum is a subcutaneous hemorrhage requiring no intervention, that is limited by suture lines 2. Subgaleal hemorrhage occurs in the subaponeurotic space, crosses suture lines, and has associated risk of mortality 3. Cephalohematoma is a risk factor for jaundice and infection, thus requires drainage in the first 24-48 hours after birth 4. All neonatal skull fractures require urgent neurosurgical referral, with vacuum extraction occurring within 6 hours after the injury

2. Subgaleal hemorrhage occurs in the subaponeurotic space, crosses suture lines, and has associated risk of mortality

Canadian Pediatric Review 2018 Perinatal-Neonatal Medicine Which of the following congenital anomalies would most likely require immediate surgical treatment in the first week of life? 1. Congenital cystic malformation of the lung 2. Tracheoesophageal fistula 3. Bilateral cleft lip and palate 4. Laryngomalacia

2. Tracheoesophageal fistula CCAMs usually get better and others repaired later (laryngomalacia not surgically repaired)

Canadian Pediatric Review 2018 Perinatal-Neonatal Medicine You have just resuscitated a 41 week baby with initial HR 40 requiring PPV by mask and chest compressions but no epinephrine. APGARS were 1, 2, 6, 8. Baby is now 3 hours old on CPAP PEEP 6 with regular non laboured respirations in 40% FiO2, RR 45, and HR 110. Initial labs have been sent off and a CXR is ordered. Other than some stiffening of the arms and legs when they drew blood work, she is overall hypotonic and with paucity of spontaneous movements. The cord gases are : arterial 6.98/80/20/9/-19 and venous 6.99/75/24/10/-18. Which of the following is the most appropriate next action? 1. Intubate the baby now with sedation and muscle relaxation 2. Turn down the overhead warmer / isolette and initiate passive cooling 3. Prepare for sterile insertion of UVC and UAC catheters 4. Discontinue respiratory support, this is severe hypoxic ischemic encephalopathy

2. Turn down the overhead warmer / isolette and initiate passive cooling Asphyxia - Hypoxic Ischemic Encephalopathy • Causes: - disruption of umbilical flow (prolapse, compression), - failure of gas placental gas exchange (abruption), - compromised fetus not tolerating labour (IUGR, anemia), - 'failure of postnatal transition' • Pathophysiology: - Initial necrosis, cell death à2nd reperfusion injury • Diagnosis: - history of perinatal depression + - acidosis, low APGAR 0-3 @ 5 min, ventilation at 10 min - neuro sx: LOC, tone, reflexes, seizures - investigations: • MRI (diffusion restriction), CFM (bedside) / full EEG, EPs HIE management (CPS statement) Therapeutic hypothermia Indications: (>36 weeks) BOTH 1&2 1. Any 2 of: - APGAR < 5 at 10 min, - ventilation 10min, - acidosis pH <7 BD >16 (on cord or at 1h) 2. Signs of moderate to severe encephalopathy Interventions: • Temperature: 34C +/- 0.5 - Passive cooling (in the community) - Active cooling (tertiary centre; servo controlled, cold compresses) - Method: total body or selective head cooling • Timing: ASAP, within 1st 6 hours • Complications: - hypotension, bradycardia, ?coagulopathy, Fat necrosis Outcomes • Prognosis: - Severe: 80% morbidity - Moderate: 30-50% - Mild: usually no deficits • Benefits of cooling - Risk reduction 25% combined mortality & major NDD - NNT 11 to prevent 1 mortality - Risk reduction 20% NDD in

Canadian Pediatric Review 2018 Perinatal-Neonatal Medicine You are called to assess a 38 week infant in level II who has been having intermittent desaturations since birth. They have applied LF O2 which is 'occasionally helpful'. You assess baby and find him placed side lying with a few rolls. For this baby, which of the following is correct? 1. You should place this baby supine, he is term and is at risk of SIDS 2. You should place this baby prone, this will facilitate breathing 3. When examining the palate, you should avoid digital palpation 4. An oral airway or LMA is contraindicated in babies with this phenotype

2. You should place this baby prone, this will facilitate breathing retro and micrognathia etc (pierre robin) - if in distress, put prone to help breathing rather than low flow because obstructing = retain Co2

the proposed cut-off for hypoglycemia in symptomatic hypoglycemia (repeated levels of less than _____ in an at-risk infant) is recommended.

2.6 mmol/L It has been known for some years that symptomatic hypoglycemia results in neuronal injury [33], making urgent intervention desirable in sick infants. Because there is no absolute level at which intervention is mandated, the proposed cut-off (repeated levels of less than 2.6 mmol/L in an at-risk infant) is recommended. CPS Hypoglycemia

At-risk babies who repeatedly have blood glucose levels of less than _____ despite subsequent feeding should also be considered for_____

2.6 mmol/L intravenous therapy CPS Hypoglycemia

Medical management of NEC fails in about _____% of patients with pneumatosis intestinalis at diagnosis; of these, ____% die

20-40 10-30 Neo Notes

Does first trimester use increase the risk of major malformations?

2005 meta-analysis No increase compared to baseline risk of 1-3% in general population >2005 inconsistent data ?paroxetine and cardiac malformations ?other teratogenicity Canadian data = incr risk of cardiac malformation on paroxetine >35mg/day 2007 meta-analysis OR of increased risk of cardiac malformations 1.72 and then re-analysis did not show an increase after reviewing more studies! 2010 study showed association w/ cardiac malformations (no specific one) and combo cardiac and non-cardiac malformations Big idea: Really hard to tell if paroxetine actually causes a problem! Different designs Control groups vary No rigorous consideration of confounders Small numbers of exposed Looking for dozens of associations but no adjustments Different doses for different illness severity 3 excellent review articles that look at 25studies each at least, study design, limitations, main findings SSRIs as a group are unlikely to be associated with increased risk of congenital malformations Evidence remains inconclusive for paroxetine and small increased risk of cardiac malformations CPS SSRI's

What is the rate of late preterm births in Canada? What is the major association with late preterm births? List 3 reasons why late preterm births increasing? What are the two leading causes of late preterm birth?

2006 - 6% live births in Canada were late preterm Most contribution is from multiple pregnancies 14% of late prem births = multiples Only 2% of all births Increasing multiple gestations Increasing obstetrical interventions Improved accuracy of gestation Obstetrical intervention needed No delivery of fetus threatens baby and mom Preterm labour Prematurity risks are low enough to proceed with spontaneous labour Large american centre study showed 80% preterm labour 20% obstetrical reason CPS facilitating discharge of late preterms infants

ANCS (steroids) should be given between _______ GA when early intensive care is a management option and, in the opinion of the obstetrical HCP, the risk of extremely preterm birth is high.

22 and 25 weeks

ANCS (steroids) should be given between _______ GA when early intensive care is a management option and, in the opinion of the obstetrical HCP, the risk of extremely preterm birth is high.

22 and 25 weeks CPS Counselling and management for anticipated extremely preterm birth

What is the percentage of pregnant women being treated? Leading to %infants in which disease prevented and mortality prevented?

22% pregnant women treated 0.2% infants disease prevented 0.01% infant mortality prevented 2 tx GBS is good = 22% mom's tx'd, 0.2% infant dz prevented CPS term infants at increased risk for early onset bacterial sepsis

>90% of term newborns pass meconium within the 1st ____; obstruction should be considered if no meconium by ____. 20% of VLBW infants do not pass meconium within the 1st ___. A breast-fed infant usually has frequent bowel movements, whereas a formula-fed infant may have 1-2 movements a day or every other day.

24 hr 24-36 hr 24 hr Neo Notes

Changing from 10% to 12.5% dextrose will increase intravenous intake by _____, as would a rate increase from _____. An increase from 100 mL/kg/day to 120 mL/kg/ day of 12.5% dextrose raises the glucose supply from ______. If this infusion rate fails to keep blood glucose levels at 2.6 mmol/L or higher, further investigation, specialist referral and/or pharmacological intervention (eg, ______) should be considered

25% 80 mL/kg/day to 100 mL/kg/day 8.7 mg/kg/min to 10.4 mg/kg/min intravenous glucagon CPS Hypoglycemia

When should you generally test "at risk infants? CPS Hypoglycemia

2h of age after initial feed Any time they are symptomatic CPS Hypoglycemia

If no significant complications, preterm infants tend to have same growth as term infants by the ______

2nd yr. VLBW infants may not catch up. (may benefit from recombinant human growth hormone therapy beginning at age 4 yr) Neo Notes

Canadian Pediatric Review 2018 Perinatal-Neonatal Medicine Pale pink newborn with HR of 88/min, actively gasping, good muscle tone, responds to nasal catheter with facial grimace. APGAR score at this moment is: 1. 3 2. 5 3. 7 4. 9

3. 7

Canadian Pediatric Review 2018 Perinatal-Neonatal Medicine Baby C, 26 weeks, 900g infant, now 20 minutes old is on CPAP PEEP 6 in 28% oxygen. The medical student on the team asks whether you plan to intubate the baby and administer surfactant. Which of the following is true? 1. Surfactant increases the incidence of air leaks in RDS 2. Administration of exogenous surfactant will inhibit endogenous surfactant production, this is why a trial of CPAP is first attempted 3. Administration of surfactant improves arterial oxygenation 4. Surfactant administration reduces the incidence and severity of chronic lung disease

3. Administration of surfactant improves arterial oxygenation RDS treatment • Transfer <32 week GA to tertiary centre • Maternal antenatal corticosteroids - For GA <34 weeks - Decreases severity of RDS, IVH, & mortality • CPAP / ventilation (improve FRC) • Surfactant therapy - Decreases mortality, pneumothorax, PIE - Decreases duration of vent support, length of stay, hospital costs - No effect on IVH, BPD, NEC, ROP Surfactant • Indications of exogenous surfactant via ETT: • Intubated preterm with RDS • Meconium aspiration sydrome, FiO2 >50%, • Consider in sick baby with pneumonia or pulmonary hemorrhage, when OI >15 [OI= FiO2xMAP / PaO2] • Consider in <29 wks GA before transport (outborn) - Timing: early is better; Natural better than synthetic • May repeat dose in 1st 72 hours (FiO2 >30%) • As early as 2h, average 4-6h after (no more than 3 doses) - Close monitoring: wean ventilation, extubation to CPAP within one hour - Risks: pneumothorax, bradycardia, blocked tube, hemorrhage (rare)

Canadian Pediatric Review 2018 Perinatal-Neonatal Medicine All of the following are recognized causes of 'floppy baby' except: 1. Trisomy 21 2. Zellweger syndrome 3. Becker muscular dystrophy 4. Spinal muscular atropy 5. Prader Willi syndrome

3. Becker muscular dystrophy (later)

Following intubation the baby is placed on settings of PIP 18, PEEP 5, rate of 40, still in 100% to achieve saturations of 88-90%. She has frequent desaturations to the 50% and is labile with handling. What is the next most appropriate action? 1. Increase the rate to 60 to improve oxygenation 2. Increase the PEEP to 7 to improve CO2 clearance 3. Continue ongoing sedation and muscle relaxation to improve ventilation and oxygenation 4. Give a small bolus of normal saline 5ml/kg, to avoid overloading the left side of the heart

3. Continue ongoing sedation and muscle relaxation to improve ventilation and oxygenation

Canadian Pediatric Review 2018 Perinatal-Neonatal Medicine Gas: 7.20/45/36/14/-9 In this baby, what is the likely etiology of the metabolic acidosis? 1. Pneumonia with systemic inflammatory response (vasodilatory shock), secondary to the meconium aspiration 2. Over-ventilation and squeezing the heart with high peak inspiratory pressure 3. Decreased left cardiac output due to high right sided pressures and subsequent poor filling 4. Adrenal insufficiency causing systemic hypotension as a consequence of the prolonged resuscitation

3. Decreased left cardiac output due to high right sided pressures and subsequent poor filling

Canadian Pediatric Review 2018 Perinatal-Neonatal Medicine You are called to assess a 4 week old, ex 25 week infant with vomiting of her last 2 NG feeds. Abdomen is distended and tender. You place the baby NPO, start antibiotics and order abdominal xrays. Which of the following are not radiological features of necrotizing enterocolitis? 1. Pneumatosis intestinalis 2. Portal venous air 3. Double bubble sign 4. Pneumoperitoneum

3. Double bubble sign NEC • Inflammatory disease with ulceration - terminal ileum, sigmoid colon (most common) • Clinical diagnosis (gold standard - pathology) - Lethargy, apnea, temp instability, bile-stained aspirates, - Abdominal distension, blood / mucous pr, shock • Risk factors: - Prematurity, ischemia (asphyxia, CHD, PDA, severe IUGR, exchange transfusions), complication Hirsprungs, Infection - Feeding (breast milk protective) • Treatment - Supportive, NPO, NG decompress, antibiotics +/- surgery • Long term - Short gut (surgical), stenosis/obstruction, recurrence

Canadian Pediatric Review 2018 Perinatal-Neonatal Medicine On day 1 of life, you are managing a 39 week infant with APGARS 3, 8. Growth parameters: Weight 2.1kg and HC 34cm. Current fluids at 10 hours of age are D12.5W at 100ml/kg/day. Serum glucose is now 1.8. What is the baby's GIR and what is the likely diagnosis? 1. GIR: 8-9 mg/kg/min; Diagnosis: inborn error of glucose metabolism 2. GIR 10-11 mg/kg/min; Diagnosis: inborn error of glucose metabolism 3. GIR 8-9mg/kg/min; Diagnosis: hyperinsulinism 4. GIR 10-11 mg/kg/min; Diagnosis: hyperinsulinism

3. GIR 8-9mg/kg/min; Diagnosis: hyperinsulinism (4-6 is D10 at NL 60-80 as a reference...) Glucose Infusion Rate = % of dextrose being infused x rate (ml/hr)/body weight (in kg) x 6 Glucose Infusion Rate (mg/kg/min) = IV rate (ml/kg/day) x % of dextrose/144

Canadian Pediatric Review 2018 Perinatal-Neonatal Medicine Which of the following are not associated with the harmful effects of oxygen? 1. Retinopathy of prematurity 2. Hypoxic ischemic encephalopathy 3. Hemolytic disease of the newborn 4. Cerebral palsy

3. Hemolytic disease of the newborn

Canadian Pediatric Review 2018 Perinatal-Neonatal Medicine An 18 hour old, Caucasian, female infant develops abdominal distension. On history: SVD, 37 weeks, GBS negative, O+ mom. Maternal diabetes and hypertension. Normal Anatomy ultrasound. IPS negative. O/E: protuberant, firm, non-tender abdomen, patent anus, no other dysmorphic features. No stool since birth. AXR shows multiple dilated loops of bowel. Of the following, a contrast enema would most likely confirm a diagnosis of: 1. Colonic atresia 2. Hirschprung's disease 3. Hypoplastic left colon syndrome 4. Midgut volvulus with malrotation

3. Hypoplastic left colon syndrome

Canadian Pediatric Review 2018 Perinatal-Neonatal Medicine 4 hours later, the ventilator settings are PIP 20 PEEP 6 rate 40, FiO2 80%. Arterial gas: 7.16/65/135/21/-4. What is the most appropriate ventilator setting change at this time? 1. Increase PIP and decrease FiO2 2. Increase PEEP and decrease FiO2 3. Increase rate and decrease FiO2 4. Decrease FiO2

3. Increase rate and decrease FiO2

Canadian Pediatric Review 2018 Perinatal-Neonatal Medicine Which of the following is correct regarding Vitamin K in the newborn? 1. Is an essential cofactor for the synthesis of coagulation factors II, VIII, IX, X 2. Is readily transported across the placenta 3. Is present in cow's milk at higher concentration than in breast milk 4. Single oral dose after delivery prevents hemorrhagic disease of the newborn

3. Is present in cow's milk at higher concentration than in breast milk 1972 (10, 9, 7, 2) 1. 7 not 8 (1972) 2. no, doesn't cross the placenta bc it's too big 4. need 2 if giving PO dose

Canadian Pediatric Review 2018 Perinatal-Neonatal Medicine You are counseling a mother in pregnancy who is worried that her baby will need breathing support. Which of the following is not a significant risk factor for the development of respiratory distress syndrome in this case? 1. This is a triplet pregnancy 2. Mother's HbA1c is 8% 3. Mother reports heroin use 4. Mother's previous two babies were born at 30 weeks and 28 weeks gestation

3. Mother reports heroin use - opiates are actually protective of RDS 1. Multiples = preterm = RDS 2. RDS is risk factor in IDM 4. Preterms = inc'd risk of preterm babies in future = RDS

Canadian Pediatric Review 2018 Perinatal-Neonatal Medicine A 34 week infant was born to an A negative mother with anti-D antibodies. Maternal titers were up to 1:256 in the pregnancy. Cord hemoglobin was 90, with a bilirubin of 130. His jaundice and hemolysis were treated appropriately. On day 4 of life, he develops feeding intolerance and abdominal distension. These GI symptoms are most likely due to: 1. Acquired sepsis from multiple blood product exposure and invasive lines 2. Portal vein thrombosis secondary to invasive catheters causing portal hypertension and ascites 3. Necrotizing enterocolitis secondary to ischemia/bowel hypoperfusion as a result of anemia or during exchange transfusion 4. An inborn error of metabolism, causing neonatal hepatitis and hyperammonemia

3. Necrotizing enterocolitis secondary to ischemia/bowel hypoperfusion as a result of anemia or during exchange transfusion

Canadian Pediatric Review 2018 Perinatal-Neonatal Medicine You are on call and receive 4 simultaneous pages to see 4 different infants. Which do you prioritize to see first? 1. Newborn at 8 hours of age who has had no urine output since birth. 2. Newborn at 2 hours of age with a murmur noted on exam by the midwife, who just breastfed well. 3. Newborn awaiting discharge at 30 hours but nurses noted has not passed stool yet. 4. Newborn 10 minutes old with a respiratory rate of 50, with sats of 99% in room air, and no work of breathing.

3. Newborn awaiting discharge at 30 hours but nurses noted has not passed stool yet.

Reducing pain from surgery

3. Pain should be routinely assessed using a scale designed for postoperative or prolonged pain in newborns. 4. Opioids should be the basis for postoperative analgesia after major surgery in the absence of regional anesthesia. 5. Postoperative analgesia should be utilized as long as pain assessment scales document that it is required. 6. Acetaminophen can be used after surgery as an adjunct to regional anesthetics or opioids, but there are inadequate data on pharmacokinetics at gestational ages under 28 weeks to permit calculation of appropriate dosages. CPS Prevention and management of pain in the neonate

Canadian Pediatric Review 2018 Perinatal-Neonatal Medicine Which of the following is true regarding screening in preterm infants in Canada? 1. ROP examinations are recommended for all babies <2000g who were ever ventilated in greater than 40% oxygen 2. Head ultrasound to screen for IVH is recommended in babies < 34 weeks gestation 3. ROP examinations should be performed after 4 weeks of age or 31 weeks corrected 4. Renal ultrasounds should be performed to screen for nephrocalcinosis in babies born at <30 weeks

3. ROP examinations should be performed after 4 weeks of age or 31 weeks corrected

Canadian Pediatric Review 2018 Perinatal-Neonatal Medicine Which statement is correct regarding Neonatal Alloimmune Thrombocytopenia? 1. Incidence is 1 in 100 births 2. Mother is often also thrombocytopenic 3. Risk of intracranial hemorrhage highest during first 96 hours 4. IVIG is an ineffective treatment 5. Expect a higher platelet count than in autoimmune thrombocytopenia

3. Risk of intracranial hemorrhage highest during first 96 hours Thrombocytopenia • Causes - Infection: bacterial, TORCH - Neonatal alloimmune thrombocytopenia - Other maternal causes • toxemia, ITP, SLE, Drugs (hydralazine, thiazides) - Consumption: DIC, Kassabach-Merrit (hemangioma) - Syndromes: IUGR, TAR, Fanconi's - Bone marrow suppression: pancytopenia, leukemia • Treatment - Transfusion if platelets < 20-30 or bleeding or procedures - NAIT - IVIG, maternal washed or antigen negative platelets, follow-up CBCs - Consider brain imaging (r/o ICH) Anemia • Causes - Physiologic anemia (HbF) - Anemia of prematurity - Hemorrhage (feto-maternal, Twin-twin, internal) - Hemolysis • Immune (Coombs +) Rh, ABO, Minor blood groups, maternal autoimmune (SLE) • Non-immune (Coombs -) DIC, infection, G6PD, pyruvate kinase deficiency, hereditary spherocytosis, alpha-thal • Investigations: - CBC, Hb, Hct, blood groups, coagulation studies - Kleihauer test on maternal blood (for fetal Hb) • Treatment - Transfusion (single vs. exchange), IVIG (immune) - Monitor CBC, jaundice

Canadian Pediatric Review 2018 Perinatal-Neonatal Medicine Which of the following are false? 1. Blood group and antibody testing during pregnancy is recommended in all mothers and bilirubin screening is recommended in all babies in the first 72 hours 2. In babies with jaundice, whose mothers are blood group O, blood group and coombs testing is recommended 3. Supplementation with dextrose gel in breast fed babies is recommended in cases of mild jaundice 4. Breastfeeding should continue during phototherapy

3. Supplementation with dextrose gel in breast fed babies is recommended in cases of mild jaundice Causes of Neonatal Jaundice • Unconjugated - Dehydration - 'Breastmilk' - Infection - Polycythemia - Hemolysis - Endocrinopathies - Extravascular blood - Genetic disorders - Increased enterohepatic circulation (GI obstruction, delayed meconium) • Conjugated - Extrahepatic obstructive • Biliary atresia, choledochal cysts - Bacterial infection - TORCH - Neonatal hepatitis • viral, bacterial, parasitic, idiopathic - Metabolic: • alpha-1 antitrypsin, IEM, endocrinopathies, CF, Iron storage disease, bile acid synthesis defects - Cholecystasis syndromes • Dubin Johnson, Byler - Toxic: hyperalimentation • ↑production - ↑breakdown of fetal erythrocytes - High RBC mass • ↓hepatic excretion: - low [binding protein] - low glucuronyl transferase fn

Canadian Pediatric Review 2018 Perinatal-Neonatal Medicine With sedation, the baby is less labile and maintaining more stable saturations between 89-92%. Current settings are 20/7 rate 50 in 100% FiO2. The blood pressure is 40/19 and a gas from the arterial line is now: pH 7.20 PaCO2 45 PaO2 36 Bicarb 14 base deficit -9. Which of the following is correct? 1. There is a significant respiratory acidosis that needs to be corrected 2. The kidneys are over-compensating for the ventilator induced respiratory alkalosis 3. The babies biggest problem is severe hypoxemia 4. The gas cannot be interpreted because there is an air bubble in the sample

3. The babies biggest problem is severe hypoxemia

Canadian Pediatric Review 2018 Perinatal-Neonatal Medicine What is true about neonatal chest compressions? 1. They should be done using the 2 thumb technique at a rate of 5 compressions to 1 ventilation 2. Compressors should compress to a depth of 2/3 of the chest AP diameter 3. They are required after 30 seconds of PPV if HR is less than 60 beats per minute 4. They should be started if baby requires intubation and HR is still < 100 beats per minute

3. They are required after 30 seconds of PPV if HR is less than 60 beats per minute 1. 3:1 not 5:1 2. 1/3 diameter not 2/3 4. wrong order

every infant ≤_____ GA regardless of birth weight, as well as any infant with a birth weight _____ to be screened for ROP Who should you screen to minimize the risk of not screening a baby with treatable disease?

30 6/7 weeks GA ≤1250 g Up to 2000g if felt to have hi risk neonatal course: Prolonged respiratory ventilation and oxygen Hypotension with inotropes Respiratory disease severe and unstable CPS ROP

The expected response to iNO is rapid, occurring in less than _____ with a PaO2 increase _____ . If there is no response, the iNO dose may be increased up to _____

30 min ≥20 mmHg 40 ppm CPS iNO

How much may breastfeeding reduce hospital admissions in the first year of life? How does breastfeeding reduce the severity of resp illnesses?

30% 4mos of breastfeeding makes URTI symps 3x less severe CPS baby friendly

Without preventive measures, gonococcal ophthalmia occurs in _____ of infants exposed during delivery and may progress quickly to _____. Infants at increased risk for gonococcal ophthalmia are those _____.

30% to 50% corneal ulceration, perforation of the globe and permanent visual impairment whose mothers are at risk for sexually transmitted infections (STIs) CPS Preventing ophthalmia neonatorum

What type of hypothermia is used in HIE, when and what are the outcomes

34 degrees >36 wks 6 hours Mild therapeutic hypothermia to a rectal temperature of 34±0.5°C initiated as soon as possible within the first 6 h of life decreases mortality and severe longterm neurodevelopmental disabilities in infants with moderate HIE who are ≥36 weeks' gestational age. Cooling may be achieved by either total body or selective head cooling. As cooling is now considered a standard of care, infants ≥36 weeks' gestational age who are depressed at birth should be assessed to determine whether they meet the criteria for cooling. There is currently no evidence that therapeutic hypothermia offers any benefit to infants <36 weeks' gestational age. CPS Hypothermia in HIE

Current guidelines recommend screening pregnant women for GBS colonization at _____ weeks' GA and providing intrapartum antibiotic prophylaxis for those who screen positive as well as for those with _______.

35 weeks' to 37 GBS bacteruria or a previous GBS-infected infant CPS term infants at increased risk for early onset bacterial sepsis

preterm and SGA infants may be vulnerable to hypoglycemia up to _____ and perhaps later, particularly if regular feeds or intravenous infusions are not yet established. The inference is that screening of preterm and SGA infants can be discontinued at _____ of age if feeding is established and blood glucose is maintained at _____ or higher

36 h of age 36h 2.6 mmol/L CPS hypoglycemia

need for oxygen at _____, together with respiratory symptoms and compatible changes on chest radiograph is one dx of BPD

36 weeks' postmenstrual age (PMA) CPS Postnatal corticosteroids to treat/prevent BPD

<32 WK - Time point of assessment for Definition of Bronchopulmonary Dysplasia: Diagnostic Criteria - Mild vs Moderate vs Severe

36 wk PMA or discharge home Treatment with >21% oxygen for at least 28 days plus Mild BPD: Breathing room air at 36 wk PMA or discharge, whichever comes first Moderate BPD: Need[*] for <30% oxygen at 36 wk PMA or discharge, whichever comes first Severe BPD: Need[*] for ≥30% oxygen and/or positive pressure (PPV or NCPAP) at 36 wk PMA or discharge, whichever comes first Neo Notes

<32 WK - Time point of assessment for Definition of Bronchopulmonary Dysplasia: Diagnostic Criteria

36 wk PMA or discharge home Treatment with >21% oxygen for at least 28 days plus Neo Notes

Late preterm infants are infants who are premature (ie, they are born at a gestational age [GA] of less than _____

37 weeks Born at 34-36+6wks 238-258days inclusive (US definition is less one day) CPS facilitating discharge of late preterms infants

SIDS is the ____ leading cause of infant mortality and is the most common cause of postneonatal infant mortality. Since the initiation of the national Back to Sleep campaign in 1994, the rate of SIDS _____

3rd progressively declined and then leveled off in 2002 at 0.57/1,000 live births. PATHOLOGY: There are no autopsy findings pathognomonic of SIDS and no findings required for the diagnosis, although there are some common findings. Petechial hemorrhages are found in 68-95% of cases. Pulmonary edema is often present. Nearly ⅔of SIDS victims have structural evidence of pre-existing, chronic low-grade asphyxia, while other studies have identified biochemical markers of asphyxia. Neo Notes

How to wean iNO Following improvement in oxygenation and after a ______ period of stability, during which the inspired oxygen concentration is decreased to _____, or the OI falls to _____, the dose of iNO should be weaned. An accepted method of weaning is to decrease the dose by ____ at _____ intervals as long as the OI remains at ____. Once a dose of 5 ppm has been attained, the dose should be decreased more gradually, by _____ and discontinued at 1 ppm if the infant remains well oxygenated in ____ oxygen with PaO2 consistently ______

4 h to 6 h 60% to 80% ≤10 50% 4 h to 6 h ≤10 1 ppm every 4 h <60% >50 mmHg Wait 4-6h Assess OI and FiO2: OI <10 FiO2 60-80% Wean 50% q4-6h At iNO 5ppm, assess PaO2 and FiO2: PaO2 >50 FiO2 <60% Wean 1ppm q4h If deterioration occurs during weaning or after treatment has been discontinued, the dose should be increased to the previous level or iNO therapy should be restarted. Once the infant has improved, weaning should be slower, taking place over a 24 h to 48 h period. CPS iNO

CPS Recommendation: How long should you screen for ROP? CPS Recommendation: How should these kids be followed up long term? CPS Recommendation: What are the responsibilities of the institution regarding ROP?

4 reasons to stop ROP exams: Complete vascularization Regression of ROP CGA 45wks w/ no prethrehold disease or worse Zone 3 vascularization w/o zone 1 or 2 ROP They are at risk for visual acuity and disturbances regardless of treatment Long term ophtho follow up If looking after infants at risk, must have ROP screening available with appropriate criteria Whatever results of ROP need to be told to parents and they need to know that it is a risk for poor visual outcome even with therapy If transferred from the unit, need to make sure that ophtho followed up is continued at the appropriate times Discharge planning needs to include arrangements for ophtho exams in the future and parents need to understand these appointments are important. CPS ROP

Canadian Pediatric Review 2018 Perinatal-Neonatal Medicine At 1 hour of age, Baby C's FiO2 was increasing so she was intubated, given surfactant, and placed on PIP 15cm H20, PEEP 5cm H2O, and rate 50. They call you to the bedside because they are having difficulty. They show you the arterial blood gas: pH 7.30 PaCO2 46 PaO2 35 Bicarb 22. What is the most appropriate ventilator setting change at this time? 1. Increase PIP 2. Decrease PEEP 3. Increase rate 4. Increase FiO2

4. Increase FiO2

Canadian Pediatric Review 2018 Perinatal-Neonatal Medicine Baby Z was born at 39 weeks to a 29 yo G1 mom, GBS negative, ROM 20 hours, no maternal fever. Mom did not receive any medication. Baby was born via SVD with cord around the neck. Required stimulation and PPV for 30 seconds then cried, sats 99% in room air, HR 154, RR 50. The most appropriate management for this baby now is: 1. Admit to L2N for observation, at risk of sepsis and perinatal depression, vital signs q 4 hours, CBC and CRP at 4 hours 2. May go to post-natal ward with Mom, vital signs q 1hour x 3 hours then q2 hours for 24 hours 3. May go to post-natal ward with Mom, vital signs q 4 hours x 24 hours, CBC and CRP at 4 hours 4. May go to post-natal ward with Mom, vital signs q 4 hours x 24 hours, reassess before discharge

4. May go to post-natal ward with Mom, vital signs q 4 hours x 24 hours, reassess before discharge - well now. CBC/CRP will help you distinguish but is not sensitive/specific. reassessment is most important • Early onset sepsis - 1st 7 days • Maternal risk factors: - GBS, PROM >18h, temp >38 (additive) - Treatment: IV PenG, ampicillin, cefazolin >4 hours • Symptomatic infant - Full septic workup if strong suspicions EOS or signs of meningitis (sz, fontanelle, irritable, altered neuro) - Ampicillin + aminoglycoside • Well-appearing, at-risk infants: - CBC might be helpful after 4 hours (WBC < 5, ANC <1.5) - CRP serial measures might be useful, but poor sensitivity early in disease - Refer to algorithm Infection in the NICU • Late onset sepsis 1st month (usually nosocomial) - CONS, Staph aureus, enterococci - Preterm: also consider fungal, ureaplasma/mycoplasma - Rx aimed to particular organism, usually gram pos/gram neg • Related to unit colonization & resistance; ex. Cloxacillin/vancomycin; gentamicin/tobramycin; fluconazole/amphotericin; erythromycin • Later onset sepsis (beyond 1st month) - GBS, gram-negative bacilli, Strep pneumonia

Canadian Pediatric Review 2018 Perinatal-Neonatal Medicine Which of the following is not a clinical sign of a patent ductus arteriosis in a preterm infant? 1. Tachycardia 2. Intermittent systolic murmur 3. Wide pulse pressure 4. Oligemic lung fields on chest x-ray

4. Oligemic lung fields on chest x-ray PDA • Ductus arteriosis: closes by 5-7 days • Risks: prematurity, asphyxia, hypoxia • Clinical features: - Preterm: 1st - 2nd week; Term: 4-6 weeks - Bounding pulses, hyperdynamic precordium, loud second heart sound, initial systolic murmur upper left sternal edge then diastolic component - Cardiac decompensation: congestive heart failure, tachycardia, tachypnea, HSM, apnea, increased oxygen requirements • Investigations: - CXR: cardiomegaly, pulmonary edema (L→R shunt) - Echocardiography: size, patency, doppler flow • Treatment - Fluid management, furosemide, transfusion - Indomethacin: prostaglandin synthetase inhibitor • Contraindications: renal insufficiency, thrombocytopenia - Surgical ligation

Canadian Pediatric Review 2018 Perinatal-Neonatal Medicine Which of the following is correct regarding intrauterine growth restriction? 1. It is commonly associated with methadone use in pregnancy 2. IUGR infants at increased risk of hemolytic disease due to ABO incompatibility 3. SGA infants may be found to have leukocytosis and elevated CRP due to in-utero inflammation 4. SGA infants may be found to have polycythemia and thrombocytopenia on blood smear

4. SGA infants may be found to have polycythemia and thrombocytopenia on blood smear 1. cocaine not opiates 2. no 3.

Canadian Pediatric Review 2018 Perinatal-Neonatal Medicine All of the following are usually associated with congenital hypothyroidism except: 1. Prolonged jaundice 2. Umbilical hernia 3. Large tongue 4. Small anterior fontanelle 5. Hypotonia

4. Small anterior fontanelle

Canadian Pediatric Review 2018 Perinatal-Neonatal Medicine Which of the following is false regarding preparing for resuscitation of preterm infants? 1. The room should be pre-warmed to 26 degrees Celsius 2. Babies <32 weeks should be placed without drying into a polyethylene bag at the start of resuscitation 3. Initiation of resuscitation in 30% oxygen may be considered for babies <32 weeks gestation 4. Therapeutic hypothermia is indicated for infants <32 weeks if the 5 minute APGAR score remains <3 and cord gases are acidotic

4. Therapeutic hypothermia is indicated for infants <32 weeks if the 5 minute APGAR score remains <3 and cord gases are acidotic - not in <32 wks

Canadian Pediatric Review 2018 Perinatal-Neonatal Medicine You are preparing for delivery of twins with antenatal history of twin-twin transfusion syndrome unsuccessfully treated with laser ablation. On fetal sampling, Twin A had a hematocrit of .75, Twin B had a Hct .35. Which of the following is true? 1. Twin A likely had a history of oligohydramnios 2. Twin B is the recipient twin 3. Twin A is at risk of high output heart failure 4. Twin B is at risk of high output heart failure

4. Twin B is at risk of high output heart failure Hydrops fetalis • Definition: - Fluid in 2 or more fetal compartments • Causes - Immune: • hemolytic disease of the newborn - Non-immune: • Chronic anemia (alpha-thal) • Cardiac failure • Renal disease • TORCH - CMV (send pleural fluid) • Congenital malformations, • Genetic syndromes (Trisomies 21,18,13, Turner, Noonan) Polycythemia • Clinical features - Hyperviscosity S&S • CNS: apnea, lethargy, tremors, irritability, sz • Renal: RVT, decreased GFR, oliguria • Cardiopulm: PPHN • GI: NEC - Other: acidosis, hypoxia, thrombocytopenia - Hyperbilirubinemia - Hypoglycemia • Treatment: - Measure central Hct (venous/arterial) - Hct > 75% repeated or symptomatic - partial exchange transfusion (controversial - rarely done) - Volume = (actual - desired Hct) x wt (kg) x 90 actual Hct

Canadian Pediatric Review 2018 Perinatal-Neonatal Medicine Which of the following is least likely to be picked up using pulse oximetry screening? 1. Pulmonary atresia with intact septum 2. Total anomalous pulmonary venous return 3. Truncus arteriosis 4. Unbalanced atrioventricular septal defect

4. Unbalanced atrioventricular septal defect

What is the risk of death in late premature infants? What is the rate of morbidity in late prems vs. term infants? What are the RR of late preterms dying from a) infection b) asphyxia c) SIDS compared to term counterparts?

4.5 times higher than term infants 13.3/1000 live births (late preterm) 3/1000 live births (term) Massachusetts 1998-2003 7 times higher morbidity vs. term 22% of late prem infants experienced life threatening neonatal morbidity CPS facilitating discharge of late preterms infants

What is the innocent declaration of 2005?

40 countries, including canada, called on governments to undertake programs to protect/promote/support breastfeeding Conclusions: All women should be enabled to practice exclusive breastfeeding Exclusively fed on breastmilk for 4-6mos Should continue until 2yrs and beyond with adequate complementary foods Create apporpriate environment of awareness and support so women can breastfeed in this manner. CPS baby friendly

neonates born before 31 weeks' gestational age (GA), approximately _____% develop some stage of ROP, 7% to 8% develop severe ROP and 5% to 6% require treatment

40% to 50 8 = 8evere 5 = fix it CPS ROP

Humidity of ____% reduces heat loss, prevents drying and irritation of resp mucosa, reduces insensible water loss

40-60 Neo Notes

Home monitoring can be safely discontinued after ____ if no events

44 wk Neo Notes

Transfusions should achieve a post-transfusion Hct of _____ and can be repeated every ______

45-55% 3-5 wk Neo Notes

When do neonates with early GBS disease present with symptoms?

5, 4's = 1/4 asymtomatic at birth, 95% by 24h, add 4% by 48's, Abx w/in 4 hrs, test at 4 wks left (test at 35-37 = 36 wks) 25->95->99% 1-2-4 Only ¼ are asymptomatic at birth 95% will have symptoms by 24hrs Additional 4% will have symptoms by 48hrs Symptoms of early onset GBS by 24hrs: Tachycardia Respiratory distress Temperature instability Poor perfusion abN WBC WBC TRIP CPS term infants at increased risk for early onset bacterial sepsis

Canadian Pediatric Review 2018 Perinatal-Neonatal Medicine A 1 week old male presenting to the emergency department with altered LOC and acidosis. What additional test should be included in the initial evaluation to help make the diagnosis and guide treatment? 1. Cortisol level 2. Serum ketones 3. Thyroid function tests 4. Serum lactate 5. Ammonia level

5. Ammonia level Clinical features suggestive of an inborn error of metabolism in the newborn Signs/symptoms • Encephalopathy • Seizures • Cardiomyopathy • Liver, GI (vomiting) • Eye (cataracts, retina) • Neonatal hydrops • Dysmorphic features Lab Abnormalities • Hypoglycemia • Acidosis • Hyperammonemia • Hyperbilirubinemia (prolonged/late)

Infants born to women with untreated chlamydia infection at delivery have a ____ risk of acquiring chlamydia, a ______ risk of developing neonatal conjunctivitis and a ______ risk of developing chlamydia pneumonia

50% 30% to 50% 10% to 20% Topical ocular prophylaxis does not prevent transmission from mother to infant, does not reliably prevent neonatal conjunctivitis and does not prevent pneumonia. Oral erythromycin prophylaxis of infants born to untreated mothers has been used in the past but has not been recommended since the association between erythromycin and pyloric stenosis was recognized. Routine prenatal screening for C trachomatis and treatment of identified infections during pregnancy is the preferred option for preventing neonatal conjunctivitis and other infections in newborns caused by this organism. Close clinical follow-up of exposed infants is recommended. The Public Health Agency of Canada recommends testing conjunctival and nasopharyngeal secretions of symptomatic infants and treating those who show positive results. CPS Preventing ophthalmia neonatorum

In hemolytic dz of the NB, Cord bilirubin is generally between _______; ______ bilirubin may also be elevated, especially if there was an intrauterine transfusion. ______ bilirubin rises rapidly to high levels in the 1st ______.

50-85 mmol/L conjugated Unconjugated 6 hr of life Neo notes

Studies of exclusively breastfed, appropriate-for-gestational-age, term babies, show that blood glucose falls immediately after birth from two-thirds of maternal levels to the 5th percentile of approximately _____ at 1 h of age

5th percentile 1.8 mmol/L There is a subsequent rise to levels over 2.0 mmol/L that is maintained for 72 h [6]. It is important to note that 12% to 14% of normal, appropriate-for-gestational-age, breastfed newborns have a blood glucose level of less than 2.6 mmol/L in the first three days of life CPS Hypoglycemia

Fetal growth can be assessed by ultrasonography as early as ____. The most accurate assessment of GA is by 1st-trimester ultrasound measurement of _____.

6-8 wk crown-rump length Check at 6-8 to see if your baby is gaining wt Neo Notes

When feeding interventions are offered for low blood glucose, levels should be rechecked in _____ to ensure that there has been a response.

60 min CPS Hypoglycemia

polycythemia, defined as a Hct ≥ ____.

65% Neo Notes

the maximal period of efficacy for ANCS is reached within ____ of the last dose

7 days CPS Counselling and management for anticipated extremely preterm birth

How many children are born premature every year in Canada?

7% of ~350,000 CPS Milk banking

♣ small premature babies may need ____mL/kg/day on day 1

70-80 ♣ VLBW have larger skin surface, immature skin, therefore increased insensible losses, also immature kidneys with decreased concentrating ability ♣ Daily weight, u/o, BUN, Na (clinical assessment unreliable) ♣ fluid overload may lead to edema, heart failure, PDA and BPD Neo Notes

Perinatal brachial plexus palsy (PBPP) -To foster realistic parental expectations, it is important to provide a clear explanation of the nerve injury and its potential sequelae. This should include the following information: - PBPP is not always preventable. - ____ of infants recover completely within the first month of life. - _____ experience permanent impairment and disability.

75% 25% CPS Perinatal brachial plexus palsy

A "physiologic" decrease in hemoglobin content is noticed at _____ in term infants and at about ____ in premature infants.

8-12 wk 6 wk Neo Notes

What is a woman's lifetime risk of developing depression? When are they at particular risk? Name complications of depression on pregnancy independent of treatment? Effect of depression in post partum period?

8-20% risk of developing symptoms of depression Childbearing years Risk of relapse during pregnancy Premature birth Low birth weight Respiratory distress Longer hospital stay Higher miscarriage rate Negative association with maternal child bonding Cognitive Emotional Behavioural consequences for the child CPS SSRI's

If increased enteral caloric intake is not effective, current practice is to provide intravenous glucose. The initial glucose infusion regime is _____ of _____, providing _____ of glucose, in keeping with studies that have measured glucose flux in newborns

80 mL/kg/day 10% dextrose 5.5 mg/kg/min CPS Hypoglycemia

The survival rate for intrauterine transfusions is ____%; the complication rate is 3%. Complications include ______

89 rupture of the membranes and preterm delivery, infection, fetal distress requiring emergency cesarean section, and perinatal death. Neo Notes

How do TcBs compare to TSBs?

95% CI ~37-78micromol/L depending on machine Ie. TcB = 100 TsB = ~137 Big idea: You need > ~37-78 micromol/L lower than the threshold to not be worried CPS Hyperbili

Prognosis of Mec Aspiration

: residual lung problems are rare, but include symptomatic cough, wheezing, and persistent hyperinflation for up to 5-10 yr o ultimate prognosis depends on the extent of CNS injury from asphyxia and the presence of associated problems such as pulmonary hypertension Neo Notes

OLIGOHYDRAMNIOS

<0.5L or AFI<5cm, 1-3% Remember = Need 5 to stay alive Intrauterine growth restriction Fetal anomalies Twin-twin transfusion (donor) Amniotic fluid leak Renal agenesis (Potter syndrome) Prune-belly syndrome Pulmonary hypoplasia Indomethacin, Angiotensin-converting enzyme inhibitors Neo Notes

CPS recommendation: when should at risk infants be intervend?

<1.8mmol/l after one effective feed <2 mmol/l at subsequent feeds <2.6mmol/l consistently Symptomatic + <2.6mmol/L Also need to look into why CPS Hypoglycemia

Low birth wt

<2500g not GA-dependent

What are characteristics of a newborn that would make them unsuitable for kangaroo care?

<27wks with: High humidification needed Chest/abdo wall or neural tube defects needing to be sterile Long abds w/ long recovery periods Immediately post op Hemodynamic instability (wide bp, swinging brady) CPS Kangeroo Care

Canadian Pediatric Review 2018 Perinatal-Neonatal Medicine Plastic Bags at Delivery (2010)

<28 wk GA now <32 weeks recommendation Don't dry prior to placing in wrap Hypothermia (<35C) = Increased mortality in preterms Decreased surfactant production Increased oxygen consumption Hypoglycemia ... avoid hyperthermia too

Recurrence rates of apnea of prematurity were higher for infants _____ GA and for infants in whom the last spell occurred at _____ PMA. For infants <26 weeks' GA, 13 days were required for 95% to remain apnea-free.

<30 weeks' >36 weeks' CPS facilitating discharge of preterms infants

CPS recommendation: At what getational age should a mother with threatened pre-term labour be sent to a tertiary care centre?

<32wks CPS surfactant

Which infants should not be routinely cooled?

<36wks >6h old Head trauma/bleed = contraindication Severe encephalopathy, congenital anomalies, abN chromosomes Need tertiary care consultation Evidence of severe head trauma or intracranial bleeding is a contraindication to cooling. There are currently no published studies showing that cooling infants >6 h of age or <36 weeks' gestational age is beneficial. Initiation of cooling for infants with very severe encephalopathy, congenital anomalies or abnormal chromosomes is best performed in consultation with a tertiary centre. CPS Hypothermia in HIE

Umbilical artery pH _____ has been associated with greater need for resuscitation and a higher incidence of complications.

<7.0 Neo Notes

3rd trimester exposure to medications/drug =

= abnormal organ or enzyme function Neo Notes

Very low birth weight

= less than 1500 g (account for >50 pecent neonatal deaths) Neo Notes

'Malignant TTN'

= reported in infants born by elective cesarean section who initially present with transient tachypnea that later develop refractory hypoxemia due to pulmonary hypertension and require ECMO support. Neo Notes

>32 WK - Time point of assessment for Definition of Bronchopulmonary Dysplasia: Diagnostic Criteria

>28 days but <56 days postnatal age or discharge home. Treatment with 21% oxygen for at least 28 days plus Neo Notes

NRP - Who can use a laryngeal mask?

>34wks >2000g (ie. If you can use a 3 tube, you can use an LMA) 2013-2014 CPS summaries - Neonatal resuscitation guidelines update: a case-based review

What do you do if mom is GBS positive?

>35wks AND otherwise well: 4hrs of PENICILLIN No therapy ?early discharge at 24h if appropriate access to community health care resources, possible immediate transport to health care facility if signs of sepsis appear No abx or <4hrs or 4hrs of NOT penicillin: CBC + diff Monitoring for 24-48h (risk of sign. Dx no justify empiric treatment 1% risk, only ¼ asymp at birth May discharge at 24h if looks well CPS term infants at increased risk for early onset bacterial sepsis

What do you define as short term neonatal morbidities that could lead to long term disability? (4) CPS Counselling and management for anticipated extremely preterm birth

>grade 3 IVH PVL (periventricular leukomalacia) ROP CLD/BPD CPS Counselling and management for anticipated extremely preterm birth

Does circumcision affect the rate of penile problems in males?

?looks like it decreases rates of penile problems overall ?more problems when younger, but less when older Penile problems later in uncircumcised penis' is it because of forced retraction of prepuce leading to tears, scarring, pain, paraphimosis? No studies to see what the effect of this is Penile problems later in uncirumcised penis' is it because of poor hygiene? No good studies of this either Ie. Retracting foreskin while bathing CPS circumcision

How should cooled infants be followed up?

@18-24mos Multidisciplinary team Look at: Psycho ed Motor Auditory Cognitive outcomes CPS Hypothermia in HIE

TERATOGENS Category A Category B

A - no risk = A'ok B - - no/some risk shown in animal studies but no human studies - B careful because it might do something but CCC (cat c) animal studies that do show risk Neo Notes

very premature infants - Does the volume of blood transfused reduce the need for further PRBC transfusions?

A higher volume of transfused blood (20 mL/kg) should be considered when transfusing a preterm baby, if the hemodynamic and respiratory status of the patient permits Transfusing a higher volume of blood may decrease the risk for exposure to >1 donor CPS Minimizing blood loss and the need for transfusions in very premature infants

Managing newborns exposed to N gonorrhoeae

A system should be established to ensure that all infants born to mothers found to have untreated N gonorrhoeae infection at delivery are treated. If the mother's test results are not available at discharge, a plan must be in place to ensure that she can be contacted promptly if the results are positive. The mother must also be advised to watch her infant for eye discharge in the first week of life and told whom to contact immediately if this symptom develops, or if the child is unwell in any way. When there is doubt about maternal compliance with this recommendation and the mother is considered to be at risk for gonococcal infection, administering one dose of ceftriaxone should be considered for the infant before discharge. Infants born to women with untreated N gonorrhoeae infection at the time of delivery, including those born by Caesarian section, should be tested and treated immediately without waiting for test results - Infants exposed to N gonorrhoeae who appear to be healthy at birth, both term and preterm, should have a conjunctival culture for N gonorrhoeae and receive a single dose of ceftriaxone (50 mg/kg to a maximum of 125 mg) intravenously or intramuscularly. The preferred diluent for intramuscular ceftriaxone is 1% lidocaine without epinephrine (0.45 mL/125 mg). This intervention is both safe and effective. Biliary stasis from ceftriaxone is not considered to be a risk with a single dose. (Ceftriaxone is contraindicated in newborns receiving intravenous calcium. A single dose of cefotaxime [100 mg/kg given intravenously or intramuscularly] is an acceptable alternative.) - If the exposed infant is unwell in any way, blood and cerebrospinal fluid cultures should also be performed. Infants with established gonococcal disease require additional investigation and therapy in consultation with a specialist in paediatric infectious diseases CPS Preventing ophthalmia neonatorum

71. 4.4 kg on day of life 2, in the NICU due to profound hypotonia requiring tube feeds. Has swelling of the hands and feet and bilateral undescended testes. Likely Cause: A. Noonan B. SMA C. Congenital myotonic dystrophy D. Prader-Willi

A. Noonan Noonan: lymphedema at birth, cryptorchidism with infertility; growth restriction

PREP 2015 Question 93 A 5-day-old term newborn is brought to the emergency department for the evaluation of jerking motions of his arms and legs. The newborn was delivered vaginally weighing 3,500 g and discharged to home 36 hours after delivery. He has been exclusively breastfed, with an increasingly poor suck, irritability, and decreased activity noted by the mother over the past 24 hours. Upon admission to the emergency department, his weight is 3,150 g and his vital signs are normal. A physical examination reveals an inconsolable, hypotonic newborn with marked jaundice that involves his legs and the palms of his hands. You note a brief jerking of his right arm, followed by bicycling movements of his upper and lower extremities. Of the following, the MOST likely cause of the jerking is A. acute bilirubin encephalopathy B. benign neonatal myoclonus C. herpes simplex encephalitis D. hypocalcemia E. hypoglycemia

A. acute bilirubin encephalopathy The jerking motions and bicycling movements of the newborn in the vignette represent findings associated with acute bilirubin encephalopathy (ABE). The newborn has marked jaundice of his legs and his palms, which suggests his serum bilirubin level is greater than 15 mg/dL (256.5 μmol/L). His clinical findings of lethargy, poor tone, irritability, and poor feeding are typically seen with ABE. The clinical signs of ABE in the neonatal period represent a spectrum. The earliest findings are often subtle and nonspecific. Moderate ABE manifests with arching of the neck and trunk, increasing lethargy, decreased feeding, and irritability with a shrill cry. Intervention at these early phases may prevent the sequelae of kernicterus. Worsening signs suggestive of severe ABE include bicycling movements of the arms and legs, inconsolable crying, inability to eat, fever, seizures, and coma. Active intervention at this point is unlikely to affect the risk of the development of kernicterus. Identification of infants at risk of developing ABE and subsequent kernicterus sequelae is essential. The American Academy of Pediatrics clinical practice guideline for the management of hyperbilirubinemia in the newborn infant of 35 weeks or longer gestation provides a framework that assists in the early identification of at-risk infants. These guidelines recommend that infants showing signs of moderate to severe ABE be given an immediate exchange transfusion, even if the total serum bilirubin is falling, acknowledging that the sequelae of kernicterus may be prevented if severe ABE does not develop. If suspected in the outpatient setting, infants exhibiting symptoms of ABE should be admitted directly to the hospital and not be referred to the emergency department where treatment may be delayed. The newborn in the vignette is manifesting the clinical findings of severe ABE and requires emergent admission for evaluation and treatment. An evaluation for the underlying cause of his hyperbilirubinemia should include a total and direct bilirubin, blood type, direct antiglobulin test (DAT), complete blood cell count, and glucose-6-phosphate dehydrogenase deficiency testing. It is appropriate to check blood glucose and serum calcium levels because of his history of poor feeding, but these are unlikely to be the cause of his jerking motions and bicycling movements. A sepsis evaluation may be performed because of his severe hyperbilirubinemia of unclear origin. Although not likely to be the cause of his clinical findings, herpes simplex should always be considered. The jerking movements of benign neonatal clonus are present only in sleep, thus excluding this as the cause of his abnormal movements. PREP Pearls The earliest findings of acute bilirubin encephalopathy (ABE) are often subtle and nonspecific. Moderate ABE manifests with arching of the neck and trunk, increasing lethargy, decreased feeding, and irritability with a shrill cry. Worsening signs suggestive of severe ABE include bicycling movements of the arms and legs, inconsolable crying, inability to eat, fever, seizures, and coma. Infants showing signs of moderate to severe ABE require an immediate exchange transfusion, even if the total serum bilirubin is falling, acknowledging that the sequelae of kernicterus may be prevented if severe ABE does not develop.

PREP 2015 Question 24 You are called to the delivery room to evaluate a newborn who is grunting, floppy, and mottled. The mother came to the emergency department in active labor with ruptured membranes. She received her prenatal care elsewhere; her group B Streptococcus status is unknown. She denies any history of sexually transmitted infections or genital lesions during pregnancy, and she has a history of anaphylaxis to penicillin. She received 1 dose of vancomycin prior to delivery. The baby's Apgar scores were 4 at one min and 6 at five min. Gestational age is estimated at 36 weeks. On physical examination, the vital signs show a temperature of 35.5°C, heart rate of 170 beats/min, and respiratory rate of 60 breaths/min with grunting and retractions. Respiratory effort is shallow with coarse breath sounds throughout. The heart has a regular rhythm without murmur, rub, or gallop; the capillary refill is about 3 seconds; and the skin contains scattered petechiae. A complete blood cell count includes the following results: White blood cell count, 2,700/µL (2.7 × 109/L), with 26% neutrophils, 17% bands, 46% lymphocytes, 8% monocytes, and 3% metamyelocytes Hemoglobin, 15.8 g/dL (158 g/L) Hematocrit, 49% (0.49) Platelets, 66 × 103/µL (66 × 109/L) Blood and urine culture specimens are obtained. The newborn is deemed too unstable for a lumbar puncture at this time. Of the following, the BEST choice for initial antimicrobial therapy is A. ampicillin and gentamicin B. ampicillin and ceftriaxone C. clindamycin and cefotaxime D. clindamycin and gentamicin E. vancomycin and gentamicin

A. ampicillin and gentamicin A number of factors in the vignette presented suggest a risk for early-onset sepsis in this newborn. These include the unknown duration of rupture of membranes, unknown group B Streptococcus status of the mother, inadequate maternal antibiotic therapy prior to delivery (only 1 dose of a nonpenicillin agent), and the baby's prematurity. In this setting, the baby's marked distress, poor respiratory effort, poor perfusion, and low white blood cell count with a left shift support the diagnosis of "sepsis syndrome" that requires rapid institution of supportive measures and antimicrobial therapy. Although group B Streptococcus is the leading cause of infection in this setting, Escherichia coli and other gram-negative organisms must be considered as well. Although less common, Listeria monocytogenes is another potential pathogen in early-onset sepsis of the newborn. Of the regimens listed, ampicillin and gentamicin provide the best coverage for these organisms with agents that are approved for use in neonates. Ampicillin and ceftriaxone cover the spectrum as well, but ceftriaxone is not indicated for use in neonates because of its high protein binding that could displace bilirubin, leading to an increased risk of kernicterus. The other regimens listed do not provide coverage for Listeria. Clindamycin does not provide coverage for gram-negative organisms. Vancomycin is indicated for coagulase-negative staphylococcal infection, which can be a pathogen in neonates, but infection with this organism does not present with early onset sepsis as seen in the infant in this vignette. Similarly, methicillin-resistant Staphylococcus aureus infection has been seen in neonates, but is not a likely cause of early onset infection. Ampicillin and cefotaxime would be another acceptable regimen in this setting, especially if ampicillin-resistant E coli are prevalent. In view of concerns over development of resistant bacteria and increased risk of candidiasis with prolonged use of cephalosporins, cefotaxime might be reserved for those with meningitis because of its excellent central nervous system penetration. PREP Pearls Ampicillin is effective against Listeria. No cephalosporin is effective against this organism. Early onset sepsis in the neonate is typically caused by group B Streptococcus or enteric gram-negative rods such as Escherichia coli. Listeria monocytogenes is a less common cause of this syndrome.

PREP 2015 Question 74 You are asked to assess a newborn who appears dusky 4 hours after birth. The mother underwent a natural childbirth at term, including immediate skin to skin bonding and delayed clamping of the umbilical cord. The pregnancy was uncomplicated. Physical examination reveals a ruddy, appropriate for gestational age newborn with perioral cyanosis, unlabored tachypnea, and mild jitters. The temperature is 37°C, the heart rate is 160 beats/min, the respiratory rate is 70 breaths/min, and the oxygenation saturation in room air is 95%. Further evaluation reveals a soft I/VI murmur at the left sternal border, clear breath sounds bilaterally, and no abdominal masses. The mother reports that her baby has not latched well onto the breast since birth. Of the following, the MOST appropriate next step in management is to obtain a(n) A. blood glucose level B. chest radiograph C. echocardiogram D. serum calcium E. venous hematocrit

A. blood glucose level A blood glucose value should be obtained immediately, as the newborn in the vignette is demonstrating both symptomatic hypoglycemia and polycythemia. Although the hypoglycemia may be secondary to polycythemia, glucose homeostasis must be achieved without delay. It is essential to monitor blood glucose values in neonates with polycythemia, because up to 40% may have hypoglycemia. If required, an intravenous glucose infusion may also assist in the management of polycythemia through hydration and dilution. Polycythemia, defined as a venous hematocrit value greater than 65% or a hemoglobin value greater than 22 g/dL (220 g/L), has been associated with delayed cord clamping. Recent studies have centered on the timing of cord clamping after birth because of the perceived neonatal benefits of increased blood volume and iron stores, which reduce the risk of physiologic anemia. A committee opinion of the American College of Obstetricians and Gynecologists does not provide the ideal timing for cord clamping because of the lack of evidence in full-term newborns. Some proponents advocate for delayed clamping up to 2 min after delivery due to the existing studies, acknowledging the increased risk of polycythemia associated with this practice. The most common features associated with polycythemia in the newborn include cyanosis, poor feeding, and hypoglycemia. The increased blood viscosity may lead to decreased microcirculatory flow. This diminished flow compromises organ function, leading to neurologic, metabolic, cardiac, renal, and respiratory complications (Item C74). A newborn with polycythemia may look cyanotic because the increased amount of hemoglobin coupled with normal oxygen extraction allows the deoxyhemoglobin level to exceed 5 g/dL (50 g/L). Although this level of deoxyhemoglobin gives the clinical appearance of cyanosis, the neonate is not hypoxic. The treatment of polycythemia remains controversial. Asymptomatic newborns with hematocrit values between 60% and 70% may be hydrated aggressively with enteral feedings or intravenous fluid, with careful monitoring for 24 to 48 hours. In symptomatic newborns with polycythemia, partial volume exchange transfusion may be performed to decrease the blood viscosity by replacing a calculated blood volume with normal saline. Necrotizing enterocolitis is a known complication of partial volume exchange transfusion. The newborn in the vignette is manifesting symptomatic hypoglycemia and his blood glucose should be checked immediately to determine if parenteral glucose is required. Further evaluation should then be pursued because of the clinical suspicion of polycythemia to guide further management, including obtaining a venous hematocrit and serum calcium level. Chest radiography may be performed at this time because of the tachypnea, as other causes should always be considered. Echocardiography may be considered if the clinical suspicion of congenital heart disease exists. PREP Pearls Polycythemia is most commonly associated with cyanosis, poor feeding, and hypoglycemia. Delayed cord clamping is associated with polycythemia.

2015 PREP Question 8 A 5-year-old boy is brought to your office by his mother for evaluation of behavioral difficulties. The mother reports he has always had a small head, motor and speech delays, and poor growth as compared to his peers. Additionally, he has poor coordination, low tone, impulsivity, hyperactivity, and attention difficulties. He has problems with reciprocal play with his peers in kindergarten. He has had significant problems with learning numbers and their usage. He has a known history of a ventricular septal defect that self-resolved as an infant. His weight is 14.5 kg (5th percentile), his height is 101 cm (5th percentile), and head circumference is 46 cm (less than 3rd percentile). Physical examination is remarkable for a shortened fifth finger, as well as the facies shown in Item Q8. (big smile, thin lips, low set ears) Of the following, the MOST likely diagnosis is A. fetal alcohol syndrome B. fetal anticonvulsant syndrome C. fragile X syndrome D. Rett syndrome E. Williams syndrome

A. fetal alcohol syndrome The patient in this vignette has classic fetal alcohol syndrome (FAS). To diagnose FAS, a physician should identify 3 cardinal features: abnormal facial features, growth deficiency, and central nervous system (CNS) problems. The possibility of prenatal alcohol exposure is not required to make the clinical diagnosis, but should be discussed with the patient's caregivers. Classic facial features include midfacial hypoplasia, smooth philtrum, thin upper lip, micrognathia, short palpebral fissures, epicanthal folds, small jaw, and microcephaly. Prenatal growth deficiency is typical as well as continued growth problems after birth. Height and weight are typically less than the tenth percentile. Central nervous system difficulties include attention-deficit/hyperactivity disorder (ADHD) problems, hypotonia, decreased impulse control, poor coordination, poor memory, learning disabilities (especially in math), speech and language delays, intellectual disability, impaired executive function, and sleep problems. Heart defects may occur and could include ventricular septal defect or atrial septal defect. There is no reported "safe" level of alcohol use during pregnancy, but large amounts of alcohol usage during any pregnancy and intake during the first 3 months of pregnancy do increase the risk of having a child with FAS. Children with fetal anticonvulsant syndrome typically display behavioral problems including ADHD, autistic features, learning difficulties, speech delay, and gross and fine motor delay. Clinical findings may include intrauterine growth retardation, microcephaly, hypertelorism, epicanthal folds, long philtrum with thin vermilion border, small mouth, flat nasal bridge, low-set ears, hypoplasia of the distal phalanges and nails, cleft palate, meningomyelocele, and congenital heart disease. The prevalence of neural tube defects in neonates exposed to valproic acid in utero is 10 to 20 times higher than in the general population. Mild to moderate intellectual deficits are common. Fragile X syndrome is an X-linked dominant disorder caused by the FMR1 gene, which is composed of a series of CGG repeats (normal number of repeats < 45). Premutation carriers for fragile X have 55 to 200 CGG repeats. Premutation carrier females can have premature ovarian failure and premutation carrier males can have fragile X tremor ataxia syndrome, which resembles Parkinson disease. Full mutation carriers have greater than 200 CGG repeats and are then clinically affected. Full mutation carriers have intellectual disability and the characteristic physical fi ndings. It can affect boys and girls, but boys are more severely affected. Affected boys typically display mild to moderate intellectual disability (only one-third of girls are intellectually disabled), anxiety, hyperactivity, attention-deficit disorder, autism spectrum disorder, and occasionally seizures. Physical features become more apparent with age and include large ears, a prominent jaw and forehead, pesplanus, long and narrow face, and macro-orchidism after puberty. Only half of girls have the physical characteristics. Rett syndrome is a neurodevelopmental disorder that predominantly affects girls. Patients with Rett syndrome typically have normal early growth and development followed by a characteristic slowing of development, loss of purposeful hand movements, distinctive hand wringing, decreased brain growth (acquired microcephaly), loss of coordination, developmental regression, autistic-like behaviors, seizures, and intellectual disability. Apraxia and breathing difficulties while awake are quite common. Williams syndrome is a genetic disorder caused by a deletion on the long arm of chromosome 7. These patients display a hoarse voice, cardiovascular anomaly, mild to moderate intellectual disability, unique personality characteristics, and distinctive facial features. Facial features include a broad forehead, short nose with a broad tip, full cheeks, and a wide mouth with full lips. The most common cardiovascular anomaly is supravalvular aortic stenosis. Affected individuals have engaging, friendly personalities. They do well with language skills and music, but have difficulty with visual-spatial tasks. Some patients exposed to alcohol in utero can have alcohol- related neurodevelopmental disorder. These patients lack the physical stigmata common with FAS, but have intellectual disabilities and problems with behavior and learning. Patients typically require specialized therapies and an individualized education plan in school. Therapies typically include speech, occupation, and physical therapy, as well as friendship training, specialized math tutoring, executive function training, and parenting and behavior management training. They oft en need involvement with a child psychiatrist to manage the behavioral problems. It is crucial for pediatricians to recognize the features consistent with alcohol exposure in utero in both FAS and alcohol-related neurodevelopmental disorder. PREP Pearls The classic phenotype for fetal alcohol syndrome includes growth deficiency, midfacial hypoplasia, smooth philtrum, thin upper lip, micrognathia, short palpebral fissures, epicanthal folds, small jaw, and microcephaly. There is no reported "safe" level of alcohol use during pregnancy. Neurologic problems in fetal alcohol syndrome include attention-deficit/hyperactivity problems, decreased impulse control, poor coordination, poor memory, learning disabilities (especially in math), speech and language delays, intellectual disability, impaired executive function, and sleep problems.

Disorders Associated with a Large Anterior Fontanel

AAn OPPen RReally TTTouchable anDerior fontantelle (Achondroplasia, Apert, Osteogenesis Imperfect, Prematurity/Phos low, Russel-Silver/Rubella, TTTrisomy/Thyroid, vit DD) Achondroplasia Russell-Silver syndrome Intrauterine growth retardation Athyrotic hypothyroidism Apert syndrome 13-, 18-, 21-trisomies Prematurity Hypophosphatasia Osteogenesis imperfecta Congenital rubella syndrome Hydrocephalus Vitamin D deficiency rickets Neo Notes

1. Description of a newborn with respiratory distress. What are your management options?

ABCs - Stimulate and suction, bag and mask, O2, HFNP, CPAP, SiPAP, intubation and ventilation. Investations - hyperoxia test, CXR, ECG, echo, blood cx, CBC, blood gas Treat as needed - antibiotics, PGE if suspecting congenital heart condition Support oxygentation -O2 Support ventilation -Respiratory support -PPV, intubation and ventilation Treat -Antibiotics in case of sepsis Investigate -Investigations -CXR, CBC, BCx

What are CPS recommendations in terms of ABO and G6PD testing in mom and baby?

ABO All mothers should be tested for ABOrh and abs during pregnancy If not done during pregnancy, take CORD blood and send for DAT If have clinical jaundice should do ABOrh and DAT if mom is O G6PD Selected at risk infants should be tested (family history, mediterranean, african american, middle east, south east asian) If have severe hyperbilirubinemia (>340) need to be tested. CPS Hyperbili

_____ is the most common cause of hemolytic disease of the newborn

ABO incompatibility Approximately 15% of live births are at risk, but manifestations of disease develop in only 0.3-2.2%. Major blood group incompatibility between the mother and fetus generally results in milder disease than does Rh incompatibility. Although ABO incompatibility occurs in 20-25% of pregnancies, hemolytic disease develops in only 10% of such offspring, and the infants are generally type A1, which is more antigenic than A2. Neo Notes

Does surfactant therapy help pulmonary hemorrhage?

ACTUALLY INCREASES incidence of pulmonary hemorrhage However bc blood products (hemoglobin/ fibrinogen) can have severe effects on surfactant production/fxn it can be used to treat Not enough RCT data likely bc it happens so quickly difficult to do study Most studies show that it helps oxygenation CPS surfactant

When should symptomatic hypoglycemia be treated?

ALWAYS! Symptomatic hypoglycemia causes neuronal damage Cut off proposed is 2.6 CPS Hypoglycemia

what is CONGENITAL AGANGLIONIC MEGACOLON (HIRSCHSPRUNG DS)

Abnormal innervation of the bowel, beginning in the internal anal sphincter and extending proximally to involve a variable length of gut. rectosigmoid in 75% of patients; 10% entire colon; total bowel aganglionosis is rare Neo Notes

Agents Acting on Pregnant Women that act as Teratogens

Accutane (isotretinoin): Facial-ear anomalies, heart disease, CNS anomalies Alcohol: Cardiac, CNS, limb anomalies; IUGR; developmental delay; attention deficits; autism Amphetamines: Congenital heart disease, IUGR, withdrawal Azathioprine, 6-MP, quinine: Abortion MP's (6-MP), AAAAre Quilling babies = 6-MP, Acutane, Alcohol, Amphetamines, Azathioprine, Quinine Neo Notes

What information have parents described as useful? (6)

Acknowledgment of stress/distress Likelihood of disability Likelihood of survival Anticipated problems Medical treatments The NICU experience CPS Counselling and management for anticipated extremely preterm birth

What should facilities and HCPs caring for mothers/infants/children do for breastfeeding? And why?

Aim to adhere to BFI practices Because known to increase initiation, duration, exclusivity of breastfeeding CPS baby friendly

What are situations where you might have secondary surfactant deficiency/dysfunction?

Albumin, blood, meconium inhibit surfactant Pulmonary hemorrhage, pneumonia, meconium aspiration syndrome CPS surfactant

What does pasteurization do to the immunologic components of breast milk?

All igM destroyed IgA = 67-100% still there IgG= 66-70% still there Lactoferrin (binds iron and decreases e.coli etc) = 20% still there Lysozyme enzyme (destroys bact cell walls) = 75% Cytokines = some reduced which may lead to more epidermal growth factor This can cause increased growth to the intestinal epithelial cells exposed to the donor breast milk CPS Milk banking

What are the CPS recommendations on who, when, how neonates should be screened for hyperbili? (8)

All neonates should have a TSB or TcB 24-72h of life TSB when clinically indicated or at metabolic screen TcB when discharged or 72h TSB venipuncture or capillary TcB okay for screening and if presents with jaundice. Needs to be interpreted with confidence intervals If visually jaundiced at 24h of age, get a bili then If not yet at treatment level, record time, TcB/TSB value, risk zone and give copy to parents, and arrange for follow up if necessary If discharged before 24h, needs r/a in 24h by someone trained in neonatal hyperbili and can do testing and admit for treatment if needed Need systematic approach to risk assessment for all infants before discharge and follow up care if infant develops jaundice If have prolonged hyperbili, need further investigations like conjugated bili CPS Hyperbili

Chlamydia and gonorrhea screening recs

All pregnant women should be screened for N gonorrhoeae and C trachomatis infections at the first prenatal visit. • Those who are infected should be treated during pregnancy, tested after treatment to ensure therapeutic success and tested again in the third trimester or, failing that, at time of delivery. Their partners should also be treated. Women who test negative but are at risk for acquiring infection later in pregnancy should be screened again in the third trimester. Rescreening for N gonorrhoeae, C trachomatis and other STIs should be considered in the third trimester for women who are not in a stable monogamous relationship. Pregnant women who were not screened during pregnancy should be screened for N gonorrhoeae and C trachomatis at delivery, using the most rapid tests available. CPS Preventing ophthalmia neonatorum

Are approved surfactant products safe? Is natural or synthetic surfactant better? What does the CPS recommend regarding natural vs. synthetic surfactant?

All produced according to microbiological standards Cannot comment regarding pathogens like prions Natural surfactant better: Decrease mortality Decrease oxygen and ventilatory support 3 days after treatment Decrease PIE/PTX (pulm air leak syndrome) No difference in BPD No data about long term outcomes This does not consider new synthetic surfactants that have better efficacy Natural surfactant is preferred over any synthetic surfactants approved at the time of the statement CPS surfactant

What is a surfactant exchange program?

Almost expiring surfactant from peripheral hospitals can be given to tertiary hospitals in exchange for longer lasting ones. CPS surfactant

What factors contribute to the success of ptx? CPS Hyperbili

Amount of skin exposed Intensity of appropriate wavelength at skin CPS Hyperbili

What is another risk of donor human milk?

Anaphylaxis But no higher than formula risk CPS Milk banking

Indications for RBC transfusion in newborns Anemia of prematurity

Anemia of prematurity The rapid decline in hemoglobin concentration is most severe in infants of shortest gestational age. More than 90% of extremely low birth weight infants are transfused, and they receive an estimated average of five transfusions each Suggested hemoglobin levels and hematocrit thresholds for transfusing infants with anemia of prematurity Postnatal age Respiratory support vs No respiratory support Week 1 115 vs 100 Week 2 100 vs 85 Week 3 and older 85 vs 75 Infants with cyanotic heart disease or similar hemodynamic disorders may require transfusion at higher hemoglobin thresholds. Transfusions should not be used to improve weight gain or to address apnea of prematurity when hemoglobin levels are already in excess of recommended levels for maintenance. CPS Red blood cell transfusion in newborn infants

What are the most common neural tube defects diagnosed at birth? What do you do with them?

Anencephaly = death Spina bifida = surgically corrected but usually motor function does not recover/improve CPS Folic Acid and Neural tube defects

Difference between - Anoxia, Hypoxia and Ischemia

Anoxia = complete lack of oxygen as a result of a number of primary causes - Hypoxia = decreased arterial concentration of oxygen - Ischemia = blood flow to cells or organs that is insufficient to maintain their normal function Neo Notes

What three medications should you consider in treatment of a mother at risk of extreme preterm birth?

Antenatal corticosteroids: Debated <24wks - not a lot of data (regression modelling says <25wks helpful Would offer it to women where active care of the infant is pursued Tocolytic Need to transfer to tertiary centre Need steroids to take effect Magnesium sulfate Glyceryl trinitrate Indomethacin is a prostaglandin synthetase inhibitor Nifedipine (Ca ch blockers) Ritodrine Nitric oxide is a potent endogenous hormone that facilitates smooth muscle relaxation in the vasculature, gut, and uterus Neuroprotection Magnesium sulphate CPS Counselling and management for anticipated extremely preterm birth

Which babies are at risk of neonatal hypoglycemia?

Any impairment of gluconeogenesis Using SGA and LGA bc representative of accelerated or restrictive growth CPS Hypoglycemia

Condition That May Cause Apparent Life-threatening Events or Sudden Death CNS

Arteriovenous malformation, seizures, congenital central hypoventilation, neuromuscular disorders (Werdnig-Hoffmann disease), Arnold-Chiari crisis, Leigh syndrome Neo Notes

What do you do for a mother that is facing extreme preterm birth?

Assess gestational age as best you can Transfer to tertiary care centre Consider medications: Steroids <24wks! Tocolytic for transfer Magnesium sulphate Talk about delivery CPS Counselling and management for anticipated extremely preterm birth

What are some suggested actions during and after death of baby?

Assure parents that it is normal to feel uncomfortable at this time Allow parents to spend as much time as they need with their baby Make repeated offers for holding the baby Name the baby Provide privacy, but do not abandon the parents Encourage relatives and friends to see the baby, according to the parents' wishes Warn about gasping and muscle contractions Reassure parents that their baby was not alone, not afraid and not in pain at the time of death Reassure parents that nothing more could be done Provide mementos to create memories Ensure that spiritual support is available Take pictures Explain the need and procedure for an autopsy Explain options and procedures for memorial services CPS perinatal loss

When should you be considering retreatment with surfactant? Why is this a problem?

At 2h if fiO2>30% Consider at 2h or more often 4-6h after the initial dose May need retreatment because some surfactant types are more prone to protein inactivation! CPS surfactant

CPS recommendation: When should asymptomatic babies be treated for hypoglycemia?

At risk babies (prem, sga, lga, idm) Check at 2h after one effective feed (should feed regularly afterwards) After 1st feed: <1.8mmol/L = IV dextrose After subsequent feeds: <2mmol/L = IV dextrose Repeated feeds: <2.6mmol/L = IV dextrose CPS Hypoglycemia

What is the most important indicator of PPV effectiveness?

Auscultating the precordial pulse and hearing a rise in HR 2013-2014 CPS summaries - Neonatal resuscitation guidelines update: a case-based review

what type of seizure syndrome: 2-4 days of age, remit at 2-15 wk of age; ocular deviation, tonic posturing, clonic jerks, motor automatisms.

Autosomal dominant benign familial neonatal seizures Neo Notes

What are the three phases of grief and mourning?

Avoidance or protest (hours - days): don't want to believe that it's gone Confrontation and disorganization: most intense grief, preoccupied with thoughts of the one lost Accommodation and reorganization (at least 2 years) CPS perinatal loss

PREP 2015 Question 55 You are called to assess a term newborn with tachypnea and temperature instability 2 hours after delivery. The maternal history is notable for 17 hours of ruptured membranes and unknown group B streptococcal status. Examination reveals a tachypneic infant with delayed capillary refill, mild grunting, and weak pulses. Vital signs include a heart rate of 180 beats/min, respiratory rate of 80 breaths/min, and a blood pressure of 50/20 mm Hg. The arterial blood sample values include a pH of 7.25, a PCO2 of 38 mm Hg, a PO2 of 70 mm Hg, bicarbonate of 16 mEq/L (16 mmol/L), lactate of 90.09 mg/dL (10 mmol/L), hemoglobin of 12 g/dL (120 g/L), and ionized calcium of 4 mg/dL (1 mmol/L). You initiate evaluation and treatment for possible sepsis. Of the following, the MOST appropriate next step in management is to A. administer hydrocortisone B. bolus normal saline C. infuse calcium gluconate D. infuse sodium bicarbonate E. transfuse packed red blood cells

B. bolus normal saline The newborn in the vignette has septic shock and requires volume expansion with normal saline to improve tissue perfusion. The history of 17 hours of ruptured membranes and temperature instability suggest neonatal sepsis, with the clinical findings of tachycardia, poor perfusion, and hypotension supporting evolving shock. The rapid recognition and treatment of septic shock, including the restoration of adequate tissue perfusion, is essential to improving outcome in affected neonates. Defining adequate perfusion in a neonate is difficult. Blood pressure continues to be the hemodynamic indicator of systemic perfusion, though it may not reflect actual tissue perfusion and oxygen delivery. The goal of maintaining the mean blood pressure greater than the gestational age in the first days after birth still remains the standard at many centers. Clinical indicators, including weak distal pulses and a capillary refill time greater than 2 seconds, may be seen with decreased tissue perfusion even before hypotension develops. Biochemical markers of the cellular effects of inadequate tissue perfusion include an increased serum lactate level and metabolic acidosis. Perfusion to the individual organs that falls below the ability of the organ to maintain blood flow will lead to decreased renal, hepatic, cardiac, and brain function. Increased oxygen consumption in an infant with septic shock may also lead to hypoglycemia if not recognized and treated. The initial step in the treatment of decreased perfusion and hypotension associated with septic shock in a full-term neonate or older preterm infant remains volume expansion with a crystalloid fluid bolus (normal saline or lactated Ringer solution). Recommendations for a preterm infant may differ based on center of care. Packed red blood cells should be reserved for the infant with evidence of acute volume loss or a hemoglobin value of 10 g/dL (100 g/L) or less. Hydrocortisone therapy is often a third-line agent for neonates who have septic shock and fail to respond to volume resuscitation and vasoactive medications. Although not the first step in the management of septic shock, the clinician should consider correction of hypocalcemia to optimize cardiac function. The newborn in the vignette has a low-normal ionized calcium level and should be monitored closely. Attempts to increase perfusion should be the first method used to correct metabolic acidosis in a neonate with septic shock, such as the newborn in the vignette. Sodium bicarbonate therapy should be reserved for correction of ongoing metabolic acidosis that is unresponsive to medical intervention in an intubated patient. PREP Pearls Poor tissue perfusion is associated with decreased tissue oxygenation, which leads to an increased serum lactate and metabolic acidosis. The full-term neonate presenting with septic shock requires volume expansion to improve tissue perfusion.

PREP 2015 Question 64 A 2,000-g, 34-week-gestation female newborn was admitted to the neonatal intensive care unit at 3 days of age because of jaundice, lethargy, and vomiting. The pregnancy was complicated by illicit drug abuse and poor prenatal care. Delivery was by cesarean birth secondary to maternal preeclampsia, and the Apgar scores were 6 and 7 at one and five minutes, respectively. She initially fed well with cow milk-based formula but was noted to develop jaundice at 24 hours after birth; the baby became increasingly difficult to feed, with multiple episodes of postprandial emesis. Because of these symptoms, a sepsis workup was performed and the baby was started on intravenous antibiotics. Physical examination demonstrates a lethargic, jaundiced infant. She has a temperature of 37.0°C, pulse rate of 130 beats/min, respiratory rate of 20 breaths/min, blood pressure of 88/56 mm Hg, and oxygen saturation of 97% on room air. The liver is palpated 5 cm below the right costal margin. The remainder of the examination findings are normal. Laboratory studies are obtained and include the following results: Total bilirubin, 15 mg/dL (256.5 mmol/L) Direct bilirubin, 5.5 mg/dL (94 mmol/L) Alanine aminotransferase, 205 U/L Aspartate aminotransferase, 170 U/L Prothrombin time, 21 s Partial thromboplastin time, 39 s International normalized ratio, 1.95 Urine test strip, positive for reducing substances Of the following, the MOST appropriate next diagnostic test is A. cell culture for cytomegalovirus from urine B. erythrocyte galactose-1-phosphate uridyltransferase C. polymerase chain reaction for herpes simplex virus in cerebrospinal fluid D. serum a-1 antitrypsin measurement with protease inhibitor typing E. urine succinylacetone

B. erythrocyte galactose-1-phosphate uridyltransferase The infant described in the vignette has direct hyperbilirubinemia, which is defined by a serum direct bilirubin concentration of more than 1.0 mg/dL (17.1 µmol/L) with total bilirubin values of less than 5.0 mg/dL (85.5 µmol/L) or greater than 20% of the total bilirubin for values greater than 5.0 mg/dL (85.5 µmol/L). Direct hyperbilirubinemia indicates cholestasis and is an abnormal finding that requires additional evaluation. In the child in the vignette, the clinical and laboratory findings do not suggest an obstructive cause of cholestasis. The infant's presentation with vomiting, lethargy, and significant hepatomegaly would be unusual in extrahepatic biliary atresia (EBA). Furthermore, this high level of total bilirubin (total/direct = 15/5.5 mg/dL) early in the neonatal course is atypical for EBA. When direct hyperbilirubinemia is diagnosed within the first few days after birth, particularly in association with other clinical symptoms, infectious and metabolic disorders must be considered. Thus, sepsis workup is an important first step in this neonate. Hepatic dysfunction (international normalized ratio of 1.95) with hepatomegaly and cholestasis during the first few days after birth (particularly, as in this case, within the first 24 hours after commencing feeding) suggest an inborn metabolic error. Although several metabolic disorders may present during the neonatal period with evidence of cholestasis and hepatic compromise, the associated finding of positive reducing substances in the urine indicate a carbohydrate metabolic disorder, the most prevalent being galactosemia. An erythrocyte cell galactose-1-phosphate uridyltransferase level will confirm the diagnosis. Galactosemia is a disorder of galactose metabolism that, untreated, leads to feeding problems, failure to thrive, life-threatening hepatocellular damage, bleeding, and sepsis. A lactose- or galactose-restricted diet initiated within the first 10 days after birth will result in a rapid resolution of symptoms and greatly reduce the risk of complications. Even with early dietary management, children with galactosemia remain at increased risk for developmental, cognitive, and motor delay. The diagnosis of galactosemia is established by measurement of erythrocyte galactose-1-phosphate uridyltransferase (GALT) activity, erythrocyte galactose-1-phosphate concentration, and GALT molecular genetic testing. In classic galactosemia, GALT activity is less than 5% of control values. Tyrosinemia type I must also be considered in neonates presenting with this constellation of clinical findings. Tyrosinemia type I results from a deficiency in fumarylacetoacetate hydrolase (FAH), the final enzymatic step in the tyrosine catabolic pathway. In affected patients, the intermediate compounds maleylacetoacetate and fumarylacetoacetate are converted to toxic metabolites succinylacetone and succinylacetoacetate, which are responsible for hepatorenal toxicity. The diagnosis is suggested by an elevated urine succinylacetone level. Succinylacetone can also inhibit the porphyrin synthesis pathway, leading to the accumulation of 5-aminolevulinate, a potent neurotoxin. Tyrosinemia type I usually presents during the neonatal period with liver involvement, marked by elevated transaminases, hypocholesterolemia, and hyperbilirubinemia. In the child in the vignette, the finding of positive urinary reducing substances suggests a carbohydrate, as opposed to an amino acid metabolic disorder. Direct hyperbilirubinemia in the neonatal period is most commonly identified during the evaluation of prolonged "physiological" jaundice, often in otherwise thriving infants. Therefore, in all infants with clinical jaundice persisting beyond 2 weeks of age direct reacting (conjugated) bilirubin should be measured. If the direct bilirubin level is elevated, biliary tract integrity should be assessed. Evaluation of hepatobiliary integrity is of paramount importance to determine if direct hyperbilirubinemia is the consequence of extrahepatic biliary tract obstruction (either EBA or choledochal cyst). Recent studies have shown the best surgical outcomes for infants with biliary atresia when the diagnosis is established by 30 to 45 days of age. A biliary tract ultrasound should be done initially to rule out choledochal cyst, and further biliary tract diagnostic evaluation, including percutaneous liver biopsy, may be performed when the infant reaches 1 month of age. Cholestasis may also result from intrahepatic bile duct paucity (eg, in α-1-antitrypsin deficiency and Alagille syndrome). Obtaining a serum α-1-antitrypsin level is part of this evaluation process. The "gold standard" for identifying extrahepatic biliary tract obstruction remains the percutaneous liver biopsy. Recent data indicate that biliary tract ultrasonography, conducted by an experienced ultrasonographer, may be helpful. The finding of a "triangular cord sign," which represents the fibrous remnant of an obliterated extrahepatic biliary tree, has been shown to be highly specific for biliary atresia. Other diagnostic modalities, including magnetic resonance cholangiopancreatography and endoscopic retrograde cholangiopancreatography, are currently being investigated. Abdominal computed tomography may suggest an absent or atretic gall bladder, but this imaging study has not proven to be of significant value in the evaluation of biliary atresia. Hepatobiliary scintigraphy, using a radiolabeled derivative of iminodiacetic acid, has long been touted as an important adjunct to the evaluation of neonatal cholestasis. Although definitive evidence of radiolabeled excretion into the small bowel may confirm biliary tract patency, the study has a significant incidence of both false-positive and false-negative results. PREP Pearls Direct hyperbilirubinemia in the first week after birth suggests a metabolic or infectious etiology. Hepatic dysfunction within a few days (often < 24 hours) of introducing breast milk or cow milk-based formulas suggests a diagnosis of galactosemia. Percutaneous liver biopsy remains the most sensitive and specific test for extrahepatic biliary atresia.

PREP 2015 Question 207 You are meeting with a first-time mother during her prenatal visit. She tells you that her pregnancy was unremarkable until 2 months ago, when ultrasonography showed that her daughter was smaller than expected. Her daughter's growth has been monitored closely since that time, but continues to be less than anticipated. The mother has had no medical problems during the pregnancy. She describes smoking 1 pack of cigarettes daily, with no drug or alcohol use. She did attempt to discontinue smoking, but was unable to stop. She is worried about her daughter's poor growth and asks if there are any other problems she may face after birth related to her smoking. Of the following, you are MOST likely to advise the expectant mother that her daughter is at increased risk for A. hearing loss B. increased attention problems C. moderate cognitive impairment D. nicotine withdrawal E. poor long-term growth

B. increased attention problems Infants whose mothers smoked cigarettes during pregnancy are at an increased risk of attention problems that extend into adulthood. Epidemiological studies have demonstrated that prenatal tobacco exposure is associated with many adverse behavioral outcomes that include attention problems, impulsivity, and hyperactivity. The link between cognition and smoking is much weaker, with some studies suggesting issues with language, reading, memory, and slightly lower IQ scores. Prenatal exposure to nicotine affects the development of neural pathways in the brain. Other compounds in smoke such as cyanide and bromide may further affect these pathways. These compounds may play a role in the adverse behavioral outcomes associated with prenatal smoking. Conversely, an increased risk of congenital abnormalities has not been seen with prenatal tobacco exposure, with the possible exception of oral facial clefts. Smoking during pregnancy has been associated with preterm birth, intrauterine growth restriction, and low birth weight. The decline in birth weight is correlated to the number of cigarettes smoked. Prenatal smoking does not appear to impact long-term growth, although some studies suggest an increased likelihood of childhood obesity, as reflected by a higher body mass index. Increased rates of sudden infant death syndrome, wheezing, and asthma have also been demonstrated in infants and children exposed to cigarette smoking during pregnancy. Exposed newborns do not experience nicotine withdrawal after delivery. Hearing and vision appear unaffected. PREP Pearls Smoking during pregnancy has been associated with preterm birth, intrauterine growth restriction, and low birth weight. Prenatal tobacco exposure is associated with many adverse behavioral outcomes that include attention problems, impulsivity, and hyperactivity. Increased rates of sudden infant death syndrome, wheezing, and asthma have been demonstrated in infants and children exposed to cigarette smoking during pregnancy.

PREP 2015 Question 112 The normal nursery staff calls you to assess a term newborn who has turned dusky twice after breastfeeding. The newborn was delivered 4 hours ago by scheduled elective cesarean delivery at term following an uneventful pregnancy. Artificial rupture of the membranes occurred at the time of delivery and revealed copious clear amniotic fluid. The newborn emerged vigorous, and no resuscitation was needed. The nurses report that the newborn has spit clear mucus several times since delivery, requiring bulb suctioning. An examination reveals a pink, well-perfused newborn with a heart rate of 140 beats/min, a respiratory rate of 70 breaths/min, and an oxygen saturation of 95% on room air. The newborn has clear lung fields with mild subcostal retractions and no murmur. Of the following, the MOST appropriate next step in management is to A. initiate nasal cannula oxygen supplementation B. insert an orogastric tube C. obtain a chest radiograph D. order an echocardiogram E. perform chest physiotherapy

B. insert an orogastric tube The newborn in the vignette should have an orogastric tube inserted to rule out esophageal atresia (EA). A fetus with EA can present in utero with polyhydramnios because of the inability to swallow amniotic fluid. When the membranes are ruptured before delivery, copious amniotic fluid may be seen. If not identified in the delivery room, newborns with EA will often have spitting, drooling, choking, and respiratory distress because of their continued inability to handle secretions or feedings. These findings should prompt a clinician to attempt to pass an orogastric tube. The inability to pass an orogastric tube into the stomach suggests an underlying diagnosis of EA. The diagnosis of EA may be confirmed with chest radiography, which demonstrates coiling of the catheter tube in the esophagus (Item C112A). Five types of congenital tracheoesophageal fistula (TEF) and EA have been described, with type C (EA with distal pouch fistula) occurring in approximately 85% of cases. Four types are associated with EA, which present in the newborn period (Item C112B). Occurring in only 4% of cases, type E (H-type fistula with a patent esophagus) may present later in infancy with refractory bronchospasm and recurrent pneumonias. More than 50% of infants with EA have other associated anomalies, with VACTERL (vertebral, anorectal, cardiac, tracheal, esophageal, renal, limb) syndrome most common. Prompt identification of EA in full-term newborns leads to a survival rate greater than 95% if no cardiac disease is present. The short- and long-term complications of TEF/EA are related both to the underlying structural abnormalities and the consequences of the surgical correction (Item C112C). A newborn with the diagnosis of EA requires transfer to a tertiary center with a pediatric surgical team. Stabilization at the referring center is focused on the maintenance of a patent airway and prevention of aspiration pneumonia. After inserting an orogastric tube in the newborn in the vignette, chest radiography should then be performed to demonstrate the location of the tube in the esophagus. Supplemental oxygen may be required if an infant with EA is hypoxic, but the newborn in the vignette has an acceptable oxygen saturation value of 95% in room air. Endotracheal intubation should be avoided, as severe gastric distention may arise if a distal pouch fistula is present. Echocardiography should be performed at the tertiary center because up to 35% of infants with EA are found to have structural heart disease. Chest physiotherapy has no role in the care of the newborn with EA. PREP Pearls Esophageal atresia should be considered in newborns with spitting, drooling, choking, and respiratory distress because of their continued inability to handle secretions or feedings. The inability to pass an orogastric tube into the stomach suggests an underlying diagnosis of esophageal atresia.

PREP 2015 Question 150 You are called by the nursery staff to inform you of a newborn admitted to your service. The mother presented in labor at 36 3/7 weeks of gestation after receiving good prenatal care that was complicated only by unknown group B Streptococcus screening status. She received 1 dose of cefazolin 4 hours prior to delivery. Spontaneous rupture of the membranes occurred 8 hours prior to delivery and the mother remained afebrile. The newborn appears clinically well 1 hour after delivery, with a glucose value of 51 mg/dL (2.8 mmol/L). The mother is requesting discharge 24 hours after delivery. Of the following, the MOST appropriate initial management is to A. observe for 24 hours with no additional evaluation B. observe for 48 hours with no additional evaluation C. obtain a blood culture and complete blood cell count with differential and observe for 24 hours D. obtain a blood culture and complete blood cell count with differential and observe for 48 hours E. obtain a blood culture and complete blood cell count with differential and treat with antibiotics

B. observe for 48 hours with no additional evaluation The newborn in the vignette should be observed in the hospital for 48 hours with no additional evaluation unless clinical signs of sepsis develop. Mothers who are admitted in labor before 37 weeks and 0 days with unknown group B streptococcal (GBS) status should receive antibiotic prophylaxis. Late preterm newborns who appear well at birth may simply be observed for 48 hours or more in the hospital if their mother received adequate antibiotic therapy starting at 4 or more hours before delivery, had ruptured membranes less than 18 hours, and did not develop chorioamnionitis. Indications for diagnostic evaluation and antibiotic treatment of a newborn at risk for early-onset GBS, regardless of gestational age, include any clinical signs of neonatal sepsis or maternal chorioamnionitis. Chorioamnionitis has commonly been defined in the literature as the development of a maternal fever associated with 2 of the following clinical findings: uterine tenderness, maternal or fetal tachycardia, and foul smelling, purulent amniotic fluid. Maternal fever is seen in 95% to 100% of cases of chorioamnionitis. The US Centers for Disease Control and Prevention cite a maternal temperature of more than 100.4°F (38°C) as an intrapartum sepsis risk factor that warrants maternal evaluation and potential management for presumed chorioamnionitis as determined by the obstetric providers. Well-appearing newborns exposed to maternal chorioamnionitis should undergo a limited evaluation (blood culture and complete blood cell count with differential and platelets) and antibiotics should be initiated. Any newborn with signs of sepsis regardless of intrapartum risk factors should undergo a full diagnostic evaluation (blood culture, complete blood count with differential and platelets, chest radiography if respiratory symptoms are present, and lumbar puncture if cardiorespiratory status is stable) and immediately be given antibiotic therapy. The revised guidelines for the prevention of perinatal GBS disease were published in 2010, including an algorithm for neonatal management (Item C150). This algorithm defines management and treatment strategies based on clinical presentation, maternal chorioamnionitis, gestational age, adequacy of antibiotic treatment, and duration of membrane rupture. Review of this algorithm supports observation of the late preterm newborn in the vignette for 48 hours. Errata: Content revised for clarification. 6/2015 PREP Pearls Indications for diagnostic evaluation and antibiotic treatment of a newborn at risk for early-onset group B streptococcal disease include any clinical signs of neonatal sepsis or maternal chorioamnionitis. Late preterm newborns who appear well at birth may simply be observed for 48 hours or more in the hospital if their mother received adequate antibiotic therapy for 4 hours or more hours before delivery, had ruptured membranes less than 18 hours, and did not develop chorioamnionitis.

In Canada, what percentage of children are disability free when assessed at 18-24mos if they were prem? CPS Counselling and management for anticipated extremely preterm birth

Basically: @23wks = 30-50% (50-70% w/ disability) @24wks = 40-50% (50-60% w/ disability) @25wks = 40-70% (30-60% w/ disability) CPS Counselling and management for anticipated extremely preterm birth

How is the donor breast milk processed once it reaches the milk bank?

Batch from up to four different mothers Milk thawed cx taken Holder pasteurization @62.5 x 30mins in industrial pasteurizer Reculture milk Any positive pathogen or >104CFU/cc of skin flora, discard Freeze the milk When ordered, mil tranported, thawed, dispensed CPS Milk banking

In what context could you possibly use steroids for CLD and why?

Bc w/ late dex therapy unknown if benefits outweigh risks for prolonged ventilator dependent or high risk CLD Only use if: Parental consent Ventilator dependent, Severe CLD, hi risk for CLD Low dose dex (0.15-0.2mg/kg/day) x 7-10d (w/ taper) Can consider inhaled steroids but effective dose and duration unknown CPS Postnatal corticosteroids to treat/prevent BPD

NRP - Why is pulse oximetry being used?

Because people evaluation is unreliable 2013-2014 CPS summaries - Neonatal resuscitation guidelines update: a case-based review

What does the CPS recommend re: intervention at 22wks? What does the CPS recommend re: discussions at 23-25wks GA? What does the CPS recommend re: active intervention in 23-24wks? What does the CPS recommend re: active treatment for 25wks?

Because survival is uncommon Recommended non-interventional approach Focus on comfort care Need to make individualize decision after counselling re: Outcomes Decision making re: active treatment Appropriate for SOME infants 25wks have Improved: survival Neurodevelopmental outcomes Active treatment is appropriate except if there are significant additional risk factors CPS Counselling and management for anticipated extremely preterm birth

Benefits and harms of circumcision?

Benefit UTI decrease HIV decrease Penile cancer decrease ?later penile problems decrease Harm UTI/HIV/Penile cancer risks ultimately very low in developed countries No cost benefit to circumcising Complications 0.2-2% from circumcision (some serious) Immediate incr in UTI post op ?decrease in finding surgically/medical followup needed problems? Painful procedure - unclear best pain control CPS circumcision

Central or peripheral nervous system hypoventilation

Birth asphyxia Intracranial hypertension, hemorrhage Oversedation Diaphragm palsy Neuromuscular disease Seizures Neo Notes

Most common cause? What can it also be caused by? CPS Perinatal brachial plexus palsy

Birth trauma Evidence to say that it happened even before delivery! CPS Perinatal brachial plexus palsy

When should you clinically suspect hemorrhagic disease of the newborn?

Bleeding <6mos regardless of whether or not got vitamin K CPS Vit K

When are blood transfusions indicated in newborns?

Blood loss/shock Prevent severe anemia in high risk prem infants CPS Red blood cell transfusion in newborn infants

What are 5 problems unique to the late preterm population? Who should be observed? Which late prems are more often readmitted?

Body looks mature but not! Usually >2500g (not lbw - <2500, vlbw - <1800, ELBW - <1000) Difference is brain weight and body composition Inadequate thermal regulation Poor suck and swallow patterns Immature and incomplete adaptation of enzyme systems (bilirubin conjugation) Poor immunological/respiratory defense mechanisms <34wks would need 24hr of observations for respiratory stability 36wks > 34/35wks Likely because discharged home more slowly at younger gestations CPS facilitating discharge of late preterms infants

How should you break bad news?

Both parents or support person Simple language Not rushed w/ plenty of times with questions Deep sense of caring Lots of time spent with child Follow up needed CPS perinatal loss

What counts as an "effective feed"? CPS Hypoglycemia

Breastfed 5-10cc/kg of formula 5-10cc/kg glucose water CPS Hypoglycemia

List three factors that has reduced SIDS?

Breastfeeding Sleeping position smoking CPS baby friendly

In Canada, what are the rates of: 1) breastfeeding initiation 2) breastfeeding at 6mos (which provinces are higher, which lower?) 3) world wide, how has the baby friendly intiative influenced breastfeeding and sustained breastfeeding?

Breastfeeding initiation 87-90% Sustained breastfeeding at 6mos exclusively - 14-24% Worldwide, breast feeding initiative has increased initiation and sustained breastfeeding Even if (like in America study) only some of the 10steps were followed! CPS baby friendly

Does supporting breastfeeding help decrease hyperbili?

Breastfeeding support No studies say that it does decrease severe jaundice But does help other aspects of breastfeeding, so overall reasonable to do Should continue breastfeeding bc risk of acute encephalopathy is low compared to benefits of breastfeeding CPS recommends that all institutions where babies are delivered should have program of breastfeeding support CPS Hyperbili

What are the goals of antenatal counselling?

Build relationship between health care professional and family Provide accurate information Help families make decisions about care Be aware of own effects/bias Understand parental experiences and values Get parents to consider their own values Foster mutual respect Build therapeutic alliance to meet mutually understood goals CPS Counselling and management for anticipated extremely preterm birth

PREP 2015 Question 131 You are assessing a newborn with hypoglycemia 8 hours after birth. She was born at 37 weeks' gestation to a 40-year-old primagravida woman whose pregnancy was notable only for the use of assisted reproductive technology. The newborn required admission to the special care nursery shortly after birth for intravenous glucose therapy to maintain her blood sugar values over 45 mg/dL (2.5 mmol/L). Your physical examination reveals a large for gestational age newborn with an abdominal wall defect (Item Q131). Of the following, the additional clinical finding this patient is MOST likely to have is A. anorectal malformation B. hypospadias C. large tongue D. patent urachus E. single umbilical artery

C. large tongue The newborn in the vignette has the clinical findings of Beckwith-Wiedemann syndrome (BWS), which is associated with macroglossia (large tongue). Beckwith-Wiedemann syndrome is related to alterations on chromosome 11p15.5, with a familial transmission rate of 15%. There is an increased incidence seen in pregnancies requiring assisted reproductive technology. Beckwith-Wiedemann syndrome is an overgrowth syndrome with affected infants often being large for gestational age at birth. Other manifestations of somatic overgrowth include macroglossia, visceromegaly, and hemihypertrophy. Islet cell hyperplasia leads to the hypoglycemia seen in up to 50% of affected infants. Abdominal-intestinal wall defects including omphalocele, umbilical hernia, and diastasis recti are commonly seen with BWS. An omphalocele is an abdominal-intestinal wall defect formed by the protrusion of bowel into the base of the umbilical cord (Item C131A), whereas a gastroschisis is formed by loops of bowel extending through a defect in the abdominal wall to the right of the umbilical cord (Item C131B). Both require immediate surgical evaluation and management in the neonatal period. The initial management in the delivery room includes covering the exposed bowel with saline-soaked sterile dressings, inserting a gastric tube to decompress the bowel, and placing the newborn in a bowel bag to the level of the axilla to minimize fluid and heat losses. Parenteral fluid resuscitation is often needed because of increased insensible losses associated with the defects. The type of abdominal-intestinal wall defect guides further evaluation. Gastroschisis is an isolated defect seen in infants with intrauterine growth restriction: approximately 10% will have coexisting intestinal atresia or stenosis. Omphalocele is associated with other anomalies in up to 70% of cases. Beckwith-Wiedemann syndrome may be seen in 10% of affected infants, whereas karyotype anomalies, including trisomy 13, 18, and 21, may be found in up to 30% of cases. Echocardiography should be performed in any infant with an omphalocele because of the 50% risk of congenital heart disease. Anorectal malformations, hypospadias, patent urachus, and single umbilical artery are not associated with BWS. They are unlikely to be found in the newborn in the vignette. PREP Pearls An omphalocele is an abdominal wall defect formed by the protrusion of bowel into the base of the umbilical cord, whereas a gastroschisis is formed by loops of bowel extending through a defect in the abdominal wall to the right of the umbilical cord. The initial management of an abdominal-intestinal wall defect in the delivery room includes covering the exposed bowel with saline-soaked sterile dressings, inserting a gastric tube to decompress the bowel, and placing the newborn in a bowel bag to the level of the axilla to minimize fluid and heat losses.

What is a full diagnostic evaluation? CPS term infants at increased risk for early onset bacterial sepsis

CBC + diff Blood culture +/- CXR (for respiratory distress) +/- LP Can be deferred if: Only resp distress and can properly be observed If unsafe now, can delay to look for low glucose or hi WBC later CPS term infants at increased risk for early onset bacterial sepsis

What infectious diseases are screened for in Canadian blood system?

CC blood transfusions are HHHella Safe (CMV, hepC, HIV, HTLV, Hep B/C, Syphilis) HIV 1, 2 antibodies and HIV p24 antigen HTLV 1, 2 HepBsAg Hep C virus Syphilis CMV Although transmission of CMV to neonatal transfusion very low, still prefer CMV NEGATIVE blood to LBW infants CPS Red blood cell transfusion in newborn infants

Clinical Manifestations of ABO incompatibility

CLINICAL MANIFESTATIONS: Most cases are mild, with jaundice being the only clinical manifestation. The infant is not generally affected at birth; pallor is not present, and hydrops fetalis is extremely rare. The liver and spleen are not greatly enlarged, if at all. Jaundice usually appears during the 1st 24 hr. Rarely, it may become severe, and symptoms and signs of kernicterus develop rapidly. Neo Notes

CDH Associated anomalies:

CNS, esophageal atresia, omphalocele, and cardiovascular lesions. Neo notes

What are the symptoms of SSRI neonatal behavioural syndrome?

CNS/motor Jittery/tremor Increased muscle tone Seizures (rare) Respiratory Tachypnea Cyanosis Persistent pulmonary hypertension (rare) GI Difficulty feeding CPS SSRI's

what is the most common cause of lower intestinal obstruction in neonates

CONGENITAL AGANGLIONIC MEGACOLON (HIRSCHSPRUNG DS) Males are affected more often than females (4:1); prematurity is uncommon. There is an increased familial incidence in long segment disease. Neo Notes

What is considered severe adverse neurodevelopmental outcomes at 18-24mos in most studies? (5) vCPS Counselling and management for anticipated extremely preterm birth

CP Cognitive impairment (>/= 2SD below mean) Seizures Blindness deafness CPS Counselling and management for anticipated extremely preterm birth

What was the previous preservative in blood? What is the new preservative in blood?

CPAD Citrate Phosphate Dextrose Adenine AS-3 Adenine saline anticoagulant/preservative CPS Red blood cell transfusion in newborn infants

How long are certain groups vulnerable to hypoglycemia?

CPS Hypoglycemia

When do certain groups become hypoglycemic and stop being vulnerable to hypoglycemia?

CPS Hypoglycemia

What kind of barbituates can be used? CPS Intubation

CPS Intubation

What opiates should be used? CPS Intubation

CPS Intubation

CPS Recommendation: When should follow up exams be done? CPS ROP

CPS ROP

ROP stages

CPS ROP

What are the zones of the retina?

CPS ROP

List 5 things on the international code of marketing breast milk substitutes?

CPS baby friendly

What are the 9 global strategies for infant/young child feeding operational targets from the declaration in 2005?

CPS baby friendly

What are the possible organisms causing early onset sepsis and what abx should you choose?

CPS term infants at increased risk for early onset bacterial sepsis

What risk does a child have of any organism sepsis if chorio is present?

CPS term infants at increased risk for early onset bacterial sepsis

CXR of TTN

CXR - prominent pulmonary vascular markings, fluid in the intralobar fissures, overaeration, flat diaphragms, and, rarely, pleural effusions Neo Notes

Why shouldn't iNO be stopped abruptly?

Can lead to severe hypoxemia because natural NO has been down regulated If this happens or pt rebounds, restart iNO at last dose and wean over 24-48h CPS iNO

What are the canadian rates of a) critical hyperbilirubinema/ exchange transfusion b) acute encephalopathy c) chronic encephalopathy

Canada 2002-2004, # needing exchange transfusion/ critical hyperbili: 4/10,000 live births Acute encephalopathy: (having neuro sign at presentation) 1/10,000 Chronic encephalopathy 1/50-100,000 CPS Hyperbili

What are the two ways glucoses can be taken?

Capillary/venous Whole blood or plasma Whole blood 10% lower than plasma Cells eat it up! Plasma= 92% water, 7% vital proteins (albumin, gamma globulin, anti- hemophilic factor, and other clotting factors, and 1% mineral salts, sugars, fats, hormones and vitamins) Whole blood = red cells, white cells, and platelets (~45% of volume) suspended in plasma (~55% of volume). CPS Hypoglycemia

Diffuse, ecchymotic, edematous swelling of the soft tissues of the scalp after delivery

Caput succedaneum • involves area presenting during vertex delivery • may extend across suture lines. • Disappears within the 1st few days of life. • Rarely may result in shock and require blood transfusion. Generally no treatment needed; but at risk for hyperbilirubinemia. Neo Notes

Agents Acting on Pregnant Women that act as Teratogens

Carbamazepine: Spina bifida, possible neurodevelopmental delay Carbon monoxide: Cerebral atrophy, microcephaly, seizures Cigarette smoking: Low birthweight for gestational age Cocaine/crack: Microcephaly, LBW, IUGR, behavioral disturbances Cyclophosphamide: Multiple malformations Neo Notes

Condition That May Cause Apparent Life-threatening Events or Sudden Death Cardiac

Cardiac Subendocardial fibroelastosis, aortic stenosis, anomalous coronary artery, myocarditis, cardiomyopathy, arrhythmias (prolonged Q-T syndrome, Wolff-Parkinson-White syndrome, congenital heart block) Neo Notes

What are the two most common congenital malformations associated with mortality in the immediate perinatal period?

Cardiac malformation = #1 Neural tube defects = #2 IN CANADA CPS Folic Acid and Neural tube defects

What does the CPS recommend re: GA/weight?

Caregivers must obtain most accurate info about GA and estimated fetal weight available CPS Counselling and management for anticipated extremely preterm birth

What do neural tube defects at the: caudal end cranial end look like?

Caudal end = spina bifida Cranial end = anencephaly CPS Folic Acid and Neural tube defects

Ecchymosis limited to the surface of one cranial bone after birth

Cephalohematoma • Subperiosteal hemorrhage • always limited to the surface of one cranial bone. • No discoloration of the scalp, no swelling until a few hours of life • Becomes a firm tense mass with a palpable rim • Most resorbed within 2 wk-3 mo but may calcify and remain for years • Underlying skull fracture in 10-25% • No treatment needed; if extensive may need photoTx for hyperbilirubinemia. Neo Notes

Molecular DNA genetic analysis of the neonate is accomplished by

Circulating fetal cells or DNA in maternal blood or plasma to get DNA go to the source of the DNA = mom to get Karyotype = go to the Korion Neo Notes

Clinical Manifestations of HIE

Clinical Manifestations: • Severity depends on duration and timing of injury o Symptoms develop over days, must do SERIAL neuro exams • IUGR (chronic hypoxia) • Signs of fetal distress during labour (slowing HR, decreased variability, early or late decals, scalp blood pH <7.20, meconium-stained amniotic fluid) • At birth Depressed with poor respiratory effort Neo Notes

When can you stop empirical therapy? CPS term infants at increased risk for early onset bacterial sepsis

Clinical course AND cultures negative Try and use minimal abx because most abx used in kids who do NOT have sepsis Gentamycin q24h Ampicillin q12h CPS term infants at increased risk for early onset bacterial sepsis

Reducing pain from bedside care procedures

Clinical implications 1. Care protocols for neonates should incorporate a principle of minimizing the number of painful disruptions in care as much as possible. 2. A combination of oral sucrose/glucose and other nonpharmacological pain reduction methods (nonnutritive sucking, kangaroo care, facilitated tuck, swaddling and developmental care) should be used for minor, routine procedures. 3. Topical anesthetics can be used to reduce pain associated with venipuncture, lumbar puncture and intravenous catheter insertion when time permits, but are ineffective for heel stick blood draws. Repeated use of topical anesthetics should be limited. 4. The routine use of continuous infusions of morphine, fentanyl or midazolam in chronically ventilated preterm neonates is not recommended due to concern about short term side effects and lack of long-term outcome data CPS Prevention and management of pain in the neonate

What are the features of chronic bilirubin encephalopathy?

Clinical sequelae of acute encephalopathy Athetoid cerebral palsy with or without: Seizures Mental deficiency Developmental delay Hearing deficit Oculomotor disturbances Dental dysplasia Athetoid = basal ganglia!, hyper and hypotonia that can't be controlled. Choreathetoid = movements face and extremities Dystonic = sustained contractions A chorea is a rapid, involuntary, non-stereotypical, semi-purposeful (some reference uses non-purposeful), dance-like movements that involved the distal muscle group more than proximal. An athetosis is a slow, involuntary, non-stereotypical, non-purposeful, writing movements, with a propensity to affects the upper limbs. CPS Hyperbili

What are the features of acute bilirubin encephalopathy?

Clinical syndrome associated with severe hyperbilirubinemia Initial: Hypotonia, lethargy, poor suck Progress: Hypertonia, opisthotonus/retrocollis (spasm of the muscles causing backward arching of the head, neck, and spine, as in severe tetanus, some kinds of meningitis, and strychnine poisoning), shrill cry Ultimately: Coma, seizures CPS Hyperbili

o focal or multifocal (several body parts, migratory). GTC seizure uncommon in neonatal period (? Due to decreased connectivity and myelination)

Clonic Seizures Neo Notes

Why does donor breast milk help preterm infants?

Cochrane review of 8 studies: Decrease rates of NEC if breast milk fed But unable to draw conclusion because feeding practices did not match our current practices (more raisin) Decreased rates of nec if breastmilk/donor milk/human milk fortifier vs. bovine fortifier in extremely prem kids weighing less than 1250g CPS Milk banking

Do the rate and cost of circumcision complications compare to rates/cost of complications of not circumcising? What are complications of circumcision and rate of complications?

Comparable Complications to surgery vs. UTI/ bacteremia/renal failure/meningitis Must be researched more Complications Bleeding Amputation of glans Renal failure UTI Sepsis/bacteremia Death Local infection Rate of complication 2-10% realistically CPS circumcision

When should you see complete recovery? Is surgery better than conservative watch and wait? CPS Perinatal brachial plexus palsy

Complete recovery by 3-4wks If incomplete recovery by then, likely will not recover Unknown No RCTs looking at nonoperative management No evidence to say that primary surgical exploration of the brachial plexus is preferable to conservative management Only some evidence to say those most severe may do better with surgical repair CPS Perinatal brachial plexus palsy

What is gestational age or postmenstrual age?

Completed days from first day of last menstrual period CPS Counselling and management for anticipated extremely preterm birth

How do you manage stillbirths and miscarriages?

Complicated just like perinatal death because the baby but also all that it meant (status, self esteem, existential loss) and self esteem - can't trust body to succesfully give birth Need to give mom and parents time with the baby even if delayed by mom's GA etc Considerately offer spiritual care and information CPS perinatal loss

13. A 27 wk prem is born at your hospital and intubated. You are awaiting transport. What is one thing you can do for each of the following to avoid the complication? **see next question! a. Hypoglycemia b. Hypothermia c. BPD d. ROP

Complication: Management: Hypothermia/Hyperthermia - maintain 'neutral thermal environment', use incubator/radiant warmer, minimal handling; continuous monitoring of temperature to make adjustments (Nelson's) IVH - careful management of respiratory status (avoid acidosis, hypoxia, hypocarbia, pneumothorax) - careful management of fluid/electrolyte status (avoid changes in BP) - low-dose indomethacin (some evidence) Hypoglycemia - Start fluid at 70-80mL/kg/day - Screen at 2 hours, and then q3-6hours (until 36 hours minimum in preterm) (CPS Statement) BPD - surfactant ROP - administer oxygen only as necessary; and adjust FiO2 according to PaO2 or non-invasive pulse oximetry (Nelson's)

14. A 2 hour old 27 week premature baby is born in your community center. You are taking care of her before the transport team arrives. What one management plan will you do to prevent each of these complications:

Complication: Management: Hypothermia/Hyperthermia - maintain 'neutral thermal environment', use incubator/radiant warmer, minimal handling; continuous monitoring of temperature to make adjustments (Nelson's) IVH - careful management of respiratory status (avoid acidosis, hypoxia, hypocarbia, pneumothorax) - careful management of fluid/electrolyte status (avoid changes in BP) - low-dose indomethacin (some evidence) Hypoglycemia - Start fluid at 70-80mL/kg/day - Screen at 2 hours, and then q3-6hours (until 36 hours minimum in preterm) (CPS Statement) BPD - surfactant ROP - administer oxygen only as necessary; and adjust FiO2 according to PaO2 or non-invasive pulse oximetry (Nelson's) Nelson's: BPD definition: oxygen requirement for 28 days postnatally; further characterized as mild/moderate/severe based on supplemental oxygen and gestational age - if <32 weeks PMA: a) mild - breathing room air at 36 weeks PMA b) moderate - <30% supplemental oxygen at 36 weeks PMA c) severe - >30% supplemental oxygen at 36 weeks PMA, and mechanical support, CPAP, or ventilation - if >32 weeks PMA: a) mild - breathing room air at 56 days of life b) moderate - <30% supplemental oxygen at 56 days of life c) severe - >30% supplemental oxygen at 36 weeks PMA, and mechanical support, CPAP, or ventilation Nelson's: IVH - risk inversely associated to gestational age and birthweight - IVH occurs in the gelatinous, vascular region of the subependymal germinal matrix (periventricular) - combo of immature vessels and poor tissue vascular support - Predisposing factors: prematurity, RDS, HIE, repferfusion injury of damaged vessels, increased or decreased cerebral blood flow, reduced vascular integrity, increased venous pressue, pneumothorax, thrombocytopenia, hypervolemia, hypertension - Grading of severity: a. Grade I: bleeding isolated in supependymal area b. Grade II: bleeding within ventricle, no dilatation c. Grade III: bleeding within ventricle + dilatation d. Grade IV: intraventricular and parenchymal hemorrhage - Screening by routine US in babies born <32 weeks GA - Prevention: - minimize traumatic brain injury (judicious management of cephalopelvic disproportion, and assisted (forceps, vacuum) delivery - treatment of maternal ITP/alloimmune thrombocytopenia - care of newborn's respiratory status, fluid and electrolyte management: o avoidance of acidosis, hypocarbia, hypoxia, hypotension, wide fluctuations in blood pressure or PCO2, and pneumothorax - single course of antenatal corticosteroids in pregnancies 24-34 weeks GA (decrease risk of death, grade II and IV IVH, PVL)

Cps recommendation pasteurized human donor breast milk should be prioritized for whom?

Compromised preterm infants Selected Ill term newborns CPS Milk banking

How does phototherapy work? CPS Hyperbili

Conformational change in bilirubin molecule Makes it water soluble CPS Hyperbili

Methemoglobinemia can be caused by

Congenital (hemoglobin M) Acquired (nitric oxide) Neo Notes

o Neonate asymptomatic when awake; cyanosis, prolonged apnea and decreased chest movement when asleep o Child pulmonary hypertension, cyanosis, and digital clubbing o Both tachycardia, low heart rate variability, multiple ganglioneuroblastomas of the sympathetic chain and the adrenal medulla

Congenital Central Hypoventilation Syndrome (Ondine Curse) o Abnormal autonomic nervous system o Labs: ↑Paco2 when asleep (80-90 mm Hg) that normalizes when awake o Treatment: Supportive; Phrenic nerve pacing is used if >2 yo o Prognosis: Some grow and develop normally, although hypoventilation abnormalities can persist Neo Notes

How do you deal with the siblings of a death? CPS perinatal loss

Consider letting them spend time after baby dies/memorial services Can develop symptoms Can blame themselves (wished them dead), trying to live up to the infant to console parents May feel over nurtured or pushed away by parents CPS perinatal loss

How should the ventilator settings be weaned after surfactant therapy?

Consider rapid weaning of ventilatory support and extubation within 1hr to nasal CPAP Studies up to this point show that not a significant change in outcomes if slow or fast Often the weaning quickly is done without any bloodwork and just based on pts clinical status CPS surfactant

What should care providers do if pregnant women, or women considering pregnancy are on paroxteine?

Consider: switching them to another antidepressant Reducing the dose CPS SSRI's

Before doing an exchange transfusion, what should you do?

Continue: Intensive phototherapy (no diaper, fibreoptic blanket) Supplemental fluids IVIG if isoimmunization Take extra blood: Chromosomes Metabolics Pyruvate kinase/Glucose6-phosphate dehydrogenase deficiency Red blood cell fragility Hb electrophoresis Retake TSB prior to doing exchange transfusion bc significant morbidity CPS Hyperbili

CPS recommendation: what must centres have if want to administer surfactant to newborn infants?

Continuous on site personnel trained and licensed to deal with complications of assisted ventilation and surfactant therapy CPS surfactant

What is the difference between conventional and intensive phototherapy?

Conventional: Single bank of fluorescent lights above the incubator with diaper in place Intensive High intensity light (>30microW/cm2/nm) to most amount of skin possible Usually two photo units (fluorescent) ~10cm away from infant Diaper on, in bassinet If approaching exchange range: Diaper off/ptx wavelength transmitting diaper Add fibreoptic blanket under infant CPS Hyperbili

Does surfactant lavage work for meconium aspiration syndrome?

Could be effective Not enough research One small RCT Maybe acute physiologic benefit No clinically significant benefits Compared to restricted rescue surfactant therapy CPS surfactant

When discharging a baby with risks of developing sepsis (but has been observed for the appropriate amount of time), what must you do with parents?

Counsel re signs of sepsis Only dischargeable if can promise to immediately transport baby to health care facility if clinical signs of sepsis develop CPS term infants at increased risk for early onset bacterial sepsis

1. Picture of a baby with an elongated skull in AP diameter. Mother brings in her baby with concerns regarding the shape of his head. What should you consider? On exam, what do you look for to confirm?

Craniosynostosis; assess position of ears relative to plagiocephaly, ridging of sutures, head circumference - ultimately need Head CT

Canadian Pediatric Review 2018 Perinatal-Neonatal Medicine Baby J is a 36 week infant who required CPAP for 12 hours for TTN/RDS and D10W by IV for hypoglycemia for 48 hours before breastfeeding was established. This is Ms. J's second late preterm infant and she eagerly asks on rounds when Baby J can have her car seat test. Which of the following regarding 'infant car seat challenge' is true? A. The CPS recommends an infant car seat challenge for infants <37 weeks gestational age prior to discharge home. B. The CPS recommends an infant car seat challenge for infants <37 weeks gestational age who required respiratory support for greater than 4 hours prior to discharge home. C. Clinicians can feel confident that an infant's passing an ICSC test indicates safety for discharge regarding respiratory stability. D. The ICSC test to screen other populations before discharge (ex. Infants with neurological impairment, hypotonia, or low birth weight) is up to the individual clinician.

D. The ICSC test to screen other populations before discharge (ex. Infants with neurological impairment, hypotonia, or low birth weight) is up to the individual clinician. Going home: facilitating discharge (CPS position statement, reaffirmed 2017) • Physiologic maturity - Thermoregulation (around 1700g to cot) - Control of breathing • Spell free period off caffeine: 5-7 days at least - Respiratory stability • 25% of <1500g infants on O2 at 36weeks CGA; Target sats 90-95% - Feeding skills and weight gain • Cue based feeding: earlier to full feeds, ¯ length of stay • Iron 2mg/kg/d, vitamin D 400-100 IU supplements • GERD: little evidence of association with pathology • Investigations / treatment - assessment for respiratory syncytial virus (RSV) prophylaxis - cranial imaging at near-term, if indicated by gestational age - retinopathy of prematurity (ROP) screening, if indicated - immunizations according to chronological age - pre-discharge P/E: weight, length and head circumference.

PREP 2015 Question 36 You attend the cesarean delivery of a term neonate due to breech presentation. The mother received good prenatal care with no history of substance abuse. General anesthesia is used due to maternal scoliosis with rod placement. Artificial rupture of the membranes occurs at the time of delivery revealing clear amniotic fluid. After breech extraction, the neonate emerges limp with a weak cry. The neonate is placed on the warmer, dried, stimulated, and the airway cleared. Your assessment at 30 seconds after birth reveals a hypotonic neonate with a heart rate of 90 beats/min, intermittent apnea, and central cyanosis. Of the following, the BEST next management step for the neonate is to A. administer naloxone B. deliver blow-by oxygen C. flick the soles of the feet D. initiate positive-pressure ventilation E. start chest compressions

D. initiate positive-pressure ventilation Positive-pressure ventilation should be initiated if the assessment of a newborn at 30 seconds after delivery reveals apnea, gasping, or a heart rate less than 100 beats/min, as seen in the neonate described in the vignette. The establishment of air exchange by effective ventilation of the lungs is the most important action in neonatal resuscitation. Any decrease in blood flow or oxygen delivery in utero may lead to fetal compromise, which often manifests as apnea at the time of delivery. Perinatal stress will initially lead to a period of rapid breathing, followed by apnea and a falling heart rate in the fetus (Item C36A). This primary apnea often responds to gentle stimulation of the affected newborn by drying or slapping of the feet immediately after birth. Ongoing perinatal stress leads to a brief period of gasping respirations before the fetus enters secondary apnea, with continued bradycardia and loss of blood pressure (Item C36A). A newborn experiencing secondary apnea will not respond to gentle stimulation and requires the initiation of positive pressure ventilation. It is difficult to determine whether a newborn is in primary or secondary apnea at birth and delays in initiating effective resuscitation may cause further compromise. Once effective ventilation is established, most newborns will respond with a rise in heart rate, followed by the development of respiratory effort. Health care providers attending the cesarean delivery of the neonate in the vignette should be prepared for the potential need for resuscitation because of the intrapartum risk factors of breech presentation and general anesthesia. The neonate remains apneic with a heart rate less than 100 beats/min at 30 seconds after birth and positive-pressure ventilation should be initiated. Continued stimulation by flicking of the feet is unlikely to initiate spontaneous respiratory effort and will lengthen the time the neonate is compromised. Blow-by oxygen is not indicated, as the neonate is not spontaneously breathing. It is important to note that room air resuscitation is recommended in full-term neonates, with the placement of a pulse oximeter on the right hand at the time positive pressure ventilation is begun to guide the future use of supplemental oxygen. Chest compressions are indicated when the heart rate remains below 60 beats/min after 30 seconds of effective positive-pressure ventilation (Item C36B). Naloxone is no longer considered a primary resuscitation drug in the delivery room, but may be used in a neonate with a normal heart rate and oxygen saturation who fails to breathe spontaneously and whose mother received a narcotic drug within 4 hours of delivery and has no history of drug use. Neonates whose respiratory effort is depressed by maternal magnesium sulfate or general anesthesia do not respond to naloxone. PREP Pearls The establishment of air exchange by effective ventilation of the lungs is the most important action in neonatal resuscitation. It is difficult to determine whether a newborn has primary or secondary apnea at birth, and delays in initiating effective resuscitation may cause further compromise.

1. You are called to see a 2.5 kg term newborn with jitteriness. You do a bedside glucose, which is 1.7. You decide to insert an IV and give the baby a bolus of 2 cc/kg of D10. Please write an ongoing fluid order for this baby, to be started after the bolus is complete.

D10% @ 80mL/kg/day = 8.3mL/hour • D10W IV at 8.3 mL/hr (TFI 80 mL/kg/day) and recheck glucose in 30 minutes • See CPS Statement on Hypoglycemia in Neonates

o Routine Screening HUS in all infants <32wks

DIAGNOSIS: o Routine Screening HUS in all infants <32wks o <1000g D3-7 D7-14 Term o <1500g D7-14 Term Neo Notes

Dx of hirschprungs dz

DIAGNOSIS: Rectal manometry and rectal suction biopsy are the easiest and most reliable indicators of Hirschsprung disease. The radiographic diagnosis is based on the presence of a transition zone between normal dilated proximal colon and a smaller-caliber obstructed distal colon caused by the nonrelaxation of the aganglionic bowel. Full-thickness rectal biopsy can be performed at the time of surgery to confirm the diagnosis and level of involvement. Neo Notes

Agents Acting on Pregnant Women that act as Teratogens Danazol, progesterone Lithium Misoprostol Penicillamine Phenytoin Prednisone

Danazol, progesterone: Virilization Lithium: Ebstein anomaly, macrosomia Misoprostol: Arthrogryposis, cranial neuropathies (Möbius syndrome), equinovarus Penicillamine: Cutis laxa syndrome Phenytoin: Congenital anomalies, IUGR, neuroblastoma, bleeding (vitamin K deficiency) Prednisone: Oral clefts Neo Notes

When should comfort care be used? If parents make a decision not to resuscitate, what should be done? CPS Counselling and management for anticipated extremely preterm birth

Decision not to resuscitate Discontinuing life-sustaining therapies Unsuccessful resuscitation Inform them that infant will still be cared for w/ comfort measures: Parents can hold baby Infant may survive for several days CPS Counselling and management for anticipated extremely preterm birth

Overall, what are the effects of cooling in HIE?

Decrease mortality Decrease major neurodevelopmental disability Different degrees depending on the degree of HIE vs surfactant which doesn't decrease BPD incidence but does decrease mortality (vs HIE where neurodev disability is also dec'd in addition to mortality = morbidity and mortality) CPS Hypothermia in HIE

What has caused people to change their minds since 2002 re: steroids for CLD prevention/treatment?

Decrease not IMMEDIATELY associated with increase in mortality or SHORT term morbidity (like needing oxygen at 36wks PMA) HOWEVER - now starting to see increase in CLD in Canada and elsewhere CLD is independent risk factor for poor neurodevelopmental outcome Could there be some use in: Lower dose of dex Shorter course of dex Alternative to dex Use in high risk infants CPS Postnatal corticosteroids to treat/prevent BPD

List 5 benefits of using human breast milk for the preterm infant?

Decreased severe infections Decreased NEC Decreased colonization with pathogenic organisms ?decreased length of hospital stay Improved neurodevelopmental outcome (hard to control for other factors that could have lead to prem birth) CPS Milk banking

Those with severe hyperbilirubinemia, how should they be followed?

Definitely need hearing screen with brainstem auditory evoked potentials bc. Of high risk of neurosensory hearing loss CPS Hyperbili

CPS recommendation: Who should be routinely screened for hypoglycemia? How frequently and when should they start and stop testing for hypoglycemia?

Definitely the following 3 Sga (<10th%) Prem IDM LGA (>90th %) considered AT RISK. 2h of age after one effective feed Q3-6h before feeds IDM/LGA stop at 12h if >/= 2.6 SGA/Prem stop at 36h if >/= 2.6 Symptomatic, test right away CPS Hypoglycemia

What are 10 factors that may increase the risk of acute encephalopathy?

Dehydration Prematurity Hypoalbuminemia Hyperosmolarity Resp distress Hydrops Acidosis Hypoxia Seizures sepsis CPS Hyperbili

What are the 6 side effects of phototherapy?

Dehydration (may not be clinically significant in term newborns who are drinking well) Temperature instability Intestinal hypermotility Diarrhea Interference with maternal/infant interaction Possible bronze discolouration of the skin CPS Hyperbili

Why is breastfeeding an important preventative health measure for lactating mothers?

Delays ovulation Increases post-partum weight loss Decreases breast/ovarian cancers CPS baby friendly

What are the problems with transcutaneous bilirubin measurements? x4 Why do we still use them?

Depends on skin thickness Depends on skin colour Wide variability of accuracy on devices Not reliable once phototherapy started We still use them because more accurate at lower levels of bilirubin, so okay as a screen! CPS Hyperbili

What should people do about pacifiers?

Depends on where you are Overall it decreases SIDS If n NICU, helps with pain relief and sucking training CPS recommends that you can consider delaying use until after breastfeeding is established CPS baby friendly

CPS recommendation: What kind of continuing education programs for health professionals should there be re: FAS/FAE?

Designed to enhance counselling skills to motivate and support lifestyle change for at-risk drinkers Should involve community resources to provide support and motivation for change for at-risk drinkers CPS Preventing FAS

What is the baby friendly initiative? How many hospitals in the world/in canada have a baby friendly designation?

Developed in 1991 Designation given to a maternity facility if follow all ten steps of the WHO/UNICEF successful bfing for at least 80% of all women and babies it cares for Given by "breastfeeding committee for canada" 20,000 maternity facilities in the world 12 hospitals/birthing centres w/ 25 community health centres/authorities in Canada as of may 2012 Most are in quebec with provincial bfing strategy CPS baby friendly

If you are starting iNO on a neonate, what must you also consider?

Different kind of ventilation HFO, JET ?surfactant ?ECHO for function, pressures, r/o cardiac disesae CPS iNO

How do you deal with parents who find out pre/postnatally that their baby has congenital anomalies? CPS perinatal loss

Difficult Surprise it isn't normal Difficult to rethink future Guilty that you hope it doesn't survive If decide to terminate: some feel it was the right thing, others feel guilty May cause marital distress and possible separation. Each experiencing it differently (man bystander, rationalize, keep feeling to themselves) May persist CPS perinatal loss

Canadian Pediatric Review 2018 Perinatal-Neonatal Medicine Discontinuation of resuscitation

Discontinuation of resuscitation "Consider stopping if no detectable HR after 10 minutes of appropriate resuscitation" • Consider: - Etiology of arrest - Gestation - Presence/absence confounders - Potential role of hypothermia - Parental wishes

What are the benefits of autopsy? CPS perinatal loss

Discuss with parents New clinically significant data discovered in up to 44% of cases CPS perinatal loss

Why does IM vitamin K still fail?

Does not prevent all cases Likely breast fed and low amounts of vitamin K in the diet! High risk: Prolonged diarrhea Liver disease Failure to thrive CPS Vit K

What is SSRI neonatal behavioural syndrome? Why not just call it neonatal abstinence syndrome?

Don't know if it is from withdrawal Could it be in utero toxicitiy Could it be both? CPS SSRI's

What's the difficulty in interpreting survival data? CPS Counselling and management for anticipated extremely preterm birth

Don't know the denominator! Still births? Live births? Only admitted to NICUs Not count the ones who never made it (decision not to resusc or unsuccessful resusc) May over estimate survival! Don't know the management? Some don't give steroids Others decide to not resuscitate and they don't count CPS Counselling and management for anticipated extremely preterm birth

What are 7 ways that you could prevent non-physiologic anemia and prevent risks associated with RBC transfusions??

Don't take so much Delayed cord clamping Reduced blood draws Return dead space volume after sampling Microtechnique Better monitoring than multiple gases Support/medically encourage Human recombinant erythropoietin Vitamins/iron supplements Transfuse correctly: Use RBCs that were screened/handled properly from regular/designated donors Use RBC multipack units that were appropriately collected Consider collecting/transfusing umblical cord blood (autologous blood transfusion) CPS Red blood cell transfusion in newborn infants

How are donors and their milk screened?

Donor = interview, serologic testing, physician consent Repeated q 6mos Must have altruistic reason No smoking, drinking, meds, cannot take OTCs Taught safe collection/storage of milk Frozen milk brought to milk bank Milk: HIV, Hep B, Hep C, HTLV Properly collected, stored, pasteurized, cultured CPS Milk banking

What is the big idea behind SSRIs and congenital malformations?

During first trimester use = no increase in risk of congenital malformations Paroxetine and cardiac malformations may have increased risk but studies inconclusive Other SSRIs and birth defects possible but better studies needed CPS SSRI's

CPS recommendation: What should be done for data collection?

Each province/territory - need to put in place long term standardized data collection system for tracking breastfeeding: initiation Duration exclusivity CPS baby friendly

What should be an essential part of developing an optimal feeding plan for a hospitalized neonate?

Education of parents! Benefit of human breast milk Benefit of pasteurized human donor breast milk This way can make an informed decision CPS Milk banking

When are situations where you can acceptably intubate an infant without premedication? In what situations would you think of not giving premeds but you should try harder? What 3 components make up an acceptable protocol for premedication prior to intubation?

Emergency resuscitation Getting airway, ventilation, appropriate heart rate NOT if can resuscitate with bag mask and can get IV access later Difficult iv access w/ lots of discomfort Use inhalants Nasal mucosa - fentanyl Planned difficult airway Remember key is to keep spontaneous ventilation Think LMA Bronchoscopic intubation Think transfer! Analgesic Vagolytic Muscle relaxant CPS Intubation

Condition That May Cause Apparent Life-threatening Events or Sudden Death Endocrine-metabolic

Endocrine-metabolic Congenital adrenal hyperplasia, malignant hyperpyrexia, long- or medium-chain acyl coenzyme A deficiency, hyperammonemias (urea cycle enzyme deficiencies), glutaricaciduria, carnitine deficiency (systemic or secondary), glycogen storage disease type I, maple syrup urine disease, congenital lactic acidosis, biotinidase deficiency TABLE 372-3 -- Differential Diagnosis of Recurrent Sudden Infant Death in a Sibship Idiopathic Recurrent true sudden infant death syndrome CNS Congenital central hypoventilation, neuromuscular disorders, Leigh syndrome Cardiac Endocardial fibroelastosis, Wolff-Parkinson-White syndrome, prolonged Q-T syndrome, congenital heart block Pulmonary Pulmonary hypertension Endocrine-metabolic See Table 372-2 Infection Disorders of immune host defense Child abuse Filicide, infanticide, Munchausen syndrome by proxy Neo Notes

Canadian Pediatric Review 2018 Perinatal-Neonatal Medicine Medications and the prolonged code

Epinephrine • Concentration = 1:10,000 • Preferred route: IV • UVC • Intraosseous catheter • Doses: • IV = 0.1 ml/kg (0.01 mg/kg) • {ETT = 1.0 ml/kg (0.1 mg/kg)} Not recommended • Naloxone 0.1mg/kg • Sodium bicarbonate • Ringers lactate Not improving? • Volume expansion - Normal saline - O negative blood (hemorrhage) • Pneumothorax - Shift of apical impulse, decreased breath sounds /asymmetry chest wall movement - Rx: needle decompress 2nd intercostal, mid clavicular line

Waiter's tip posture - arm held in adduction and internal rotation with pronation of the forearm _____ reflex absent Able to ______ Poor prognosis if ______

Erb-Duchenne C5-C6 Moro and Biceps extend forearm, grasp Poor prognosis Damage due to transaction Involvement of deltoid - results in shoulder drop Entire arm Neo notes

hydrops fetalis

Erythroblastosis fetalis Hemolytic dz of the newborn When small quantities of Rh+ fetal blood containing D antigen inherited from an Rh+ father enter the maternal circulation, antibody formation against D antigen may be induced in the unsensitized Rh- recipient mother. Once sensitization has taken place, considerably smaller doses of antigen can stimulate an increase in antibody titer. Initially, a rise in IgM antibody occurs, which is later replaced by IgG antibody; the latter readily crosses the placenta and causes hemolytic manifestations May range from only laboratory evidence of mild hemolysis (15% of cases) to severe anemia with massive enlargement of the liver and spleen. When the compensatory capacity of the hematopoietic system is exceeded, profound anemia occurs and results in pallor, signs of cardiac decompensation (cardiomegaly, respiratory distress), massive anasarca, and circulatory collapse. This clinical picture of excessive abnormal fluid in two or more fetal compartments (skin, pleura, pericardium, placenta, peritoneum, amniotic fluid), termed hydrops fetalis Neo Notes

What is the role of the health care provider? CPS perinatal loss

Establish an attachment and bonding Name of patient, he/she, personality, mementos, be part of bedside rounds Optimize immediate interaction w/ parents Get parents involved even if bad news Dying process and afterwards: Allow parents to spend time throughout the process, ask about friends and family Reassure that baby is not alone or in pain Religious/spiritual practices inquired and sough after Get mementos/photographs CPS perinatal loss

Why should we decrease pain and discomfort when intubating neonates?

Ethically imperative to administer analgesia before a planned painful intervention Do so until proven harmful Decreasing pain and discomfort will likely affect physiologic sequelae CPS Intubation

Who should be tested for G6PD deficiency?

Ethnicity Family history Boys AND girls with severe hyperbili (higher likelihood of needing exchange transfusion) CPS Hyperbili

Etiology of HIE

Etiology: • Most due to perinatal events • Maternal factors: o inadequate oxygenatio, low BP, inadequate relaxation of the uterus, premature separation of the placenta; impedance to the circulation of blood through the umbilical cord; and placental insufficiency • Fetal factors: o failure of oxygenation due to cardiac or pulmonary disease; severe anemia; shock Neo Notes

How do you handle congenital anomalies? CPS perinatal loss

Even if they have anomalies, mother's still say helpful to see and hold Can cover up/hide Mementos and pictures still helpful Still need services, religious, autopsy Parents see the beautiful things CPS perinatal loss

CPS recommendation: Why should information be provided to health professionals regarding EtOH and pregnancy?

Facilitate early recognition of at-risk drinking and early intervention CPS Preventing FAS

What are common cited reasons for not using premedication? CPS Intubation

Fear of complications Studies showing rarely an issue: Increased tone of chest wall musculature (other increased tone) Easily fixed with opioid antagonist/muscle relaxant Fear of long term respiratory effects Frequently intubate to quickly give surfactant and then extubate Fentanyl can be safely used in this situation! Can be extubated within 1h o administration Need to think about remifentanil CPS Intubation

What are the 7 psychological steps of becoming a parent?

Feelings of procreativity/generativity Sense of continuity through generation Fears and expectation about baby Effect on expectant parents' relationship Response to quickening bodily changes Attachments to real/idealized aspects of the infant Self esteem building CPS perinatal loss

Congenital Diaphragmatic Hernia - _____ 2x as commonly affected; defects more common on the ____ (85%)

Females left Neo notes

What analgesia is recommended? CPS Intubation

Fentanyl No RCT compared to others Watch for rare chest freeze (slow admin, with or shortly followed by muscle relaxant/naloxone) Need to quickly intubate bc suppress resp drive! Morphine No help if alone! Likely bc of 10mins onset! Must have more data before recommending: Remifentanil Thiopental methohexital CPS Intubation

Canadian Pediatric Review 2018 Perinatal-Neonatal Medicine Fetal genetic sampling

Fetal genetic sampling - Chorionic villus (9-12 wks) • Chromosomes • Risk: 1% pregnancy loss = Chorion for chromosomes - Amniocentesis (> 16 weeks) • Chromosomes, lung maturation, infection, renal • Risk: 0.5% pregnancy loss • Anatomic Ultrasound - 18-20 weeks • Other tests available prn - Fetal echocardiogram - Fetal MRI - Periumbilical blood sampling • Rh disease / hemolysis / renal/ chromosomes

Identifiable Causes of Preterm Birth Fetal, placental, uterine, maternal, other

Fetal: Fetal distress, Multiple gestation, Erythroblastosis, Nonimmune hydrops Placental: Placental dysfunction, Placenta previa, Abruptio placenta Uterine: Bicornate uterus, Incompetent cervix (premature dilatation) Maternal: Pre-eclampsia, Chronic medical illness, Infection, Drug abuse Other: Premature rupture of membranes, Polyhydramnios, Iatrogenic, Trauma Neo Notes

What is the classic triad of chorio?

Fever Lower uterine tenderness Left shift in WBC CPS term infants at increased risk for early onset bacterial sepsis

What's the problem with the diagnosis of chorio?

Fever is VERY common in labouring women Esp if have epidural analgesia Fever alone and chorio histology are often not consistent Chorio distinctions possible = fever Definite = fever, lower uterine tenderness, left shift in WBC CPS term infants at increased risk for early onset bacterial sepsis

How do the interests of mothers and fetus' change during pregnancy?

First trimester = well being, autonomy of mother take precedence Survival and health of fetus and newborn start to take over Problem is the legal status of fetus and newborn in Canada After birth best interests of infant takes precedence CPS Counselling and management for anticipated extremely preterm birth

Possible fetal alcohol effects

Fit some criteria but not all of fetal alcohol syndrome Alcohol is being considered as one of the possible causes of a child's birth defects Ex. Delayed growth of baby, single birth defect, developmental/learning/behavioural disorders that are noticeable months/years after birth It is NOT a miler form of FAS CPS Preventing FAS

Perinatal brachial plexus palsy (PBPP)

Flaccid paralysis of the arm at birth Affects different nerves of the brachial plexus supplied by C5 to T1 also known as obstetrical brachial plexus palsy, is a flaccid paralysis of the arm at birth, affecting different nerves of the brachial plexus supplied by C5 to T1 PBPP has been associated with shoulder dystocia, infants who are large for gestational age, maternal diabetes and instrumental delivery; however, there are no proven causative correlations. CPS Perinatal brachial plexus palsy

What vitamin supplementation significantly decreases the RECURRENCE risk of neural tube defects?

Folic acid CPS Folic Acid and Neural tube defects

How is milk banking in canada monitored?

Follow all guidelines from HMBANA Adhere to health canada regulations for food substances Inspected regularly by canadian food inspection agency CPS Milk banking

What is the alternative to folic acid supplementation? What is supplemented in UK vs. Canada?

Fortification of commonly ingested products! Vitamin d in milk Iodine in salt Fluoride in water Uk = bread Canada (maybe not everyone at high risk eats bread) = flour (already a little bit is added) CPS Folic Acid and Neural tube defects

Management of fractures from delivery

Fractures of the skull • Due to pressure from forceps or maternal bony prominences • Linear # are the most common: asymptomatic, no treatment required • Depressed #: asymptomatic unless associated intracranial injury; should elevate severe depressions to prevent cortical injury from sustained pressure. • Occipital #: fatal hemorrhage b/c of disruption of the underlying vascular sinuses; may result during breech deliveries from traction on the hyperextended spine Neo Notes

What bilirubin value should you be measuring?

Free bilirubin commercial assays not available and clinically uncertain how these levels correlate It is free bilirubin that theoretically crosses blood brain barrier and causes kernicterus Conjugated bilirubin sometimes not available in a total serum bili value. Needs to be estimated in clinical jaundice bc there are causes of neonatal jaundice that are conjugated Worry about it if >20% of unconjugated or >18micrmol/L Total serum bilirubin is what you care about CPS Hyperbili

Is there any increased risk of adverse neurodevelopmental outcomes after prenatal exposure to SSRIs?

Further studies needed No evidence currently CPS SSRI's

What are ineffective treatments of severe hyperbili in context of hemolysis?

G6PD: Phenobarbitone Tin-mesoporphyrin Synthetic analogue of heme oxygenase that inhibits its activity and thus decreases production bilirubin Previously shown to decrease need for ptx in g6pd deficiency but RCTs have not shown that benefit Prophylactic phototherapy Used for abo isoimmunization No help CPS Hyperbili

GI manifestations/signs of NEC

GASTROINTESTINAL Abdominal distention Occult/gross blood in stool Abdominal tenderness Change in stool pattern/diarrhea Feeding intolerance Abdominal mass Delayed gastric emptying Erythema of abdominal wall Vomiting Neo Notes

What other factors influence prognosis? (5) CPS Counselling and management for anticipated extremely preterm birth

Gestational age most important Female sex Multiplicity Tertiary care centre Antenatal steroids New prognostic tools for outcomes include these, but still imprecise! CPS Counselling and management for anticipated extremely preterm birth

What is G6PD deficiency?

Glucose 6 phosphate dehydrogenase deficiency X linked, recessive mutation Enzyme v. important in red blood cell metabolism and allows the RBC to deal with oxidative stresses Males mostly affected Female heterozygotes are affected too bc >50% of rbc affected with x chromosome random inactivation Incr risk of developing severe hyperbili CPS Hyperbili

Which vagolytic is recommended? CPS Intubation

Glycopyrrolate or atropine None directly compared Caution Glycopyrrolate dose unknown for small prems Atropine must be in the right dosage (10-20micrograms) CPS Intubation

CPS recommendation: what should we do about the different recommendations?

Gov't, health care facilities, health care providers Need to make every attempt to adhere to international code of marketing of breast-milk substitutes and relevant world health assembly resolutions CPS baby friendly

What is the leading cause of neonatal early onset sepsis?

Group B streptococcus (streptococcus agalactiae) CPS term infants at increased risk for early onset bacterial sepsis

Infectious Agents Detected in Milk and Newborn Disease CONTRAINDICATED IN BREASTFEEDING

HIV HTLV-1, HTLV-2 Mycobacterium tuberculosis - Active ds or <2 wks Rx CONTINUE BREASTFEEDING Rubella - Wild type/ Vaccine Epstein-Barr virus Mycobacterium tuberculosis - PPD+/CXR- Cytomegalovirus - term infant HHV-6, HHV-7 Toxoplosma gondii Syphilis[3] CONDITIONALLY OK TO BF Group B streptococci[*] Listeria monocytogenes[*] Mastitis- if no B/L abscess Herpes simplex virus - if no breast vesicles Cytomegalovirus - preterm infant Hepatitis C virus[†] Hepatitis B virus[1] Varicella-zoster virus[2] - cover vesicles † Provided that the mother is HIV-seronegative. Breast milk transmission of hepatitis C virus has not been documented. 1 Immunize and immune globulin at birth. 2 Provide appropriate antivaricella therapy or prophylaxis to newborn. 3 Treat mother and child if active disease. neo notes

NRP - What should you check before starting another intervention?

HR respirations 2013-2014 CPS summaries - Neonatal resuscitation guidelines update: a case-based review

When is conjugated hyperbili worrisome?

HSM/ persistent jaundice >18micromol/L >20% of TSB concentration CPS Hyperbili

Treatment of neonatal anemia by blood transfusion CPS STATEMENT

Hct 30-35% and Hgb 100-120 - if extreme illness; Hct 20-30%, Hgb 60-100 if severe illness, Hct <20 and Hgb <6 if retics <120 Neo Notes

What kind of cooling techniques are available?

Head cooling + mild systemic hypothermia Fontanelle to 30deg using head cooling cap Rectal temp to 34deg +/- 0.5deg w/ radiant body heating Prob: no EEG, harder to control, expensive/labour intensive, scalp edema/skin breakdown Full body cooling Body blankets, cooling packs Adv: eeg, less expensive, easier to use No evidence to say that one is better than another re: death and disability CPS Hypothermia in HIE

What is the US national institute on alcohol abuse and alcholism (NIAAA) definition for heavy and moderate drinking? 1 drink?

Heavy drinking = >/= 2 drinks/day Moderate drinking= 1-2 drinks/day 1 drink 14g/17cc of absolute alcohol 12oz bottle of beer 5oz wine 1.5 oz 80 proof liquor CPS Preventing FAS

Should IVIG be used to treat mild to moderate hyperbilirubinemia?

Helps in immune hemolytic jaundice Studies show decreased exchange transfusion IVIG 500mg/kg, 1g/kg Exact indication unknown CPS recommends: Kids with +ve DAT Predicted severe hyperbilirubinemia At risk of needing exchange CPS Hyperbili

Erythroblastosis fetalis

Hemolytic Disease of the Newborn (Erythroblastosis Fetalis) Neo Notes

1. Baby born at home at 38 weeks by midwife. Now presents at 7 days with melena. Hgb 70, MCV 112, plts normal. What is the most likely diagnosis?

Hemorrhagic Disease of Newborn Hemorrhagic disease of the newborn (classic form) due to lack of Vitamin K administration at delivery Hemorrhagic disease of the newborn (classic form) N p 773 Classic disease -2-7 days -GI, mucosal, intracranial, circumcision, cutaneous bleeding -risks vitamin K deficiency and breastfeeding -prevented by parenteral vitamin K at birth -oral vitamin K regimens require repeated dosing over time -incidence ~ 2% if not given vitamin K

Indications for RBC transfusion in newborns Hemorrhagic shock

Hemorrhagic shock A general but unevaluated approach to emergency treatment includes partial restoration of the circulation with normal saline, at 10mL/kg to 20 mL/kg, while awaiting the emergency provision of group O Rh-negative packed RBCs, which should be administered in similar volumes. A hemoglobin level only provides guidance when chronic bleeding has occurred. The collection of cord or pretransfusion blood for typing is helpful. The administration rate for emergency transfusion depends on the critical state of the circulation, and may range from an initial 1 min push of 20 mL to later stabilization rates of 10 mL/kg/h, depending on the state of recovery. A major risk of rapid and massive transfusion is hyperkalemia. Saline, adenine, glucose and mannitol-prepared blood has an additional supernatant potassium content of approximately 1 mmol/L/day of storage and, therefore, may approach 50 mmol/L at 42 days. This potassium load is rapidly distributed after transfusion and subsequent RBC uptake may even cause hypokalemia. A threshold hemoglobin level for further transfusion has not yet been determined. A theoretical and unevaluated minimum is in the order of 60 g/L. Group O Rh-negative blood may be used in the emergency transfusion of newborns. Otherwise, either group O Rh-compatible or group-specific Rh compatible blood must be used. Starting at four months postnatal age, cross-matching of donor blood is required. In cases of massive hemorrhage, for which a large volume of blood may be required, care should be taken to avoid hyperkalemia and dilution of coagulation factors, using combined replacement with fresh frozen or frozen plasma, as necessary. CPS Red blood cell transfusion in newborn infants

Breast feeding and antimicrobials

High-dose metronidazole Discontinue breastfeeding for 12 h to 24 h to allow excretion of dose Chloramphenicol Trimethoprim/sulfamethoxazole, sulfisoxazole, dapsone Proceed with caution if nursing infant has jaundice or G6PD deficiency, and also if the child is ill, stressed or premature Primaquine, quinine Contraindicated during breastfeeding unless both mother and baby have normal G6PD levels Antibiotics Group 1: Penicillins, cephalosporins, carbapenams, macrolides, aminoglycosides, quinolones Continue breastfeeding Group 2: High-dose metronidazole Discontinue breastfeeding for 12 h to 24 h to allow excretion of dose Group 3: Chloramphenicol Caution: Possible idiosyncratic bone marrow suppression Group 4: Trimethoprim/sulfamethoxazole, sulfisoxazole, dapsone Proceed with caution if nursing infant has jaundice or G6PD deficiency, and also if the child is ill, stressed or premature Antitubercular drugs Isoniazid, rifampin, streptomycin, ethambutol Continue breastfeeding. Infants only need pyridoxine supplementation if receiving isoniazid themselves Antiparasitics Group 1: Chloroquine, quinidine, ivermectin; maternal topical diethyltoluamide or icaridin/picaridin Continue breastfeeding Group 2: Primaquine, quinine Contraindicated during breastfeeding unless both mother and baby have normal G6PD levels Antifungals Fluconazole, ketoconazole Continue breastfeeding Antivirals Acyclovir, valacyclovir, amantadine Continue breastfeeding. If considering prolonged use of amantadine, observe for milk suppression, as it can suppress prolactin production CPS contraindications to breastfeeding

What are contraindications to breastfeeding based on CPS?

Hiv in canada Radioactive isotopes/radiation therapy Cytotoxic chemotherapy Others from other CPS statement: HTLV Human T-cell lymphotropic virus (HTLV) Mastitis with pus Active TB not treated for 2wks Active HSV lesions on the breast CPS baby friendly

_______ may be associated with severe hemolytic anemia and a clinical picture resembling hydrops fetalis; it can be distinguished for hemolytic dz of the NB (RH or severe ABO) by a _____

Homozygous α-thalassemia negative direct Coombs. Anemia and jaundice may occur in infancy from hereditary spherocytosis and other red cell membrane defects, and, if untreated, can result in kernicterus. Hemolytic anemia producing jaundice in the 1st wk of life may also be secondary to congenital deficiencies in RBC enzymes, such as pyruvate kinase or G6PD. Neo Notes

What evidence and suggestions are there for dexamethasone use for CLD?

Hydrocortisone good bc: Anti-inflammatory prevent adrenal insufficiency and thus decr CLD Early hydrocortisone therapy: 15mg/kg x2 doses or 1-2mg/kg x 15 days Some studies were for CLD, others for decr bp No decrease in: Mortality CLD Combo mortality, cld Survivors on home O2 Failure to extubate CP CP or mortality No studies re: late hydrocortisone Looking at one study where hydrocortisone given 1-2wks of age to prems needing o2 and on ventilator compared with healthy counterparts: If treated: Needed less o2 and Weaned from ventilation No difference in neurodevelopmental outcomes: Head circumference Neurological outcome Psychomotor development School performatnce MRI at 8y no diff to other prems in terms of volumes/grey or white matter volumes Big idea: No evidence at this time to show safety or effectiveness But ?promising alternative? CPS Postnatal corticosteroids to treat/prevent BPD

______ is an indication for umbilical vein transfusion in infants with pulmonary immaturity

Hydrops or fetal anemia (Hct <30%) Neo Notes

o Jitteriness or tremors, apathy, episodes of cyanosis, convulsions, intermittent apneic spells or tachypnea, weak or high-pitched cry, limpness or lethargy, difficulty feeding, and eye rolling. o Episodes of sweating, sudden pallor, hypothermia, and cardiac arrest and failure also occur.

Hypoglycemia Neo Notes

Other abnormality found in severe hydrops/hemolytic dz of the newborn

Hypoglycemia occurs frequently in infants with severe isoimmune hemolytic disease and may be related to hyperinsulinism and hypertrophy of the pancreatic islet cells in these infants. Neo Notes

Hypomagnesemia in neonates

Hypomagnesemia Usually assoct with low calcium Seen in IDM; also results in tetany; watch out in exchange transfusions- binds to citrate Tx: IM injections of MgSO4 Calcium should also correct Neo Notes

What are the 4 CPS recommendations for when to give RBC transfusions?

Hypovolemic shock with acute blood loss Extreme illness where transfusion may increase oxygen delivery to vital organs: Hb 100-120 Hct 30-35% Severely ill +/- mechanical ventilation + compromised oxygen delivery Hb 60-100 Hct 20-30% FTT, no weight gain, tachycardia >180, tachypnea/supplemental O2, lethargy Hb <60 Hct <20 Retics <100 (low plasma epo) CPS Red blood cell transfusion in newborn infants

What are the 4 risk factors when giving surfactant?

Hypoxemia Bradycardia - when giving it Obstructing ET tube Pulmonary hemorrhage once given Mortality due to pulmonary hemorrhage not increased Relative risk varies in studies CPS surfactant

PATHOGENESIS of Intrauterine fetal demise

Hypoxia, acidosis, infection, lethal anomaly Neo Notes

1. A baby is born to a mother who did not receive any prenatal care. (I can't remember the other details - maybe something about the bones or a pseudoparalysis?)

I think the answer was 'congenital syphilis'

It is recommended that ______ be routinely screened for neonatal hypoglycemia

IDMs (gestational or otherwise), preterm infants (less than 37 weeks) and SGA infants (weighing at less than the 10th percentile) CPS Hypoglycemia

Environmental Factors Associated with Increased Risk for SIDS INFANT RISK FACTORS:

INFANT RISK FACTORS: Age (peak 2-4 mo, but may be decreasing) Male gender Race/ethnicity (African-American and Native American) Growth failure No pacifier Prematurity Prone and side sleep position Recent febrile illness Smoking exposure (prenatal and postnatal) Soft sleeping surface, soft bedding Infant-mother or -parent bed sharing Thermal stress/overheating Colder season, no central heating NO EFFECT: URTI Immunizations Breastfeeding Neo Notes

Penicillin-allergic women with a high risk of anaphylaxis should be treated with _____ for GBS

IV clindamycin when the GBS isolate is sensitive to clindamycin and erythromycin OR with IV vancomycin when the isolate is resistant to clindamycin or susceptibilities are unknown Because the efficacy of the latter two regimes has not been confirmed in clinical trials, they should be considered inadequate IAP when managing the neonate CPS term infants at increased risk for early onset bacterial sepsis

What treatment methods should be used for symptomatic hypoglycemia?

IV therapy! (always account for breastfeeds in TFI) 2cc/kg 10% bolus Infusion 80cc/kg/day of 10% (5.5mg/kg/min) Check in 30mins should be 2.6 If not raise infusion rate or concentration If refractory: Consider referral (endo/IEM) Start IV glucagon Glucagon bolus 0.1-0.3mg/kg Glucagon infusion 10-20microgram/kg/hr Longer term: Diazoxide, octreotide, hydrocortisone CPS Hypoglycemia

Who should get an immediate full diagnostic work up? CPS term infants at increased risk for early onset bacterial sepsis

If newborn infant has any clinical signs of sepsis Full diagnostic work up: Cbc + diff Blood culture +/- LP +/- CXR (esp if resp distress) No Ucx Start therapy immediately! CPS term infants at increased risk for early onset bacterial sepsis

When should you use surfactant deficiency tests?

If not a candidate for prophylaxis but you are concerned re: RDS, could use these CPS surfactant

CPS recommendation: When should you offer po regimen of vitamin K?

If parents refuse IM dose Oral dose 2mg vitamin K1 at first feeding It's actually the parenteral formulation given orally! Rpt vitamin K1 2mg po @: 2-4wks 6-8wks Counsel that still at risk of HDNB (intracranial hemorrhage, gi bleed, ecchymoses Counsel that it is very important to come back for next doses! CPS Vit K

How and when should hyperbilirubinemia be worked up?

If required intensive phototherapy (ie. Mild-moderate hyperbilirubinemia and beyond) Hx Pex Investigations CPS Hyperbili

CPS recommendation: When should surfactant be given?

If significant risk of RDS: Prophylactic surfactant Minutes after intubation and stabilization CPS surfactant

When should MFM and neonatology be notified? CPS Counselling and management for anticipated extremely preterm birth

If woman is 22wks and at risk of premature delivery CPS Counselling and management for anticipated extremely preterm birth

Postnatal Diagnosis of hemolytic dz of the NB

Immediately after the birth of any infant to an Rh- woman, blood from the umbilical cord or from the infant should be examined for ABO blood group, Rh type, Hct and hemoglobin, and reaction of the direct Coombs test. If the Coombs test is positive, a baseline serum bilirubin level should be measured, and a commercially available RBC panel should be used to identify RBC antibodies present in the mother's serum. The direct Coombs test is usually strongly positive in clinically affected infants and may remain so for a few days up to several months. Neo Notes

Are there concerns with natural surfactants?

Immunologic effects from exposure to surfactant protein A and B from bovine/ porcine sources No evidence that this is a problem Kids with RDS naturally have abs to surfactant proteins Actually less if treated with surfactant! Familial preference bc animal product CPS surfactant

Meconium Ileus in Cystic Fibrosis

Impaction of meconium causes intestinal obstruction. The absence of pancreatic enzymes in CF limits normal digestive activities in the intestine, and meconium becomes viscid and mucilaginos. Treatment is high Gastrografin enemas and laparotomy. 50% of these infants have associated intestinal atresia, stenosis, or volvulus that requires surgery. MECONIUM PERITONITIS: Perforation of the intestine may occur in utero or shortly after birth. Perforations occur most often as a complication of meconium ileus in infants with CF, but are occasionally due to a meconium plug or in utero intestinal obstruction of another cause. Characteristically noted are abdominal distention, vomiting, and absence of stools. Treatment consists primarily of elimination of the intestinal obstruction and drainage of the peritoneal cavity. Neo Notes

_____, in the setting of hypoxia, is the main mechanism causing brain injury following intrapartum hypoxia-ischemia

Impaired cerebral blood flow CPS Hypothermia in HIE

CPS recommendation: When should you consider your first treatment as failure and retreat? CPS surfactant

In 2hrs FiO2 is >30% persistent/recurrent 2hrs is a minimum. Usually more like 4-6hrs. CPS surfactant

What is a "limited diagnostic evaluation"? CPS term infants at increased risk for early onset bacterial sepsis

In mom's room Cbc + diff Q4h HR, RR, temp x 24h if wbc <5x109/L Full septic work up ?treatment CPS term infants at increased risk for early onset bacterial sepsis

In older patients, the _____ triad must be considered when considering hirschprungs dz

In older patients, the Currarino triad must be considered (anorectal malformations—ectopic, anus, rectal stenosis; sacral bone anomalies—hypoplasia, poor segmentation; or presacral masses—anterior meningoceles, teratoma, cysts). Neo Notes

Why are G6PD levels normal sometimes even if you are deficient? Who does this most often happen to?

In presence of illness and hemolysis, can have normal values Esp african american type where new rbc are not deficient Esp heterozygote females w/ varying levels CPS Hyperbili

What does pasteurization do for pathogens in breast milk?

Inactivate bacterial/viral contaminants like CMV Spore-forming bacillus species may persist but they are an uncommon contaminant in breast milk unlike cow's milk and are detectable in the double culture done CPS Milk banking

What is the incidence, mortality, of GBS early onset sepsis? Why don't we know the true incidence of GBS disease in Canada?

Incidence invasive early onset sepsis w/ GBS= 2/1000 live births if intrapartum abx not given Mortality = 2-13% No centralized reporting system CPS term infants at increased risk for early onset bacterial sepsis

What were 4 problems with previous storage of blood?

Increase extracellular K+ Increase extracellular pH Decrease in RBC enzyme (2,3-diphosphogycerate) Too long kept and K+ would increase CPS Red blood cell transfusion in newborn infants

How does breastfeeding affect intelligence?

Increase performance of neurocognitive testing CPS baby friendly

Risk estimation for anticipated mortality or long-term NDD: Moderate-to-high likelihood of mortality or moderate-to-severe NDD Suggested level of care: Intensive care or palliative care are both usual care options Clinical examples that usually meet the risk estimation

Infant born at 23-24 weeks GA, irrespective of most additional risk factors*** • Infant born at 25 weeks GA, with signs of fetal anemia and abnormal placental blood flow CPS Counselling and management for anticipated extremely preterm birth

Risk estimation for anticipated mortality or long-term NDD: Low likelihood of mortality or moderate-tosevere NDD Suggested level of care: Intensive care is recommended** Clinical examples that usually meet the risk estimation

Infant born at 25 weeks GA, without additional risk factors*** • Infant born late in 24th week of gestation (e.g., 245), well grown with ANCS given, born in a tertiary care centre CPS Counselling and management for anticipated extremely preterm birth

How to do kangaroo care

Infant in diaper and cap Held upright prone against bare chest of parent Covered with clothing/blanket 1-3h Cardioresp/temp monitoring Usually stable and no ventilation, can do in as light as 600g, </= 26wks, newly born Should have a protocol for each centre CPS Kangeroo Care

When should infants at risk of RDS receive surfactant therapy? (prophylactic or rescue)

Infants at risk of RDS should receive surfactant within minutes of stabilization Ie. Prophylactically to infants at risk of RDS Studies show decrease in pulmonary leak syndrome and mortality Studies show that it should be used soon after stabilization because even minor delays increases mortality Nobody really knows the effect of antenatal steroids it does change the time of respiratory distress physiology. Would give it prophylactically to those without antenatal steroids CPS surfactant

Hypermagnesemia in neonates

Infants of mother's treated with MgSO4 during labour and in delayed passage of MEC, can result in CNS depression Tx: IV Calcium and diuresis Exchange transfusion if really severe Neo Notes

15. A neonate is born to a mom who is an IVDU who received no prenatal care. List two possible infections that are vertically transmitted, and the treatment for each disease.

Infection Treatment Hepatitis B Hepatitis B vaccine at birth, 1-2 mo, and 6 months; and IVIG at birth. Post-vaccination testing at 9-12 months. HIV CPS statement: urgent consultation with HIV expert; anti-retroviral therapy within 6-12 hours of birth (triple combination ART or Zidovudine for 6 weeks + 3 doses of Nevirapine in first week of life) Hepatitis C Serology at 12-18 months; no specific therapy

What kind of consent is needed before giving donor breast milk?

Informed consent All risks and benefits of human donor milk Health advantages compared to bovine milk All the options Need written informed consent CPS Milk banking

What must be done before giving the donor milk?

Informed consent by parents Order by medical provider CPS Milk banking

What is iNO? How does it work?

Inhaled nitrous oxide Potent vascular muscle tone regulator (dilates pulmonary artery) Primarily used in newborns with persistent pulmonary htn of the newborn where increased pulm pressures cause shunting of blood right to left over pfo or pda L arginine NO by NO synthases in the lung endothelium NO diffuses into Lung vasculature and activating enzymes that vasodilate pulm vasculature and improve ventilation/perfusion matching CPS iNO

What dose of surfactant should be used?

Initial dose of ~100mg of phospholipid/kg body weight Some evidence to say that lower for prophylaxis, higher for rescue CPS surfactant

1. You are called to the newborn nursery to see a 4 hour old newborn female in respiratory distress. The RR is 70 and you note increased work of breathing. The nurses have also noted that the baby has a significant amount of oral secretions. a. What one test is required to make the diagnosis?

Insertion of orogastric tube • Diagnosis = Esophageal atresia o Insert an NG and take an CXR (NG tube coiled in esophagus)

10 steps to baby friendly/breast feeding

Integrated 10 Steps Practice Outcome Indicators [35] Step 1: Have a written breastfeeding policy that is routinely communicated to all health care providers and volunteers. Step 2: Ensure all health care providers have the knowledge and skills necessary to implement the breastfeeding policy. Step 3: Inform pregnant women and their families about the importance and process of breastfeeding. Step 4: Place babies in uninterrupted skin-to-skin contact with their mothers immediately following birth for at least an hour or until completion of the first feeding or as long as the mother wishes: Encourage mothers to recognize when their babies are ready to feed, offering help as needed. Step 5: Assist mothers to breastfeed and maintain lactation should they face challenges including separation from their infants. Step 6: Support mothers to exclusively breastfeed for the first six months, unless supplements are medically indicated. Step 7: Facilitate 24-hour rooming-in for all mother-infant dyads: mothers and infants remain together. Step 8: Encourage baby-led or cue-based breastfeeding. Encourage sustained breastfeeding beyond six months with appropriate introduction of complementary foods. Step 9: Support mothers to feed and care for their breastfeeding babies without the use of artificial teats or pacifiers (dummies or soothers). Step 10: Provide a seamless transition among the services provided by the hospital, community health services and peer support programs. Apply principles of primary health care and population health to support the continuum of care, and implement strategies that affect the broad determinations that will improve breastfeeding outcomes. CPS baby friendly

When should irradiated RBC be used? 4

Intrauterine transfusions If received a previous intrauterine transfusion Immunocompromised infants Infant receiving blood from a relative 1st or 2nd degree relative blood needs to be irradiated to PREVENT GRAFT VS. HOST disease Leukocyte reduction filters not enough NOT YOU NORMAL SMALL RBC TRANSFUSIONS! CPS Red blood cell transfusion in newborn infants

How does epo physiology differ intrauterine and extrauterine?

Intrauterine, erythropoietin made in liver Extrauterine, changed to kidney primarily Switch determined genetically, magically :D In prem kids low oxygen sensors on kidney and liver likely cause low epo production Thus, contributes to anemia of prematurity CPS Red blood cell transfusion in newborn infants

CPS recommendation: In what 6 situations should newborns receive surfactant therapy?

Intubated + RDS Intubated + RDS + before transport Intubated + MAC Sick + Pneumonia + OI 15 Intubated + pulmonary hemorrhage + clinical deterioration Intubated + </=29wks in periphery CPS surfactant

What and how much of vitamins/iron should anemic newborns take?

Iron 4-4.5mg/kg/day This is the dose to prevent late anemia Don't know what the optimal dose is Vitamin E 25IU Prevent hemolytic anemia Vitamin B12 and folate Make sure it is part of the diet. CPS Red blood cell transfusion in newborn infants

What are the 8 risk factors that increase your risk of developing severe hyperbili and thus needs to be treated at lower TSB [] on the normogram for PTX?

Isoimmune hemolytic disease (ABO/rh) G6PD Asphyxia Sepsis Respiratory distress Temperature instability Extreme lethargy acidosis CPS Hyperbili

What are symptoms of neonatal hypoglycemia?

Jittery/tremors Convulsions Cyanosis Difficulties feeding Eye rolling Sweating Pallor Hypothermia Cardiac arrest/failure CPS Hypoglycemia

How do you deliver accurate and consistent information to parents? (6)

Joint neonatal/obstetric counselling Interdisciplinary guidelines/staff info cards Case management rounds Social workers/spirtiual care workers Clear documentation Easily available consults with updated management plans Labour management Mode of delivery Care of infant CPS Counselling and management for anticipated extremely preterm birth

Chorionic villus biopsy is used for:

Karyotype, molecular DNA genetic analysis, enzyme assays Korionic = Karyotype Neo notes

Paralyzed hand + ipsilater ptosis and miosis (Horner syndrome)

Klumpke C7-T1 Paralyzed hand + ipsilater ptosis and miosis (Horner syndrome) Treatment Partial immobilization to prevent contractures Gentle massage If persists beyond 3-6mos then neuroplasty, neurolysis Neo Notes

What are five critical attributes to attentive care provider? Do you need to follow up with the parents? How can pediatricians or obstetricians play a role? CPS perinatal loss

Knowing - empathy Caring Doing what you would want for yourself (mementos/pictures etc) Enabling - anticipatory guidance Maintain belief that a path filled with meaning will be chosen Yes Maintain healthy grieving started in hospital Grief counsellor/health care professional that they met in hospital Pedatricians/obs = esp if no time spent in NICU etc May need to have visit before and after death Explain results of autopsy Follow up appointment in a few weeks Talk about parents and siblings Referral to regional counsellers/bereavement specialists CPS perinatal loss

Canadian Pediatric Review 2018 Perinatal-Neonatal Medicine Oxygen and alternate airways

LMA Size 1 wt>2kg GA > 34 wks OXYGEN Start in room air TERM (term infant = improved survival) 21-30% PRETERM Target saturations Preterm end-target: 88-94% increase to 100% when starting compressions

What are the 6 categories affected in HIE?

LOC Spontaneous activity Neuromuscular activity Primitive reflexes Autonomic system Seizures Seizure PLANS (prim reflexes, LOC, Autonomic, Neuromusc, Spont activity) CPS Hypothermia in HIE

Late complications of hemolytic dz of the NB

Late Complications: Late anemia may be hemolytic or hyporegenerative. Treatment with supplemental iron, erythropoietin, or blood transfusion may be indicated. A mild graft vs host reaction may be manifested as diarrhea, rash, hepatitis, or eosinophilia. Neo Notes

How is po vitamin K related to late hemorrhagic disease of the newborn?

Late hemorrhagic disease of the newborn Usually with breastfed infants Rise in frequency w/ po vitamin K Why? If not enough vitamin K, induced protein (PIVKA-II) is still seen in the blood IM decreases this value faster, po takes 5 days to clear but no difference at 5d between route At 4-6wks, more po vitamin K kids show signs of vitamin K deficiency (?PIVKA-II) vs. IM vitamin K kids (20% vs .5%) Have tried a micelle form of po vitamin K but difference persists Big question is how do these biochemical markers correlate with risk of bleeding? CPS Vit K

How should glucose testing be done? CPS Hypoglycemia

Less point of care capillary bedside glucose strips: Not accurate at low glucose levels Prone to sample/observer error Formal lab analysis better Remember whole blood, plasma, cap, venous, delayed sampling can affect the value Delayed sampling a problem because cellular component of whole blood will use glucose for glycolysis! CPS Hypoglycemia

Level of consciousness and spontaneous activities in categories in HIE

Level of consciousness Mild: Hyperalert Moderate: Lethargic Severe: Stupor or coma Spontaneous activity Mild: Normal Moderate: Decreased Severe: No activity CPS Hypothermia in HIE

Why is it important to test for G6PD deficiency?

Lifelong illness w/ implications More likely to develop severe hyperbili More likely to need exchange transfusion CPS Hyperbili

CPS recommendation what is the preferred nutrition for newborns in the hopsital?

Like all newborns, should be their own mother's milk If not available and hospitalized human donor milk is a recommended alternative CPS Milk banking

What are the likely organisms for early onset sepsis? What is the best choice for empirical therapy? CPS term infants at increased risk for early onset bacterial sepsis

Likely organisms: Listeria monocytogenes E.coli Group B strep regular = amp and gent Meningitis signs or +ve LP = cross bbb CPS term infants at increased risk for early onset bacterial sepsis

Baby well w/ GBS NEGATIVE mother with Risk factors at delivery?

Limited diagnostic work up Cbc + diff Observation x 24h Or none! According to recommendation section! This is to catch the VERY rare case of GBS neg but turned positive since testing and risk factors present. ****Big idea! If you have GBS unknown ask immediately about risk factors, if positive you are gbs +Ve! well + colonized + other risk factors = individualize (q4h vitals x24h + dc counselling) well + unknown + risk factors 1 -> IAP Abx yes = NL care/dc counselling @ 24h well + unknown + risk factors 1 -> IAP Abx no = examine + q4h vitals x24 + dc counselling well + unknown + risk factors >1/Chorio = -> individualize = q4h vitals x24h and consider CBC @ 4h • Symptomatic infant - Full septic workup if strong suspicions EOS or signs of meningitis (sz, fontanelle, irritable, altered neuro) - Ampicillin + aminoglycoside • Well-appearing, at-risk infants: - CBC might be helpful after 4 hours (WBC < 5, ANC <1.5) - CRP serial measures might be useful, but poor sensitivity early in disease - Refer to algorithm Adequate IAP consists of at least one dose given at least 4 h before birth of: IV penicillin G (initial dose 5 million units) or ampicillin (initial dose 2 grams) OR IV cefazolin (initial dose 2 grams) if the mother is allergic to penicillin but at low risk for anaphylaxis Penicillin-allergic women with a high risk of anaphylaxis should be treated with IV clindamycin when the GBS isolate is sensitive to clindamycin and erythromycin OR with IV vancomycin when the isolate is resistant to clindamycin or susceptibilities are unknown. CPS term infants at increased risk for early onset bacterial sepsis

Should a single dose be used or multiple doses? What does the CPS recommend?

Limited evidence Multiple doses in 24-72hrs seemed to: Decrease mortality Decrease pulmonary leak syndrome If an infant with RDS has persistent oxygenation and ventilatory requirements in first 72h, should receive additional surfactant. More than 3 doses have not shown benefit CPS surfactant

What two things must you consider in the decision to resuscitate an extremely preterm infant?

Long term disability Risk of mortality CPS Counselling and management for anticipated extremely preterm birth

Feeding late preterms Which mothers have the hardest time with feeding late prems? How do you observe that feeding is effective? Should late prems be given fortifier? What are 6 aspects of feeding needed before discharge home of late prem?

Longer feeding times Shorter feeding intervals Because suck and swallow immature Poor suck Poor latch Leads to poor lactation Recommendations • Twenty-four hours of successful feeding of late preterm infants must be established before discharge home. • First-time mothers, in particular, require careful supervision and, when infants are leaving from an intensive care environment, should have a rooming-in experience. Older First time mothers C-section Observe feed Is infant satisfied How often are they feeding? Is baby peeing enough Is baby gaining weight? No evidence that it helps long term Does increase weight gain faster If do fortify look for higher protein/mineral formula or fortifier Observe for 24hrs Not more than 6h in the day taken up by feeding Not longer than 20mins feed If first time mom, needing a rooming in experience if baby was in intensive care Early weight loss can't exceed 10% body weight Need to consider health, parenting, feeding skills of mother and supports at home CPS facilitating discharge of late preterms infants

When do breastfed infants lose the most weight and how much weight do they tend to lose?

Lose most weight on day 3 Up to 6-8% normal >10% needs evaluation CPS Hyperbili

What is the unique aspect of neonatal death?

Loss of the baby but everything that baby meant to the people's lives: Lost future life Lost parental role Loss of a dream Loss of family Hits hard regardless of age Hits each of the parents differently with different phases of grief at different times Usually takes 2-4yrs, sometimes 5 CPS perinatal loss

What are other possible neonatal effects of fetal exposures to SSRIs?

Lower birth weight Lower gestational age Respiratory distress Admission to NICU CPS SSRI's

What about <1000g birth weight babies? (followed into adolesence and adulthood?) CPS Counselling and management for anticipated extremely preterm birth

Lower education attainment Increased neurosensory impairment Functional limitations in cognition/sensation/ mobility/ selfcare No difference: Self esteem Quality of life CPS Counselling and management for anticipated extremely preterm birth

Environmental Factors Associated with Increased Risk for SIDS MATERNAL AND ANTENATAL RISK FACTORS

MATERNAL AND ANTENATAL RISK FACTORS Elevated 2nd trimester serum α-fetoprotein Smoking Alcohol use Drug use (cocaine, heroin) Nutritional deficiency Inadequate prenatal care Low socioeconomic status Younger age Lower education Single marital status Shorter interpregnancy interval Intrauterine hypoxia Fetal growth restriction Neo Notes

Fetal fibronectin

MATERNAL CERVIX Indicates risk of preterm birth Neo Notes

What is oxygenation index?

MFP FiO2 x MAP x 100 ---------------- PaO2 CPS iNO

What is the international code of marketing breast milk substitutes?

Made by the WHO Protect breastfeeding by ensuring ethical marketing of breast milk substitutes by the industry CPS baby friendly

Major risk factors for Development of Severe Hyperbilirubinemia in Infants > 35 weeks

Major risk factors Predischarge TSB or TcB level in the high-risk zone Jaundice observed in the first 24 hr Blood group incompatibility with positive direct antiglobulin test, other known hemolytic disease (G6PD deficiency) Gestational age 35-36 wk Previous sibling received phototherapy Cephalohematoma or significant bruising Exclusive breastfeeding, particularly if nursing is not going well and weight loss is excessive East Asian race Neo Notes

Can end-tidal carbon monoxide be used to predict those with severe hyperbili?

Make sense bc during hemolysis, have incr CO But does not correlate well and not anymore useful when added to TSB measurements CPS Hyperbili

Should all babies have blood group done and DAT done?

Makes sense OR of severe hyperbili with mom O is 2.9 (OR = odds of having it if exposed) Bc baby w/ jaundice and mom O, likely A/B w/ isoimmunization ALSO, if DAT+ve will need ptx more than DAT -ve BUT, Testing universally (BG/DAT) does not improve outcomes: more than testing group O mothers alone More than testing babies with group O mothers and clinically jaundice Big idea: Do a DAT and blood group if : mother is O and baby has jaundice Infant is showing increased risk of developing severe hyperbili bc of timed serum bili [] If positive, leads to higher risk on normogram CPS Hyperbili

How does the timing of circumcision affect UTIs? Does circumcision affect your risk of sexually transmitted diseases?

Males in general: Incr rate of UTI in 1st month Males w/ circumcision: Incr rate of UTI immediately post op (9-20d) Girls' rates constant throughout UNCLEAR Extra risk of STD w/ uncircumcised penis ?incr HIV association in multiple studies but then not in others (lots in Africa etc) Unclear of robustness of studies Believe: No extra risk of STD w/ circumcision from studies (postulated risk of incr non-gonococcal urethritis) CPS circumcision

CPS recommendation: What should provincial/territorial ministries of health do about breastfeeding?

Mandate development of strategy to implement BFI in all health care facilities/community health centres/physician offices Need to appoint coordinator to develop/coordinate these strategies CPS baby friendly

NRP - What does MRSOPA mean?

Mask readjustment Reposition airway Suction mouth and nose Open mouth Pressure increase Alternative airway 2013-2014 CPS summaries - Neonatal resuscitation guidelines update: a case-based review

HIGH RISK INFANTS (~9% of all births) Socioeconomic

Maternal age <16 or >40y, Illicit drug, alcohol or cigarette use, Poverty, Unmarried, Stress neo notes

______ are the factors most commonly associated with EOS.

Maternal group B streptococcal (GBS) colonization in the current pregnancy, GBS bacteruria, a previous infant with invasive GBS disease, prolonged rupture of membranes (≥18 h), and maternal fever (temperature ≥38oC) These risk factors are additive; the presence of more than one factor increases the likelihood of EOS. CPS term infants at increased risk for early onset bacterial sepsis

How do we screen for neural tube defects in the prenatal period?

Maternal serum alpha protein + ultrasound at 15-20wks Catches 85-90% of affected pregnancies If previously affected: Amniocentesis to test amniotic fluid for alpha fetoprotein Catches >95% of cases w/ 0.5-1% risk CPS Folic Acid and Neural tube defects

How should you hydrate infants on ptx?

May need IVF if dehydrated significantly Should still enterally feed because: Rehydration Provide energy Reduce enterohepatic reuptake of bili CPS Hyperbili

Which ethnic groups should be tested for G6PD?

Mediterranean Middle east Southeast asian african CPS Hyperbili

Autonomic system in categories in HIE

Mild Autonomic system: Sympathetic Pupils: Mydriasis Heart Rate: Tachy Respiration: NL Secretions: Sparse Seizures: None Electroencephalogram: Mild depression Moderate Autonomic system: ParaSympathetic Pupils: Miosis Heart Rate: Bradycardia Respiration: Periodic Secretions: Profuse Seizures: Common Electroencephalogram: Moderate depression Severe Autonomic system: None Pupils: Nonreactive Heart Rate: Variable Respiration: Apnea Secretions: Variable Seizures: Uncommon Electroencephalogram: Severe depression CPS Hypothermia in HIE

Processing of donating breast milk

Milk processing in North America follows guidelines set out by the HMBANA. Processing of human breast milk in Canada must also adhere to Health Canada regulations for food substances and must be inspected regularly by the Canadian Food Inspection Agency. All member banks of the HMBANA are not for profit and supply milk to NICUs on a cost-recovery basis. Each free-standing milk bank must have a medical director and a governing board that includes physicians, dieticians, lactation consultants, nursing and infection control representatives. Mothers are not accepted if they are taking medications, smoke or drink. They are temporarily excluded during periods of over-the-counter medication use. the milk is batched from up to four different mothers to blend constituent variations. The milk is then thawed, and a bacterial culture is taken. The milk then undergoes Holder pasteurization (62.5°C for 30 min) in an industrial grade pasteurizer, and is recultured. Any milk that is culture positive for any pathogen or for greater than 104 colony-forming units/mL of skin flora before pasteurization or any positive culture after pasteurization is discarded. It may be prescribed for a variety of medical conditions such as preterm birth, gastrointestinal surgery, malabsorption or feeding intolerance, and immunodeficiency. The process of pasteurizing human breast milk inactivates bacterial and viral contaminants such as cytomegalovirus. Spore-forming Bacillus species are known to survive routine Holder pasteurization but, unlike cow's milk, this is a rare contaminant of human breast milk and is detectable from the surveillance cultures performed before and after pasteurization Many of the nutritional components are not altered or only minimally reduced in content through the process of pasteurization. Carbohydrates, fats and salts are unchanged. Thirteen per cent of the protein content is denatured. Fat-soluble vitamins are unchanged. There are effects on immunological factors. Along with inactivation of all viruses and most bacteria through pasteurization, all beneficial immune cells are also inactivated. Secretory immunoglobulin (Ig) A, which binds microbes within the digestive tract, is found at 67% to 100% of its original activity. Targeted IgG antibodies are reduced at 66% to 70%. IgM antibodies are completely removed. Lactoferrin, which binds iron required by many bacteria, thus reducing their growth, is reduced to 20% of its original level. Lysozyme enzyme, which attacks bacterial cell walls, drops to 75% activity. A reduction in certain cytokines by pasteurization permits an expanded function of epidermal growth factor, which may lead to increased growth of intestinal epithelial cells exposed to pasteurized human donor breast milk CPS milk banking

What follow up should be offered for newborns at high risk for severe hyperbilirubinemia?

Monitoring until feeding/weight gain established Definitely need hearing screen with brainstem auditory evoked potentials bc. Of high risk of neurosensory hearing loss CPS Hyperbili

What kind of light sources are used?

Most common = fluorescent Can use: Halogen but cannot be placed to close to baby Fibreoptic Good because mom can breastfeed, no eye pads Bad because not as high light intensity CPS Hyperbili

Neonatal meningitis bugs

Most common: GBS, E. coli, and L. monocytogenes. Other: Pneumococcus, non-typable Haemophilus influenzae, staphylococci, Klebsiella, Enterobacter, Pseudomonas, T. pallidum, & Mycobacterium tuberculosis Neo Notes

Contraindications to breastfeeding

Most notably, mothers who are HIV-positive in Canada are recommended to formula-feed their infants. Mothers receiving cytotoxic chemotherapy should be counselled to discontinue breastfeeding for the duration of treatment [53]. Mothers receiving radioactive isotopes or radiation therapy may also be counselled to temporarily suspend breastfeeding during their treatment course. Infants with classic galactosemia should not receive breast milk [54]. While breastfeeding was considered contraindicated in phenylketonuria in the past, current practice in many, if not most, metabolic disease treatment centres encourages breastfeeding to supplement a low-phenylalanine formula, along with strict monitoring of phenylalanine levels CPS baby friendly

What does pasteurization do to the nutritional content of breast milk?

Most things are unchanged! Some water soluble vitamins studied can degrade Thirteen percent of protein is denatured CPS Milk banking

CPS recommendation: When should multiple doses of surfactant be considered? How many doses is acceptable?

Multiple doses considered if: Persistent oxygenation and ventilatory requirements over 72h life Max 3 doses CPS surfactant

seizures that are bilateral jerking associated with flexion of upper and occasionally lower extremities (more often EEG changes)

Myoclonic Seizures: rapid nonrhythmic jerks - Generalized Neo Notes

what is the most common life-threatening emergency of the gastrointestinal tract in the newborn period

NECROTIZING ENTEROCOLITIS (NEC) Neo Notes

Should breastfeeding be stopped for physiologic or breast milk jaundice?

NO Breast-milk jaundice: Watch latch Teach appropriate skills Monitor feeding closely May need certified lactation consultant CPS baby friendly

Have there been any cases of transmission of disease through donor breast milk? What is needed before you give?

NO But risk is not 0 Need parental consent before you give? CPS Milk banking

Should we use oral agar to treat mild-moderate hyperbilirubinemia?

NO Does not prevent enterohepatic reuptake of bilirubin CPS Hyperbili

Should we be using erythropoeitin regularly as a treatment for neonatal anemia?

NO Recent study showing variable results! Don't know safety Don't know dose Don't know timing Don't know nutritional support CPS Red blood cell transfusion in newborn infants

What is methemoglobin?

NO absorbed into blood and binds ion of heme protein Nitrosyl hemoglobin formed and oxidized to methemoglobin w/ release of nitrates Should be <2.5% (not usually a problem if at suggested doses) Prem infants more at risk but usually oka if <20ppm CPS iNO

Does CPS endorse sharing of unprocessed human milk?

NO! CPS Milk banking

Is clinical assessment of jaundice accurate?

NO! Not visible w/ TSB <68micromol/L Only 50% of infants w/ TSB >128micrmol/L visibly jaundiced One study showed 100micromol/L difference in what MD visually guessed and actual amount CPS Hyperbili

Should breastfeeding be interrupted for hyperbilirubinemia treatment?

NO! Stopping more associated with stopping bf at 1mos Stopping not associated with: poorer outcomes Longer treatment (observation that slower to decrease in first 24h) Decreased frequency of normalizing bilirubin in 48h CPS Hyperbili

Should tin-mesoporphyrin be used to treat mild-moderate hyperbilirubinemia?

NO! Heme oxygenase synthetic analogue Blocks production of bilirubin DOES NOT HELP CPS Hyperbili

Is universal screening between 24-72h w/ TSB and TcB enough?

NO- still needs to be assessed for jaundice repeatedly between 0-72h There can still be significant jumps in bili esp if there is weight loss! Person assessing needs to be trained to recognize, able to do a tsb and tcb quickly and refer/admit to a hospital if required CPS Hyperbili

what NIPPV is useful for obstructive apnea

Nasal CPAP (2-5 cm H2O) and high-flow humidified nasal cannula (1-2.5 L/min) are effective for mixed or obstructive apnea. Neo Notes

CPS recommendation: Which type of surfactant should be used?

Natural surfactant preferred over any artificial surfactant at time of publication (2005 and reaffirmed 2013) CPS surfactant

What type of surfactant is prefered

Natural surfactants should be used in preference to any of the artificial surfactants available at the time of publication of this statement The incidence of BPD is not different in babies given natural or synthetic surfactants, but because mortality is reduced in babies given natural surfactants, the combined outcome of death or BPD is reduced natural surfactants improve survival without BPD and with a lower incidence of airleak CPS surfactant

OSCE NB with hypoglycemia. Describe mngt step by step with answers from examiner.

Nec -Amp, Gent, Flagyl -Consult GenSx -Transfer to tertiary care center RF for GBS Know GBS HUS -day 3, day 10, day 42 and @ term, if > 1250g skip day 3 Grade 1 and 2 -no impact on prognosis, Grade 3 or 4 -impact likely Grading likely to be changed in near future and Grade 1 and Grade 2 on HUS doesn't necessarily rule out PVL harder to see on U/S Likely will see PVL on day 42 HUS

What kind of research is needed?

Need more prospetive studies evaluating benefits of banked human breast milk in preterm infants in the NICU CPS Milk banking

What follow up should be arranged for pts who have had exchange transfusion or neurologic abnormalities?

Need referral to regional multidisciplinary follow-up programs CPS Hyperbili

CPS recommendation: What should be done for the HCPs, managers, volunteers who work at hospitals/community services that care for mothers and children?

Need to create an educational strategy for them CPS baby friendly

What are CPS recommendations regarding hypoglycemia and thermal instability?

Need to demonstrate euglycemia before going home Need to make sure home envirmonment is warm enough to safely support infant without excessive clothing/bedding CPS facilitating discharge of late preterms infants

open neural tube defects screening

Need to get A's between 15-18 to get open offers to university! (open = gut/abdomen = gastroschisis/ompahlocele) Second-trimester screening (15-18 wk) of maternal serum α-fetoprotein (MSAFP) levels is used to screen for open neural tube defects. About 90% of affected pregnancies can be detected by an elevated MSAFP level. Gastroschisis, omphalocele, congenital nephrosis, twins, and other abnormal conditions can also be identified. Neo Notes

CPS recommendation: what should governments do about breastfeeding and why?

Need to increase: breastfeeding initiation Duration Exclusivity rates Because: Important health, immunological, emotional, cognitive benefits for infant/children CPS baby friendly

How do you deal with the death of a twin? CPS perinatal loss

Need to rejoice and mourn at the same time Avoid regarding surviving twin as consolation bc parents saw them together. Might be harder than a singleton CPS perinatal loss

What do you need to tell parents? CPS Perinatal brachial plexus palsy

Need to set up realistic expectations Provide a clear explanation of nerve injury and sequelae C5-T1 from spine, bundle together to innervate different muscles of the arm. This has been damaged and results in the paralysis of your baby's arm 3 must says: pBPP is not always preventable 75% of infants will recover in 1st month of life 25% will have permanent impairment and disability CPS Perinatal brachial plexus palsy

1. You are treating a 38 week old newborn who was flat at birth but improved with 2 minutes of bag valve mask. Now at 12 hours of age the baby is tachypneic at 20, BP is 38/20 (mean 32). You take the following CXR. (CXR with L sided pneumothorax, RUL collapse but no mediastinal shift). What is your management?

Needle decompression and Chest tube insertion • Needle thoracocentesis - 2nd intercostal space in the MCL o It's surprising that there's no mediastinal shift on CXR, but the baby is becoming hemodynamically unstable (hypotension) so better to assume a tension pnuemothorax and perform needle decompression than wait • Procedure: o Second intercostal space (21-23 gauge scalp vein needle or 22-24 gauge angiocath with 20 ml syringe w/ 3-way stopcock) and formal chest tube placement later ♣ Should hear a rush of air ♣ Repeat CXR • Typical Symptoms of Tension pneumothorax: o Asymmetric chest (bulging of affected side) o Abdominal distention (d/t downward displacement of diaghram) o CVS changes: ♣ Initially increase in systolic BP then narrow pulse pressure hypotension, decreased HR ♣ Shift of apical impulse away from affected side (most consistent finding) o Resp: cyanosis, hypoxia, incr RR, decr breath sounds on affected side • Typical signs of tension pneumothorax: o Low PaO2, Increased PCO2 o Transillumination of chest (usually only works on thin prems however) o AP CXR: ♣ Mediastinal shift away from pneumothorax ♣ Depressed diaphragm on same side as pneumothorax ♣ Displacement of lung on pneumothorax side away from chest wall by band of air o Lateral CXR: ♣ Rim of air around lung(aka pancaking) ♣ lat decub view: pneumo side up - can detect a small pneumotx not seen on routine CXR • Asymptomatic pneumothorax • Seen in 1-2% of neonates o Note that a pneumothorax occurs in 15-30% of infants who receive PPV o Of these 15% were meconium stained. • Management: o Can observe with f/u CXR q 8-12 hr or sooner if symptomatic ♣ Usually resolves in 48 hrs o Can give 100% O2 x 8-12 hr for faster resolution (nitrogen washout- less N is able to enter lungs and absorption of N from extrapleural space is increased and exhaled; total gas tension decreases which help absorption of N by blood ♣ Only to be done with full term babes that don't have ROP ♣ Controversial whether nitrogen washout actually works

Why is it so important to educate early about breastfeeding?

Needs to be perceived as the NORM! University aged parents know that it is important but feel like breastfeeding is something you do in private (which detracts from bf) CPS baby friendly

1. 36 wk baby 2.1 kg (5lbs) at birth, day 7 jittery, irritable, on exam HR 218, T 37.5, RR 70 bp 90/60. Face is flushed, eyes wide open, alert but irritable, normal tone and normal cry, jaundiced, DTR's 2/4 and symmetric. There is hepatosplenomegaly. What is the likely diagnosis and list 2 tests to confirm the diagnosis?

Neonatal Graves Disease; TSH, T4 or fT4

Canadian Pediatric Review 2018 Perinatal-Neonatal Medicine Neonatal Seizures

Neonatal Seizures • DDX: jitteriness, sleep myoclonus • Types: myoclonic, tonic, clonic, other • Etiology - Clues: GA, onset - HIE: usually 1st 24 hours; often term baby - IVH: preterm, 1st 2 days - Metabolic: ↓Gluc ↓Ca ↓ Mg; in IEM intractable seizures + other derangements (encephalopathy, acidosis, ammonia) - Other: stroke, drugs (SSRI), NAS (opiates, benzodiazepines), infection, brain malformations, benign neonatal seizures (family hx/exclusion) • Investigate: EEG, EPs, Imaging (US/MRI/CT) - Work up etiology • Treatment: - Treat the cause! - Anti-convulsants: Phenobarbital 20mg/kg, phenytoin 20, Lorazepam • Prognosis: - bad for malformations, severe IVH / HIE; IEM - hypoglycemia, SAH, hypocalcemia usually normal - neonatal stroke - unilateral 1/3 normal -1/3 mild - 1/3 severe

1. A newborn baby present with the following rash. He has thrombocytopenia and mild hepatitis. What is the diagnosis?

Neonatal lupus erythematosus - get cutaneous lesions in 50% of babies, rash is annular plaques (precipitated by sunlight, usually face/peri-orbital). Get heart block, hepatic involvement/hepatitis/hepatomegaly in 10%, get anemia, thrombocytopenia

NRP Updates

Neonatal resuscitation • 10% of newborns need assistance; 1% need resuscitation • 3 questions: TERM gestation? TONE, BREATHING or CRYING? • If answer to questions is no → NRP - Stabilization (warm 36.5-37.5), dry, clear airway, stim) - Ventilation (if HR<100, not breathing) - Chest compressions (if despite ADEQUATE ventilation HR<60) - Medications (epinephrine), consider hypovolemia/pneumothorax Updates in 2015 à2107 THE MOST IMPORTANT PART OF NRP IS VENTILATION OF THE BABY'S LUNGS • Consider cardiac monitor when PPV begins • Ensure ventilation that moves the chest • Intubate prior to chest compressions • Use cardiac monitoring to assess HR during chest compressions • Team debriefing before and after resuscitation • No routine intubation of non-vigorous post-mec baby before starting NRP steps

Neuromuscular control in categories in HIE

Neuromuscular control Mild Tone: Normal Posture: Mild distal flexion Stretch reflexes: Overactive Segmental myoclonus: Present Moderate Tone: Mild hypotonia Posture: Strong distal flexion Stretch reflexes: Overactive Segmental myoclonus: Present Severe Tone: Flaccid Posture: Decerebrate Stretch reflexes: Absent Segmental myoclonus: Absent CPS Hypothermia in HIE

In updated literature did the benefits of early steroid therapy outweigh the adverse effects?

New systematic reviews/cochrane reviews "EARLY START FOR SET TIME" Increase short term effects: Hyperglycemia/hypertension/GI perforation/bleed Increase long term: Neurodevelopmental impairment Cerebral palsy Big idea: Benefits of early steroid therapy, esp dex, did not outweigh the adverse effects CPS Postnatal corticosteroids to treat/prevent BPD

What does the CPS recommend re: elective C/S? CPS Counselling and management for anticipated extremely preterm birth

No c/s for <24wks unless maternal indications CPS Counselling and management for anticipated extremely preterm birth

Should surfactant be given to newborns with hypoplasia and congenital diaphragmatic hernia?

No conclusions can be made CPS surfactant

Is using donor breast milk cost effective?

No direct canadian studies on economic evaluation Evidence available that it is cost effective because reduces: Length of stay Sepsis NEC Theoretical benefit = promoting breastfeeding awareness in community at larage CPS Milk banking

How long can you use iNO for?

No known time frame Mean 2-4days 90% off within 1 wk If cannot wean after 1wk look into another reason like lung / heart pathology CPS iNO

What are the 5 key changes to the 2011 NRP guidelines?

No need to ask about meconium = gestation, tone, breathing Establish effective ventilation before providing chest compressions= MRSOPA, laryngeal mask, endotracheal airways AIR at 21% for all babies and titrate based on pulse oximetry (preductal). Can use self-inflating resusc bags w/o a resevoir to deliver higher [oxygen] than previous suggested. If persistent bradycardia secondary to cardiac lesion, usual compression to breath ratio of 3:1 can be increased to minimize interruption of chest compressions post-successful resuscitation, baby still likely needs care! Ie. Moderate - severe encephalopathy = needs hypothermia 2013-2014 CPS summaries - Neonatal resuscitation guidelines update: a case-based review

Why is breastfeeding cost-effective?

No need to purchase bottles No need to purchase formula Saves society money on loss of productivity secondary to illness CPS baby friendly

How long should the cooling last for HIE?

No optimal duration known Duration in studies = 48-72h Most lasted 72h The optimal duration of treatment is unknown. The duration of hypothermia in trials lasted between 48 h to 72 h. Most used 72 h of cooling. CPS Hypothermia in HIE

What is an ideal analgesic? (5) CPS Intubation

No respiratory effects Rapid onset Short duration Effective analgesia and sedation Reliable kinetics in all children CPS Intubation

Baby well, mother GBS unknown?

No risk factors: No interventions Risk factors: makes you a mother GBS +Ve Received abx x 4h of penicillin = no intervention No abx = limited diagnostic eval w/ minimum 24h observation ****Big idea! If you have GBS unknown ask immediately about risk factors, if positive you are gbs +Ve! CPS term infants at increased risk for early onset bacterial sepsis

What do updated studies say about late steroids?

No significant increase in: (although many limited studies, not enough power to detect increased rates, open label contamination) Neurosensory disability Cerebral palsy Combined cp and death Decrease in: Failure to extubate within 7 days Mortality by 28d CLD by 36wks Mortality or death at 36wks Number of infants discharged home on oxygen Increase in: Hypertension/hyperglycemia Dex hypertrophic cardiomyopathy and ROP (but no blindness) BIG idea: Both beneficial and harmful effects Reserve for infants that cannot be weaned from mechanical ventilation CPS Postnatal corticosteroids to treat/prevent BPD

Why is neural tube defect primary prevention so important!?

No treatment in utero! Termination may not be acceptable High cost to patient and society CPS Folic Acid and Neural tube defects

What are the complications of premedicating an infant for intubation?

None of the randomized controlled trials, however, have demonstrated serious complications from premedication given before intubation. Infants are now frequently intubated for the purpose of administering surfactant, with a plan to extubate as soon as they have responded adequately. In such a circumstance, one priority for a premedication regimen should be to avoid prolonged adverse respiratory effects. Although fentanyl has a prolonged serum halflife in the newborn, averaging 10 h or more, it causes only short-lasting respiratory depression, and infants can be safely extubated less than 1 h after its administration. One potential advantage of ultrashortacting agents such as remifentanil is a very short serum half-life of only a few minutes and, thus, there is less concern about potential residual effects. CPS Intubation

NL development of retinal vessels

Normally: At wk 16, retinal angiogenesis normally proceeds from the optic disc to the periphery, reaching the outer rim of the retina (ora serrata) nasally at about 36 wk and extending temporally by approximately 40 wk. Neo Notes

What are the 3 approaches to defining normal glucose values?

Normative ranges Breastfed, appropriate for gestational age At birth BG falls from 2/3rds mother to 5th% (1.8mmol/L) at 1h Subsequently rises to >2 for first 72hrs ~15% healthy newborns BG <2 for 72hrs Presence/absence of sequelae At risk term, preterm, SGA Short/long term neurological /neuroimaging changes at <2.6mmolL IDM Long term probs at <1.6mmol/L Don't know if association or cause of problems Prospective trials (none exist) - checking if intervention outweighs short/long term risks CPS Hypoglycemia

What's the big idea with long term disability in the extremely prem? CPS Counselling and management for anticipated extremely preterm birth

Not a lot of difference in short term morbidity and long term disability in infants born at 22-25wks of age Significant increase in short term morbidity and long term disability compared to more mature prems CPS Counselling and management for anticipated extremely preterm birth

How should surfactant be administered?

Not a lot of evidence Only one study compared bolus to small aliquots showing that slow infusion as effective as bolusing No evidence that shifting position of infant is helpful CPS surfactant

Does surfactant replacement therapy help in neonatal pneumonia?

Not adequately studied Shown some benefit ?decr need for ECMO if used in kids with sepsis CPS surfactant

How are milk banks run?

Not for profit Provide milk on cost-recovery basis to NICU Run by lactation consultants day - day but can also have clerical staff/dietary technicians Have a board (of Mds, dieticians, lactation consultants, nursing, infection control) that review the collection policies regularly CPS Milk banking

Can we use midazolam? CPS Intubation

Not recommended! Problems: Non-analgesic sedative! Serious adverse effects seen: Hypotension Decr cardiac output Decr cerebral blood flow velocity Pharmacokinetics long half life >22h Prolonged infusions = neurologic effects It causes hypotension, decreased cardiac output and decreased cerebral blood flow velocity, has variable kinetics with a half-life that can exceed 22 h and, when used as a prolonged infusion, has been associated with an increase in adverse neurological outcomes. Midazolam should not be used for intubation purposes in the newborn CPS Intubation

what are the 5 cases of steroids use for CLD that the CPS does NOT RECOMMEND?

Not recommended: Using steroids in the first 7 days to PREVENT CLD (dex/hydrocort/ inhaled ) High dose dex (>0.5mg/kg/day) to PREVENT/TREAT CLD Low dose dex (0.15-0.2mg/kg/day) to all infants on ventilation >7dol to PREVENT/ TREAT CLD Hydrocortisone to TREAT CLD Inhaled steroids to PREVENT CLD CPS Postnatal corticosteroids to treat/prevent BPD

CPS recommendation: What should you do with ventilation once surfactant is given?

ONE OPTION= Rapid weaning and extubation at 1h to nasal CPAP CPS surfactant

What tests MUST be done before treating for early onset sepsis?

ONLY THE BLOOD CULTURE No other test has a high negative predictive value Proportion of population with negative tests that don't have the disease Not even a cbc! Negative likelihood ratio of normal CBC is 0.7 Normal CBC is only 1.4x more likely if you don't have sepsis CPS term infants at increased risk for early onset bacterial sepsis

Should neonates exposed to SSRIs in utero be treated differently?

Observe 48h if: Late exposure to SSRIs (>20wks) Look for: SSRI neonatal behavioural syndrome Respiratory distress Parents should be counselled before birth CPS SSRI's

When should pasteurized human donor breast milk be prescribed?

Only after written informed consent from parent/guardian CPS Milk banking

NRP - When should cord clamping take place and how long?

Only if not requiring resuscitation At least 1 min 2013-2014 CPS summaries - Neonatal resuscitation guidelines update: a case-based review

Is there a risk of pulmonary hypertension with in utero SSRI exposure?

Only one study has shown this ?rare Only associated with: SSRI's Second half of pregnancy (>20wks) CPS SSRI's

HEMORRHAGIC DISEASE OF THE NEWBORN 2) Early-onset life-threatening vitamin K deficiency-induced bleeding

Onset 0-24 hr occurs if the mother has been treated with drugs (phenobarbital, phenytoin) that interfere with vitamin K function. Site of Hemorrhage Cephalohematoma Subgaleal Intracranial Gastrointestinal Umbilicus Intra-abdominal Etiology/risks Maternal drugs (phenobarbital, phenytoin, warfarin, rifampin, isoniazid) that interfere with vitamin K Inherited coagulopathy Prevention Posible vitamin K at birth or to mother (20 mg) before birth Avoid high-risk medications Neo Notes

HEMORRHAGIC DISEASE OF THE NEWBORN 1) Classic form: due to

Onset 2-7 days transient deficiency of vitamin K-dependent factors due to lack of free vitamin K from the mother and absence of the bacterial intestinal flora normally responsible for the synthesis of vitamin K. It occurs in all newborn infants by 48-72 hr after birth and gradually resolves by 7-10 days of age. It is responsive to and prevented by vitamin K therapy. Site of Hemorrhage Gastrointestinal Ear-nose-throat-mucosal Intracranial Circumcision Cutaneous Gastrointestinal Injection sites Etiology/risks Vitamin K deficiency Breast-feeding Prevention Prevented by parenteral vitamin K at birth. Oral vitamin K regimens require repeated dosing over time Neo Notes

HEMORRHAGIC DISEASE OF THE NEWBORN 3) Late onset

Onset >2 wk (1-6 mo) is associated with vitamin K malabsorption, as noted in neonatal hepatitis or biliary atresia. Site of Hemorrhage Intracranial Gastrointestinal Cutaneous Ear-nose-throat-mucosal Injection sites Thoracic Etiology/risks Cholestasis—malabsorption of vitamin K (biliary atresia, cystic fibrosis, hepatitis) Abetalipoprotein deficiency Idiopathic in Asian breast-fed infants Warfarin ingestion Prevention Prevented by parenteral and high-dose oral vitamin K during periods of malabsorption or cholestasis Neo Notes

NRP - What are the objectives of debriefing? x5

Open ended questions Minimal facilitator statements Away from the location of the event to decrease emotional load As soon as feasible A safe learning environment with no punitive manner 2013-2014 CPS summaries - Neonatal resuscitation guidelines update: a case-based review

______ has been shown to be one of three independent predictors of poor neurodevelopmental outcome at 18 to 24 months.

Oxygen dependency at 36 weeks' PMA CPS Postnatal corticosteroids to treat/prevent BPD

What recommendations and evidence is there for steroids in infants at high risk of CLD?

Oxygen dependency at 36wks of age 1 of 3 independent predictors of poor neurodevelopmental outcome at 18-24mos One regression study = higher CLD rates in control group, less difference in rate of death or CP Big idea: dex may be beneficial to infants at high risk for CLD but not yet been studied in clinical trials CPS Postnatal corticosteroids to treat/prevent BPD

When should an unintubated newborn be treated with surfactant therapy?

Oxygenation index >15 Pneumonia sick CPS surfactant

Diaphragmatic paralysis

PHRENIC NERVE C3-C5 Diaphragmatic paralysis - irregular and labored resps, asymmetric chest motion Confirmed by fluoroscopy or ultrasound Supportive - O2, gavage feeds Spontaneous recovery in 1-3mos Surgical plication if persists To remember: Phret if your phrenic nerve doesn't nerve, C 3-5 ribs different on affected side (not fully expanded) Neo Notes

Does circumcision play a role in penile cancer? Does circumcision affect the risk of cervical carcinoma?

POSSIBLY Higher incidence of penile cancer in uncircumcised males in developed countries Lower incidence of penile cancer in uncircumcised males in CHINA! One of MULTIPLE risk factors! (HPV is big, +/- smoking, more sexual partners, previous STD symps (rash/wart/tear), +/-smoking) Rates of penile cancer is SO LOW in developed countries and not that much higher in non developed countries US 1/100,000 men annually Undeveloped 3-6/100,000 men annually Need more research! NO EVIDENCE YET HPV 16+18 and HSV2 Wives of men who were previously married to women with cervical cancer have a higher risk of cervical cancer themselves Earlier sexual activity, multiple sexual partners are risks CPS circumcision

What is the most concerning possible association of exposure to SSRIs during the second half of pregnancy? (>20wks)

PPHN Absolute rate likely less than 1% CPS SSRI's

What is the purpose of phototherapy?

PREVENT severe hyperbili if have mild/moderate elevated bili Initial therapy for severe hyperbilirubinemia CPS Hyperbili

o infants with birthweight <1,000 g have incidences of severe neurologic impairment:

PROGNOSIS: o infants with birthweight <1,000 g have incidences of severe neurologic impairment: o Grade I or no IVH: 40% o Grade II: 50% o Grade III: 55%, o Grade IV: 70% o PVL, cystic PVL, and progressive hydrocephalus requiring shunt insertion are each independently associated with a poorer prognosis. o 10-15% of LBW neonates with IVH demonstrate hydrocephalus: asymptomatic then may get enlarging head circumference, lethargy, a bulging fontanel or widely split sutures, apnea, and bradycardia Neo Notes

Signs of pain in neonates

Pain o Signs of pain in newborn: increases in heart rate, respiratory rate, blood pressure, and intracranial pressure, changes in skin color, vomiting, gagging, hiccupping, diaphoresis, dilated pupils, and palmar and forehead sweating, o Need to observe facial expression, body movements, crying o Management: sucrose, pacifiers, opioids, massage, kangaroo care Neo Notes

Condition That May Cause Apparent Life-threatening Events or Sudden Death Gastrointestinal

Pancreatitis, diarrhea and/or dehydration, gastroesophageal reflux, volvulus Neo Notes

What are some barriers to shared decision making?

Parents don't even know how to get involved Don't want to carry the burden of these decisions Some want to be involved and have the expertise to do so Some health professionals are uncomfortable engaging families in decision making Would rather just give info than address family values and preferences CPS Counselling and management for anticipated extremely preterm birth

HIGH RISK INFANTS Past Medical History

Past Medical History Genetic ds, DM2, Hypertension, Asymptomatic bacteriuria, Rheumatologic illness (SLE), Medication use Neo Notes

What is kernicterus?

Pathological finding Deep yellow staining brain stem and basal ganglia Neurons and neuronal necrosis CPS Hyperbili

How does circumcision affect UTI rates?

Pathophysiology: Normal flora from mom is okay bc get flora and get her immunoglobulins Bacterial colonization on mucosal surface of prepuce w/ fimbriated bact strains (proteus, e.coli) and non fimbriated Pseudomonas, klebsiella, serratia Definite evidence that UTI rates incr in uncircumcised males ?increased bacteremia ?w/ decr rates of circumcision ?incr rates of UTIs? OR of UTI in uncircumcised male = 12-13 Big question though: Are the UTI rates in newborn males overall high enough to warrant routine circumcision Are there other strategies we can use? 2 mentioned: Colonize w/ non pathogenic bacteria in newborn More rooming in with mom and family to increase colonization between mom and baby Possible benefit to females too! Right now, meta-analysis No routine circumcision for UTI bc of lack of evidence Must warn about this risk when counselling for circumcision CPS circumcision

What are 6 factors that are different about hyperbilirubinemia in late preterm infants? What is the incidence of kernicterus in Western countries? When does the CPS recommend that late prems get their bili's checked? 1) What is considered as extreme hyperbilirubinemia (surrogate for risk of kernicterus)? 2) what is the earliest that a late prem has reached this level? How often should late prems be seen and evaluated for jaundice?

Peaks later (at 7days rather than 5days) Peaks higher (207micromol/L vs. 190 micromol/L) Risk of reaching extreme hyperbilirubinemia (surrogate for kernicterus) doubles ever week shorter than 40wks Lasts longer Even more affected by breast feeding (dehydration, poor wt gain incr enterohepatic circulation) At risk of kernicterus at lower level Needs to be treated/observed at a lower level Risk of developing extreme hyperbilirubinemia not well predicted in first 48h 4/million births Universal screening within 72hours Transcu or serum For late prems want within 48h if low-intermediate zone: Re-evaluate in 24-48h 428micrmol/L ~5days Repeatedly x 10days Feeding, weight gain, no jaundice is established CPS facilitating discharge of late preterms infants

What is the current strategy for GBS early onset sepsis prevention? Why is this not an international strategy?

Penicillin at least 4 hours before delivery Screen with bacterial rectovaginal culture at 35-37wks Treat if positive at delivery Not international Bacterial resistance Not worth it Need 24,000 cultures + 7000 pregnant women in labour to PREVENT one neonatal death CPS term infants at increased risk for early onset bacterial sepsis

What does the provider training workshop consist of?

Performance skills station - earn the different roles Integrated skills station = megacode Simulated scenarios with debriefing 2013-2014 CPS summaries - Neonatal resuscitation guidelines update: a case-based review

Can early cooling be done at community hospitals?

Periphery = only after consulting with trained neonatologist while awaiting transport Turn off overhead warmer gel/ice cool packs Keep careful eye on temperature Tertiary centre ideally: NICU to deal with multi-organ failure and sequelae of cooling Place with u/s, ct, mri Place with neuro eeg/evoked potential recordings Early cooling, before 6 h of age, by trained physicians in community hospitals may be beneficial and should only be initiated following consultation with a level III neonatal intensive care unit. Mild hypothermia can be achieved passively by turning off the overhead warmer, or with ice or cool gel packs while awaiting transport, and can be continued during transport. Careful control of body temperature is important. Hypothermia should be provided in level III neonatal intensive care units where resources are available to treat multiorgan failure that may be associated with HIE, as well as possible complications of the hypothermia treatment such as cardiac arrhythmias and bleeding diathesis. Centres providing hypothermia should have access to ultrasound, computed tomography and magnetic resonance neuroimaging, as well as be able to perform electroencephalograms (EEGs) and neurosensory evoked potential recordings. CPS Hypothermia in HIE

Canadian Pediatric Review 2018 Perinatal-Neonatal Medicine Periventricular leukomalacia

Periventricular leukomalacia • "softening of white matter around ventricles" • PV echoes or cysts seen at 10-21 days • Risk factors: - Twin-twin transfusion - Chorioamnionitis - Asphyxia - Severe lung disease - Hypocarbia - NEC - Postnatal dexamethasone • Prognosis: cerebral palsy

Predisposing factors for PPHN

Persistent Pulmonary Hypertension of the Newborn • Predisposing factors: birth asphyxia, meconium aspiration pneumonia, early-onset sepsis, RDS, hypoglycemia, polycythemia, maternal use of NSAIDS and SSRIs, and pulmonary hypoplasia as a result of diaphragmatic hernia, amniotic fluid leak, oligohydramnios, or pleural effusions. o Often IDIOPATHIC Neo Notes

What does NRP recommend in terms of mandatory individuals at a delivery?

Person 1 = care of newborn, capable of initiating resuscitation, skilled in provision of ppv and chest compressions Person 2 = skilled in more advanced resuscitation procedures needs to be readily available to assist 2013-2014 CPS summaries - Neonatal resuscitation guidelines update: a case-based review

What else may be present on exam in hydrops/hemolytic dz in the newborn

Petechiae, purpura, and thrombocytopenia may also be present in severe cases as a result of decreased platelet production or the presence of concurrent disseminated intravascular coagulation. Jaundice may be absent at birth because of placental clearance of lipid-soluble unconjugated bilirubin, but in severe cases, bilirubin pigments stain the amniotic fluid, cord, and vernix caseosa yellow. Indirect-reacting bilirubin accumulates postnatally and may rapidly reach extremely high levels and present a significant risk of bilirubin encephalopathy Neo Notes

Potential benefits of circumcision

Phimosis treatment Phimosis is defined as a scarring and thickening of the foreskin that prevents retraction back over the glans. Phimosis may occur secondary to recurrent infections, inflammation or lichen sclerosis. Phimosis needs to be differentiated from the normal nonretractile foreskin UTI reduction The preputial sac provides an environment for colonization of the urethra with uropathogenic organisms that can cause UTI in infant boys STI reduction The inner surface of the foreskin is rich in Langerhans and other HIV target cells that are exposed to infection during sexual intercourse, which is speculated to be one mechanism leading to HIV acquisition Cancer reduction Female partners of circumcised men have a reduced cervical cancer risk, with ORs ranging from 0.18 to 1.61 depending on the sexual-behavioural risk level of their partner. Penile cancer is rare in developed countries. Squamous cell carcinoma of the penis occurs almost exclusively in uncircumcised men, with phimosis being the strongest associated risk factor CPS circumcision

How should you treat severe hyperbilirubinemia?

Phototherapy Recheck bilirubin in 2-6h If no response: Prepare for exchange transfusion Supplemental fluids IVIG if isoimmunization CPS Hyperbili

Why do preterm births often limit mom's ability to breastfeed?

Physical = baby transported far away from mom Mom can't produce enough milk Too ill Too stressed CPS Milk banking

What is the difference between physiologic anemia vs. anemia of prematurity vs. non-physiologic anemia?

Physiologic anemia = Term baby Declines 8-12wks Plateau w/o symptoms Increases progressively Anemia of prematurity = Premature baby Declines 4-12wks Symptomatic Non physiologic anemia = (multiple causes, most commonly, iatrogenic phlebotomy) Inappropriately low epo concentrations Low hematocrit Bm hypoplasia reticulocytopenia CPS Red blood cell transfusion in newborn infants

What treatment methods are available for asymptomatic hypoglycemia?

Po: (recheck in 60mins) frequent breastfeeding Supplementation Formula IV dextrose: (recheck in 30mins) Minibolus 2cc/kg of 10% dextrose Not helpful unless incr infusion rate bc glucose v. short acting! Infusion of 80cc/kg/day of 10% (5.5mg/kg/mins) CPS Hypoglycemia

Postdischarge problems of the late preterm

Poor growth/feeding Hyperbilirubinemia Hypoglycemia/temperature control Apnea and sids Sepsis PHATS Hyperbilirubinemia Late preterm infants are over-represented among infants with extreme hyperbilirubinemia. Bilirubin levels in late preterm infants peak later (at seven rather than at five days), stay elevated for longer and reach higher mean values (207 μmol/L versus 190μmol/L) compared with term infants. The late preterm infant is probably at risk of kernicterus at lower levels of bilirubin than the term infant Exclusive breastfeeding increases the risk of extreme hyperbilirubinemia by a factor of approximately six; almost all the infants in the kernicterus registry and with extreme hyperbilirubinemia were breastfed. This very strong association exists particularly in the late preterm infant and is attributed, in part, to dehydration and poor weight gain due to poor feeding. With reduced enteral feeding, the enterohepatic circulation of bilirubin is greatly increased. Late preterm infants who score in or above the low intermediate zone must be re-evaluated within 24 h to 48 h. The median age at which infants with extreme hyperbilirubinemia were reported to have reached or exceeded 428 μmol/L was 4.5 days; these were not necessarily associated with early rapid rises in bilirubin levels Recommendations • Late preterm infants must have an assessment of their serum bilirubin levels within 48 h of birth, and be evaluated using current guidelines for detection, management and prevention of hyperbilirubinemia for late preterm newborn infants. • Late preterm infants should be assessed for feeding, weight gain and jaundice repeatedly in the first 10 days of life until consistent weight gain without jaundice has been established. CPS facilitating discharge of late preterms infants

What 7 aspects of morbidity does surfactant therapy decrease?

Poor oxygenation Incidence of pulmonary air leaks (PTX, PIE - pulmonary interstitial emphysema) Duration of ventilatory support Likelihood of surviving without bronchopulmonary dysplasia (mostly by improving survival overall) Lower cost of intensive care treatment Shorter hospital stays No increase in adverse neurodevelopmental outcome CPS surfactant

Vascular complication of hemolytic dz of the NB

Portal vein thrombosis and portal hypertension may occur in children who have been subjected to exchange transfusion as newborn infants. It is probably associated with prolonged, traumatic, or septic umbilical vein catheterization. Neo Notes

How should you use a CBC when determining who is at risk of early onset sepsis? CPS term infants at increased risk for early onset bacterial sepsis

Positive predictive value (proportion of population with positive test with disease) LOW esp if patient looks WELL! Left shift Wbc count Many dif algorithms (wbc <5 or >30 + anc <1.5 + immature/mature >0.2) but still low positive predictive value HIGHEST positive predictive value = wbc <5 Positive likelihood ratio = 10-20 (10-20x more likely to have this if you have disease) Post test probability of sepsis = 10-20% Only 22-44% w/ sepsis will have low wbc count! If pt looks well but this result, justified to do a full diagnostic work up Big idea: CBC low predictive value esp if kid looks well Only really helpful if wbc <5, seriously consider: Do full diagnostic workup Start abx CPS term infants at increased risk for early onset bacterial sepsis

Can we predict who will develop severe hyperbilirubinemia with timed serum blood concentrations of bilirubin?

Possible Look at total serum bilirubin at a certain time (usually 18h-72h) And Compare to gestational age! CPS Hyperbili

Can we use propofol? CPS Intubation

Possible but need further research re: single dose use One study showing use with morphine, atropine, succ faster intubation, better maintained sats, shorter recoverytime Problems: Need to use it with analgesia Hypotension in older indivs Limited pharmacokinetics show increased individual variability in clearance! Multiple/prolonged doses = problems CPS Intubation

What are possible signs of early onset sepsis? What should be done if pt showing signs? CPS term infants at increased risk for early onset bacterial sepsis

Possible early signs: (TRIP) Tachycardia Respiratory distress Temperature instability Poor perfusion No unique signs to GBS Show ANY signs, immediately treat after a full diagnostic workup Any delay can increase the risk of poor outcome CPS term infants at increased risk for early onset bacterial sepsis

What are 5 must do's with intubation besides premedication?

Pre-oxygenate Observation and monitoring (pulse oximetry) Limit duration of attempts (30s) Confirm tube placement with carbon dioxide detection Performed or supervised by individual with adequate training/experience CPS Intubation

Minor risk factors for Development of Severe Hyperbilirubinemia in Infants > 35 weeks

Predischarge TSB or TcB level in the high intermediate-risk zone Gestational age 37-38 wk Jaundice observed before discharge Previous sibling with jaundice Macrosomic infant of a diabetic mother Maternal age ≥25 yr Male gender Neo Notes

How do steroids affect adrenal insufficient? CPS Postnatal corticosteroids to treat/prevent BPD

Prems as they are may be experiencing relative adrenal insufficiency Putting them no steroids postnatally may further exacerbate this by suppressing the hypothalamic-pituitary-adrenal axis One study said 14d of low dose dex did not cause this, prems were pretty stable in this study Must think about this esp if kids are being exposed to surgery/sepsis/nec CPS Postnatal corticosteroids to treat/prevent BPD

What is hemorrhagic disease of the newborn?

Presents as unexpected bleeding. Most often: GI hemorrhage Intracranial bleed Ecchymosis Classic hemorrhagic disease of newborn First week of life Late hemorrhagic disease of newborn 3-8wks of life Almost exclusively with breast fed infants CPS Vit K

What are indications of breast milk?

Preterm birth Gastrointestinal surgery Malabsoprtion/feeding intolerance immunodeficiency CPS Milk banking

What have SSRIs been linked to in pregnancy?

Preterm birth IUGR Congenital malformations Altered APGAR scores Changes in newborn behaviour CPS SSRI's

CPS recommendation: When should prevention efforts start? What should be involved?

Prevention should target women before and during childbearing years Efforts should be: Family centred Culturally sensitive Comprehensive (draw on all services) CPS Preventing FAS

HIGH RISK INFANTS Previous Pregnancy

Previous Pregnancy Fetal/neonatal demise, Prematurity, IUGR, Congenital malformation, Incompetent cervix, Blood group sensitization, Neonatal thrombocytopenia, Hydrops Neo Notes

How many are estimated to have residual deficits/

Previously 10% Now more like 20-30% CPS Perinatal brachial plexus palsy

What is the recommendation for folic acid supplementation?

Primary prevention = 0.4-0.8mg/day Prevention of recurrence = 0.8-4mg/day (optimal dose not known) CPS Folic Acid and Neural tube defects

What are the three levels of prevention and how do they apply to FAS/FAE?

Primary prevention = action to avert health problem before it appears Educate generalized public about effects of prenatal alcohol exposure Secondary prevention = action to identify those at risk Screening/intervention programs for pregnant women or childbearing potential Tertiary prevention = action to prevent recurrence and lessen impact of disease Diagnosis w/ programs for FAS/FAE kids Treatment for women and partners who already have an affected child and want more CPS Preventing FAS

Primary reflexes in categories in HIE

Primary reflexes Mild Suck: Weak Moro: Strong Oculovestibular: Normal Moderate Suck: Weak or Absent Moro: Weak Oculovestibular: Overactive Severe Suck: Absent Moro: Absent Oculovestibular: Absent CPS Hypothermia in HIE

Is there a difference in primary repair and secondary surgical reconstruction results?? When should a referral be made? And to whom? How should you decide on treatment and prognosis? What does secondary surgical reconstruction entail? Does it help? CPS Perinatal brachial plexus palsy

Primary repair is better Secondary repair will still improve function with severe impairment If incomplete recovery by 1 month on physical exam Send to multidisciplinary team that deals with brachial plexus palsy Team should have: Neurologist +/- physiatrist +/- rehab therapists +/- plastic surgeons NO evidence to say nonsurgical management is any better than primary surgical exploration Based on multidisciplinary team's consideration of: Physical exam Diagnostic imaging Electrodiagnostic tests history Soft tissue and bone reconstruction Inferior to primary intervention Does improve function in kids with significant impairment CPS Perinatal brachial plexus palsy

What do you do about the late prem infant?

Proceed exactly as above Gbs +Ve = proceed Gbs -ve = proceed Gbs unknown = risk factors +ve risk factors everytime bc <36wks Limited diagnostic work up, observation x 24h Only difference is must not discharge until 48h No risk factors: No interventions Risk factors: makes you a mother GBS +Ve Received abx x 4h of penicillin = no intervention No abx = limited diagnostic eval w/ minimum 24h observation Big idea! If you have GBS unknown ask immediately about risk factors, if positive you are gbs +Ve! CPS term infants at increased risk for early onset bacterial sepsis

Prognosis in HIE

Prognosis: • Correlates to timing and severity of insult o Cord or initial blood pH <6.7 have a 90 percent risk of death or severe neuro outcome ♣ Also Apgar 0-3 at 5 mins, high base deficit (>20), decerebrate posture and lack of spontaneous activity o Combined use of early EEG and MRI are useful in predicting outcome in term infants with HIE • Brain death: no universal agreement on definition o coma unresponsive to pain, auditory, or visual stimulation o apnea with Pco2 40 >60 mm Hg without ventilatory support o absent brainstem reflexes (pupil, oculocephalic, oculovestibular, corneal, gag, sucking) o Must occur in the absense of hypothermia, hypotension, and elevated levels of depressant drugs o Absence of cerebral blood flow on scans and of electrical activity on EEG is inconsistently observed in clinically brain-dead infants Neo Notes

Ways to improve communication w/ parents expecting a pre-term/low birth wt child

Providing written information improves parental understanding and recall [66]. Consistency and accuracy in provided information is crucial for expectant parents. Communication between obstetrical and neonatal teams concerning consultations, along with clear documentation of the joint plan in the mother's medical chart, promotes consistency and adherence to the plan. Communicating with parents about periviability, potential outcomes and difficult decisions requires specialized training: trainees must demonstrate expert competence before performing consultations without supervision. Involving trained peer counsellors may provide further support to parents. Shared decision making (SDM) is the best approach for preference-sensitive decisions, which include those made when no clear evidence supports one treatment over another, options have different inherent benefits/risks, or parental values are involved. SDM can mitigate parental grief around end-of-life decisions, enhance knowledge of and satisfaction with care, aid decision making that is consistent with parental values and foster collaboration with medical teams CPS Counselling and management for anticipated extremely preterm birth

What is the first 4 steps when faced with early preterm newborn?

Provision needs to be made for short term observation of late prems for cardioresp/thermal/feeding stability before you send them to low risk/intermediate/high risk nursery CONFIRM age w/ careful documentation Best is early obstetric ultrasound Monitor for: Thermal: wrap and take core temp Wait to bathe after achieve 36.5 Temperature of room, bedding, clothes, exposure Metabolic Glucose at 2h Cardiac Vital signs Feeding check: Early feeding should be attempted Support to mom and baby Evidence that it can happen effectively regardless of method Breastfeeding may be VERY difficult bc mothers are sickest or have many demands at home Need to be supported in decision CPS facilitating discharge of late preterms infants

CPS recommendation: What should be the message to community leaders/members? What role do health professionals play re: women at risk of FAS/FAE? How should alcohol and drug addiction treatment centres help FAS/FAE issue? What should you tell mom's about EtOH exposure?

Prudent to not drink alcohol during pregnancy Identifying at risk women Should use screening methods to find these women at high risk BEFORE pregnancy Should inform women of risk and refer to to services/interventions Make it a priority for pregnant women Incorporate needs of women Transportation daycare If small amounts occasionally effect should be minimal Related to: Amount exposed to Blood type Nutritional health Other lifestyle characteristics of the mother If exposure occurred, stopping anytime will have benefits CPS Preventing FAS

Condition That May Cause Apparent Life-threatening Events or Sudden Death Pulmonary

Pulmonary Pulmonary hypertension, vocal cord paralysis, aspiration, laryngotrachael disease Neo Notes

Only severe complication associated with surfactant treatment

Pulmonary Hemorrhage Neo Notes

1. 7 day old baby presents with cyanosis and tachypnea. O2 sats 80% after given 100% O2. CXR - clear. S1 and single S2. ECG shows right axis deviation. Systolic murmur Grade III/VI over LSB. What is the most likely diagnosis?

Pulmonary atresia/Tetralogy of Fallot TGA- single S2, no murmur ToF - single S2, systolic murmur

When should newborns with pulmonary hemorrhage be given surfactant?

Pulmonary hemorrhage Clinical deterioration CPS surfactant

How frequently should at risk infants be screened?

Q3-6h before feeds CPS Hypoglycemia

Should transfusions be given for severe anemia in high risk term/preterm infants?

Questionable Benefits: Decrease cardiac output Decrease heart rate Decrease lactate Decrease apnea spells Improve growth parameters Problems: First two likely due to incr oxygen carrying capacity but the rest, are they even related to anemia? Is there a clinical benefit or a long term benefit? Improving these things, is it worth the risks of blood transfusions? CPS Red blood cell transfusion in newborn infants

What is the optimal muscle relaxant for intubation? What are the recommended muscle relaxants? CPS Intubation

Rapid onset Short duration Few side effects Rapid acting should be considered Short duration is best Succinylcholine of 2mg/kg is considered best choice Succinylcholine; watch for: Hypertension w/ depolarization Malignant hyperthermia (autosomal dominant disorder of skeletal muscle that remains asymptomatic until meds given) Hyperkalemia Think about keeping it around if using opiate in case of chest freeze Mivacurium Optimal Onset of 8-12mins (tube fixation, weaning, extubation) NOT AVAILABLE IN NORTH AMERICA Rocuronium Rapid onset of action Muscle relaxation duration too long (~1hr) CPS Intubation

CPS recommendation: what rate of male UTIs in infants? What rate of reduction by circumcision?

Rate of UTI = 1-2% Rate of reduction by circumcision = 12fold CPS circumcision

What are some of the obstacles facing parents who lose a baby? CPS perinatal loss

Recognition is hard - no big physical loss, defies nature Reacting is hard - not just to the loss but all the meaning behind it Recollecting/re-experiencing the relationship is hard because a tangible one didn't exist Relinquishing old attachments very hard because mostly identity of baby was in the parent Readjustment to the world is hard - hard to lose being a mother than being mothered Hard to reinvest in a similar relationship unlike a new spouse etc CPS perinatal loss

What do studies say and CPS recommends re: low dose dex?

Recommend = insufficient evidence to demonstrate safety of routine low dose dex Lower doses = DART trial of 0.15mg/kg/day x 10d (total 0.89mg/kg) vs. placebo Before 0.5-1mg/kg/d tapered over varying days Probs: Declining enrollment (no power to look at long term) No significant diff in morbidity or mortality Overall hi dose = better at preventing death and CLD , low dose no difference in neurodevelopmental sequelae! CPS Postnatal corticosteroids to treat/prevent BPD

What is the CPS recommendation of breastfeeding?

Recommend exclusive breastfeeding for 6mos Then continued breastfeeding with appropriate complementary foods up to 2yrs and beyond CPS baby friendly

Recs on frenotomy

Recommendations Based on available evidence, frenotomy cannot be recommended for all infants with ankyloglossia. Clear criteria are needed for the diagnosis of ankyloglossia, along with specific attention to characteristics of infants for whom a frenotomy would be of value to improve feeding. Identifying the specific characteristics of ankyloglossia that may guide the clinician in determining which infants are more likely to benefit from frenotomy is crucial for prognosis. CPS Ankyloglossia

Apnea and SIDS in late preterms Are late prems more likely to have SIDS? IS sids related to apnea of prematurity What are CPS recommendations re: monitoring for apnea and SIDS? Are late prems at risk for apnea of prematurity?

Recommendations • Late preterm infants of 34 weeks' GA may be considered for a period of cardiorespiratory monitoring in a neonatal intensive care unit before transfer to a low-risk nursery. • Infants who manifest apnea require a diagnostic evaluation. If apnea of prematurity is diagnosed, infants should receive cardiorespiratory monitoring in a neonatal intensive care unit until they have achieved eight days of freedom from apnea [43]. • Special care should be exercised to ensure that guidelines for the prevention of SIDS are followed in late preterm infants SIDS Yes more likely Not related to apnea of prematurity Usually happens weeks after term SIDS monintoring Late prems of 34wk may need a period of observation in NICU w/ cardioresp monitoring before transfer to low risk nursery If apnea observed need full workup. If apnea of prematurity diagnosed, need cardioresp monitoring in NICU until 8d of no apnea Especially need to follow strict guidelines to prevent SIDS Apnea of prematurity = decreases with brain maturation More common in late prems then term Rarely cause for readmission More common in </=34wks Some say: Observe 12-24h on cardioresp monitoring If see it exclude other causes May treat with caffeine but do not discharge Want at least 8 days apnea free before discharge (does not mention on or off caffeine) CPS facilitating discharge of late preterms infants

Sepsis in late preterm infants Are late prems more at risk of sepsis and infection? What does the CPS recommend for families of late prem infants to prevent sepsis and infection?

Recommendations • Late preterm infants of less than 36 weeks' GA should be considered at risk of infection and managed according to current guidelines for prevention of group B streptococcal infection [46]. • Exposure to people with active upper respiratory tract infections or other viral infections should be avoided. • The benefits and techniques of hand washing in the prevention of infection should be taught on discharge. Yes! Sepsis - more susceptible at birth and after discharge Most concerning = GBS late and early Likely delivered without status known or for maternal chorioamnionitis Must adhere to guidelines No discharge sooner than 48h No IPA = cbc w/ 24h of q4h temp, rr, hr IPA = watch Viral infections Need to counsel that for 1st year of life, avoid those with viral URTI symps esp in winter Strictly follow sepsis guidelines Avoid active viral infections or URTI people Teach benefits and techniques of hand washing for infection prevention CPS facilitating discharge of late preterms infants

Hypoglycemia and temp control in late preterms Are late prems at risk for hypoglycemia?

Recommendations • Late preterm infants should have been demonstrated to be euglycemic before discharge. • The home environment must be adequately warm to support the infant's thermal environment without recourse to excessive clothing or bedding. Should not be at higher risk if not sga, severely prem, IDM, LGA Same critical level of 2.6mmol/l Should follow documented guidelines CPS facilitating discharge of late preterms infants

Discharge follow up for late preterms What 3 things should be considered (according to the CPS) before discharge?

Recommendations • Mother and infant separation at discharge should be avoided through the provision of flexible accommodation arrangements for parents. • A follow-up appointment within 48 h of discharge should be arranged with a community-based health care provider before the infant is discharged home. • Infant discharge should be flexible and reversible; provisions should be made to incorporate accessible community services into nurseries. Mother and infant separation at discharge should be avoided through the provision of flexible accommodation arrangements for parents Follow up appointment with community based health care provider within 48h of discharge should be arranged BEFORE the infant is discharged home Infant discharge should be flexible and reversible. Provisions should be made to incorporate accessible community services into nurseries CPS facilitating discharge of late preterms infants

Car seat challenge

Recommendations • The Canadian Paediatric Society does not recommend routine ICSC testing before discharge for preterm infants • Parents and other care providers should be counselled regarding safe car seat use and be able to demonstrate appropriate technique and practice - preferably using their own car seat - before their infant is discharged from hospital • Infants should only be placed in a car seat for travel in a moving vehicle and removed promptly once the destination is reached CPS car seat challenge

What does the CPS recommend re: transfer to tertiary care hospital? When should you not transfer to a tertiary care centre? CPS Counselling and management for anticipated extremely preterm birth

Recommended for all women: At least 23wks Threatened preterm birth Active management chosen If: consulted tertiary care centre Informed decision by family Not to resuscitate May agree to not transfer CPS Counselling and management for anticipated extremely preterm birth

Benefits of Kangaroo care

Reduce: Mortality at discharge Severe illness Infections Length of hospital stay Improve: Bonding Breastfeeding Maternal satisfaction Born premature = premature abrupt interruption in neurobehavioural development = disorganization of nervous system Kc helps to reorganize nervous system faster Increase sleep time More quiet sleep Less REM/Arousal sleep Less irritable and fussy More alert and responsive More organized sleep-wake cycle Improve long term development Helps breastfeeding: Increase duration Increase volumes of milk expressed exclusive breastfeeding Incr bf at time of hospital discharge which in turn: Decrease incidence of infections Decrease NEC Improved growth Improved neurodevelopmental outcome Increase parental bonding Those with KC more likely to touch Show positive affect Adapt to infant's signals Better home environment More sensitve to infant Reduces pain for bedside procedures Modulates baby's perception of pain Baby hears maternal heart sounds, rhythmic breathing, wamth which stimulates auditory, tactile, vestibular, thermal sensory systems Cardiorespiratory and temperature stability, sleep organization and duration of quiet sleep, neurodevelopmental outcomes, breastfeeding and modulation of pain responses appear to be improved for preterm infants, decreases the incidence of nosocomial infection, who have received KC during their hospital stay. CPS Kangeroo Care

Why should milk banking be adopted as a cost-effective nutritional source for hospitalized neonates?

Reduces disease incidence/severity Reducing resources during hospitalization CPS Milk banking

What is leukodepletion?

Removal of leukocytes from stored/ circulating blood THEORETICAL BENEFITS decreases risk of TRALI reduction in non-haemolytic transfusion reactions reduction in CMV transmission improved chance of finding an organ transplant match if required reduction in storage lesion effect reduction in graft vs host disease CPS Red blood cell transfusion in newborn infants

What 2 different approaches are there to treating hypoglycemia?

Replacement (po/iv) Mobilize existing stores (steroids/counter-regulatory hormones like glucagon) CPS Hypoglycemia

FACIAL NERVE paralysis due to birth trauma presents with

Results from pressure over CN7 in utero Rarely due to agenesis Involves entire face, incl forehead, eye & mouth (Central palsy- forehead can move b/c contralat innervation) Can have assoc CN6 palsy Must be differentiated from congenital absence of the obicularis oris muscle Better prognosis if nerve injured by pressure than if nerve is torn Eye care Resolves within weeks Neuroplasty if persistent R CN7 palsy - absent nasolabial fold, corner of mouth immobile Neo Notes

The use of specialized digital retinal photography (____), which captures images that can be transmitted electronically, improves availability of screening in the community,[24] a practice that is family-centred and may be cost efficient

RetCam CPS ROP

With the use of _____ to prevent Rh sensitization, nonimmune (nonhemolytic) conditions have become frequent causes of hydrous.

RhoGAM Neo Notes

Systemic manifestations/signs of NEC

SYSTEMIC Lethargy Glucose instability Apnea/respiratory distress Poor perfusion/shock Temperature instability DIC Acidosis (metabolic and/or respiratory) Positive results of blood cultures Neo Notes

What is the big idea behind SSRIs and neonatal behavioural syndrome?

Seen in 10-30% of newborns if SSRIs used late in gestation Generally mild and self limited ~10% can be more severe CPS SSRI's

Condition That May Cause Apparent Life-threatening Events or Sudden Death Infection Trauma Poisoning

Sepsis, meningitis, encephalitis, brain abscess, hepatitis, pyelonephritis, bronchiolitis (RSV), infant botulism, pertussis Child abuse, suffocation, physical trauma, Munchausen syndrome by proxy Boric acid, carbon monoxide, salicylates, barbiturates, ipecac, cocaine, insulin Neo Notes

How do you define fetal alcohol syndrome?

Set of alcohol related disabilities associated with the use of alcohol during pregnancy Minimum criteria: Prenatal and/or postnatal growth restricition CNS involvement - neurological abnormalities, developmental delays, behavioural dysfunction, learning disabilities, other intellectual impairments, skull/brain malformations Characteristic facial features: short palpebral fissures (eye slits), thin upper lip, flattened cheek bones, indistict groove between upper lip and nose CPS Preventing FAS

What are the CPS recommendations for treatment with intensive ptx?

Severe hyperbilirubinemia levels At very high risk of developing severe hyperbili CPS Hyperbili

How should decisions be made? CPS Counselling and management for anticipated extremely preterm birth

Shared decision making between health professionals and families At a minimum, families must be asked how they want to participate. CPS Counselling and management for anticipated extremely preterm birth

What does the CPS recommend re: counselling for parents facing extremely premature birth? What does the CPS recommend re: type of information and how delivered? What does the CPS recommend re: decision making team? What does the CPS recommend re: dealing with management decisions? What does the CPS recommend re: ongoing counselling CPS Counselling and management for anticipated extremely preterm birth

Should be given opportunity to have face-to-face discussion with: Obstetrical care provider Neonatologist Pediatrician If possible, do so more than once Accurate information re: Survival Long term outcome Compassionately and clear manner Supplementary written information is encouraged Must verify parental understanding All decision making between parents and health professionals Informed/shared process Decision aids helpful Clearly written and easily available plan for: Labour Mode of delivery Intensity of neonatal intervention Counselling should be ongoing If decisions about the infant changes, theses must be documented Parents need updated information as the pregnancy continues CPS Counselling and management for anticipated extremely preterm birth

How should the infant be rewarmed?

Should be slow Recommended is 0.5deg q2h Any seizure or worsening encephalopathy seen, recool The speed of rewarming is controversial and varies between increasing rectal temperatures by 0.5°C every hour to every 4 h. The consensus is that rewarming should be slow. Most centres rewarm infants by 0.5°C every 2 h. Worsening of encephalopathy and seizures on rewarming has been reported, in which case infants may require recooling CPS Hypothermia in HIE

Does the CPS think vagolytic agents are necessary?

Should be strongly CONSIDERED ATROPINE 20micrograms/kg effective and safe if given once No minimum dose exists 10micrograms/kg may be enough CPS Intubation

What are 4 associations with brachial plexus palsy?

Shoulder dystocia Large for gestational age Gestational diabetes Instrumental delivery CPS Perinatal brachial plexus palsy

Is hypothermia safe?

Side effects: Bradycardia Mild hypotension Arrhythmias Mild thrombocytopenia Sclerema (hardening of subcu tissue)/edema NO SERIOUS SIDE EFFECTS Mild hypothermia is safe with no serious side effects reported. Mild bradycardia, mild hypotension, arrythmias, mild thrombocytopenia and sclerema/edema have been described CPS Hypothermia in HIE

What are three ethnicities at higher risk for neural tube defects?

Sikh Irish welsh Sikh the WEL(sh) in the neural tube - IRISH it wasn't there CPS Folic Acid and Neural tube defects

What should mothers do about smoking/etOH and breastfeeding?

Smoking = still breastfeed because helps to protect from negative effects of smoking Counsel re: Cessation Avoid smoking in presence of infant Avoid smoking in the home EtOH needs to be watched because flows freely into breastmilk CPS baby friendly

Can you prevent late HDNB with repeated doses of vitamin k?

Sounds like a good idea Breast fed babies should get additional doses of vitamin K Still not as effective as 1 IM dose ?compliance CPS Vit K

sudden generalized jerks lasting 1-2 sec, distinguished from generalized tonic spells by their shorter duration and by the fact that spasms are usually associated with a single, very brief, generalized discharge.

Spasms Neo Notes

What are the effects of premedication on the physiological responses? CPS Intubation

Specifically, bradycardia can be largely prevented by the use of atropine; systemic hypertension can be reduced by adequate analgesia, which also reduces endocrine and endorphin responses; and intracranial hypertension can be avoided by the use of muscle relaxants Morphine appears not to reduce the occurrence of severe hypoxia with bradycardia during intubation, in comparison with placebo, probably because of the delayed onset of action. It is likely that fentanyl is more effective because of the more rapid onset of action. Other newer agents that are even faster acting may also be more effective. CPS Intubation

How should iNO be dosed?

Start 20ppm Watch for response in 30 mins Expected incr of PaO2 >20mmHg No response, raise to 40ppm Have seen anywhere from 1-80ppm Concentration of NO2 in inspired mixture should be as low as possible <0.5ppm CPS iNO

When do symptoms of SSRI neonatal behavioural syndrome start and when do they resolve?

Start within hours Mild usuall Resolve within 2wks CPS SSRI's

Agents Acting on Pregnant Women that act as Teratogens Statins Tetracycline Thalidomide Toluene (solvent abuse) Valproate Vitamin D Warfarin (Coumadin)

Statins: IUGR, limb deficiencies, VACTERL Tetracycline: Retarded skeletal growth, pigmentation of teeth, hypoplasia of enamel, cataract, limb malformations Thalidomide: Phocomelia, deafness, other malformations Toluene (solvent abuse): Craniofacial abnormalities, prematurity, withdrawal symptoms, hypertonia Valproate: CNS, facial and cardiac anomalies, limb defects, impaired neurologic function Vitamin D: Supravalvular aortic stenosis, hypercalcemia Warfarin (Coumadin): Fetal bleeding and death, hypoplastic nasal structures Neo Notes

When should do at risk infants become hypoglycemic?

Studies showing variable times for checking based on when children become hypoglycemic! (remember that all children dip in 1st 1-2hrs to a NATURAL trough!) Little sequelae from asymptomatic hypoglycemia IDM hypoglycemic at ~1h LGA ~3h SGA ~6h CPS Hypoglycemia

How should these babies be delivered? 3 risks w c/s CPS Counselling and management for anticipated extremely preterm birth

Study elective CS vs. expectant management Cs = more maternal complications (hemorrhage and infection) NO benefit to baby Problem w/ CS: <24wks, likely need classical incision Incr hemorrhage Incr fertility issues Risk of rupture in future pregnancies But may be needed for obstetrical issues Must make a decision between mother and obstetrician CPS Counselling and management for anticipated extremely preterm birth

Should supplemental fluids be used to treat mild to moderate hyperbilirubinemia?

Supplemental fluids = IV or oral! Restricted to: infants w/ elevated risk of exchange transfusion who are already on photothearpy CPS Hyperbili

Potential risks of circumcision

Surgical procedures, including circumcision, are painful. Even with procedural analgesia, individuals experience postprocedural pain that must be treated. Newborns who experience procedural pain have altered response to later vaccinations, with demonstrated higher pain scores. Acute complications of neonatal circumcision include minor bleeding, local infection and an unsatisfactory cosmetic result. Severe complications, such as partial amputation of the penis and death from hemorrhage or sepsis, are rare occurrences The most common late complication of circumcision is meatal stenosis (2% to 10%), which may require surgical dilation. This condition can be prevented almost completely by applying petroleum jelly to the glans for up to six months following circumcision. Partial re-adherence of the penile skin to the glans is not uncommon. Such adhesions often resolve spontaneously by puberty CPS circumcision

Breast feeding and infections

TB - tx for 2 wks and give infant ppx Brucellosis Human T-cell lymphotrophic virus Galactosemia chemotherapy radiation illicit drugs/alcohol Mastitis and breast abscesses Continue breastfeeding unless there is obvious pus, in which case pump milk and discard from the infected breast and continue to breastfeed from the other breast Tuberculosis(TB) Main route of transmission is airborne, not via organisms in milk. With active untreated TB, delay direct breastfeeding until mother has received 2 weeks of appropriate anti-TB therapy; provide TB prophylaxis for infant.* Infant can be fed expressed breast milk during the 2-week period. Brucellosis Discontinue breastfeeding with untreated maternal brucellosis; infections might be passed through breast milk Malaria Continue breastfeeding While the antimalarials chloroquine, hydroxychloroquine and quinine are found in variable quantities in breast milk, all three are regarded as compatible with breastfeeding unless the infant has glucose-6-phosphate dehydrogenase (G6PD) deficiency in which case withdrawal of quinine is advised.[12] Similarly, primaquine should not be used unless both mother and infant have normal G6PD levels. Cytomegalovirus (CMV) Continue breastfeeding with latent or active maternal CMV infection Hepatitis Hepatitis A virus Continue breastfeeding; immunoglobulin prophylaxis for the infant. Hepatitis B virus Continue breastfeeding; routine prevention of infant HBV infection with HBIG at birth; immunization with HBV vaccine Hepatitis C virus Continue breastfeeding; immunization with HBV vaccine Herpes simplex virus HSV-1, HSV-2 Continue breastfeeding. Practice meticulous hand hygiene. Cover oral labial lesions with a mask. If there are lesions on the breast/ HSV mastitis, verify that it is HSV not varicella-zoster virus. Interrupt direct breastfeeding until lesions are crusted over. Use expressed breast milk Chickenpox, shingles Varicella-zoster virus (VZV) Continue breastfeeding. For perinatal VZV, give VZIG; for postpartum, consider VZIG Enterovirus Continue breastfeeding. Practice meticulous hand hygiene HIV Breastfeeding and expressed breast milk both contraindicated. See text for details. Human T-cell lymphotrophic virus type I or II Breastfeeding and expressed breast milk both contraindicated Parvovirus Continue breastfeeding West Nile virus Continue breastfeeding Even maternal therapy with tetracycline, aminoglycosides or quinolones is not an indication to withhold breastfeeding. CPS contraindications to breastfeeding

Recomendations for transport of critically ill newborns

Teams used to transport newborns to tertiary neonatal-perinatal centres should be specifically dedicated for transport, based at a tertiary hospital and have expertise in the care of newborns. The inclusion of maternal and paediatric populations should depend on patient volumes, resources and needs. • A collaborative practice model, with one RN working with either another RN, an RT or an EMT/paramedic with expertise in neonatal or paediatric transport, is the optimal neonatal transport team. Medical 'on line' control is best provided by an experienced neonatologist with expertise in transport medicine. • Specific training in airway management and other procedural skills using effective teaching methods and validated assessment tools is recommended. Refresher skills courses and ongoing clinical assessment help to ensure maintenance of competencies. • Transport teams must have the equipment and supplies necessary to provide intensive care and meet all land and air ambulance specifications for safety. Transport safety measures for thermal regulation and noise reduction must be employed. Dedicated team vehicles permit the storage of equipment, supplies and the hydraulic lifts or stretchers needed for team and patient safety. • Policies and procedures must be in place to guide team performance and ensure optimal patient outcomes. A single access point, with provincial/territorial coordination and integration of transport modes, must ensure the immediate availability of medical advice, the rapid dispatch of a transport team, and the identification of a receiving hospital. Communication with families, referral and receiving staff from first contact to admission, is essential. • Transport teams must have a database that captures both the severity of illness, and clinical and utilization metrics, including transport times, which can be used for benchmarking, quality improvement and research. CPS interfacility transport of critically ill newborns

When should exchange transfusion be used?

Term infant w/ no other risk factors: 375-425 Follow normogram Infant with any sign of acute encephalopathy Hypotonia, poor suck, lethargic Hypertonic, shrill cry, retrocollis/opisthotonus Coma/seizures CPS Hyperbili

At what number does acute encephalopathy become more prominent?

Term infants, unlikely until more than: 340micromol/l Very rare until above: 425micromol/l In US kernicterus: ¾ = >515 2/3 = >600 But even >500, some may escape encephalopathy, reasons unknown? CPS Hyperbili

_______ ointments have been considered to be acceptable alternatives for preventing gonococcal ophthalmia. However, N gonorrhoeae strains isolated in Canada in 2012 showed considerable resistance to these agents, with tetracycline at ____ and erythromycin at ____.

Tetracycline and erythromycin 30% 23% In Canada, erythromycin has been the only antibiotic eye ointment available for use in neonates since tetracycline ophthalmic ointment became unavailable. Povidone-iodine has been considered for prophylaxis, but this agent may not be effective and has been associated with a 5% rate of chemical conjunctivitis CPS Preventing ophthalmia neonatorum

Clinical Manifestations of hirschsprungs

The clinical symptoms usually begin at birth with the delayed passage of meconium. Some infants pass meconium normally but subsequently present with a history of chronic constipation. Failure to thrive, with hypoproteinemia from a protein-losing enteropathy is less common. Breast-fed infants may not suffer as severe a disease as formula-fed infants. Failure to pass stool leads to dilatation of the proximal bowel and abdominal distention. As the bowel dilates, intraluminal pressure increases, resulting in decreased blood flow and deterioration of the mucosal barrier. Stasis allows proliferation of bacteria, which can lead to enterocolitis with associated sepsis and bowel obstruction. Hirschsprung disease in older patients must be distinguished from other causes of abdominal distention and chronic constipation. The history often reveals increasing difficulty with the passage of stools, starting in the 1st few weeks of life. The stools, when passed, may consist of small pellets, be ribbon-like, or have a fluid consistency; the large stools and fecal soiling of patients with functional constipation are absent Neo Notes

Phimosis tx

The first-line medical treatment of phimosis involves applying a topical steroid twice a day to the foreskin, accompanied by gentle traction. This therapy serves to thin the tissue and release adhesions, allowing the foreskin to become retractable in 80% of treated cases, thus usually avoiding the need for circumcision. Topical steroid treatment is also useful to hasten foreskin retraction in boys with nonretractile foreskins. A number of steroid preparations have been used, including betamethasone 0.05% to 0.1%, triamcinolone 0.1% and mometasone furoate 0.1%. CPS circumcision

posthitis

The foreskin can become inflamed or infected (posthitis), often in association with the glans (balanoposthitis) in 1% to 4% of uncircumcised boys CPS circumcision

Discharge readiness: Infant competencies (physiological maturity)

The four most important physiological competencies are: • Thermoregulation • Control of breathing • Respiratory stability • Feeding skills and weight gain CPS facilitating discharge of preterms infants

When should you screen babies for ROP? (it's not a benign exam)

The more prem the baby, the longer ROP takes to develop </=27wks -> 31wks >27wks -> 4wks chronological age CPS ROP

Milk banking recs

The preferred nutrition for the newborn is his/her own mother's milk. When this is not available or is limited, pasteurized human donor breast milk is a recommended alternative for hospitalized neonates. • The use of pasteurized human donor breast milk should be prioritized to compromised preterm infants and selected ill term newborns. • Pasteurized human donor breast milk should only be prescribed following written informed consent from a parent or guardian. • Education of parents about the benefits of human breast milk or pasteurized human donor breast milk is essential to parental choice and informed decision making in prescribing an optimal feeding plan for hospitalized neonates. • Milk banking should be adopted as a cost-effective nutritional source for hospitalized neonates because it reduces disease incidence and severity, thus reducing resource use during the hospitalization. • Recognized functions of the human milk bank should include the promotion of breastfeeding and ongoing human milk research. • There is a need for prospective studies to evaluate the benefits of banked human breast milk in preterm infants in the NICU. • The Canadian Paediatric Society does not endorse the sharing of unprocessed human milk. CPS milk banking

What is the target temperature to be reached in cooling for HIE?

The range of rectal or esophageal temperature in the trials varied between 32.5°C to 35°C using either method of cooling. CPS: The optimal rectal or esophageal temperature appears to be 34±0.5°C CPS Hypothermia in HIE

swallowed blood syndrome presentation and dx

The swallowed blood syndrome, in which blood or bloody stools are passed, usually on the 2nd or 3rd day of life, may be confused with hemorrhage from the gastrointestinal tract. The blood may be swallowed during delivery or from a fissure in the mother's nipple. Differentiation from gastrointestinal hemorrhage is based on the Apt test: the infant's blood contains mostly fetal hemoglobin, whereas swallowed blood from a maternal source contains adult hemoglobin. A yellow-brown color indicates that the blood is maternal in origin; a persistent pink indicates that it is from the infant. Neo Notes

NEC triad

The triad of intestinal ischemia (injury), enteral nutrition (metabolic substrate), and pathogenic organisms has classically been linked to NEC. The greatest risk factor for NEC is prematurity Many factors may contribute to the development of a necrotic segment of intestine, gas accumulation in the submucosa of the bowel wall (pneumatosis intestinalis), and progression of the necrosis to perforation, peritonitis, sepsis, and death NEC rarely occurs before the initiation of enteral feeding and is much less common in infants fed human milk. Aggressive enteral feeding may be a risk factor. Neo Notes

What about early inhaled steroids? What about late inhaled steroids? What is CPS opinion of inhaled steroids?

Theoretically better bc less systemic absorption 2007 meta-analysis looking at many dif steroids x 2-4wks Compared to placebo no decrease in: Cld Death Cld and death Side effects (hyperglycemia, hypertension, infection) Ie. No help but no harm No difference between inhaled budesonide/ beclomethasone vs. systemic dex re: Cld Death Cld or death combo Duration of mechanical ventilation Side effects Even for studies where inhaled steroids used only for oxygen needs (no ventilator), no difference in long term respiratory outcomes ?all that is shown is that systemic steroids work faster but inhaled steroids better growth Big idea: Inhaled steroids may help acute pulmonary mechanics, no evidence that it consistently helps with inflammatory mediators No evidence comparing it to placebo No long term evidence from kids who have been treated Little evidence to support routine use to prevent CLD Does not appear to offer significant benefits over systemic steroids for treatment of infants who are ventilator dependent CPS Postnatal corticosteroids to treat/prevent BPD

Can you use cord blood hemoglobin/HCT to predict severe hyperbilirubinemia?

Theoretically makes sense. Bilirubin comes from blood, more blood, more bili DOES NOT WORK CPS Hyperbili

What is a normal neonatal glucose value?

There isn't one! Depends on gestational age, size, clinical condition Bid idea: 1.8 may be normal in the first few hours of age BUT in high risk infants, <2.6mmol/L can lead to short and long term adverse outcomes CPS Hypoglycemia

Main concerns of later preterm?

Thermal regulation Feeding Late severe hyperbilirubinemia CPS facilitating discharge of late preterms infants

Which babies are most affected? CPS SSRI's

Those exposed later in gestation Late in gestation carried overall risk ratio of 3 Those on fluoxetine/paroxetine CPS SSRI's

CPS recommendation: when should women get antenatal steroids?

Threatened delivery <34wks Regardless of availability of postnatal surfactant therapy CPS surfactant

>32 WK - Time point of assessment for Definition of Bronchopulmonary Dysplasia: Diagnostic Criteria - Mild vs Moderate vs Severe

Time Point of assessment: >28 days but <56 days postnatal age or discharge home. Treatment with 21% oxygen for at least 28 days plus Mild: Breathing room air by 56 days' postnatal age or discharge, whichever comes first Moderate: Need[*] for <30% oxygen at 56 days' postnatal age or discharge, whichever comes first Severe: Need[*] for Υ⇑30% oxygen and/or positive pressure (PPV or NCPAP) at 56 days' postnatal age or discharge, whichever comes first Neo Notes

♣persistent posturing of a limb or posturing of trunk or neck in an asymmetric way often with persistent horizontal eye deviation ♣more common, bilateral tonic limb extension or tonic flexion of upper extremities often associated with tonic extension of lower extremities

Tonic Seizures Focal and General Neo Notes

What is severe hyperbilirubinemia? What is critical hyperbilirubinemia?

Total serum bilirubin concentration >340micromol/L in first 28d of life >425micromol/L during the first 28days of life CPS Hyperbili

Transfusion goal calculations

Transfusion volumes of 20 mL/kg given at 7 mL/kg/h can be tolerated by newborns but the evaluation of this practice is limited; a conservative maximum would be 5 mL/kg/h. An alternative approach is to calculate the volume of blood transfused by hemoglobin target selection, which has conventionally been 150 g/L in the early newborn period. Attempting to reach this target later in life (when the transfusion threshold has fallen to 75 g/L) results in unacceptably large transfusion volumes; a lower target (eg, 130 g/L) may, therefore, be selected. Smaller and more frequent blood transfusions have become more acceptable as the risks of donor exposure have declined. Given the current uncertainty regarding the neurodevelopmental effects of anemia (see Anemia, below), it may be preferable to keep hemoglobin levels relatively constant. CPS Red blood cell transfusion in newborn infants

bleeding from the fetal into the maternal circulation

Transplacental hemorrhage with bleeding from the fetal into the maternal circulation has been reported in 5-15% of pregnancies. The diagnosis is confirmed by demonstrating significant amounts of fetal hemoglobin and red blood cells in maternal blood on the day of delivery by the Kleihauer-Betke test. Acute blood loss usually results in severe distress at birth, initially with a normal hemoglobin level, no hepatosplenomegaly, and early onset of shock. In contrast, chronic blood loss in utero produces marked pallor, less distress, a low hemoglobin level with microcytic indices, and, if severe, heart failure. Neo Notes

How do you treat an infant with mom who had chorio?

Treat as a risk factor And even if mom is GBS negative, perform limited evaluation (cbc + diff) and monitorx 24hrs CPS term infants at increased risk for early onset bacterial sepsis

Tx of HIE

Treatment • Systemic or selective cerebral hypothermia to 33.5ºC within 6 hr of birth x 72h o Reduces mortality or major neurodevelopmental impairment (term) o Decreases rate of apoptosis and suppresses production neurotoxic mediators • Supportive care Neo Notes

Tx of hemolytic dz of the NB after delivery

Treatment of a Liveborn Infant: Fresh, low-titer, group O, leukoreduced, and irradiated Rh- blood cross matched against maternal serum should be immediately available. If clinical signs of severe hemolytic anemia (pallor, hepatosplenomegaly, edema, petechiae, ascites) are evident at birth, immediate resuscitation and supportive therapy before proceeding with exchange transfusion may save some severely affected infants. Neo Notes

Screens in 2nd TM for trisomy 21

Triple Test hCG, AFP, uE3 2nd T Quadruple Test hCG, AFP, uE3, inhibin A 2nd T 2nd trim x2 = quad test = Inhibit (inhibin A) HAFp (hCG, AFp) of Ur (uE3) concern about Trisomy 21 in the 2nd trimester Neo Notes

MATERNAL SERUM α-FETOPROTEIN - decreased in

Trisomies, aneuploidy Neo notes

What are the different ways of determining screening bilirubin concentrations?

Tsb Venipuncture = capillary (cap more in Canada) Transcutaneous CPS Hyperbili

Canadian Pediatric Review 2018 Perinatal-Neonatal Medicine Endotracheal intubation

Tube size: >35 weeks 3.5-4.0; Preterm > 1kg 3.0; <1kg 2.5 Depth: weight + 6cm (oral) Confirm placement: • Visualize through cords • Chest movement • Bilateral air entry • Heart rate improves! End-tidal CO2 colorimeter Recommended by NRP Ventilation of the lungs (whether by mask or ETT) is the single most important step in the resuscitation of the compromised newborn

MATERNAL SERUM α-FETOPROTEIN Elevated in

Twins, neural tube defects (anencephaly, spina bifida), intestinal atresia, hepatitis, nephrosis, fetal demise, incorrect gestational age Down in downs and up in open (NTDs) Neo Notes

Tx of twin-to-twin transfusion

Tx = maternal digoxin, aggressive amnioreduction for polyhydramnios, selective twin termination, or ablation of the anastomosis. Twin-twin transfusion syndrome, artery from one twin acutely or chronically delivers blood that is drained into the vein of the other. The latter becomes plethoric and large, and the former is anemic and small. Neo Notes

The terms 'type 1' and 'type 2' ROP are now used to differentiate eyes with significant changes of ROP that require treatment type ___ from eyes with significant changes that do not require treatment but must be carefully monitored, type ____

Type 1 requires tx, Type 2 does not require tx but just monitoring 1 = 1 chance to save the eyes 2 = 2 good 2 tx CPS ROP

What are the common causes of unconjugated hyperbilirubinemia?

Unconjugated = inc'd production/dec'd processing Conjugated = dec'd excretion (cholestasis) Breast milk, breast feeding Blood: DAT +ve (ABO, RH, abs) DAT -ve (g6pd/PK, sickle cell, spherocytes) Too much blood (cephalohematoma, bruising) Not enough albumin Sepsis Torch Metabolic diseases Physiologic/premature/crigler-nijjar/Gilberts CAUSES of UNCONJUGATED (INDIRECT) HYPERBILIRUBINEMIA: 1. Increased lysis of RBCs (i.e., increased hemoglobin release) •Isoimmunization (blood group incompatibility: Rh, ABO and minor blood groups) •RBC enzyme defects (e.g., G6PD deficiency, pyruvate kinase deficiency) •RBC structural abnormalities (hereditary spherocytosis, elliptocytosis) •Infection (sepsis, urinary tract infections) •Sequestered blood (e.g., cephalohematoma, bruising, intracranial hemorrhage) •Polycythemia •Shortened life span of fetal RBCs (80 vs. 120 d) 2. Decreased hepatic uptake and conjugation of bilirubin •Immature glucuronyl transferase activity in all newborns: term infants have 1% of adult activity, preterm infants have 0.1%. •Gilbert Syndrome •Crigler Najjar Syndrome (Non-hemolytic Unconjugated Hyperbilirubinemia): inherited conjugation defect (very rare) •Pyloric stenosis (mechanism is unknown) •Hypothyroidism •Infants of Diabetic Mothers (polycythemia is also common) •Breastmilk Jaundice (pregnanediol inhibits glucuronyl transferase activity) 3. Increased enterohepatic reabsorption •Breast feeding jaundice (due to dehydration from inadequate milk supply) •Bowel obstruction •No enteric feedings EVALUATION of JAUNDICE (UNCONJUGATED) 1. Initial evaluation: •Total and direct bilirubin •Blood type and Rh (infant & mother) •Hematocrit •Direct Antiglobulin (Coombs) Test on infant 2. Later evaluation (as indicated): •RBC smear, reticulocyte count (if evidence or suspicion of hemolytic disease) •Blood culture, urinalysis, urine culture •Thyroid function tests, G6PD assay, Hgb electrophoresis vs CONJUGATED (DIRECT) HYPERBILIRUBINEMIA (CHOLESTASIS): Clinically, jaundice is green compared to jaundice due to unconjugated hyperbilirubinemia (yellow). 1. Hepatocellular diseases: A. Hepatitis: •Neonatal idiopathic hepatitis •Viral (Hepatitis B, C, TORCH infections) •Bacterial (E. coli, urinary tract infections) B. Total parenteral nutrition C. Hepatic ischemia (post-ischemic damage) D. Erythroblastosis fetalis (late, "Inspissated Bile Syndrome") E. Metabolic disorders (partial list): •Alpha-1 antitrypsin deficiency •Galactosemia, tyrosinemia, fructosemia •Glycogen storage disorders •Cerebrohepatorenal disease (Zellweger) •Cystic fibrosis •Hypopituitarism 2. Biliary tree abnormalities: A. Extrahepatic biliary atresia: In first 2 weeks,, unconjugated bilirubin predominates; elevated conjugated bilirubin is late. B. Paucity of bile ducts (Alagille's vs. non-syndromic) C. Choledochal cyst D. Bile plug syndrome EVALUATION and MANAGENMENT of CHOLESTASIS: 1. Initial evaluation: •Total and direct bilirubin •AST, ALT, GGT, urine reducing substances •Hepatic ultrasound 2. Later evaluation (as indicated): •Hepatitis B and C serology •α1-antitrypsin deficiency studies •Very long chain fatty acids •Brain sonogram •HIDA scan •Cholangiogram 3. Management: •Conjugated bilirubin is not toxic. •Management is treatment of cause. •Phototherapy will cause "bronzing" with conjugated hyperbilirubinemia. CPS Hyperbili

NRP - When should you consider discontinuing resuscitative efforts?

Undetectable HR x 10mins 2013-2014 CPS summaries - Neonatal resuscitation guidelines update: a case-based review

Who should have their bilirubin screened?

Universal screening No cost benefit analysis Idea is to PREDICT who will have a peak hyperbili that is SEVERE. Previously only: Clinical jaundice kids in first 4 dol Kids not jaundiced but w/ risk factors This was a problem: Clinical jaundice very difficult to judge Clinical jaundice peaks at 3-5days of age (already discharged) CPS Hyperbili

What do you need to do when using ptx? 3 CPS Hyperbili

Use eye patches to protect developing retina Animal studies = risk Reassure parents Most think that ptx means serious disease Reassure that with proper intervention/follow up, can prevent consequences of hyperbili CPS Hyperbili

What are the characteristics of an acceptable protocol for premedication?

Vagolytic Glycopyrrolate and atropine are both effective and have not been directly compared. It should be noted that there is no minimum total dose - 10 μg/kg to 20 μg/kg is effective and safe. Analgesia The optimal analgesic for intubation would have a very rapid onset, no effect on respiratory mechanics, a short duration of action with good sedation, and reliable kinetics. None of the currently available agents fit this profile. Fentanyl, the most widely used analgesic agent, blunts physiological disturbance during intubation in adults and older children, and has a good safety profile. According to a number of studies [1][4], morphine is the most commonly used drug for intubation; however, it does not improve physiological stability during intubation when used alone. This may well be because at least 10 min are required for good analgesia after IV administration, suggesting it may not be the optimal drug for analgesia before intubation. Muscle relaxation The optimal muscle relaxant for intubation would have a rapid onset, short duration of action and few side effects. Succinylcholine has been most widely used, but has rare serious side effects and causes an increase in blood pressure after use, simultaneously with the depolarization. Hyperkalemia may occur, but major elevations are uncommon and usually seen in association with significant tissue injury. Succinylcholine may trigger malignant hyperthermia, a rare autosomal dominant disorder of skeletal muscle that remains asymptomatic unless triggering substances are given. Succinylcholine should not be used in infants with hyperkalemia or a family history of malignant hyperthermia. Of the nondepolarizing agents, mivacurium most closely fits the ideal profile. The duration of action of approximately 8 min to 12 min is reasonable for allowing tube fixation after intubation, and will allow rapid weaning and extubation if the infant was intubated for a brief procedure such as surfactant administration. However, mivacurium is not currently available in North America and alternative agents (eg, cisatracurium) should be investigated. Rocuronium has been investigated and has the advantage of a rapid onset of action, but for most purposes, the duration of muscle relaxation (of up to 1 h) is too long and would not be appropriate. CPS Intubation

What two antiepileptics are associated with neural tube defects?

Valproic acid carbamazepeine VAL drove a CAR through the neural tube CPS Folic Acid and Neural tube defects

What are 4 reasons for emergent blood loss during delivery? When should you use blood to resuscitate?

Vasa previa Placental abruption Cord accidents Maternal-fetal transfusion Use it if hct significantly decreased CPS Red blood cell transfusion in newborn infants

The volume to be exchanged is calculated from the following formula:

Volume of exchange (mL) = Blood volume × (Observed - Desired hematocrit)/Observed hematocrit Neo Notes

Congenital adrenal hyperplasia

Vomiting, diarrhea, dehydration, hyperkalemia, hyponatremia, shock, ambigious genitalia, hypertension Neo Notes

What is the golden minute?

Warm Clear airway prn Dry Stimulate HR Breathing PPV start ALL WITHIN ONE MINUTE 2013-2014 CPS summaries - Neonatal resuscitation guidelines update: a case-based review

1. Newborn term, AGA baby, cried at birth, Apgars 9 and 9. Few hours later found to be in respiratory distress. RR 80, HR high. Cap refill 4-5 seconds, BP 48/32. Hyperinflated chest with minimal indrawing. Cannot hear breath sounds on left, cannot hear heart sounds. Baby is turning cyanotic. What investigation do you do? What is your possible diagnosis? Baby's heart rate is now 80 and is more cyanotic. What one investigation do you want to do (1)

What investigation do you do? (1) ?CXR, needle decompression, What is your possible diagnosis? (1) tension pneumothorax, Baby's heart rate is now 80 and is more cyanotic. What one investigation do you want to do (1) needle decompression

What is loss? What is grief? What is mourning? What is bereavement?

What is loss? Two categories: Physical symbolic What is grief? Psychological/behavioural/social/physical reaction to loss What is mourning? Cultural/public display of grief through behaviour What is bereavement? Entire process precipitated by loss of a loved one through death CPS perinatal loss

1. Baby born at home at 38 wks by midwife. Now presents at 7 days with melena. Hb 70, MCV 112, plts normal.

What is the most likely diagnosis? Hemorrhagic Disease of Newborn

CPS circumcision

While there may be a benefit for some boys in high-risk populations and circumstances where the procedure could be considered for disease reduction or treatment, the Canadian Paediatric Society does not recommend the routine circumcision of every newborn male. CPS circumcision

How should you handle withdrawal of aggressive support? CPS perinatal loss

Whole NICU team with parents need to discuss the issue Take away the burden of the decision Once decided Pre warn parents about exactly what will happen and changes in baby to expect Remove as much of the tubes as possible Make sure baby and parents can spend quiet time and optimized time together privately Find the right spiritual support that family wants CPS perinatal loss

When should pts with isoimmunization be rechecked for hb?

Will experience severe anemia bc abs persist Check in: 2wks if low at discharge 4wks if normal at discharge CPS Hyperbili

Canadian Pediatric Review 2018 Perinatal-Neonatal Medicine Non-initiation of resuscitation

With discussion with parents, appropriate for • Limits of viability - (ex. GA <22 weeks) • Severe congenital anomalies - (ex. anencephaly, bilateral renal agenesis)

When was acute bilirubin encephalopathy first seen?

With rhesus disease Now largely preventable Still happens in healthy term infants without risk factors Need to: Clinical assessment Proper interpretation of TSB Proper and fast treatment CPS Hyperbili

How quickly should you see changes after surfactant? What do you need to do about them?

Within minutes: Increase in static compliance (bc incr in FRC) No change to dynamic compliance Improved gas exchange Need to quickly: Wean pressures (or else over distension) Can also get hyperventilation so watch carefully CPS surfactant

29. 3 day old infant cyanosis with crying, investigation: a) ECG b) CXR c) ABG d) BCx e) Echo

Would do CXR, ECG and Echo, must be more to question to allow you to pick the difference C (first thing) 3 most common causes of central cyanosis in newborns (admitted to a NICU): • Respiratory Distress Syndrome • Sepsis • Cyanotic heart disease Cyanosis due to respiratory disease may be indistinguishable from cyanosis due to cardiac causes • However, cyanosis that occurs when crying (increased R L shunting) suggests cardiac cause (similar to TET spells); cyanosis due to resp causes often improves with crying Realistically should probably do all of these... but would probably start with a CXR?

1. Neonate with glucose of 0.9. Not interested in feeding. You decide to treat & give a 2cc/kg bolus of D10W. Write your orders for fluids to run next (be specific.)

Write your orders for fluids to run next (be specific.) - 80mL/kg/day of D10% (GIR 5.5mg/kg/min); recheck BG in 30 minutes. - Direct from CPS statement - Target BG 2.6mmol/L or higher, start infusion at 80mL/kg/day of D10, then increase as needed (D% or volume) - May continue BF as volume of colostrum is small (if IV & po intake >100mL/kg/day, must observe for dilutional hyponatremia)

What are the ten steps to successful breastfeeding by WHO and UNICEF made in 1989?

Written breastfeeding policy that's routinely communicated to all health care staff Train all health care staff in skills necessary to implement the breastfeeding policy Inform all pregnant women about the benefits and management of breastfeeding Help mothers initiate breastfeeding within a half hour after birth Show mothers how to breastfeed and maintain lactation even when they are separated from their infant Give newborns no food or drink other than breastmilk, unless medically indicated Practice rooming-in - allow mothers and infants to remain together 24 a day Encourage breastfeeding on demand Give no artificial teats/pacifiers (also called dummies/soothers) to breasfeeding infants Foster the establishment of breastfeeding support groups and refer mothers to them at discharge from the hospital or clinic CPS baby friendly

Because surfactant therapy is available, should we still need antenatal steroids?

YES Any mother at risk of delivering a baby <34wks GA should be given 1 course of antenatal steroids regardless of availability of postnatal surfactant therapy STILL helps! Increases survival without ventilation, without intraventricular hemorrhage Decreases disease severity CPS surfactant

Are SSRIs safe in breast feeding?

YES Excreted into breastmilk No evidence of adverse effects on infant CPS SSRI's

Should you give surfactant before transport? If you want to give surfactant (before transport or anywhere really), what must you have!? At what age should you prophylactically get surfactant right away in a peripheral hospital? After surfactant in a peripheral hospital who should be sent to a tertiary hospital?

YES Intubated infants With RDS Receive exogenous surfactant therapy before transport Data shows that it's safe. No good data to say that it's better. Maybe better bc less air leak like ptx that is very difficult to manage on route Continuous presence (on-site availablity) of personnel trained/competent/licensed to deal with acute complications of assisted ventilation and surfactant therapy. 29wks Consider immediate intubation, surfactant administration after stabilization As long as continuous onsite trained personnel available. Should be everyone even though residual lung disease may be low! Likely because they will have immaturity of other organ functions requiring close monitoring and care CPS surfactant

Is there a difference in UTI rates among adults who are circumcised?

YES! Less in circumcised males 30% of UTI males uncircumcised 12% of UTI males circumcised CPS circumcision

If a pregnant woman was GBS negative at screening OR if she received full intrapartum abx, could her infant still have early onset sepsis? CPS term infants at increased risk for early onset bacterial sepsis

YES: NON GBS SEPSIS: Intrapartum abx DOES NOT decrease frequency of sepsis from other organisms Became positive after 35-37wks Very rarely still gbs sepsis despite appropriate abx CPS term infants at increased risk for early onset bacterial sepsis

Can neurotoxic effects occur with bilirubin levels below severe/critical levels?

YES: Neurotoxic and cerebral injury Long term complications But exact levels UNKNOWN CPS Hyperbili

Do late preterms face higher risks of neurodevelopmental complications?

Yes At <2y follow-up, higher: Cerebral palsy (3x higher rate) Mental retardation/developmenal delay (1.25x higher rate) At early school age (even if no acute perinatal morbidity) higher: Developmental delay Special needs CPS facilitating discharge of late preterms infants

Is leukodepletion done in Canada? Should it be used in newborns?

Yes - across Canada No benefit to newborns but also no harms CPS Red blood cell transfusion in newborn infants

Does surfactant therapy help? In what situations

Yes - benefits morbidity and mortality Rescue therapy Prophylactic CPS surfactant

Can parents donate blood to be used in their infants? What are 7 issues with parental/relative directed donation?

Yes - some units are capable of doing that No data but probably no safer (everyone is going through the same testing) Identifying a compatible donor Mom may have anemia, fever/infection, medical problems, not be ABO compatible Delay in taking blood, do grouping/other tests) Needs irradiation May have problems with storage and not available when needed Ethical issues like what if you find out a family member is HIV +ve? Complying with parental wishes/reducing anxiety CPS Red blood cell transfusion in newborn infants

Can you use cord blood total bilirubin to predict severe hyperbilirubinemia?

Yes but: Low positive predictive value (proportion of people with a certain bilirubin level that will have severe hyperbili) Low specificity TSB in cord blood of >30micromol/L correlated with peak TSB >300 PPV 4.8% for term infants PPV 10.9% for prem infants CPS Hyperbili

CPS recommendation: does circumcision decrease HIV/penile cancer risk? Should routine neonatal circumcision be done for this reason? CPS recommendation: Does the CPS recommend routine circumcision for newborn males?

Yes evidence that circumcision decreases incidence of penile cancer and HIV transmission There is inadequate information to recommend circumcision as a public health measure to prevent these diseases NO - benefits and risks and complications equally balanced CPS circumcision

53. Child with brachial plexus injury. How long before if no change in exam is prognosis poor? a) 1 month b) 3 month c) 9 month d) 1 year

a) 1 month CPS: 75% recover within 1 month; 25% experience permanent disability - if PE shows incomplete recovery by end of first month, referral yo multidisciplinary team should be made - Nelson's: full recovery occurs in most patients; prognosis depends on whether nerve was merely injured or lacerated - if paralysis was due to edema and hemorrhage about the nerve fibers, function should return within a few months; if due to laceration, permanent damage may result - decision for surgical intervention by 3 months if function has not improved Perinatal brachial plexus injury • C5 - T1 • 0.4 - 5 infants per 1000 live births • no prospective studies on cause or prevention, birth trauma is the most common cause • Associated with o shoulder dystocia o large for gestational age o maternal diabetes o Instrumental delivery • No proven causative correlations • 20% to 30% of these infants will have residual deficits • If physical examination shows incomplete recovery by three to four weeks, full recovery is unlikely o results from nonrandomized studies children with severe injuries may do better with surgical repair • OVERALL o BPP is not always preventable o 75% of infants recover completely within the first month of life o 25% experience permanent impairment and disability o If exam shows incomplete recovery by the end of 1st month, refer to a multidisciplinary team A CPS Statement: Perinatal brachial plexus injury. Updated 2012 "If physical examination shows incomplete recovery by three to four weeks, full recovery is unlikely." "If the physical examination shows incomplete recovery by the end of the first month, referral to a multidisciplinary brachial plexus team should be made. The team should include neurologists and/or physiatrists, rehabilitation therapists and plastic surgeons." RF: Should dystocia, LGA, IDMs, Instrumental delivery Erb palsy: ▪ C5, C6 ▪ Abducted and internally rotated shoulder, pronated forearm, and wrist and fingers flexed ("waiter's tip" position) ▪ Absent biceps reflex, asymmetric Morro, preserved palmer grasp Klumpke palsy: ▪ C7, C8, T1 ▪ Weakness of flexor muscles of the wrist and small muscles of the hand ("claw hand") ▪ Horner syndrome in up to 1/3 of patients ▪ Good prognosis; 95% normal function by 1y Neonatal brachial plexus injuries occur in <0.5% of deliveries. PS p466

59. A premature infant was born at 30 weeks and is now 39 weeks CGA. He was treated early in life for apnea of prematurity with 5 days of caffeine. He now continues to have apneas with occasional significant bradycardias about 5 times per day. What is the most likely cause of his apneas: a) Apnea of prematurity b) GERD c) Inborn error of metabolism

a) Apnea of prematurity

16. A mother is given Demerol 90 minutes prior to delivery. The baby is suctioned and stimulated but remains apneic with a pulse of 80. Next step in management: a) Bag and mask b) More stimulation c) Narcan d) Antibiotics

a) Bag and mask a. Bag and mask b. More stimulation - do not delay ventilation for ongoing stimulation, if stimulation was going to work it would have on 1st attempt c. Narcan - perhaps but ABCs 1st (no longer routine in NRP) d. Antibiotics - no role in neonatal resuscitation A NRP - Remember MRSOPA Ventilation is essential before and during administration of Narcan as an antidote. If depression is due to other anesthetics or analgesics, artificial respiration should be continued until the infant is able to sustain ventilation. - N18 p723

20. The outcome in neonates since the introduction of surfactant is best described as: a) Decreased mortality b) Decreased BPD c) Increased air leaks d) Increased IVH d) Decreased PDA

a) Decreased mortality Surfactant adminstration: • Improves survival and reduces air leak but has not consistently reduced the incidence of BPD • Natural surfactants appear superior to synthetic • Rescue treatment is initiated as soon as possible in the first 24 hrs of life ♣ Repeat dosing every 6-12 hr for total of 2-4 doses • Complications - transient hypoxia, bradycardia and hypotension, blockage of ETT and pulmonary hemorrhage A Surfactant is a surface-active material that is made up of a mixture that is rich in phosphatidylcholine (64%), phosphatidylglycerol (8%), and lesser amounts of proteins and other lipids. Surfactant acts as an antiatelectasis factor in the alveolar linning by lowering surface tension at diminished lung volumes and increasing I at high volumes. This allows for the maintenance of functional residual capacity, which acts as a reservoir to prevent wide fluctuation in arterial PO2 and PCO2 during respiration. In patients with RDS, surfactants have been shown to: ♣ Decrease the need for supplemental oxygen therapy ♣ Lower mortality rates ♣ Decrease the incidence of air-leak syndromes The incidence of BPD has not been substantially reduced owing to increased survival of extremely preterm infants, who are most likely to develop this disorder. PS p474 Complications of surfactant therapy include: transient hypoxia, hypercapnia, bradycardia and hypotension, blockage of the endotracheal tube, and pulmonary hemorrhage. Predisposing factorsforIVHinclude: prematurity, respiratory distress syndrome, hypoxic ischemicorhypotensiveinjury,reperfusioninjuryofdamagedvessels, increasedordecreasedcerebralbloodflow,reducedvascularintegrity,increased venouspressure,pneumothorax,thrombocytopenia,hypervolemia, and hypertension. CPS Statement FN 05-01: Recommendations for neonatal surfactant therapy (Updated 2012 ♣ Mothers at risk of delivering babies with less than 34 weeks gestation should be given antenatal steroids according to established guidelines regardless of the availability of postnatal surfactant therapy (grade A). ♣ Intubated infants with RDS should receive exogenous surfactant therapy (grade A). ♣ Intubated infants with meconium aspiration syndrome requiring more than 50% oxygen should receive exogenous surfactant therapy (grade A). ♣ Sick newborn infants with pneumonia and an oxygenation index greater than 15 should receive exogenous surfactant therapy (grade C). ♣ Intubated newborn infants with pulmonary hemorrhage which leads to clinical deterioration should receive exogenous surfactant therapy as one aspect of clinical care (grade C). ♣ Natural surfactants should be used in preference to any of the artificial surfactants available at the time of publication of this statement (grade A). ♣ Infants who are at a significant risk for RDS should receive prophylactic natural surfactant therapy as soon as they are stable within a few minutes after intubation (grade A). ♣ Infants with RDS who have persistent or recurrent oxygen and ventilatory requirements within the first 72 h of life should have repeated doses of surfactant. Administering more than three doses has not been shown to have a benefit (grade A). ♣ Retreatment should be considered when there is a persistent or recurrent oxygen requirement of 30% or more, and it may be given as early as 2 h after the initial dose or, more commonly, 4 h to 6 h after the initial dose (grade A). ♣ Options for ventilatory management that are to be considered after prophylactic surfactant therapy include very rapid weaning and extubation to CPAP within 1 h (grade B). ♣ Intubated infants with RDS should receive exogenous surfactant therapy before transport (grade C). ♣ Centres administering surfactant to newborn infants must ensure the continuous on-site availability of personnel competent and licensed to deal with the acute complications of assisted ventilation and surfactant therapy (grade D). ♣ Mothers with threatened delivery before 32 weeks gestation should be transferred to a tertiary centre if at all possible (grade B). ♣ Infants who deliver at less than 29 weeks gestation outside of a tertiary centre should be considered for immediate intubation followed by surfactant administration after stabilization, if competent personnel are available (grade A). ♣ Further research into retreatment criteria and the optimal timing of prophylactic therapy is required.

63. A 2.2 kg term newborn breast feeds well at birth and then is seen at 2 hours of age with a BS of 1.9. He is asymptomatic. What is the best initial management? a) Feed and recheck BS in 1 hour b) IV dextrose at 4-6mg/kg/hr c) IV D10 2cc/kg then dextrose at 4-6 mg/kg/hr d) Supplement BF with bottle and recheck BS

a) Feed and recheck BS in 1 hour

9. Which of the following are true? a) Fetal pO2 is 25-30 b) The incidence of asymptomatic PFO in the adult population is 10%

a) Fetal pO2 is 25-30 • Umbilical pO2: 30-35% • Average fetal pO2: 26-28% • PFO incidence: A valve-competent but probe-patent foramen ovale may be present in 15-30% of adults. Fetal Circulation N18 p1855 -right and left ventricles exist in a parallel circuit, as opposed to the series circuit of a newborn or adult -placenta provides for gas and metabolite exchange -lungs do not provide gas exchange -vessels in the pulmonary circulation are vasoconstricted -3 structures unique to the fetus maintain this parallel circulation: ductus venosus, foramen ovale, ductus arteriosus -oxygenated blood from the placenta flows to the fetus thru the umbilical vein with a PO2 of 30-35 mmHg -50% umbilical venous blood enter the hepatic circulation -50% mixes with poorly oxygenated inferior vena cava blood -PO2 26-28 mmHg -and enters the right atrium -directed across the foramen ovale to the left atrium left ventricle ascending aorta upper part of the fetal body (coronary, cerebral arteries, upper extremities) -only 10% of the blood in the ascending aorta is sent to the descending aorta -fetal superior vena cava blood is less oxygenated -PO2 12-14 mmHg directed to right atrium right ventricle -PO2 18-22 mmHg -rather than the foramen ovale pulmonary artery 10% to the lungs and 90% via ductus arteriosus to the descending aorta to perfuse the lower part of the fetal body -ie. the right ventricle is pumping against systemic blood pressure and is performing a greater volume of work than the left ventricle -the upper part of the fetal body is perfused with a slightly higher PO2 than the blood perfusing the lower part of the fetal body -65% of descending aortic blood flow returns to the placenta, 35% perfused the fetal organs -right ventricle is perfusing lower body -left ventricle is perfusing upper body PFO N18 p1883 -a valve competent but probe-patent foramen ovale may be present in 15-30% of adults -an isolated PFO does not require surgical treatment, although it may be a risk for paradoxical (right to left) systemic embolization -device closure of these defects is considered in adults with a history of thromboembolic stroke EMed - Patent Foramen Ovale 2012 -PFO is a flaplike opening b/w the atrial septa primum and secundum at the location of the fossa ovalis that persists after 1 yr of age and has the potential for RL shunt -w increasing evidence that PFO is the culprit in paradoxical embolic events, the relative importance of the anomaly is being reevaluated -present in 10-15% of the population by transthoracic echo -most w isolated PFO are asxatic

73. A 3 day old is tachypneic, cyanosed despite 100% O2. Bilateral crackles on exam with weak peripheral pulses and no heart murmur. What is the diagnosis? a) HLHS b) Sepsis c) AV fistula

a) HLHS • Failed hyperoxic test = need R to L shunt (cyanotic heart defect or PPHN) • HLHS L sided lesion presenting with weak pulses • AV fistula = acyanotic lesion; present with murmur similar to PDA but louder. • Sepsis - must always think of but with other constellation of symptoms, HLHS more common.

17. Term newborn. Apneic at a few hours of age with decreased perfusion. First step in management is: a) Intubate b) Antibiotics c) Prostaglandins d) Observe for further deterioration

a) Intubate a. Intubate b. Antibiotics -Y, but not 1st step -sepsis is on ddx c. Prostaglandins -Y, but not 1st step -CHD is on ddx A Respiratory failure/insufficiency - Remember ABCs. Took the term 'apneic' to mean poor respiratory effort as in a newborn resuscitation rather than spells of apnea as seen in apnea of prematurity.

18. 16 week male born at 28 weeks gestational age. Hospitalized with RSV bronchiolitis pCO2 60, pO2 94 in 50% oxygen. CXR shows RML infiltrated. Tachypneic. Best management: a) Intubate and ventilate b) Humidified oxygen and monitor closely c) Ribavirin d) Steroids e) Antibiotics

a) Intubate and ventilate Evidence of poor oxygenation and ventilation, at risk of apnea ... would probably try BiPap first RSV treatment = symptomatic • Humidified oxygen and suctioning for hospitalized infants who are hypoxic o disagreement among experts regarding the usefulness of epinephrine or β2-agonist ♣ most patients do not receive lasting benefit from prolonged therapy • Fluids/nutrition o Many infants are slightly to moderately dehydrated, and therefore fluids should be carefully administered in amounts somewhat greater than those for maintenance o Often intravenous or tube feeding is helpful when sucking is difficult because of tachypnea • Corticosteroid therapy is not indicated except in older children with an established diagnosis of asthma o its use is associated with prolonged virus shedding o no proven clinical benefit. • In nearly all instances of bronchiolitis, antibiotics are not useful, and their inappropriate use contributes to development of antibiotic resistance • Ribavirin : Early small trials of its use suggested a modest beneficial effect on the course of RSV pneumonia, with some reduction in the duration of both mechanical ventilation and hospitalization o Subsequent studies failed to document a clear beneficial effect of ribavirin. • Palivizumab is licensed for prophylaxis in high-risk infants o small clinical trials using the antibody as a therapy during established infection have not shown benefit to date a. Intubate and ventilate - signs suggest impending respiratory failure b. Humidified oxygen and monitor closely c. Ribavirin - No d. Steroids - No e. Antibiotics - No Staff Neo agreed with a - signs suggest impending respiratory failure A Hypercarbia, high FiO2, Respiratory distress

1. A GP calls you about a 36 week baby that is now 3 days old. The physical exam is normal except for jaundice. The bilirubin on day 3 is 280. The family is of East Asian origin. What are four questions you would ask him? What are two pieces of advice you would give him to manage this patient?

a) Is bilirubin conjugated or unconjugated? Is breastfeeding established? Weight? FMHx of hyperbilirubinemia (or G6PD)? Mom and baby blood type? a) Start Phototherapy with repeat bili in 6-8 hours, Test for G6PD Deficiency, DAT

64. A 24 hour old newborn is seen with a report of scant UOP, the creatinine is 85. What is this most consistent with? a) Maternal creatinine b) Creatinine cannot be interpreted without a urea c) Creatinine is elevated for this newborn d) Creatinine cannot be interpreted without a urine creatinine

a) Maternal creatinine A First24hrsoflife, 0 - 1cc/kg/hrofUOPduetonaturalsurgeofADHininfant causingwaterretentionanddecreasedUOP.SothescantUOPisnot surprising/abnormal.

13. Which of the following pertain to the transition from fetal to neonatal life? a) PFO persists in 10% of adults b) Umbilical veins close before umbilical arteries

a) PFO persists in 10% of adults • The muscular umbilical arteries contract readily, but the vein does not. The vein retains a fairly large lumen after birth. • The blood vessels are functionally closed but anatomically patent for 10-20 days. o The arteries become the lateral umbilical ligaments; the vein, the ligamentum teres; and the ductus venosus, the ligamentum venosum. • The umbilical cord usually sloughs within 2 wk. Delayed separation of the cord, after more than 1 mo, has been associated with neutrophil chemotactic defects and overwhelming bacterial infection • PFO present in 15-30% of adults Wrong Wrong! Arteries close before vein. Forms the basis of delayed cord clamping. At the time of birth, the placental vessels may contain up to 33% of the fetal-placental blood volume. Constriction of the umbilical arteries limits blood flow from the infant, but the umbilical vein remains dilated. The extent of drainage from the placenta to the infant is very dependent on gravity. The recommendation is to keep the baby at least 20 to 40 cm below the placenta for about 30 to 60 seconds before clamping the cord. PS p423

39. What are risk factors for unconjugated hyperbilirubinemia in a neonate? a) Prematurity b) LGA c) Male

a) Prematurity - CPS Statement: Both prematurity and male are RFs for 'severe' hyperbilirubinemia, but short gestation has higher odds ratio than male sex Risk Factors for Development of Severe Hyperbilirubinemia in Infants > 35 weeks: Major risk factors Predischarge TSB or TcB level in the high-risk zone Jaundice observed in the first 24 hr Blood group incompatibility with positive direct antiglobulin test, other known hemolytic disease (G6PD deficiency) Gestational age 35-36 wk Previous sibling received phototherapy Cephalohematoma or significant bruising Exclusive breastfeeding, particularly if nursing is not going well and weight loss is excessive East Asian race Minor risk factors Predischarge TSB or TcB level in the high intermediate-risk zone Gestational age 37-38 wk Jaundice observed before discharge Previous sibling with jaundice Macrosomic infant of a diabetic mother Maternal age ≥25 yr Male gender Decreased risk TSB or TcB level in the low-risk zone Gestational age ≥41 wk Exclusive bottle feeding Black race Discharge from hospital after 72 hr Additional risk factors include polycythemia, infection, PREMATURITY, and having a diabetic mother. RF for Jaundice (unconj HBR) ♣ Prematurity ♣ Sibling with severe HBR ♣ Bruising ♣ Cephalohematoma ♣ Dehydration ♣ Asian decent ♣ Exclusive breastfeeding ♣ IDM ♣ G6PD deficiency ♣ Maternal blood group O or Rhesus negative EN p248, SK p588

6. Prolonged apnea in ex-25 week baby, now corrected age 39 wks. Past hx of IVH. What is the cause. a) Prolonged apnea of apnea of prematurity b) PVL c) GERD

a) Prolonged apnea of apnea of prematurity • Apnea of prematurity usually resolves by 36 weeks o May be prolonged especially in very premature babies • The onset of apnea in a previously well premature neonate after the 2nd wk of life or in a term infant at any time is a critical event that warrants immediate investigation • Data do not support a causal relationship between Gastroesophageal reflux and apneic events or the use of antireflux medications to reduce the frequency of apnea in preterm infants. • PVL doesn't usually cause apnea...

49. A baby has direct hyperbilirubinemia. He was treated for an E. coli sepsis. He has hepatomegaly on exam. Which test will likely yield the diagnosis? a) RBC GALT function b) G6PD level c) Osmotic fragility d) RBC glucose-phosphate 1 deficiency

a) RBC GALT function (deficient) Not b) G6PD level (assess for G6PD) c) Osmotic fragility - diagnoses hereditary spherocytosis - RBCs are incubated in progressive dilutions of an iso-osmotic buffered salt solution; - exposure to hypotonic saline causes RBCs to swell and spherocytes lyse more readily than biconcave cells in hypotonic solutions d) RBC glucose-phosphate 1 deficiency (not a thing?) - direct hyperbilirubinemia = conjugated - Nelson's: Galactosemia - deficiency of galactose-1-phosphate uridyl transferase (can have complete or partial deficiency) - classic typically presents by 2nd half of first week of life, infant receives high amounts of lactose (glucose + galactose) in breast milk/formula -> unable to metabolize galactose-1-phosphate -> accumulates and injures kidney, liver, and brain - Clinical manifestations of complete deficiency: jaundice, hepatomegaly, vomiting, hypoglycemia, seizures, lethargy, irritability, feeding difficulties, poor weight gain, nuclear cataracts, vitreous hemorrhage, hepatic failure, liver cirrhosis, ascites, splenomegaly, or mental retardation. - increased risk for e. coli sepsis (onset of sepsis often precedes diagnosis of galactosemia) - partial deficiency generally asymptomatic, more common, diagnosed usually by NBS - Diagnosis: - demonstration of reducing substance in several urine specimens collected while patient is receiving lactose (can be identified by chromatography or by enzymatic test specific for galactose) - direct enzyme assay using erythrocytes - hemolysates of erythrocytes, see deficient activity of galactose-1-phosphate uridyl transferase (GALT), and increased concentrations of substrate galactose-1-phosphate Galactosemia! Direct Conjugated Hyperbilirubinemia: • > 2 mg/dL or >20 percent total bilirubin • Never physiologic, should always be investigated o Liver enzymes, bacterial/viral cultures, metabolic screening tests, hepatic ultrasound, sweat chloride and occasionally liver biopsy o Dark urine + Pale stools suggests biliary atresia (after second week of life) • Not neurotoxic, but signifies serious underlying disorder involving cholestatis or hepatic injury COMMON UNCOMMON Hyperalimentation cholestasis CMV infection Other TORCH infections Inspissated bile from prolonged hemolysis Neonatal hepatitis Sepsis Hepatic infarction Inborn errors of metabolism (Galactosemia and tyrosinemia) CF Biliary atresia Choledochal cyst Alpha anti-trypsin deficiency Neonatal iron storage disease Alagille syndrome Byler disease Galactosemia A Galactosemia ♣ Also known as GALT ♣ Autosomal recessive ♣ Inability to metabolise galactose or lactose (glucose + galactose) and accumulation of galactose-1-phosphate that causes damage to the brain, liver and kidney. ♣ Non-glucose-reducing substances positive ♣ Reduced galactose 1-phosphate uridyl transferase in RBCs ♣ Associated with Escherichia coli sepsis ♣ Hepatomegaly, Jaundice, FTT, cataracts, neurocognitive problems, ovarian problems. ♣ Increased conjugated bilirubin, increased INR, RTA, hemolytic anemia ♣ Positive urine-reducing substances ♣ RBC GALT before blood transfusions ♣ Diet therapy: restriction of galactose and lactose, soy formula alternative, calcium supplements SK p338 & 548 Glucose-6-phosphatase (Glc-6-Pase) deficiency ▪ Also termed GSD type I or von Gierke disease, is a rare form of GSD. ▪ The hydrolysis and transport of glucose 6-phosphate requires a hydrolase and microsomal transporters, pyrophosphate and glucose. ▪ Type Ia results from a deficiency in the glucose 6-phosphate hydrolase activity, and makes up more than 80% of cases. ▪ Types Ib (glucose-6-phosphate transporter deficiency), Ic, and Id are allelic defects in the translocase associated with glucose-6-phosphatase. ▪ Patients who lack G-6-phosphatase are not able to produce glucose from glycogenolysis. ▪ They have fasting hypoglycemia that is unresponsive to exogenous glucagon or epinephrine administration. ▪ Galactose and fructose are normally converted to glucose by way of glucose-6-phosphate. ▪ When G-6-phosphatase is lacking, administering galactose or fructose (as lactose or sucrose) aggravates the lactic acidosis without raising the blood glucose level. ▪ Diagnosis is made by enzyme assay in blood, liver or muscle biopsy. Genetic testing is also available, which is especially useful for family screening. Liver biopsy remains the criterion standard for diagnosis. Presentation: ▪ Parental consanguinity positive (autosomal recessive). ▪ Positive Family history ▪ Hypoglycemia (symptomatic or asymptomatic). ▪ Failure to thrive. ▪ Delayed pubertal onset and development. ▪ Seizures (hypoglycemia). ▪ Abdominal pain (hepatomegaly, uric acid nephrolithiasis, or pancreatitis). ▪ Hematuria. ▪ Bony pain or pathological fractures (osteopenia). ▪ Gouty arthritis (uric academia). ▪ Recurrent bacterial infections. Management: ▪ Compliance with appropriate dietary therapy is the first line of treatment with avoidance of fasting. o Institute frequent feedings of glucose polymer solutions. o Use gastric drip-feeding through an enteral feeding tube. o Feed uncooked cornstarch, which is a complex carbohydrate that undergoes slow intestinal hydrolysis and provides a constant source of glucose over several hours. EMed - Glucose-6-Phosphatase deficiency 2012

62. What is the most common cause of central apnea in a 34 week prem who is now 8 weeks of age? a) RSV bronchiolitis b) Inborn error of metabolism c) Apnea of prematurity

a) RSV bronchiolitis A "Fever is an inconstant sign in RSV infection. In young infants, particularly those who were born prematurely, periodic breathing and apneic spells have been distressingly frequent signs, even with relatively mild bronchiolitis. Apnea is not necessarily caused by respiratory exhaustion, but rather appears to be a consequence of alterations in central control of breathing." - Nelson's chapter 252 (clinical manifestions) CPSStatementsRiskfactorsforsevereRSV

42. A newborn with 37.3 axillary temperature. What should be done next? a) Repeat after 20 minutes of unbundling b) Do a tympanic temperature - not recommended in newborns/infants c) Do CBC d) Full septic W/U

a) Repeat after 20 minutes of unbundling - CPS Statement Temperature Measurement in Pediatrics. - Normal temps: - Axillary: to 37.5 - Oral: to 37.5 - Rectal: to 38 - Tympanic: to 38 - Recommended techniques: - Birth to 2 years: rectal (gold standard), axillary (screening) - 2-5 years: rectal (definitive); axillary, tympanic (screening) - >5 years: oral (definitive), axillary, tympanic (screening) CPS Statement: Temperature Measurement in Pediatrics (Feb 2011) • Normal body temperature 37°C (98.6°F) • Generally accepted that 38°C (100°F) per rectum represents a fever Route Celsius Farenheit Rectal 36.6 - 38°C 97.9 - 100.4°F Ear 35.8 - 38°C 96.4 - 100.4°F Oral 35.5 - 37.5°C 95.9 - 99.5°F Axillary 34.7 - 37.3°C 94.5 - 99.1°F Rectal Axillary Oral Tympanic **Gold standard** Limitations: Slow to change in relation to core temp, Stay elevated after core temp begins to fall & vice versa Affected by: Depth of measurement, Conditions affecting local blood flow, Stool, perforation has been described (<1 in 2 million) **Inaccurate estimate of core temperature in children** Relies on the thermometer remaining directly in place over the axillary artery Affected by: Environmental conditions Low sensitivity and specificity in detecting fever BUT recommended by AAP for screening for fever in neonates 20 risk of rectal perforation (small risk) **Accuracy between axillary and rectal, & accuracy may increase with age** Limitations: Influenced by recent ingestion of food, drink and mouth breathing, Relies on the mouth remaining sealed, with the tongue depressed for 3-4 min (cannot for unconscious or uncooperative patients) **Promising tool for screening children at low risk in the ER, not recommended for home or hospital use** Measures thermal radiation from TM blood supply is similar to blood bathing the hypothalamus Crying, otitis media or earwax have not been shown to change tympanic readings Affected by: Variable canal structure, probe design and position, probe must be small enough to be deeply inserted into the meatus to allow orientation of the sensor against the TM Meatal diameter 4 mm at birth, 5 mm at 2 years Age Recommended Technique Birth - 2 years 1. Rectal (definitive) 2. Axillary (screening low risk children) 2 - 5 years 1. Rectal (definitive) 2. Axillary, Tympanic (or Temporal Artery if in hospital) (screening) 5 years 1. Oral (definitive) 2. Axillary, Tympanic (or Temporal Artery if in hospital) (screening)

74. What is true about neonatal chest compressions? a) Required after 30 seconds of PPV with HR less than 60 b) Chest compressions to be done at a ratio of 5:1 with every breath c) 2 finger technique is preferred d) Required after no response to stimulation

a) Required after 30 seconds of PPV with HR less than 60 NRP ratio is 3:1 (90 compressions to 30 breaths)

61. A 12 hour old infant born at term had mild polyhydramnios on 20 week U/S. Now the baby is grunting intermittently with lots of secretions. He improves with intermittent suctioning. What diagnosis is this most consistent with: a) TEF / esophageal atresia b) Duodenal atresia

a) TEF / esophageal atresia

19. In counseling a woman who has had a child with a meningomyelocele, what would you tell her as regards to her next pregnancy: a) Take folic acid prior to conception and then for 10 weeks afterwards b) Ultrasound at 16 weeks c) Amniocentesis at 16 weeks d) Alpha-fetoprotein at 16 weeks

a) Take folic acid prior to conception and then for 10 weeks afterwards Daily folic acid intake of 400mcg reduces the risk of spina bifida by as much as 70% - PS p581 CPS: previous infant with a NTD, recommendations: 1. Take folic acid prior to conception and afterwards 2. Screening with 16 week U/S and AFP OR amniocentesis with AFP levels of amniotic fluid • Women who take folic acid supplements before and during pregnancy lower their risk for delivering a child with major malformations • 4-5 mg of folic acid daily for women who have had a child with a neural tube defect • 0.4 mg for all other women planning pregnancy • 2-5 mg of folic acid daily to women with epilepsy who are taking anticonvulsants, beginning as long as 3 months before conception until 12 weeks' gestation

24. You are discussing with a mother the prognosis of her child born at 31 weeks gestation. There was thin meconium at birth. APGARS were 8 and 9. Head ultrasound showed a small intraventricular hemorrhage at one week of age. She is concerned because he is not yet walking and has stiff legs. a) The child likely has spastic diplegia which is often associated with prematurity and IVH b) The history is unusual - children with CP usually have a history suggestive of birth asphyxia c) IVH would cause PVL which would result in hemiplegia d) There will be a progressive decline in his development

a) The child likely has spastic diplegia which is often associated with prematurity and IVH Cerebral Palsy: • Prevalence increased among low birthweight infants (esp <1,000 g) o Primarily due to intracerebral hemorrhage and periventricular leukomalacia (PVL) • PVL : o hypoxia-ischemia, venous obstruction from an IVH, or undetected fetal stress decreased perfusion to the brain periventricular hemorrhage and necrosis o Causes CP/motor abnormalities because corticospinal tracts descend through the periventricular white matter A TheprevalenceofCPhasincreasedsomewhatduetotheenhancedsurvivalofvery prematureinfantsweighing<1,000g,whogoontodevelopCPatarateof approximately15/100.ThemajorlesionsthatcontributetoCPinthisgroupare intracerebralhemorrhageandperiventricularleukomalacia(PVL).Althoughthe incidenceofintracerebralhemorrhagehasdeclinedsignificantly,PVLremainsamajor problem.PVLreflectstheenhancedvulnerabilityofimmatureoligodendrogliain prematureinfantstooxidativestresscausedbyischemiaorinfectious/inflammatory insults. Approximately30%ofprematureinfants<1,500ghaveIVH.Theriskisinverselyrelate togestationalageandbirthweight,withthesmallestandmostimmatureinfantsbeing atthehighestrisk;7%ofinfants1,0011,500ghaveasevereIVH(gradeIIIorIV), comparedwith14%ofinfants7511,000gand24%ofinfants750g.In3%ofinfants <1,000g,periventricularleukomalacia(PVL)develops. N18 IVH ♣ IVH originates in the supependymal germinal matrix, a region of the developing brain that regresses by term ♣ Site of proliferation and differentiation of neuroblasts and glioblasts that migrate to the cerebral cortex ♣ Neuronal differentiation complete at 20 weeks ♣ Glial differentiation ongoing until 32 weeks ♣ Germinal matrix regression is nearly complete by 32 weeks ♣ Germinal matrix is prone to hemorrhage and / or ischemia -metabolically active differentiating cells with high O2 needs; supplied by a primitive and fragile capillary network; located in a vascular watershed zone prone to hypoxic-ischemic insult ♣ Grades: o I - isolate subependymal or germinal matrix hemorrhage o II - hemorrhage into lateral ventricles, no ventricular enlargement o III - hemorrhage into lateral ventricles, ventricular enlargement o IV - intraparenychmal hemorrhage

11. A well newborn presents with the following rash that appears particularly in the areas covered by clothes and in the creases. (Not exact photo) a) What is the diagnosis? b) What do you advise the parents in terms of management?

a) What is the diagnosis? Miliaria rubra (heat rash) - occurs in covered/sweaty areas (creases, covered by clothes); small groups of erythematous papules and pustules b) What do you advise the parents in terms of management? - light, loose clothing; cool baths - symptoms will resolve spontaneously

1. Jittery neonate age 6 hr with glucose 1.8 What to do:

a) feed and recheck in 1 hour b) bolus with D50 2 cc/kg c) IV D10 at TFI 80 Recheck within 30 minutes after any change to IV infusion If seizing, would give 2mL/kg 10% dextrose at start of infusion Hypoglycemia defined blood glucose <2.6mmol/l and specific interventions are recommended with this reference - SK p574 CPS Statement: Screening guidelines for newborns at risk for low blood glucose (2004), Reaffirmed in 2012 Protective factors for Hypoglycemia ♣ Early feeding ♣ AGA ♣ Term delivery Risk factors for Hypoglycemia ♣ Prematurity ♣ Hypothermia ♣ Hypoxia ♣ Maternal diabetes ♣ Maternal hypertension ♣ Maternal glucose infusion in labor ♣ IUGR/SGA ♣ RDS ♣ Polycythemia ♣ Hormonal deficinecies PS p455-456, SK p576 Critical Sample: to be drawn in the presence of hypoglycemia when persistent / at risk baby: ♣ Acylcarnitine, Amino acids, Ammonia ♣ Blood gas, -hydroxybutyrate ♣ Cortisol ♣ Free Fatty Acids ♣ Glucose ♣ Growth Hormone ♣ Insulin ♣ Ketones (urinary) ♣ Lactate ♣ Organic Acids (urinary) ♣ TSH and Free T4 SK p574

4. Infant born to mom with no PNC. Babe has mydriasis. What did mom take: a) heroin b) cigarettes c) cocaine d) alcohol

a) heroin Neonatal abstinence syndrome: • withdrawal from heroin: earliest sign is yawning, followed by lacrimation, mydriasis, restlessness, insomnia, "goose flesh," cramping of the voluntary musculature, bone pain, hyperactive bowel sounds and diarrhea, tachycardia, and systolic hypertension • Withdrawal from alcohol and cocaine is rare (sympathomemtic) Effects of Cocaine: HTN, tachycardia, cardiac ischemia, hyperthermia, respiratory distress, intestinal infarction, rhabdomyolysis, mydriasis, psychomotor agitation, coma seizures, cerebralvascular accident. - SK p70 Examples of other Sympathomimetics: Amphetamines, Methamphetamines,MDMA (Ecstasy), Decongestants (ephedrine), Caffeine, Theophylline, withdrawal from sedatives/hypnotics. - SK p60

3. Shown photo of gastroschisis (same photo as previous). What is an associated defect? a) intestinal atresia b) renal defect c) cardiac defect

a) intestinal atresia • Gastroschisis: o Herniated abdominal contents, free with no sac o Usually to the right of the umbilicus o Often have prolonged ileus o 10-15% intestinal atresia • Omphalocele: o herniation of abdominal contents into the base of the umbilical cord o sac is covered with peritoneum without overlying skin. o saline-soaked sterile dressings should be applied immediately o Immediate surgical repair, o majority (≈75%) have associated congenital anomalies/syndromes: ♣ Beckwith-Wiedemann syndrome (omphalocele, macrosomia, hypoglycemia), ♣ other chromosomal (29%, including trisomies 13 and 18) ♣ isolated congenital anomalies (musculoskeletal, 24%; urogenital, 20%; cardiovascular, 15%; and central nervous system, 9%).

7. Neonate with dehydration and mom was IDDM. Baby develops hematuria. What's the dx: a) renal vein thrombosis b) ??

a) renal vein thrombosis - polycythemia more common in IDM than born to non-diabetic mothers - RVT presents in neonates and infants; often have predisposing factors - dehydration, asphyxia, sepsis/shock, congenital hypercoagulable states, IDDM - on exam: acute gross hematuria, flank mass (usually unilateral) - US: dilated kidney; confirmed on Doppler Infants of diabetic mothers; • LGA, marcrosomia o Increased risk birth trauma • Hypoglycemia: 25-50% o Usually of shorter duration (recovery may being 4-6 hr) • Higher incidence of RDS (antagonistic effect of insulin on surfactant synthesis) • Asymmetric septal hypertrophy cardiomegaly heart failure (5-10%) • Hematologic: increased incidence of hyperbilirubinemia, polycythemia and RV thrombosis • Congenital anomalies 3x greater risk (cardiac defects, NTDs, renal anomalies, etc) Remember: Palpable flank mass, hematuria, and thrombocytopenia = RVT until proven otherwise PS p446

1. Ex 29 week prem with hyperreflexia and gross motor delay. Had history of Grade II IVH and PVL. a. Advise the mother about the likely cause of CP in this child. b. What would you see on a CT that is specific to this?

a. Advise the mother about the likely cause of CP in this child. PVL b. What would you see on a CT that is specific to this? Loss of periventricular white matter and ventricular enlargement. a. Advise the mother about the likely cause of CP in this child: CP likely due to hx of prematurity, IVH and PVL b. What would you see on a CT that is specific to this? • CT - necrosis in periventricular white matter • MRI - increased periventricular intensity on T2-weighted scans, ventriculomegaly, a bumpy configuration of the lateral ventricles, high-intensity areas in the white matter adjacent to the trigone body of the lateral ventricles, and deep cortical sulci o These findings result from glial scarring and inadequate myelination of periventricular axons o MRI findings in children with PVL are related to clinical outcome

19) In an infant who has had a previous Acute Life Threatening Event, but a normal workup, which is the most appropriate course of action: a. Baby should sleep in parents room b. Baby should sleep in a separate room, but be checked on every hour c. Audio monitor with baby in next room d. Suggest that baby sleep on its side e. No intervention

a. Baby should sleep in parents room - cosleeping not bedsleeping A CPS definitions: Bedsharing refers to a sleeping arrangement in which the baby shares the same sleeping surface with another person. Cosleeping refers to a sleeping arrangement in which an infant is within arm's reach of his or her mother, but not on the same sleeping surface. Sleeping in the same room (ie, room-sharing), but not in the same bed, is cosleeping. Summary of evidence reviewed by CPS: ♣ Sleeping on the back carries the lowest risk of SIDS. ♣ Room-sharing lowers the risk of SIDS . ♣ The risk of SIDS is increased when infants bedshare with mothers who smoke cigarettes. ♣ Bedsharing with an adult who is extremely fatigued or impaired by alcohol or drugs (legal or illegal) that impair arousal can be hazardous to the infant. ♣ The use of soft bedding, pillows and covers that can cover the head increase the risk of death in all sleeping environments. ♣ Sleeping with an infant on a sofa is associated with a particularly high risk of sudden unexpected death in infancy. ♣ An infant is more at risk of sudden unexpected death if he/she bedshares with people other than his/her parents or usual caregiver. CPS Statement: Recommendations for safe sleeping environments for infants and children 2004 (reaffirmed in 2012) ◦ Infants should sleep on their back, in cribs meeting the Canadian Government's safety standards. This is the recommended sleeping arrangement for the first year of life, under all circumstances. ◦ The infant sleep environment must be free of quilts, comforters, bumper pads, pillows and pillow-like items. Dressing infants in sleepers should be considered to eliminate the need for any covers over the baby, other than a thin blanket. ◦ Parents should also be aware that room-sharing is protective against SIDS and that this type of sleeping arrangement is a safer alternative to bedsharing. This may be particularly appealing to mothers who breastfeed and want their baby to be near them without sharing the same bed surface. ◦ Effective counselling to prevent maternal smoking should begin at the onset of pregnancy, and ideally, well before that. Mothers who smoke during their pregnancy should be informed that their infant has a greater risk of SIDS. Passive exposure to environmental tobacco smoke is also associated with an increased risk of SIDS. When there is exposure to cigarette smoking, pre- or postnatally, the risk of SIDS is further increased with bedsharing. ◦ Hospitals should not allow mothers to sleep in the same bed with their newborns in view of the effects of postpartum maternal weakness or fatigue, analgesia or postanesthesia. This policy will also serve to educate parents on safe sleeping practices. However, it must not compromise in any way the maternal-infant interaction necessary for the initiation of successful breastfeeding. ◦ Parents should not place infants on waterbeds, air mattresses, pillows, soft materials or loose bedding, even for temporary sleeping arrangements (eg, during travel). Car seats and infant seat carriers must not replace the crib as a sleep surface due to the risk of the harness straps causing upper airway obstruction. ◦ Sleeping with an infant, or letting the infant sleep alone on any type of couch, recliner or cushioned chair is dangerous, placing infants at substantial risk for asphyxia or suffocation. Any makeshift bed is dangerous as well. Apnea monitors and SIDS: Apnea, SIDS and home monitoring AAP 2003 ♣ Epidemiological studies have not been able to demonstrate an impact of home monitoring on the incidence of SIDS ♣ AAP recommends that home monitors not be prescribed to prevent SIDS. ♣ Indication for monitoring include the following: o Premature infants with persistent apnea and bradycardia o Technology-dependent infants o Infants with neurologic or metabolic disorders that affect respiratory control o Infants with CLD, especially those requiring O2, CPAP, and/or mechanical ventilation PS p479

1. Newborn, cardiac history. CXR given. (Egg on a string) a. Diagnosis? b. What do you do?

a. Diagnosis? Transposition of the great arteries b. What do you do? Start Prostaglandin and call cardiothoracic surgery!

1. A 4.3kg neonate is born at 39 weeks. He has a 1.5cm omphalocele and a slightly protuberant tongue. His glucose is 0.8mmol/L and IV glucose is started. a. Name 3 hypoglycemic conditions in which the glucose requirement is expected to be normal for a term baby. b. Name 3 hypoglycemic conditions in which you would expect the glucose requirements to be higher than expected for a term baby. c. Hypoglycemia with normal glucose requirement d. Hypoglycemia with increased glucose requirement

a. Name 3 hypoglycemic conditions in which the glucose requirement is expected to be normal for a term baby. SGA; metabolic (IEM, GSD), Endo (GH deficiency, cortisol deficiency) - Normal GIR = 5-6 b. Name 3 hypoglycemic conditions in which you would expect the glucose requirements to be higher than expected for a term baby. Hyperinsulinism, sepsis, asphyxia - Higher GIR > 10 c. Hypoglycemia with normal glucose requirement - d. Hypoglycemia with increased glucose requirement -

1. Infant born at 33 weeks, apneic, HR < 50 a. What 2 things do you do in your initial management? b. CXR shown - name 2 abnormalities (bad x-ray - hazy white-out of all lung-field. What is the underlying problem?

a. What 2 things do you do in your initial management? Warmer, dry + stimulate, PPV b. CXR shown - name 2 abnormalities (bad x-ray - hazy white-out of all lung-field. What is the underlying problem? - ?RDS (ground-glass, reduced lung fields) - bilateral pleural effusions - consider chylothorax

1. You are taking care of a newborn ventilated baby born at 26 weeks who is now 29 weeks CGA with pulmonary interstitial emphysema on CXR. He is currently ventilated with the following settings: rate 40, PIP 18, PEEP 4, FiO2 60%. You receive the following blood gas: pH 7.2, pCO2 56, HCO3 19, base deficit - 6. a. What change in ventilator settings do you make? 48 hours later, the baby is suddenly mottled, and has the following bloodwork: Na 139, K 6.8, pH 7.18, pCO2 38, pO2 68, HCO3 12, base deficit -12. The baby's urine output is 2.5 cc/kg/h. b. What complication has most likely occurred?

a. What change in ventilator settings do you make? Increase PEEP, Increase RR, Increase PIP (can safely increase) (PEEP 4 is less than physiologic, want to lower CO2) 48 hours later, the baby is suddenly mottled, and has the following bloodwork: Na 139, K 6.8, pH 7.18, pCO2 38, pO2 68, HCO3 12, base deficit -12. The baby's urine output is 2.5 cc/kg/h. b. What complication has most likely occurred? Sepsis (metabolic acidosis) - other complications = PDA, bleed (metabolic acidosis), tension pneumothorax Tension pneumothorax (should give other VS changes) a. What change in ventilator settings do you make? Decrease PIP, Increase RR 48 hours later, the baby is suddenly mottled, and has the following bloodwork: Na 139, K 6.8, pH 7.18, pCO2 38, pO2 68, HCO3 12, base deficit -12. The baby's urine output is 2.5 cc/kg/h. b. What complication has most likely occurred? Tension pneumothorax a. What change in ventilator settings do you make? Increase PIP and increase rate (may not want to increase PEEP b/c of interstitial emphysema) b. 48 hours later, the baby is suddenly mottled, and has the following bloodwork: Na 139, K 6.8, pH 7.18, pCO2 38, pO2 68, HCO3 12, base deficit -12. The baby's urine output is 2.5 cc/kg/h. What complication has most likely occurred? Tension Pneumothorax; also think of NEC, septic shock What change in ventilator settings do you make? Increase the rate and increase PEEP 48 hours later, the baby is suddenly mottled, and has the following bloodwork: Na 139, K 6.8, pH 7.18, pCO2 38, pO2 68, HCO3 12, base deficit -12. The baby's urine output is 2.5 cc/kg/h. What complication has most likely occurred? ASK Neo NEC ?? -will make you acidotic and hyperkalemia Staff neo doesn't know

1. Polycythemic newborn. Hb 240, Hct 0.75. Wt 2000g. Child requires a partial exchange transfusion. a. What fluid do you use as the diluent? b. How much blood do you replace to decrease the Hct to 0.5?

a. What fluid do you use as the diluent? Normal Saline b. How much blood do you replace to decrease the Hct to 0.5? Formula from Nelson's

1. Polycythemic newborn. Hgb 240. Hct 0.75. Wt 2000g. Child requires a partial exchange transfusion. a. What fluid do you use as the diluents? b. How much blood do you replace to decrease the hematocrit to 0.5?

a. What fluid do you use as the diluents? Saline b. How much blood do you replace to decrease the hematocrit to 0.5? 53mL - Blood volume ~ 160mL (8% of 2000g) - [160mL x (75-25)]/75 = 53 How much blood do you replace to decrease the hematocrit to 0.5? Volume of exchange (mL) = Blood volume x (Observed -Desired hematocrit) / Observed hematocrit = 160 cc x (0.75-0.5)/0.75 = 53.3 cc Nelson's: Polycythemia defined by Hct >65% - Incidence increased at high altitudes, post-term, SGA, twin-to-twin recipient, delayed cord clamping... - CP: **often asymptomatic; irritability, lethargy, tachypnea, respiratory distress, cyanosis, feeding disturbances, hyperbilirubinemia, hypoglycemia, thrombocytopenia - Severe complications: seizures, stroke, pulmonary hypertension, necrotizing enterocolitis, renal vein thrombosis, renal failure - Treatment if symptomatic: partial exchange transfusion with normal saline o Volume of exchange = [blood volume x (observed - desired hematocrit) ]/ observed hematocrit o Blood volume in neonate = 85-90mL/kg, or 8% of body weight a. What fluid to you use as the diluents? Normal Saline b. How much blood do you replace to decrease the hematocrit to 0.5? Volume of exchange (ml) = Blood volume x (observed -desired hct) / observed hct = (85ml/kg x 2) x (0.75-0.5) / (0.75) = 57ml Note: Blood volume in term infants 80ml/kg

1. Newborn term, AGA baby, cried at birth. Apgars 9 and 9. Few hours later found to be in respiratory distress. RR 80. HR high. Cap refill 4-5 seconds. BP 48/32. Hyperinflated chest with minimal indrawing. Cannot hear breath sounds on left, cannot hear heart sounds. Baby is turning cyanotic. a. What investigation do you do? b. What is your possible diagnosis? Baby's heart rate is now 80 and is more cyanotic. c. What one investigation do you want to do now?

a. What investigation do you do? CXR b. What is your possible diagnosis? Pneumothorax Baby's heart rate is now 80 and is more cyanotic. c. What one investigation do you want to do now? Needle decompression

1. A one hour old boy has a plethoric face, marked acrocyanosis, and respiratory distress. His hematocrit is 75%. a. What is his diagnosis? b. How would you treat him? c. What are 3 complications of the problem?

a. What is his diagnosis? Polycythemia b. How would you treat him? Partial exchange transfusion c. What are 3 complications of the problem? Seizures, renal vein thrombosis, stroke, pulmonary hypertension

1. 6 week old baby presents with bruising and lethargy. Mom had minimal antenatal care and had a home delivery. She has not been to see a healthcare practitioner. The baby's labs show a normal CBC, but INR of 4.2 and PTT of 57? a. What is the diagnosis? b. What one test would you do right now? c. What is the best way to prevent this? (be specific) d. If parents refuse, what is the alternative and be specific? (be specific):

a. What is the diagnosis? Hemorrhagic Disease of the Newborn (Late) b. What one test would you do right now? CT Head c. What is the best way to prevent this? (be specific) IM Vitamin K administration at birth (within 6 hours) d. If parents refuse, what is the alternative and be specific? (be specific): Repeated oral vitamin K administration

1. Picture of baby with back with a large red firm plaque; in question mentions baby was healthy term but had a difficult and prolonged delivery a. What is the diagnosis? b. What one blood test would you check for?

a. What is the diagnosis? Subcutaneous fat necrosis b. What one blood test would you check for? Calcium Nelson's- Subcutaneous fat necrosis: - cause often unknown; may be caused by ischemic injury (pre-eclampsia, birth trauma, asphyxia) - inflammatory disorder of adipose tissue - occurs primarily in first 4 weeks of life - lesions are rubbery, erythematous to violaceous, plaques or nodules on cheeks, buttocks, back, thighrs, or upper arms - if uncomplicated, typically involute spontaneously within weeks to months - calcium deposition may occur within areas of fat necrosis - hypercalcemia - rare but life-threatening; manifests as lethargy, poor feeding, vomiting, FTT, irritability, seizures, or renal failure - Treatment: usually self-resolved; can help enhance renal calcium excretion with hydration and Lasix - limit dietary calcium & vit D, can give corticosteroids/pamidronate

1. 36 week baby 2.1 kg at birth, day 7 jittery, irritable on exam. HR 218. T 37.5. RR 70. BP 90/60. Face is flushed, eyes wide open, alert but irritable, normal tone and normal cry. Jaundiced. DTR's 2/4 and symmetric. There is hepatosplenomegaly. a. What is the likely diagnosis? b. List 2 tests to confirm your diagnosis.

a. What is the likely diagnosis? Neonatal Graves Disease b. List 2 tests to confirm your diagnosis. TSH, T4 Nelson's: Neonatal Graves disease - Caused by transplacental passage of thyroid autoantibodies (clinical course can be altered by presence of antithyroid medications) - Prenatal diagnosis: Fetal tachycardia, goiter - CP: o Goiters o Restless, irritable, appears anxious, unusually alert o Eyes open widely (appear exophthalmic) o Tachycardia, tachypnea, may have elevated temperature o Severe: weight loss, hepatosplenomealy, jaundice - Diagnosis: Elevated T3/T4, suppressed TSH - Treatment: oral propranolol and methimazole; usually remits spontaneously Neonatal hyperthyroidism (Graves) • Infants of mothers with Graves disease at risk(2%) • Clinical features: o Tachycardia and goiter may allow prenatal diagnosis o Prematurity, IUGR o Extremely restless, irritable, hyperactive, unsually alert o Microcephaly with ventricular enlargement, craniosynostosis common o Tachycardia, tachypnea and elevated temperature, hypertensive (may cause cardiac decompensation) o HSM, Jaundice • Diagnosis- T4 elevated, TSH suppressed • Treatment - Propranolol and PTU or methimazole; remits spontaneously within 6-12 weeks o If delayed can have osseous maturation, microcephaly and MR What is the likely diagnosis? Neonatal thyrotoxicosis List 2 tests to confirm your diagnosis. TSH suppressed Free T4 elevated N p 2336 Neonatal hyperthyroidism -Neonatal Graves -d/t transplacental passage of antithyroid drugs taken by the mother -mothers of these infants have active Graves, Graves in remission, or rarely hypothyroidism and a history of lymphocytic thyroiditis -occurs in only 2% born to women with Graves -fetal tachycardia and goiter may allow prenatal diagnosis -usu remits spontaneously w/I 6-12 wks -many are premature and have intrauterine growth restriction -extremely restless, irritable, hyperactive, appear anxious, unusually alert -microcephaly, ventricular enlargement -eyes open widely -extreme tachycardia and tachypnea, elevated temperature -ravenous appetite, poor wt gain -HSM, jaundice -severe hypertension, cardiac decompensation may occur -the infant may die if therapy is not instituted promptly -T4 is elevated, TSH is suppressed -advanced bone age -cranial synostosis are common -Rx -propranolol and PTU or methimazole -advanced osseous maturation, microcephaly, and mental retardation occur when treatment is delayed -intellectual development is normal in most treated infants with neonatal Graves disease

1. Newborn has significant respiratory distress and CXR that is consistent with pneumonia. She is ventilated with PIP 36, PEEP 5, rate 60, FiO2 1.0, she is not saturating very well. a. What is the likely diagnosis? b. What intervention should you start now?

a. What is the likely diagnosis? PPHN b. What intervention should you start now? iNO 1. a. What is the likely diagnosis? Persistent Pulmonary Hypertension of the Newborn b. What intervention should you start now? Inhaled NO, may require ECMO

10. You diagnose a child clinically with craniosynostosis. The child is not dysmorphic. a. What is the most common type of craniosynostosis? b. The xray confirms your diagnosis. What would your next step be?

a. What is the most common type of craniosynostosis? Sagittal (produces long narrow skull, scaphocephaly) - occurs most commonly in boys b. The xray confirms your diagnosis. What would your next step be? Consult neurosurgery for consideration of surgery - would be for cosmetic purposes only (fusion of only one suture rarely causes neurological deficit)

1. Newborn's TSH screen comes back as 25 (N less than 20). a. What is the most likely cause of a false positive screen (1 line). b. What is the most likely cause of a false negative screen (1 line) c. List 3 investigations to work up congenital hypothyroidism.

a. What is the most likely cause of a false positive screen (1 line). Collecting specimen too early b. What is the most likely cause of a false negative screen (1 line) Very sick infant or post-transfusion c. List 3 investigations to work up congenital hypothyroidism. TSH, FT4, T4

1. You are asked to see a term newborn in the delivery room after an uneventful delivery and resuscitation. The baby has a RR of 70 and is cyanotic. a. What is the most likely diagnosis? b. What one treatment will you start immediately?

a. What is the most likely diagnosis? Congenital Heart Disease b. What one treatment will you start immediately? Prostaglandin Causes of central cyanosis in the neonate: Right-to-left shunt Intracardiac level: cyanotic disease, anomalous systemic venous connection to left atrium Great vessel level: persistent pulmonary hypertension of the newborn Intrapulmonary level: pulmonary arteriovenous malformation Ventilation/perfusion mismatch Airway disease: pneumonia, aspiration, cystic adenomatoid malformation, diaphragmatic hernia, pulmonary hypoplasia, labor emphysema, atelectasis, pulmonary hemorrhage, hyaline membrane disease, transient tachypnea of the newborn Extrinsic compression of lungs: pneumothorax, pleural effusion, chylothorax, hemothorax, thoracic dystrophy Alveolar hypoventilation Central nervous system depression: asphyxia, maternal sedation, intraventricular hemorrhage, seizure, meningitis, encephalitis Neuromuscular disease: Werdnig-Hoffman disease, neonatal myasthenia gravis, phrenic nerve injury Airway obstruction: choanal atresia, laryngotracheomalacia, macroglossia, Pierre Robin syndrome Hemoglobinopathy Methemoglobinemia: congenital or secondary to toxic exposure Other hemoglobinopathies Diffusion impairment Pulmonary edema: left-sided obstructive cardiac disease, cardiomyopathy Pulmonary fibrosis Congenital lymphangiectasia a. What is the most likely diagnosis? TGA b. What one treatment will you start immediately? • Prostaglandin E1 (Alprostadil) infusion 0.05mcg/kg/min and titrate to lowest dose needed o MOA: Causes vasodilation by means of direct effect on vascular and ductus arteriosus smooth muscle o Side effects: Apnea (usually occurs within 1 hour of starting infusion), hypotension, flushing, fever, tachycardia ♣ Ensure you have second IV for resuscitation o Note: If patient deteriorates once starting Alprostadil might indicate a left sided obstructive lesion like: TAPVR with obstruction o Effect of PGE1 decreases after 96 hrs What is the most likely diagnosis? Transposition of the great arteries What one treatment will you start immediately? Prostaglandin E1 0.1mcg/kg/min as an intial dose

1. You are called to see a 2 day old newborn male with jitteriness. At birth he was found to have a cleft palate, but has been bottle feeding well since. On physical examination, you note the baby has a harsh systolic murmur. a. What is the most likely diagnosis? b. What is the reason for the baby's jitteriness?

a. What is the most likely diagnosis? DiGeorge Syndrome b. What is the reason for the baby's jitteriness? Hypocalcemia a. What is the most likely diagnosis? • DiGeorge syndrome o 22q11.2 deletion o Diagnosis: ♣ Cytogenetic study using FISH for detection of deletion of 22q11.2 o Characteristics: ♣ Congenital heart disease (74% • Conotruncal malformations: TOF, Interrupted aortic arch, VSD, Truncus arteriosus ♣ Palatal abnormalities (69%) • Velopharyngeal incompetence (VPI), submucosal cleft palate, and cleft palate • Significant feeding problems (30%) ♣ Characteristic facial features (present in the majority of Caucasian individuals) • Hypertelorism, downturning eyes, hooded eyelids, low- posteriorly rotated ears, widened area below nasal bridge, bulbous nose tip, micrognathia, short philtrum ♣ Learning difficulties (70-90%) ♣ Immune deficiency (77%) • Secondary to thymic hypoplasia ♣ Hypocalcemia (50%) ♣ Renal anomalies (37%) ♣ Other: hearing loss, laryngotracheoesophageal anomalies, growth hormone deficiency, autoimmune disorders, seizures, skeletal abnormalities (tapered fingers) CATCH 22 Cardiac defects, abnormal facies, thymus hypoplasia, cleft palate, hypokalemia, 22q11 microdeletion b. What is the reason for the baby's jitteriness? • Note: Jitteriness is defined as rhythmic tremors of equal amplitude, can be normal, can be initiated by external stimuli such as handling. It can be manifestation of hypoglycemia, hypocalcemia, drug withdrawal from maternal drug use, and perinatal asphyxia. In the infant with persistent or exaggerated tremors, possible causes must be investigated • Hypocalcemia o Muscle cramps o Carpopedal spasm (Trousseau sign) o Facial twitch (Chvostek sign) o Paresthesias o Tetany, seizures o Laryngeal stridor o ECG abnormalities (shortened QT, bradycardia, widened T waves) What is the most likely diagnosis? CATCH 22 syndrome (velocardiofacial syndrome) (diGeorge) What is the reason for the baby's jitteriness? Hypocalcemia

1. Neonate is jittery and has cleft palate and heart murmur. a. What is the reason for his jitteriness? b. What is the underlying condition?

a. What is the reason for his jitteriness? - Hypocalcemia b. What is the underlying condition? - DiGeorge Syndrome (Thymic Hypoplasia) Nelson's DiGeorge: o Thymic hypoplasia, anomalies of great vessels (right-sided aortic arch), CHD (conotruncal), bifid uvula (submucosal cleft), esophageal atresia, and facial features o Diagnosis often first suggested by hypocalcemic seizures in infancy (hypocalcemia secondary to hypoplasia of parathyroids)

1. Newborn weighs 1.8 kg. Jittery and found to have glucose 1.3. Repeat glucose is 0.8. a. What is this baby's glucose requirement in mg/kg/min? b. Write your IV order.

a. What is this baby's glucose requirement in mg/kg/min? 5.5mg/kg/min b. Write your IV order. Bolus 2mL/kg D10%, then D10% at 80mL/kg/day a. What is this baby's glucose requirement in mg/kg/min? 4-8 mg/kg/min b. Write your IV order: IV D10W @ 80cc/kg/day = 6ml/hr

1. Neonate born to mom who just revealed HIV positive status. a. What treatment(s) would you start this baby on (1 line) b. When would you start the treatment? (1 line) c. For how long would you treat? (1 line)

a. What treatment(s) would you start this baby on (1 line) Zidovudine x 6 weeks and 3 doses of Nevirapine - assume high-risk, Mom not on active treatment b. When would you start the treatment? (1 line) Within 6-12 hours of birth (no later than 72 hours) c. For how long would you treat? (1 line) 6 weeks CPS statement: urgent consultation with HIV expert; anti-retroviral therapy within 6-12 hours of birth (triple combination ART or Zidovudine for 6 weeks + 3 doses of Nevirapine in first week of life) Low Risk of Perinatal HIV Transmission Mothers received standard ART during pregnancy with sustained viral suppression near delivery and no concerns related to adherence 4 weeks of ZDV Higher Risk of Perinatal HIV Transmissiona,b Mothers who received neither antepartum nor intrapartum ARV drugs Mothers who received only intrapartum ARV drugs Mothers who received antepartum and intrapartum ARV drugs but who have detectable viral load near delivery, particularly if delivery was vaginal Mothers with acute or primary HIV infection during pregnancy or breastfeedingc Combination ARV prophylaxis with 6 weeks ZDV and 3 doses of NVP (prophylaxis dosage, with doses given within 48 hours of birth, 48 hours after first dose, and 96 hours after second dose) or Empiric HIV therapy consisting of ZDV, 3TC, and NVP (treatment dosage)d Presumed Newborn HIV Exposure Mothers with unknown HIV status who test positive at delivery or postpartum or whose newborns have a positive HIV antibody test ARV management as above (for higher risk of perinatal HIV transmission). ARV management should be discontinued immediately if supplemental testing confirms that mother does not have HIV. Newborn with Confirmed HIVe Confirmed positive newborn HIV virologic test/NAT 3 drug combination ARV regimen at treatment dosage Low Risk of Perinatal HIV Transmission Mothers received standard ART during pregnancy with sustained viral suppression near delivery and no concerns related to adherence 4 weeks of ZDV Higher Risk of Perinatal HIV Transmissiona,b Mothers who received neither antepartum nor intrapartum ARV drugs Mothers who received only intrapartum ARV drugs Mothers who received antepartum and intrapartum ARV drugs but who have detectable viral load near delivery, particularly if delivery was vaginal Mothers with acute or primary HIV infection during pregnancy or breastfeedingc Combination ARV prophylaxis with 6 weeks ZDV and 3 doses of NVP (prophylaxis dosage, with doses given within 48 hours of birth, 48 hours after first dose, and 96 hours after second dose) or Empiric HIV therapy consisting of ZDV, 3TC, and NVP (treatment dosage)d Presumed Newborn HIV Exposure Mothers with unknown HIV status who test positive at delivery or postpartum or whose newborns have a positive HIV antibody test ARV management as above (for higher risk of perinatal HIV transmission). ARV management should be discontinued immediately if supplemental testing confirms that mother does not have HIV. Newborn with Confirmed HIVe Confirmed positive newborn HIV virologic test/NAT 3 drug combination ARV regimen at treatment dosage https://aidsinfo.nih.gov/guidelines/html/2/pediatric-arv/510/antiretroviral-management-of-newborns-with-perinatal-hiv-exposure-or-perinatal-hiv

1. 10 day old baby with failure to thrive, jaundice, hepatomegaly, blood culture positive for E.Coli. a. What underlying disorder may the child have? b. What test can you do to confirm this diagnosis?

a. What underlying disorder may the child have? Galactosemia b. What test can you do to confirm this diagnosis? RBC GALT

1. 10 day old baby with failure to thrive, jaundice, hepatomegaly, blood culture positive for E. coli. a. What underlying disorder may the child have? b. What test can you do to confirm this diagnosis?

a. What underlying disorder may the child have? Galactosemia b. What test can you do to confirm this diagnosis? RBC GALT/RBC galactose 1 phosphate uridyl transferase Think E. Coli-ctosemia (E. Coli)

1. A newborn baby has a brachial plexus injury after a traumatic birth. a. What would you tell his mother about his prognosis? b. You see the baby in one month and there is no improvement in his arm movements. What would you do now?

a. What would you tell his mother about his prognosis? Most resolve within 1-3 months (75% by 1 month) b. You see the baby in one month and there is no improvement in his arm movements. What would you do now? If no improvement - rehab to surgeons, plastics, OT, PT

1. Baby with PPHN, 7.32, pCo2 48, Bicarb 18, fio2 0.8, sat 91 preductal, 82 postductal a. What's your diagnosis? b. What 4 things can you do while waiting for the transport team:

a. What's your diagnosis? PPHN b. What 4 things can you do while waiting for the transport team: - Intubate & Optimize ventilation (avoid hypercarbia, give oxygen) - Keep baby calm/sedated (paralysis is controversial!) - Start iNO - CXR

The prevention of pain is important not only because it is an ethical expectation, but also because of potential deleterious consequences of repeated painful exposures. These consequences include

altered pain sensitivity (which may last into adolescence) and permanent neuroanatomical and behavioural abnormalities, as found in animal studies. CPS Prevention and management of pain in the neonate

Blood for neonatal transfusion is often issued as group ____ packed RBCs with compatible infant Rh type Alternatively, non-group ___ infants may receive nongroup ___ RBCs if passive maternal ______ is not detected in an infant's serum or plasma

anti-A or anti-B O = Open to all and no pesky Rh to cause problems All birthing centres must be supported by transfusion services prepared to issue uncross-matched group O Rh-negative blood in a perinatal emergency. The hematocrit of packed RBCs with buffy coat preparation is approximately 60%. CPS Red blood cell transfusion in newborn infants

If GBS status is unknown, intrapartum antibiotic prophylaxis should be offered if ______.

any other risk factors (Table 1) are present CPS term infants at increased risk for early onset bacterial sepsis

Enteral supplementation may be used in _____ with blood glucose levels of _____ to augment caloric intake, rechecking levels in 60 min to identify persistent hypoglycemia

asymptomatic infants 1.8 mmol/L to 2.5 mmol/L CPS Hypoglycemia

Threshold ROP is defined as

at least five contiguous or eight cumulative clock hours of stage 3 ROP in zones I and II in the presence of plus disease Threshold ROP, as well as more severe forms of prethreshold ROP, require treatment and are incorporated in the type 1 ROP category. 5x3 = 8 = (eight takes the cake, threshold to tx) CPS ROP

Adequate intrapartum antibiotic prophylaxis consists of _____

at least one dose given at least 4 h before birth of: • IV penicillin G (initial dose 5 million units) or ampicillin (initial dose 2 grams) OR • IV cefazolin (initial dose 2 grams) if the mother is allergic to penicillin but at low risk for anaphylaxis CPS term infants at increased risk for early onset bacterial sepsis

Asymptomatic, at-risk babies should receive ______ before a blood glucose check at ____ of age and should be encouraged to feed regularly thereafter.

at least one effective feed 2 h CPS Hypoglycemia

When should asymptomatic hypoglycemia be treated?

at risk infants Bg <2.6 Evidence to say BG <2, asymptomatic, at risk causes long term developmental outcomes (worse the longer someone is hypoglycemic). IDMs can handle lower (1.5). Some say <2 is a problem but when intervening, aim for 2.6 CPS Hypoglycemia

The oxygen available to neonatal tissue is lower than that in adults, but a neonate's erythropoietin response is ______ for the degree of anemia and, as a result, hemoglobin and reticulocyte levels are _____

attenuated low. Neo Notes

32. 3 week male infant with vomiting, lethargy, decreased po intake. Na 118, K 8. Most important lab test? a) Vasopressin b) 17-O-HP c) Cortisol d) Calcium e) Renin

b) 17-O-HP - salt-wasting CAH Hyponatremia and Hyperkalemia suggests Adrenal Insufficiency: • Due to Congenital Adrenal Hyperplasia • Clinical manifestations: o Result from inadequate secretion or action of glucocorticoids, mineralocorticoids or both ♣ Depending on site of enzymatic defect, also can have over or underproduction of Androgens o Dominant feature = hyponatremia and hyperkalemia ♣ Vomiting, dehydration and acidosis ♣ Hypotensive shock o In females: ambiguity of external genitalia is an obvious clue; may go undiagnosed in males until they present with adrenal crisis • Deficiency of 21-Hydroxylase is the most common form (see Endo section) diagnosed with elevated 17-OHP levels B R/O CAH - most like 21-hydroxylase deficiency Vasopressin should not cause hyperkalemia Cortisol deficiency could give rise to a similar picture but even with a low level this still could be CAH. Therefore need a definitive test. Hypo/Hyper-calcemia do not fit this clinical picture. Renin again would not yield a definitive diagnosis

26. Expected survival rate for an infant born at 25 weeks gestation: a) > 90% b) 75% c) 50% d) 25% e) < 10%

b) 75% Predicting Neonatal Mortality: • 23 weeks - 15 % • 24 weeks - 56% • 25 weeks - 79% • Increased chance of survival - antenatal steroids, female sex and singleton pregnancy • Birthweight-specific neonatal diseases (IVH, GBS, Pulmonary hypoplasia) also contribute to poor outcome o Scoring systems - Score for Neonatal Acute Physiology (SNAP) or Clinical Risk Index for Babies (CRIB) collected in first 24 hours of life can be used to help provide more accurate assessment of the risk of death B CPS Statement: Management of the woman with threatened birth of an extremely low gestational age infant "Canadian statistics are reported by the Canadian Neonatal Network (CNN) for infants admitted to 29 Canadian NICUs. From 2005 to 2010, of a total of 2893 infants, survival was 8% at ≤ 22 weeks, 36% at 23 weeks, 62% at 24 weeks and 78% at 25 weeks. Note that these data do not include delivery room deaths or infants not admitted to a CNN site."

69. What is an indication of seizure activity in a neonate? a) Tachycardia b) Abnormal eye movements c) Irregular breathing d) Irritability e) Vomiting

b) Abnormal eye movements B Staff neo -abnormal eye movements, irregular breathing and apnea are not specific enough, abnormal eye movements if seen in isolation would make you think seizure

58. You are examining a newborn infant, born at full term 2600g. He has a flat nasal bridge, a thin upper lip and a smooth philtrum. He is mildly hypotonic. Which of the following ingestions is it important to inquire about in your history of the mother's pregnancy: a) Cocaine b) Alcohol c) Marijuana d) Heroin

b) Alcohol - small, thin upper lip, smooth philtrum, hypotonia - Nelson's: Characteristics of fetal alcohol syndrome include prenatal onset and persistence of growth deficiency; Facial abnormalities including short palpebral fissures, epicanthal folds, maxillary hypoplasia, micrognathia, smooth philtrum, and thin smooth upper lip; cardiac defects (septal defects); minor joint and limb abnormalities; developmental delay/mental retardation Features described suggest FAS see genetics notes! FAS B FETAL ALCOHOL SYNDROME ♣ Caused by the teratogenic effects of maternal excessive alcohol ingestion during gestation. ♣ Pre- and postnatal growth deficiency. Microcephaly, FTT, short palpebral fissures. Long smooth philtrum, thin upper lip. Cognitive and behavioural delay.

37. Ex-30 week prem, now at 39 weeks corrected has had an uneventful course. He had apnea of prematurity treated with caffeine. He is still having apneas with occasional bradycardia 5-6 times a day. What is the likely cause? b) Apnea of prematurity c) Seizures d) GERD

b) Apnea of prematurity GERD may also occur in infants with apnea of prematurity • No support for a causal relationship • Use of antireflux medications has not been shown to decrease frequency of apnea A N18 p 729 Apnea of prematurity ♣ The immaturity of the brainstem respiratory centers is manifested by an attenuated response to carbon dioxide and a paradoxical response to hypoxia that results in apnea rather than hyperventilation ♣ AOP usuallyresolvesby36weeks, canhaveprolonged,up to44weeks? ♣ AOP usually resolves by 36 wk of postconceptional age (PCA) and does not predict future episodes of sudden infant death syndrome (SIDS). ♣ Some infants with persistent apnea are discharged as long as cardiorespiratory monitoring can be performed at home. ♣ In the absence of significant events, home monitoring can be safely discontinued after 44 wk PCA. Seizures-Apnea can mimic Sz but AOP is more common and therefore more likely. GERD - may also occur in infants with apnea of prematuriy. data do no support a causal relationship b/w gastroesophageal reflux and apneic events or the use of antireflux medications to reduce the frequency of apnea in preterm infants

75. 2 month old baby with poor feeding, hepatomegaly, crackles, soft systolic murmur. What is the most likely diagnosis? a) Congenital infection b) CHF c) Sepsis

b) CHF

68. Full term baby delivered after traumatic forceps delivery. Now 1 month old with vomiting, lethargy, and red plaque on back of hand. What lab test would you check? a) Glucose b) Calcium c) Potassium d) ALP e) Creatinine

b) Calcium Subcutaneous fat necrosis • cause unknown: o May be due to ischemic injury under various circumstances, such as maternal preeclampsia, birth trauma, asphyxia, and prolonged hypothermia o Often no provocative factors are not identified • Occurs primarily in the 1st 4 wks of life in full-term or post-term infants • Clinically lesions are asymptomatic, rubbery to firm, erythematous to violaceous plaques or nodules on the cheeks, buttocks, back, thighs, or upper arms o May be focal or extensive and are generally asymptomatic, although they may be tender during the acute phase. • Histopathologic changes are diagnostic and consist of necrosis of fat o Granulomatous cellular infiltrate composed of lymphocytes, histiocytes, multinucleated giant cells, and fibroblasts; and radially arranged clefts of crystalline triglyceride within fat cells and multinucleated giant cells. o Calcium deposits are commonly found in areas of fat necrosis and this may sometimes result in rupture and drainage of liquid material. • A rare but potentially life-threatening complication is hypercalcemia o Presents at 1-6 mo of age with lethargy, poor feeding, vomiting, failure to thrive, irritability, seizures, shortening of the QT interval, or renal failure o The origin of the hypercalcemia is unknown. o Treatment of hypercalcemia: ♣ Increase renal calcium excretion (hydration and furosemide) ♣ Limit dietary calcium and vitamin D intake • Corticosteroids • DDx: sclerema neonatorum, panniculitis, cellulitis, or hematoma. • Treatment: o Usually self-limited no therapy required ♣ Uncomplicated lesions involute spontaneously within weeks to months, usually without scarring or atrophy. o Needle aspiration of fluctuant lesions may prevent rupture and subsequent scarring.

54. 1 mo 2 d old with a vesicle noted on the upper lip? a) Sucking blister (present at birth, resolve rapidly) b) HSV c) Varicella d) Epidermolysis bullosa

b) HSV CPS Statement - initial symptoms of NHSV present within first four weeks of life; can present up to six weeks after birth - SEM disease - neonatal varicella acquired from mother usually occurs in first 1-2 weeks - "sucking pads (calluses)" are found on lips in first few months, due to combined intracellular edema and hyperkeratosis; should be distinguished from sucking blisters Not a) Sucking blister (present at birth, resolve rapidly) c) Varicella d) Epidermolysis bullosa - heterogeneous group of congenital, hereditary blistering disorders that are characterized by induction of blisters by trauma and exacerbation of blistering in warm weather • Sucking Blister: o Solitary or scattered superficial bullae present at birth on the upper limbs of infants o Presumably induced by vigorous sucking on the affected part in utero. o Common sites: radial aspect of the forearm, thumb, and index finger o Resolve rapidly without sequelae • Sucking Pad: o Calluses which are found on the lips in the first few months of life are due to suckling o Caused by combined intracellular edema and hyperkeratosis. Usually occur on labial tubercle of upper lip • HSV: o HSV 2 predominant cause of neonatal disease (75-80%) o Clinical manifestations ♣ Three categories of disease • Localized to skin, eye and mouth • Encephalitis with or without localized mucocutaneous disease • Disseminated infection with multiple organ involvement o Skin, eye and mouth ♣ 50% have localized disease ♣ Vesicles appear on 6-9th day of life ♣ Clusters of vesicles often develop on presenting part of body ♣ Significant morbidity can occur with up to 10% later showing neurologic impairment despite having no evidence of disseminated disease o CNS Infection ♣ 1/3 present with encephalitis in the absence of disseminated disease ♣ As many as 60% of these infants do not have mucocutaneous vesicles ♣ 10-14 days of life develop lethargy, seizures, temp instability, hypotonia ♣ Mortality 15% with treatment ♣ 2/3 surviving infants impaired neurodevelopmental outcomes o Disseminated infection ♣ Most severe, 50 percent mortality ♣ Accounts for 22% of cases ♣ Present within 7 days of life. ♣ Seizures, encephalitis, shock, resp distress, DIC, resp failure ♣ 40% have long term morbidity

15. An ex 1500 g premature infant presents with a prolonged apnea spell at 5 weeks of age. Extensive laboratory workup is unremarkable. On sending the parents home, you suggest: a) Infant should always sleep on side b) Infant should sleep in parents room c) Infant should sleep in own room, but parents should check on infant hourly d) An audio monitor should be placed in infant's room e) Parents should not use heavy blankets in the crib

b) Infant should sleep in parents room (6 months per CPS statement) or e) Parents should not use heavy blankets in the crib Newborn sleep advice/SIDS prevention: • Position: Supine, side sleeping is not recommended o Tummy time while awake and observed o Alternating the placement of the infant's head as well as his or her orientation in the crib to reduce positional plagiocephaly • Location: in the same room as their parents but in their own crib or bassinette o close to mom to facilitate nursing and contact • Environment: o sleep on a firm mattress o avoid soft materials: pillows, comforters, quilts, sheepskins, cushion-like bumper pads, and stuffed toys o loose bedding may be hazardous; blankets, if used, should be tucked in around the crib mattress o avoid overheating and overbundling • Devices advertised to maintain sleep position, "protect" a bed-sharing infant, or reduce the risk of rebreathing are not recommended • Home respiratory, cardiac, and O2 saturation monitoring may be of value for selected infants who have extreme instability, but there is no evidence that monitoring decreases the incidence of SIDS and it is therefore not recommended for this purpose. • Consider offering a pacifier at bedtime and naptime. The pacifier should be used when placing the infant down for sleep and not be reinserted once it falls out o For breast-fed infants, delay introduction of the pacifier until breast-feeding is well established. • Mothers should not smoke during pregnancy and infants should not be exposed to secondhand smoke. a. Infant should always sleep on side - NO supine b. Infant should sleep in parents room - YES, CPS recommends co-sleeping but not bedsharing for the 1st 6 months c. Infant should sleep in own room, but parents should check on infant hourly - NO d. An audio monitor should be placed in infant's room - NO e. Parents should not use heavy blankets in the crib - YES Staff Neo would go with e since it is so wrong Ask Neo please

11. Baby born to heroin addicted Mom. Apneic despite bag and mask ventilation. Best management? a) Narcan b) Intubate until baby breathes on his own c) Bag and mask ventilation until baby breathes on his own

b) Intubate until baby breathes on his own • Failure to initiate or sustain respiratory effort is common at birth. o primary apnea: respond to stimulation o secondary apnea: need ventilatory assistance; ♣ originates in the CNS as a result of asphyxia or peripherally because of neuromuscular disorders. ♣ Prematurity alone is seldom a causative factor, except in infants weighing <1,500 g. ♣ Intrapulmonary problems may result in poor ventilation despite strong respiratory efforts. • Narcosis: o results from administration of morphine, meperidine, fentanyl, barbiturates, or tranquilizers to the mother shortly before delivery or from maternal anesthesia given during the 2nd stage of labor. o Treatment: ♣ initial physical stimulation and securing of a patent airway. ♣ If effective ventilation is not initiated, artificial breathing with a bag and mask must be instituted. ♣ if the respiratory depression is due to an opiate, Narcan 0.1mg/kg IV or IM ♣ Naloxone is contraindicated in infants born to mothers with opiate addiction because it precipitates acute neonatal withdrawal with severe seizures. a. Narcan - No, as will precipitate acute withdrawal and seizures b. Intubate until baby breathes on his own - has failed bag and mask ventilation c. Bag and mask ventilation until baby breathes on his own B Narcan is contraindicated in infants born to mothers with opiate addiction because it precipitates acute neonatal withdrawal with severe seizures. - N18 p723

28. In utero exposure to cocaine results in which of the following in the newborn: a) Hearing deficits b) Microcephaly c) Hypotension d) Spinal dysraphism

b) Microcephaly - spontaneous abortion, fetal demise, placental abruption, prematurity/preterm labour, LBW, SGA, reduced fetal length and head size Maternal cocaine use is associated with: • Preterm labour • Abruptio placentae • Neonatal irritability • Decreased attentiveness • Small for gestational age • Microcephaly Usually it requires no treatment. Effects of cocaine on the fetus: Microcephaly, LBW, IUGR, behavioral disturbances, ?association with Gastroschisis

46. Newborn weighs 4.5kg. He is plethoric and lethargic with poor feeding. Venous bloodwork shows hemoglobin of 170, Hct of 0.72, and glucose of 3.4, with rest of bloodwork normal. What do you do? a) Give 4cc/kg D10W b) Partial exchange transfusion c) Start antibiotics

b) Partial exchange transfusion - Nelson's: Infant polycythemia defined as Hct >0.65 (central; heel-poke value will be higher than central) - clinical manifestations include lethargy, feeding disturbances and more - Treat with partial exchange transfusion if Hct >.7-.75, or symptoms of hyperviscosity present Symptomatic hyperviscosity (>70%) Calculation: Vol to exchange = 72 - 50 / 72 x 360 = 110ml NS Refer to Q39

71. 2 month old ex 32 weeker who was ventilated with Ua/Uv lines. Now has HSM but has an otherwise normal physical exam. What is the likely cause? a) Hepatic hemangioma b) Portal vein thrombosis c) Hereditary spherocytosis d) Congenital CMV e) Fungal infection

b) Portal vein thrombosis Not a) Hepatic hemangioma - hepatomegaly c) Hereditary spherocytosis - splenomegaly • Increased risk for PVT in infants that have had umbilical catheterization and severe illness o can be asymptomatic or have symptoms of portal hypertension including HSM • Hepatic hemangioma = abdo mass, HSM, jaundice, pneumonia, multiple skin hemangiomas and cardiac insufficiency • Hereditary spherocytosis = mild pallor, jaundice, splenomegaly, usually no hepatomegaly • Congenital CMV = microcephaly, periventricular calcifications, IUGR, retinitis, blueberry muffin-like rash, sensorineural hearing loss, hepatitis, hyperbilirubinemia, thrombocytopenia

47. 12 h old newborn has a sat of 80%. Increases to 85% with 100% O2. Mild tachypnea, RR 65. CXR has no abnormalities. What next initial mngt should be done? a) Intubate and ventilate b) Prostaglandins c) Antibiotics

b) Prostaglandins - Nelson's: Hyperoxia test - neonates with cyanotic congenital heart disease are not able to raise arterial PaO2 during administration of 100% oxygen; exclude intracardiac right-to-left shunt if PaO2 >150mmHg (not perfect) - no specific value for percent saturation change, but expect more significant than 5% Failed hyperoxia test: if PaO2 increase < 30mmHg, PaO2 < 70 mmHg when giving 100% oxygen for 10 mins • Congenital heart disease (cyanotic lesions) • Primary Pulmonary Hypertension of the Newborn If worried about cardiac disease - start Prostaglandins If concerned may be PPHN - intubate and ventilate (probably reading too much into this one) Failed hyperoxia test c/w CHD PaO2Greaterthan250mmHg-nocardiaclesion 100250ismixing Lessthan100RotoLshuntinghighlypredictiveofCHD N18Ch423

72. 3 week old with previous E. coli sepsis and persistent jaundice. What is the likely problem? a) Increased osmotic fragility b) RBC galactose phosphate uradyl transferase deficiency c) RBC glucose 1 phosphate dehydrogenase deficiency

b) RBC galactose phosphate uradyl transferase deficiency

36. What is the number one cause of central apnea at 8 weeks of age? a) Seizure b) RSV c) Apnea of prematurity

b) RSV B "Fever is an inconstant sign in RSV infection. In young infants, particularly those who were born prematurely, periodic breathing and apneic spells have been distressingly frequent signs, even with relatively mild bronchiolitis. Apnea is not necessarily caused by respiratory exhaustion, but rather appears to be a consequence of alterations in central control of breathing." - Nelson's chapter 252 (clinical manifestions) CPSStatementsRiskfactorsforsevereRSV Apnea • Many causes, including Apnea of Prematurity • Cessation of pulmonary airflow for a specific time interval (usually 10-20 sec) o Central - complete cessation of airflow and respiratory efforts with no chest wall movement o Obstructive - absence of noticeable airflow but with the continuation of chest wall movements o Mixed - most frequent type • Apnea of prematurity usually resolves by 36-40 weeks postconceptional age (GA + [postnatal age) o Preterm infants respond paradoxically to hypoxia by developing apnea rather than by increasing respirations (as do mature infants), also attenuated response to carbon dioxide o Methylxanthines (caffeine) are the mainstay of treatment • Acute bronchiolitis (RSV) "Apnea may be more prominent than wheezing early in the course of the disease, particularly with very young infants (<2 mo old) or former premature infants"

45. Newborn with axillary temperature of 37.8, well, normal exam. What do you do? a) Full septic work up and antibiotics b) Rectal temperature c) Take off all clothes for 20 minutes and recheck temperature d) CBC and differential

b) Rectal temperature CPS Statement: Fever in newborn axillary = above 37.5; axillary = screening, gold standard = rectal ** see above, except temp is now a little higher... consider doing a rectal temperature (gold standard in this age group)

44. What is true about surfactant therapy in premature infants? a) Use of surfactant decreases morbidity b) Use of surfactant decreases mortality c) There is no benefit

b) Use of surfactant decreases mortality CPS Statement: "Surfactant therapy reduces mortality, and several aspects of morbidity in babies with RDS" - increases likelihood of surviving without BPD, largely by improving survival, rather than incidence of BPD Surfactant replacement therapy: • One of the major advances in the care of preterm infants • Prophylactic and rescue administration have reduced adverse outcomes o Including mortality ♣ Specifically reduced with prophylactic (also reduceds air leak) • No reduction in the rates of BPD (probably due in part to the survival of infants with severe RDS who would have died without surfactant adminstration

5. Kleihauer-Betke Test—how does it work. a) test baby for mom's blood b) test mom for baby's blood c) test cord blood

b) test mom for baby's blood Transplacental hemorrhage: • bleeding from the fetal into the maternal circulation • Kleihauer-Betke test: demonstration of significant amounts of fetal hemoglobin and red blood cells (RBCs) in maternal blood on the day of delivery (quantitative) • Fetal hemoglobin F: has gamma instead of beta chains o After the 8th wk: Hb F is the predominant hemoglobin o 24 wk: constitutes 90% of the total hemoglobin. o During the 3rd trimester: a gradual decline occurs o Synthesis of Hb F decreases rapidly postnatally o by 6-12 mo of age, only a trace is present • Apt test: (qualitative) o Can be used if vaginal bleeding to determine if fetal in origin o Can be used in baby if blood stools/NG aspirate to test if fetal or maternal o Positive = fetal, negative = maternal Kleihauer-Betke test detects the precense of fetal cells in the maternal circulation. From the % of fetal RBCs and the estimated maternal blood volume, the size of the hemorrhage can be determined. 1% fetal cells in the maternal circulation indicates a bleed of about 50ml. PS 444

82. Mom at 28 wks gestation. Household has husband and a 2 and 5 year old sibling. She ask for best advice on prevention of influenza in her future baby for the first 6mo. a. vaccinate mom with inactivated vaccine after delivery b. vaccinate mom now with inactive vaccine c. vaccinate husband and the kids

b. vaccinate mom now with inactive vaccine CPS statement for 2016/2017 season - vaccinate pregnant women in all trimesters to protect them & fetus; live vaccine not recommended as theoretical risk to fetus

what type of seizure syndrome: 5th day of life, apneic and focal motor seizures, good response to medications and a good prognosis.

benign idiopathic neonatal seizures (fifth day fits) Neo Notes

The _____ diameter is used to assess gestational age beginning in the 2nd trimester.

biparietal Bi (2) the 2nd trimester, brain is measured to see if things are on PAR(ietal) for the course for growth Neo Notes

The use of premedication in intubation reduces the adverse physiological responses of

bradycardia, systemic hypertension, intracranial hypertension and hypoxia. Perhaps more importantly, premedication decreases the pain and discomfort associated with the procedure. CPS Intubation

60. What advice should you give to parents to prevent positional plagiocepaly: a) Children should be put to sleep in car seats b) Children should be put to sleep on their sides c) Children should be put on their stomachs as much as possible when awake

c) Children should be put on their stomachs as much as possible when awake • Positional plagiocephaly prevention: o Alternate between placing head to the foot or to the head of the bed encouraging them to lie on the side that allows them to look around the room o Tummy time 3x/day 10-15min each N18p1741 Infants should have some time in the prone position (tummy time) while awake and observed. Alternating the placement of the infant's head as well as his or her orientation in the crib can also minimize the risk of head flattening from supine sleeping (positional plagiocephaly). AandBarebothNOTrecommendedasincreasedriskofSIDS. CPSstatement: Recommendationsforsafesleepingenvironmentsforinfantsandchildren.

67. Baby with symmetric IUGR. Cause? a) Smoking b) Preeclampsia c) Congenital infection d) Placental insufficiency

c) Congenital infection C - only cause of symmetric IUGR NelsonsChapter91.2PrematurityandIUGR Table914FactorsassociatedwithIUGR Fetal • Chromosomeabn • Chronicinfection • Congenitalabn • Irradiation • Multiplegestation • Pancreatichypoplasia • Insulindeficiency • InsulinlikeGHtype1deficiency Placental • Decplacentalwt,cells,orboth • Decsurfacearea • Placentitis • Infarct • Tumour • Placentalseparation • TwintwinTxsyndrome Maternal • Toxemia • HTN,renaldz • Hypoxemia(highaltitude,maternalcardiac/lungdz) • Malnutrition • Chronicillness • Sicklecell • Drugs(narcotics,EtOH,cigarettes,cocaine,antimetabs)

66. A term newborn is born to a heroin addicted Mom. On initial assessment the baby is apneic but HR is greater than 100. After 30 seconds, the baby continues to be apneic despite adequate bag and mask ventilation. What is the next step in your management? a) IM Narcan b) Intubate and give Narcan down the ETT c) Continue to support ventilation until baby breathes on his own d) Epinephrine 1:10,000 -no indication

c) Continue to support ventilation until baby breathes on his own

77. Which of the following predisposes to late hemorrhagic disease of the newborn: a) Breastfeeding b) Prematurity c) Cystic fibrosis d) Maternal phenytoin e) Oral antibiotics

c) Cystic fibrosis - LATE disease associated with malabsorption with vitamin K Hemorrhagic Diseases of the Newborn EARLY ONSET CLASSIC DISEASE LATE ONSET Age 0-24 hr 2-7 days 1-6 mo Site of hemorrhage Cephalohematoma Gastrointestinal Intracranial Subgaleal Ear-nose-throat-mucosal Gastrointestinal Intracranial Intracranial Cutaneous Gastrointestinal Circumcision Ear-nose-throat-mucosal Umbilicus Cutaneous Injection sites Intra-abdominal Gastrointestinal Thoracic Injection sites Etiology/risks Maternal drugs (phenobarbital, phenytoin, warfarin, rifampin, isoniazid) that interfere with vitamin K Vitamin K deficiency Cholestasis Malabsorption of vitamin K (biliary atresia, cystic fibrosis, hepatitis) Inherited coagulopathy Breast-feeding Abetalipoprotein deficiency Idiopathic in Asian breast-fed infants Warfarin ingestion Prevention Treatment Posible vitamin K at birth or to mother (20 mg) before birth Prevented by parenteral vitamin K at birth. Oral vitamin K regimens require repeated dosing over time Prevented by parenteral and high-dose oral vitamin K during periods of malabsorption or cholestasis Avoid high-risk medications Incidence Very rare ≈2% if not given vitamin K Dependent on primary disease

21. Which of the following is associated with polyhydramnios: a) IUGR b) Hirschsprung's disease c) Esophageal atresia d) Renal agenesis

c) Esophageal atresia Polyhydramnios: defects that interfere with fetal swallowing and/or absorption of fluid ♣ gastrointestinal obstruction due to duodenal, esophageal, or intestinal atresia ♣ neuromuscular disorders, such as anencephaly or myotonic dystrophy ♣ Increased urine production may occur with a high cardiac output state, or, rarely, from entities such as fetal Bartter syndrome ♣ Other causes: Maternal diabetes, Fetal anemia, Multiple gestation Associations with Polyhydramnios: ♣ Diabetes ♣ Hydrops fetalis ♣ Multiple gestation Trisomy 18 / 21 ♣ Anencephaly ♣ Hydrocephaly ♣ Meningomyelocele ♣ Esophageal and duodenal atresia ♣ Congenital diaphragmatic hernia ♣ Cleft palate ♣ Werdnig-Hoffmann syndrome (SMA) ♣ Beckwith-Weidemann syndrome (BWS) ♣ Conjoined twins ♣ Cylothorax ♣ Cystic adenomatoid lung malformation (CCAM) ♣ Gastroschisis ♣ Sacral teratoma ♣ Myotonic dystrophy ♣ Placental chorioangioma

51. You are seeing a 1 week old Chinese boy with 1 day of jaundice. His bili is 270 (mostly indirect), Hgb 95, retics 9%. Mom is AB+ and he is B+. He otherwise looks well. What is the diagnosis? a) Sepsis b) Thalassemia c) G6PD d) ABO incompatability

c) G6PD Not d) ABO incompatability (Mom can't be AB+, usually O, or could be A or B, and have maternal antibodies against other) - unconjugated hyperbilirubinemia, hemolysis - thalassemia - microcytic anemia, less commonly hyperbilirubinemia Sepsis should always be on the differential for jaundice. ABO incompatabilitiy is not possible in this case (Mom is AB+ and therefore should not have B antibodies, both are Rh +) G6PD Deficiency • X-linked, over 140 mutations described o 4.9 percent global prevalence o African Americans, Mediterranean, Asians • G6PD catalyzes conversion of Glucose 6-phosphate to 6-phosphogluconic acid o This reaction produces NADPH which maintain glutathione in the reduced state protecting cells from oxidant threats from certain drugs and infx that would otherwise cause precipitation of hemoglobin (Heinz bodies) or damage the RBC membrane • Two clinical syndromes: o Episodic hemolytic anemia o Chronic nonspherocytic hemolytic anemia • Most common manifestations: o Neonatal jaundice o Episodic acute anemia induced by infections, certain drugs, and, rarely, fava beans. ♣ Asymptomatic, but hemolysis ensues in 24-48 hr after ingestion of oxidative substance • Triggers o Drugs aspirin, sulfonamides, rasburicase, and antimalarials (primaquine) • Fava beans contain compounds that lead to production of hydrogen peroxide • Laboratory findings: o Anemia secondary to hemolysis o Haptoglobin saturated with Hgb o Free Hgb may appear in the plasma and urine if haptoglobins become saturated o Heinz bodies - precipitated Hgb, seen only within the first 3-4 days of illness because they are rapidly cleared from the blood o Smear: Bite cells, plychromsia representing reticulocytosis • Diagnosis o Demonstration of reduced activity of G6PD in RBCs Direct measurement of enzyme activity is <10% of normal o During an acute episode may get falsely low normal results as young RBCs and reticulocytes have higher enzyme activity Glucose-6-phosphate dehydrogenase deficiency ▪ Most common hemolytic anemia caused by an RBC enzymatic defect. ▪ The enzyme G6PD is a key component of the pentose phosphate pathway, which ordinarily generates sufficient nicotinamide adenine dinucleotide phosphate hydrogen (NADPH) to maintain glutathione in a reduced state. ▪ Reduced glutathione protects sulfhydryl groups in the RBC membrane from oxidation. ▪ The deficiency is inherited in an X-linked recessive fashion. In patients who are deficient (most commonly those of African, Mediterranean, or Asian ancestry), oxidant stresses (particularly certain drugs or SBI) can result in hemolysis. ▪ When a patient with G6PD is exposed to significant oxidant stress, hemoglobin is oxidized forming precipitates of sulhemoglobin (Heinz bodies), which are visible on specially stained preparations. ▪ The most common variants of G6PD deficiency have been found in areas where malaria is endemic. ▪ G6PD deficiency protects against parasitism of the erythrocyte. ▪ The RBC morphology during episodes of acute hemolysis is striking: o RBCs appear to have "bites" (cookie cells) taken out of them. o These are areas of absent hemoglobin that are produced by phagocytosis of Heinze bodies by splenic macrophages. o As a result, the RBCs appear blistered. o Clinically evident jaundice, dark urine resulting from bilirubin pigments, hemoglobinuria when hemolysis is intravascular, and decreased haptoglobin levels are common during hemolytic episodes o Early on, the hemolysis usually exceeds the ability of the bone marrow to compensate, so the reticulocyte count may be low for 3 to 4 days. ▪ The amount of G6PD enzymatic activity depends on the age of the RBC. Older RBCs have the least, and reticulocytes have the most. ▪ In an acute hemolytic episode, the older cells are destroyed first; younger ones may remain, and reticulocytes may increase. ▪ If G6PD levels are measured at this point, the result may be misleadingly near or above the normal range. ▪ If clinical suspicions remain, repeating the test when the reticulocyte count is reduced (i.e. patient is in a steady-state condition) will give a more accurate measurement. Alternatively testing the mother or performing electrophoresis to identify the precise variant present aids the diagnosis. ▪ The treatment of G6PD deficiency is supportive. EN p570-571, PS p320

52. Baby 6 days old presents in shock. Glucose 1.6 and cardiomegaly on CXR. What is the most likely etiology of the shock? a) Congenital heart disease b) Overwhelming sepsis c) IEM

c) IEM - severely ill newborn infant - think sepsis or inborn error of metabolism - glucose dysregulation can occur as a manifestation of stress with any form of shock (can occur with sepsis, heart failure, shock, inborn error of metabolism) -primary literature: 'comparing neonates presenting with bacterial sepsis/meningitis, or left-sided obstructive lesions' - cardiomegaly is strongly specific, and quite sensitive for CHD - inborn errors of metabolism causing cardiomyopathy - fatty acid oxidation defects, glycogen storage diseases (most common), lysosomal storage disorders - ultimately chose IEM because CHD less likely to cause 'shock' at 6 days • Hypoglycemia more common in CHO and FFA oxidation disorders • More common disorders with hypoglycemia are: o hepatic glycogen storage diseases (hypoglycemia, hepatomegaly, lactic acidosis) o Pompe Disease - macroglossia, hypotonia, cardiomegaly with congestive heart failure, and hepatomegaly. **Cardiomegaly (hypertrophic cardiomegaly) is the most striking feature and may be apparent in the neonatal period, NOT hypoglycemia because glycogen accumulates within lysosomes as a result of deficiency of the enzyme acid maltase BUT cytoplasmic glycogen is NORMAL o galactosemia and hereditary fructose intolerance, although symptoms of the latter disorder occur only after fructose (sucrose) has been introduced in the diet B Hypoglycemia, shock (likely academia), and cardiomegaly ?which IEM initially though GSD II (Pompe) but hypoglycemia is not a feature although cardiomegaly is ?BWS, present with hypoglycemia in neonatal period but ??shock Galactosemia - if shock secondary to sepsis. Hypoglycemia would fit but again cardiomegaly is not a feature FAOD present with hypoglycemia and may have cardiomyopathy but ?cardiomegaly and ?shock

56. Baby with bili of 280 and conjugated 200? What is the most likely cause based on incidence? a) Breastfeeding jaundice b) Hemolysis ABO c) Neonatal hepatitis d) Galactosemia

c) Neonatal hepatitis Not a) Breastfeeding jaundice (unconjugated) b) Hemolysis ABO (unconjugated) d) Galactosemia (incidence 1/60 000 per Nelson's) Up to Date: Most common causes of neonatal cholestasis - extrahepatic biliary atresia = 25% - idiopathic neonatal hepatitis = 25% - infectious hepatitis = 11% - parenteral nutrition-associated - metabolic disease - alpha-1 antitrypsin deficiency - alagille syndrome - progressive familial intrahepatic cholestasis - Neonatal cholestasis = prolonged elevation of serum levels of conjugated bilirubin beyond 14 days of life - extrahepatic (biliary atresia) and intrahepatic causes (metabolic, viral, hepatitis) • Breast feeding jaundice and ABO incompatability causes unconjugated hyperbilirubinemia • Galactosemia does cause both but incidence is 1 in 60 000 • Biliary atresia incidence is 1/10,000−15,000 live births • Idiopathic neonatal hepatitis 1/5,000−10,000. Intrahepatic bile duct paucity less common: 1/50,000−75,000 live births • Conjugated Hyperbilirubinemia o CB levels >15% of total bilirubin level o Due to failure to secrete CB from hepatocyte to the duodenum cholestasis o Associated with hepatomegaly, splenomegaly, pale stools, dark urine o DDx: ♣ Liver cell injury (normal bile ducts) ♣ Excessive Bilirubin load (inspissated bile syndrome) ♣ Bile flow obstruction (biliary atresia, extrahepatic or intrahepatic) ♣ Liver cell injury (normal bile ducts) • Infection o Viral: Rubella, CMV, Hepatits, EBV, Adenovirus, Echoviruses 14 and 19 o Bacterial: Syphilis, E. coli, GBS, Listeria, TB, Staph • Toxic o Prolonged use TPN due to lipid o Sepsis o Ischemic necrosis • Metabolic o Alpha-1 antitrypsin deficiency o Storage disease: ♣ Gaucher, Niemann-Pick, Glycogenosis Type IV, Wolmans) o Dubin-Johnson o Zellweger o Trisomy 18 o Rotor syndrome ♣ Excessive Bilirubin Load • Severe hemolytic disease • Erythroblastosis fetalis who have been tx with intrauterine transfusions • Those on ECMO ♣ Bile Flow Obstruction • Biliary atresia • Extrahepatic: Choledochal cyst, trisomy 13 or 18, polysplenia • Intrahepatic: Alagille syndrome, choledochal cyst, bile duct stenosis, lymph node/hemangioma/tumour compression bile duct, inspissated bile, CF Breastfeeding and ABO incompatibity result in unconjugated HBR Galactosemia does result in conjugated HBR but this is higher than would espect Neonatal Hepatitis ▪ Characterized by an ill-appearing infant with an enlarged liver and jaundice. ▪ No specific laboratory test, but if liver biopsy is performed, the presence of hepatocyte giant cells is characteristic. ▪ Hepatobiliary scintigraphy typically shows slow hepatic uptake with eventual excretion of isotope into the intestine. ▪ These infants have a good prognosis overall, with spontaneous resolution occurring in most. EN p489

41. Which of the following scenarios is an indication to intubate and suction for meconium delivery? a) Thick meconium b) Apgars < 5 c) No spontaneous respiratory effort

c) No spontaneous respiratory effort - none with new NRP guidelines! (Used to be no spontaneous respiratory effort) - Should resuscitate as with other babies; may ultimately require intubation/suction if airway is obstructed

38. 3 week old infant has jaundice. His birth weight was 3250g and he now weighs 3490g. He is breastfeeding well. Hemoglobin is 127 and total bilirubin is 270, conjugated bilirubin is 8. Coombs test is negative. What should be done? a) Admit for phototherapy b) Referral to gastroenterology c) Reassess in 1 week d) Recommend switching from breastfeeding to formula

c) Reassess in 1 week Breast milk jaundice: • Significant elevation in unconj bili develops in ~ 2 percent of breast-fed term infants o After 7th day of life o Maximal levels usually reached during the 2-3rd week o If breastfeeding is continued, bilirubin gradually decreases, but may persis for 3-10 weeks at lower levels • If BF is discontinued, level falls rapidly, reaching normal range within a few days o When resumed, bilirubin seldom returns to previously high levels • Etiology not completely clear but may be attributed to the presence of glucuronidase in some breast milk This infant is thriving and his bilirubin level is below phototherapy cutoff - since he is likely at the peak bili, he could be followed up to ensure level starts dropping. C CPS Statement: Guidelines for detection, management and prevention of hyperbilirubinemia in term and late preterm newborn infants. Updated 2011 Although breastfed infants are at a higher risk for developing severe hyperbilirubinemia than are formula-fed infants, the known risks of acute bilirubin encephalopathy are very small when weighed against the substantial known benefits of breastfeeding [17][63]. Support of the breastfeeding mother by knowledgeable individuals increases the frequency and duration of breastfeeding. It is difficult to find reliable evidence that the risk of severe jaundice can be minimized by a program of breastfeeding support, but other aspects of breastfeeding difficulty can be reduced by such programs, and providing such support is reasonable Only in rare metabolic disorders e.g. galactosemia should breastfeeding be permanently discontinued with respect to HBR. PS p452 Basically, should support breastfeeding; there is no evidence to support switching to formula and/or interrupting breastfeeding

30. 3 week old who is brought to the office because mother thinks he is too yellow. Breastfed. Otherwise well. Total bili is 180. Direct is 8. What do you do? a) Septic workup b) Investigate for blood group incompatability c) Reassure mother that condition may last for 4-12 weeks d) Investigate for metabolic disease

c) Reassure mother that condition may last for 4-12 weeks (Breast milk jaundice) Breastmilk Jaundice: • Develops in 2 percent of breast-fed term infants after day 7 o Maximum bilirubin levels in 2-3rd week of life, may persis up to 10 wks • Presence of glucuronidase in some breast milk • If BF discontinued - bili falls, with resumption levels seldom return to high levels o Phototherapy may be of benefit C Likely Breastmilk jaundice HBR in breastfed infants during the 1st week of life is called breastfeeding jaundice and is thought to be the result of poor caloric intake and/or dehydration. HBR in breastfed infants after the 1st week of life is known as breast-milk jaundice. The cause of breast-milk jaundice is uncertain; however, possible etiologies include an increased enterohepatic circulation of bilirubin as a result of the presence of -glucuronidase in human milk and/or the inhibition of the hepatic glucuronosyl transferase by a factor such as free fatty acids in some human milk samples.

14. Intubated baby with bilateral pulmonary infiltrates who desaturates with handling and improves with bagging. What do you do next? a) Stat CXR b) Cardiac catheterization c) Sedate and moderate hyperventilation d) Antibiotics e) PGE

c) Sedate and moderate hyperventilation

63. 28 week infant, 32 weeks currently. Feeding well on gavage feeds. Using HF 4L/min room air. What should his transfusion threshold be? a. 100 b. 75 c. 85 d. 115

c. 85

As both dexamethasone and hydrocortisone administration within the first seven days of life is associated with an increased risk of ______, early postnatal corticosteroid therapy is not recommended to prevent CLD

cerebral palsy CPS Postnatal corticosteroids to treat/prevent BPD

The relative immaturity of the late preterm infant is reflected in longer term outcomes. Rates of _____ are three times higher, and rates of ____ when compared with term infants in a large retrospective cohort study

cerebral palsy developmental delay or mental retardation are modestly increased (HR 1.25) CPS facilitating discharge of late preterms infants

Infants born _____ may have a lower hematocrit than do those born _____

cesarean section vaginally Neo Notes

Tx of NEC

cessation of feeding, nasogastric decompression, and administration of intravenous fluids Careful attention to respiratory status, coagulation profile, and acid-base and electrolyte balance are important. Once blood has been drawn for culture, systemic antibiotics should be started immediately. If present, umbilical catheters should be removed while maintaining good intravenous access. Ventilation should be assisted in the presence of apnea or if abdominal distention is contributing to hypoxia and hypercapnia. Intravascular volume replacement with crystalloid or blood products, cardiovascular support with volume and/or inotropes, and correction of hematologic, metabolic, and electrolyte abnormalities are essential to stabilize the infant. Neo Notes

The risk of kernicterus developing from hemolytic disease is greater than from _____

comparable nonhemolytic hyperbilirubinemia Neo Notes

Neonatal ophthalmia, a relatively common illness, is defined as

conjunctivitis occurring within the first four weeks of life. Originally, this term only referred to cases caused by N gonorrhoeae, but the term currently encompasses any conjunctivitis in this age group. N gonorrhoeae now accounts for <1% of reported cases of neonatal ophthalmia in the United States, while that due to Chlamydia trachomatis ranges from 2% to 40%. Other bacteria such as Staphylococcus species, Streptococcus species, Haemophilus species and other Gram- negative bacterial species account for 30% to 50% of cases. Much less commonly, neonatal conjunctivitis is caused by viral infections (herpes simplex, adenovirus, enteroviruses). CPS Preventing ophthalmia neonatorum

First trimester _______ is the most accurate for dating within 3 to 8 days

crown-rump length The degree of imprecision increases with advancing GA (± 10 days at 16 to 22 weeks and ± 2 weeks at 24 weeks) CPS Counselling and management for anticipated extremely preterm birth

50. Which population will you screen for retinopathy of prematurity? a) < 1500g regardless of gestational age b) ? c) ? d) < 1250g or < 31 weeks gestational age

d) < 1250g or < 31 weeks gestational age

50. Mother with + GBS never got treated with intrapartum abx. Baby born 32 weeks now 9 days. He has been having apneas in the past 24 hours. Blood culture shows GP cocci in clusters after 18 hours of culture. What is the most likely diagnosis? a) GBS b) Strep viridians c) Staph aureus d) Coag neg staph

d) Coag neg staph (more common than S. aureus as nosocomial infection) - CPS statement comments on most common bacteria in term infants (GBS, E. coli, Streo viridans, Strep pneumo, Enterococcus, Enterobacter, S. aureus, H. flu); no comment on preterm - Gram staining: - GP bacteria retain crystal violet and appear purple - GN bacteria do not retain crystal violet but take up safranin counterstain and appear pink - GP cocci in clusters = Staphylococcus - GP cocci in chains = Streptococcus - Nelson's: - in neonates, CONS bacteremia (most common cause of nosocomial bacteremia) may be manifested as apnea, bradycardia, temperature instability, abdominal distension, hematochezia, meningitis in absence of CSF pleocytosis, cutaneous abscesses, and persistence of positive blood cultures despite adequate antimicrobial therapy; usually indolent and not associated with overwhelming septic shock - early vs late onset infections (~7 days, consider whether infection acquired before/during, or after delivery) - most important neonatal factor predisposing to infection is prematurity or LBW Not a) GBS (chains) b) Strep viridians (chains) c) Staph aureus Early onset sepsis = birth to 7 days • Organisms- GBS, E Coli, Klebsiella, Listeria monocytogenes and H influenza • Usually present in first 24 hours Late onset sepsis = 8-28 days • Consider bacteremia, as well as focal infections (Meningitis in 75 percent, osteomyelitis, arthritis and UTIs) Nosocomially acquired sepsis = 8 days to discharge • Risk increased by frequency of broad-spectrum antibiotics and by presence of hardware • Coagulase-negative staph is an important pathogen Early onset infection is less than 7 days, late onset 7-30 days, late late onset is greater than 30 days. Nosocomially acquired sepsis ▪ 8 days to discharge ▪ Occurs predominantly in premature infants in the NICU; many of these infants have been colonized with the multidrug resistant bacteria indigenous to the NICU ▪ Risk of such SBI is increased by frequent treatment with broad-spectrum antibiotics for sepsis and by the presence of central venous indwelling catheters, endotracheal tubes, umbilical vessel catheters, and electronic monitoring devices. ▪ Epidemics of bacterial (CONS, fungi, enteric bacteria) or viral sepsis, bacterial or aseptic meningitis, staphylococcal bullous skin infections, cellulitis, pneumonia (bacterial or caused by adenovirus or respiratory syncytial virus), omphalitis (caused by S.aurea or gram-negative bacilli), and diarrhea (staphylococcal, enterviral, or caused by rotavirus or enteropathogenic E.coli) are common in the NICU ▪ Initial clinical manifestations of nosocomial infection in a premature infant may be subtle and include o Apnea, bradycardia, temperature instability, abdominal distension, and poor feeding ▪ In later stages signs of infection are o Shock, DIC, worsening respiratory status, and local rxns (omphalitis, eye discharge, diarrhea, and bullous impetigo ▪ Treatment is usually a combination of Vancomycin / Oxacillin / Ampicillin with an aminoglycoside (Gentamicin / Tobramycin. ▪ Treatment with aminoglycosides for >3 days necessitates monitoring of serum peak and trough concentrations to optimize therapy and to avoid ototoxicity and nephrotoxicity. ▪ Persistent signs of inection despite antibacterial treatment suggest canididal or viral sepsis. EN p 258 Common pathogens that are responsible for late-onset sepsis in the newborn infant: ▪ Coagulase-negative staphylococci (48%) ▪ Staphylococcus aureus (8%) ▪ Enterococcus species (3%) ▪ Gram-negative enterics (18%) ▪ Candida species (10%) PS p 416 Major RF for nosocomial sepsis: ▪ Prematurity ▪ Use of parenteral alimentation and central lines ▪ Intravenous fat emulsions ▪ H2 blockers ▪ Steroids for BPD ▪ Prolonged duration of mechanical ventilation ▪ Overcrowding ▪ Heavy staff workloads PS p 416 CPS Statement: Management of the infant at increased risk for sepsis. Updated 2011 ▪ The risk of invasive early-onset GBS disease in an infant whose mother is GBS-positive and does not receive IAP is approximately 1%. ▪ Only one-quarter of these babies are asymptomatic at birth. ▪ This risk of significant disease probably does not justify routine empirical treatment in these circumstances, and careful observation with treatment at the first clinical sign of infection appears to be reasonable. ▪ Ninety-five per cent of infants with early-onset GBS infection present with clinical signs within 24 h (either temperature instability, tachycardia, poor peripheral perfusion, respiratory distress or abnormal CBC). ▪ Four per cent of infected infants present between 24 h and 48 h of age, with only 1% developing signs after 48 h of age. ▪ If a GBS-positive woman receives IAP less than 4 h before delivery (or receives no antibiotics or a non-penicillin regimen), then a limited diagnostic evaluation is required, and the infant should not be discharged before 24 h of age. ▪ At the time of discharge, the infant should be evaluated and the parents should be educated regarding signs of sepsis in the newborn. ▪ Discharge at 24 h to 48 h is conditional on the parents' ability to immediately transport the baby to a health care facility if clinical signs of sepsis develop. Empirical therapy for infants with positive cerebrospinal fluid (CSF) evaluation CSF findings Most common organisms Suggested expectant antimicrobials for early-onset meningitis Gram-positive cocci Group B Streptococci, less commonly: Staphylococcus species or enterococci Ampicillin or penicillin plus gentamicin Gram-positive rods Listeria monocytogenes Ampicillin plus gentamicin Gram-negative rods Escherichia coli, less commonly: Klebsiella, Pseudomonas and Citrobacter Cefotaxime plus gentamicin Gram-negative cocci Uncommon Cefotaxime Pleocytosis, or other findings strongly suggestive of meningitis, but Gram stain-negative, or too unstable to have an LP Any of the above are possible. Ampicillin plus gentamicin LP Lumbar puncture. Source : Canadian Paediatric Society 2007

65. A newborn term infant had thin meconium at delivery but had good APGAR scores and required only 2 minutes of free flow O2. Now at 12 hours of age he has increasing work of breathing. On CXR there is hyperinflation of the RUL with mediastinal shift. What is the most likely diagnosis? a) Meconium aspiration syndrome b) Neonatal pneumonia c) CCAM d) Congenital lobar emphysema

d) Congenital lobar emphysema - ? sicker child, hyperinflation (not cystic lesion) CCAM: • Adenomatoid proliferation of bronchioles = cysts • CXR: large air-filled cyst, a conglomerate of air-filled cysts, or a solid mass (ie, fluid-filled cyst) Congenital lobar emphysema: • Hyperinflation of one or more of the pulmonary lobes compression of the remaining lung tissue herniation of the affected lobe across the anterior mediastinum displacement o Left upper lobe is affected most often (40 to 50 percent) o CLE affecting multiple lobes is rare • Prevalence of 1 in 20,000 to 1 in 30,000 o Males > females, in a ratio of 3:1 • Clinical features: o Usually symptomatic in neonatal period ♣ Tachypnea and increased WOB ♣ Cyanosis o If presents later - recurrent pneumonia, poor feeding o Physical exam: respiratory distress, wheezing, decreased breath sounds + hyperresonant area ♣ cardiac point of maximal impulse may be displaced o Other congenital anomalies often accompany CLE and may be apparent on examination (Cardiac most common, renal, gastrointestinal, musculoskeletal, and cutaneous disorders also may occur) • Diagnosis: o Characteristic appearance on CXR = distension of the affected lobe + mediastinal shift o Computed tomography (CT) of the chest or magnetic resonance imaging may help establish the diagnosis of CLE in atypical cases and may demonstrate an intrinsic or extrinsic source of airway obstruction. N18 p1779andfigure389.1 Congenital lobar emphysema (CLE) Can result in severe respiratory distress in early infancy and can be caused by localized obstruction. Familial occurrence has been reported. In 50% of cases, a cause of CLE can be identified. Congenital deficiency of the bronchial cartilage, external compression by aberrant vessels, bronchial stenosis, redundant bronchial mucosal flaps, and kinking of the bronchus caused by herniation into the mediastinum have been described as leading to bronchial obstruction and subsequent CLE. Clinical manifestations usually become apparent in the neonatal period but are delayed for as long as 5-6 mo in 5% of patients. Many are diagnosed by antenatal ultrasonography. Prenatally diagnosed cases are not always symptomatic at birth. Some patients remain undiagnosed until school age or beyond. Signs range from mild tachypnea and wheeze to severe dyspnea with cyanosis. CLE affects the upper and middle lobes, with the left upper lobe the most common site. The affected lobe is essentially nonfunctional because of the overdistention, and atelectasis of the ipsilateral normal lung may ensue. With further distention, the mediastinum is shifted to the contralateral side with impaired function. A radiolucent lobe and a mediastinal shift are often revealed by radiographic examination. A- Nelsons P.743 The resulting small airway obstruction may produce respiratory distress within the 1st hours, with tachypnea, retractions, grunting, and cyanosis observed in severely affected infants. Partial obstruction of some airways may lead to pneumothorax or pneumomediastinum, or both. The typical chest roentgenogram is characterized by patchy infiltrates, coarse streaking of both lung fields, increased anteroposterior diameter, and flattening of the diaphragm D NelsonsChapter95.6MAS-ClinicalManifestations • inuteroorwiththe1stbreath,thick,particulatemeconiumisaspirated intothe lung • smallairwayobstructionproducesrespiratorydistresswithin1sthoursof life • tachypnea,retractions,grunting,andcyanosisobservedinseverelyaffectedinfants • partialobstructionmayleadtopneumomediastinum,pneumothorax,orboth • Overdistentionofthechestmaybeprominentusuallyimproveswithin72h exceptwhensevereandneedingmechvent • highrisk formortality • Tachypneamaypersistformanydaysorevenseveralweeks • typicalCXR=patchyinfiltrates,coarsestreakingofbothlungfields,increased anteroposteriordiameter,flatteningofthediaphragm. NelsonsChapter387.3CCAM hamartomatous/dysplasticlungtissuemixedwithmorenormallung,generally confinedtoonelobe presentinthenewbornperiod/earlyinfancyrespiratorydistress,recurrent respiratoryinfection,andpneumothorax. maybeconfusedwithadiaphragmatichernia smallerlesionsareusuallyasymptomaticuntilmidchildhood,whenepisodesof recurrentorpersistentpulmonaryinfectionorchestpainoccur Breathsoundsmaybediminished,+mediastinalshiftawayfromthelesion CXR=cysticmass,+/mediastinalshift.Occasionally,a/flevelsuggestsabscess NelsonChapter384-Emphysemaandoverinflation • usuallyapparentinneonatalperiodbutdelayedforaslongas56moin5%of patients • ManydiagnosedbyantenatalU/Sandnotalwayssymptomaticatbirth(May remainundiagnoseduntilschoolageorbeyond). • mildtachypnea+wheezeseveredyspnea+cyanosis. • Canaffectoneormorelobes;upperandmiddlelobes,andtheleftupperlobe= mostcommon • affectedlobe=nonfunctionalduetooverdistention,+atelectasisofipsilateral normallungcanensue • distentionmediastinumshiftstocontralateralside,withimpairedfunction seenaswell • CXR=radiolucentlobe+mediastinalshift • CT=aberrantanatomyofthelesion • MRIorMRangiography=vascularlesions,whichmightbecausingextraluminal compression. • Nuclearimaging=perfusiondefectsintheaffectedlobe

12. Most likely cause of late hemorrhagic disease of the newborn? a) Phenytoin use in mom b) Baby did not get Vit K prophylaxis c) Oral antibiotics d) Cystic fibrosis

d) Cystic fibrosis - late disease likely from Vit K malabsorption (biliary atresia, hepatitis, CF) • All newborns have moderate decrease in factors II, VII, IX, and X (Vit K dependent) by 48-72 hr after birth, with a gradual return to birth levels by 7-10 days of age o due to lack of free vitamin K from the mother and absence of the bacterial intestinal flora normally responsible for the synthesis of vitamin K o Breast milk is a poor source of vitamin K • Prolonged PT, PTT, bleeding time • Classic form : 2-7 days - due to Vit K deficiency • Early-onset : birth to 24 hr: mother has been treated with drugs (phenobarbital, phenytoin) that interfere with vitamin K function • Late onset (>2 wk) is often associated with vitamin K malabsorption, as noted in neonatal hepatitis or biliary atresia, CF a. Phenytoin use in mom - early b. Baby did not get Vit K prophylaxis - classic 2-3 days c. Oral antibiotics d. Cystic fibrosis - d/t malabsorption D Late-onset vitamin K deficiency bleeding in the newborn ♣ This usually occurs between age 2-12 weeks; however, late-onset vitamin K deficiency bleeding can be seen as long as 6 months after birth. ♣ This disease is most common in breastfed infants who did not receive vitamin K prophylaxis at birth. ♣ Vitamin K content is low in mature human milk and ranges from 1-4 mcg/L. ♣ Industrial contaminants in breast milk have been implicated in promoting vitamin K deficiency bleeding. ♣ More than half of these infants present with acute intracranial hemorrhages. ♣ In addition to breastfeeding, clinical states that are risk factors for late-onset vitamin K deficiency bleeding include: o Diarrhea o Hepatitis o Cystic fibrosis o Celiac disease o Alpha1-antitrypin deficiency o Short bowel syndrome o Intestinal bacterial overgrowth o Chronic exposure to broad spectrum antimicrobials EMed - Hemorrhagic disease of the Newborn 2012

70. Newborn was recently extubated after a course of systemic corticosteroids. What is the likely side effect? a) Hypotension b) Hypoglycemia c) Leucopenia d) Hypertrophic cardiomyopathy

d) Hypertrophic cardiomyopathy - AE's of Dexamethasone for BPD: hypertension, hyperglycemia, GI bleeding/perforation, hypertrophic cardiomyopathy, sepsis, poor weight gain and head growth Postnatal corticosteroids to treat or prevent chronic lung disease in premature infants: • Does not affect mortality by the time of discharge or length of hospitalization • Early systemic administration decreases incidence of CLD at 28 days PNA and 36 weeks PNA o Allows for earlier extubation and fewer ventilation days • Many short term adverse effects: o Hyperglycemia (often requiring insulin) o Hypertension o GI bleeding and intestinal perforation o Hypertrophic obstructive cardiomyopathy o Poor weight gain and growth o Increased incidence of PVL D CPS Statement FN 02-01 Postnatal corticosteroids to treat or prevent chronic lung disease in premature infants PedsSecrets4thEditionp.608forsideofCHRONICsteroiduse. CUSHINGOIDMAP Cataracts Ulcers Striae HTN Infectiouscomplications Necrosisofbone(avascular) Growthretardation-MAJORISSUEFORTHENEWBORNS Osteoporosis IncreasedICP DM Myopathy Adiposetissuehypertrophy-obesity Pancreatitis

33. Symmetrically IUGR baby (with microcephaly). Most likely cause? a) Maternal malnutrition b) Maternal smoking c) Maternal PIH d) Maternal infection

d) Maternal infection Symmetric IUGR - maternal infection, chromosomal abnormalities Symmetric IUGR: • Earlier onset • Associated with diseases that seriously affect fetal cell number o Chromosomal or genetic abnormalities o Teratogenic causes o Maternal infections (TORCH) o Severe maternal hypertension Asymmetric IUGR (relative sparing of head circumference) • Often of late onset • Poor maternal nutrition • Late onset or exacerbation of maternal vascular disease (preeclampsia, chronic hypertension) D - only one that gives rise to symmetric IUGR N18 p702 -IUGR is often classified as reduced growth that is symmetric (head circumference, length, and weight equally affected) or asymmetric (with relative sparing of head growth) -symmetric IUGR often has an earlier onset and is associated with disease that seriously affect fetal cell number, such as conditions with chromosomal, genetic, malformation, teratogenic, infectious, or severe maternal hypertensive etiologies -asymmetric IUGR is often of late onset, demonsrates preservation of Doppler waveform velocity to the carotid vessels, and is associated with poor maternal nutrition or with late onset or exacerbation of maternal vascular disease (preeclampsia, chronic hypertension)

57. A 6 day old infant presents in shock with a glucose of 1.8 and cardiomegaly on CXR. Which of the following is the most likely etiology of the shock: a) Cardiac b) Sepsis c) Endocrine d) Metabolic

d) Metabolic vs B Pompe disease, also referred to as GSD type II or acid maltase deficiency, is caused by a deficiency of acid -1,4- glucosidase (acid maltase), an enzyme responsible for the degradation of glycogen in lysosomes. This enzyme defect results in lysosomal glycogen accumulation in multiple tissues and cell types, with cardiac, skeletal, and smooth muscle cells being the most seriously affected. Infantile-onset Pompe disease is thought to be uniformly lethal without specific therapy. Affected infants present in the 1st few months of life with hypotonia, a generalized muscle weakness with a "floppy infant appearance," feeding difficulties, macroglossia, hepatomegaly, and a hypertrophic cardiomyopathy followed by death from cardiorespiratory failure or respiratory infection usually by 1 yr of age. GlycogenstoragediseasetypeIIorPompeonsetbirthto6months:cardiomegaly, hypotonia,hepatomegaly. These include elevated levels of serum creatine kinase, aspartate aminotransferase, and lactate dehydrogenase. In the infantile form a chest x-ray showing massive cardiomegaly is frequently the 1st symptom detected However AAP: Inborn errors of metabolism: a guide to diagnosis. PEDIATRICS Vol. 102 No. 6 December 1, 1998 Hypoglycemia is not a feature of GSD type II (Pompe disease) because cytoplasmic glycogen metabolism and release are normal in this disorder in which glycogen accumulates within lysosomes as a result of deficiency of the enzyme acid maltase. Clinical manifestations of this disorder include macroglossia, hypotonia, cardiomegaly with congestive heart failure, and hepatomegaly. Cardiomegaly is the most striking feature and may be apparent in the neonatal period. Congestive heart failure is the cause of death in most cases.

35. All of the following are normal in term newborns except: a) Hypotonia post feed b) Irregular respiratory pattern in REM sleep c) Sigh following a brief period of apnea d) Mild cyanosis with feeding

d) Mild cyanosis with feeding

22. A 32 week gestation infant with Rh incompatability has received 5 exchange transfusions. He now has vomiting, diarrhea, and abdominal distension. Most likely diagnosis: a) Sepsis b) Acute gastroenteritis c) Portal vein thrombosis d) Necrotizing enterocolitis

d) Necrotizing enterocolitis Nelson's: Complications from exchange transfusion - metabolic acidosis, electrolyte abnormalities, hypoglycemia, hypocalcemia, thrombocytopenia, volume overload, arrhythmias, NEC, infection, GVHD, death Complications of exchange transfusion: metabolic acidosis, electrolyte abnormalities, hypoglycemia, hypocalcemia, thrombocytopenia, volume overload, arrhythmias, NEC, infection, graft versus host disease, and death D Complications of Exchange transfusion: ♣ Transfusion reactions ♣ Metabolic abnormalities (hypocalcemia, hypoglycemia, hyperkalamia, hyper/hypo-thermia, acidosis) ♣ Sepsis ♣ Vessel perforation ♣ Hypotension ♣ Thrombocytopenia ♣ Anemia ♣ Hypoxia ♣ Graft-versus-host disease ♣ NEC EN p250, PS p453, SK p 592 Indications for exchange transfusion ♣ HBR above threasholds ♣ Hydrops or severe anemia ♣ Postnatal rise of BU >17mmol/l/h in Rh incompatibility ♣ Acute bilirubin encephalopathy (Kernicterus) ♣ ?applicable general rule: indirect bilirubin of 20mg/dl or abovein an infant with hemolysis weighing >2000g SK p592, EN p250

27. 2 day old infant presents with fever of 39.5 ax. He is breastfed and his weight has fallen from 3.8 to 3.5 kg. He is lethargic but rouses during the examination. His fontanelle is normal. Na 150, K 7.3, Cl 110, BUN 8, Cre 110, unconjugated bili 190, normal CBC. What is the likely diagnosis? a) Hypernatremic dehydration due to decreased fluid intake b) Hypertonic breast milk c) Diabetes insipidus d) Sepsis e) Meningitis

d) Sepsis Hyperthermia Elevations in T to 38-39 seen on day 2-3 of life; otherwise well (BF with low intake, overdressed); associated with weight loss Severe cases have been asst with SIDS, hemorrhagic shock and encephalopathy Lower environmental temperature, ensure adequate intake Dress in clothing suitable for the environment Assess for possible infection Sepsis 50 percent will not mount a fever; early manifestations include: grunting, poor feeding, pallor, apnea, LETHARGY, hypothermia, abnormal cry Requires prompt recognition and initiation of appropriate antibiotics (Ampicillin and aminoglycoside) D A - Explains the weight loss, lethargy, Na but he is FEBRILE. B - Breast milk is HYPOTONIC. C - In DI, you can be hypernatremic but shouldn't be febrile? D - Sepsis explains the fever, weight loss, lethargy. E - Fontanelle normal. Unlikely with no neurological manifestations or meningism. However will still be part of FSWU

31. You are called from a family doctor about a 5 day old jaundiced baby. Well looking. Total bili is 355, direct is normal. He is breastfed. What do you recommend? a) Septic workup b) Investigate for a metabolic disease c) Coombs test and Hgb d) Start phototherapy e) Followup in 1 week

d) Start phototherapy After 4 days of life, phototherapy cutoff is 340 in a term, low risk baby. Investigations should also be done, but should not delay initiation of treatment. D Differential list would include: Enclosed hemorrhage, Cephalohematoma, breastfeeding or breast milk jaundice, sepsis, congenital infection e.g. TORCH, polycythemia, Hypothyroidsim, Galactosemia, Crigler-Najjar syndrome

60. (REPEAT) What is the most common presentation of a 2-day old newborn with cyanotic heart disease? a. bounding/dynamic precordium b. normal pulses and quiet precordium c. decreased pulses and poor perfusion d. tachypnea and nasal flaring e. palpable thrill

d. tachypnea and nasal flaring

Role of Nitric Oxide in RDS

decreases need for ECMO Neo Notes

very premature infants - Does the timing of umbilical cord clamping at delivery influence the need for and frequency of PRBC transfusions?

delayed cord clamping (30 s to 180 s) is associated with fewer infants requiring a PRBC transfusion during their hospital stay and fewer transfusions per infant. Delayed cord clamping should be performed in all preterm infants who are not in immediate need of resuscitation Delay cord clamping (optimally for 30 s to 180s after birth) for preterm infants who are not in immediate need of resuscitation • While cord milking may be considered as an alternative to delayed cord clamping, because so few patients have been enrolled in randomized trials to date, the technique cannot be recommended as routine practice at the present time CPS Minimizing blood loss and the need for transfusions in very premature infants

23. 3 day old infant develops poor perfusion, diminished peripheral pulses, and decreased urine output. What medication would you administer? a) Atropine b) Morphine c) Bicarbonate d) Furosemide e) Prostaglandins

e) Prostaglandins Timing is suspicious for duct-dependent cardiac lesions... would give fluids first, but PGE should be started if suspicion is high E R/O duct dependent lesion If LVOTO / CoA then Atropine wont help and may even worsen condition. Morphine may lead to respiratory depression again worsening the condition. Also pain not an issue here I think. Bicarbonate only used in very specific circumstance - acidosis caused by high GI or renal losses of bicarb. If given can cause more trouble than good; worsen cardiac function, increase intracellular acidosis, alter cerebral blood flow, raise PCO2 Frusemide may further reduce the peripheral perfusion by facilitating diuresis and dropping intravascular volume

Operative intervention for hirschprungs dz

endoscopic endorectal pull-through procedure. The great majority of patients achieve fecal continence. Postoperative problems include recurrent enterocolitis, stricture, prolapse, perianal abscesses, and fecal soiling. Some children will require myectomy or a redo pull-through procedure. Neo Notes

What follow up should be arranged for all neonates?

f/u q24-48h for feeding/jaundice until feeding is established (~3-4dol) CPS Hyperbili

Late deceleration

fetal hypoxemia. Neo Notes

Perinatal brachial plexus palsy (PBPP) If the physical examination shows incomplete recovery by the end of the ____, referral to a multidisciplinary brachial plexus team should be made. The team should include neurologists and/or physiatrists, rehabilitation therapists and plastic surgeons.

first month • Because there are no randomized controlled trials evaluating nonsurgical management versus primary surgical exploration, decisions for primary surgical exploration versus nonsurgical management and prediction of prognosis have to be based on history, electrodiagnostic procedures, diagnostic imaging and physical examination by the multidisciplinary team. • Secondary soft tissue and bone reconstructive surgery may improve function in children with significant impairment, but is inferior to primary intervention. CPS Perinatal brachial plexus palsy

Jaundice caused by reduced enzyme activity

genetic deficiency, hypoxia, infection, thyroid deficiency Neo Notes

Sick newborn infants with pneumonia and an oxygenation index ______ should receive exogenous surfactant therapy • Options for ventilatory management that are to be considered after prophylactic surfactant therapy include very rapid weaning and extubation to CPAP within 1 h (grade B). • Intubated infants with RDS should receive exogenous surfactant therapy before transport

greater than 15 CPS surfactant

Screens in 1st TM for trisomy 21

hCG and PAPP-A + nuchal translucency PHind (hCG and PAPP-A) Trisomy 211111 in the 1st trimester, it'll be clear (nuchal translucency) Neo Notes

Early deceleration

head compression. Neo Notes

The cord blood ______ is usually proportional to the severity of the disease in hemolytic anemia of the NB

hemoglobin content Neo Notes

Jaundice due to Increases in bilirubin load

hemolytic anemias, polycythemia, shortened RBC lifespan, infection Neo Notes

Anemia appearing in the first few days after birth is most frequently a result of ______

hemolytic disease of the newborn Other causes are hemorrhagic disease of the newborn, bleeding from an improperly tied umbilical cord, large cephalohematoma, intracranial hemorrhage, or subcapsular bleeding from rupture of the liver, spleen, adrenals, or kidneys. Delayed anemia may develop as a result of hemolytic disease of the newborn. Neo Notes

The risk of initial sensitization of Rh- mothers has been reduced to less than 1% by IM injection of ______ within _____ of an Rh+ infant, or ______. RhoGAM administered at _____ and again at _____ is more effective than a single dose.

human anti-D globulin (1 mL of RhoGAM) 72 hr of delivery of an Rh+ infant or ectopic pregnancy, abdominal trauma in pregnancy, amniocentesis, CVS, or abortion. 28-32 wk and again birth (40 wk) is more effective than a single dose. Neo Notes

To avoid overhydration and ______ in supplemented infants, oral and intravenous intake should not exceed _____ without careful monitoring for ______.

hyponatremia 100 mL/kg/day dilutional hyponatremia CPS Hypoglycemia

Key steps in the SDM (Shared decision making) process include

identifying the decision to be made (choice talk), reviewing the options (option talk), and providing support for deciding what matters most to the parents (decision talk). The HCP's expertise lies in recognizing major biological and medical factors influencing survival and long-term prognosis, while the family knows most about the socioenvironmental and familial characteristics that will influence their infant's outcomes (e.g., finances, resource availability, support from extended family). Such characteristics are difficult to measure but must all be considered in the SDM process. Decision 'coaching', where a trained HCP provides parents with individualized, nondirective guidance, is often used in conjunction with other decision aids to facilitate SDM CPS Counselling and management for anticipated extremely preterm birth

Low MSAFP is associated with

incorrect gestational age estimates, trisomy 18 or 21, and IUGR. Neo Notes

the incidence of late preterm delivery appear to be _____

increasing CPS facilitating discharge of late preterms infants

Meconium is Associated with ______. o Aspiration pneumonia develops in ____percent

intrauterine fetal distress and hypoxia 5% Neo Notes

The long-term prognosis of polycythemic infants

is unclear. Reported adverse outcomes include speech deficits, abnormal fine motor control, reduced IQ, school problems, and other neurologic abnormalities. It is thought that the underlying etiology (chronic intrauterine hypoxia) and hyperviscosity contribute to adverse outcomes. It is unclear whether partial exchange transfusion improves the long-term outcome. Most asymptomatic infants develop normally. Neo Notes

Ankyloglossia ('tongue-tie') is a relatively common congenital anomaly characterized by an abnormally short _____, which may restrict tongue tip mobility.

lingual frenulum Ankyloglossia can be classified based on the degree of fusion remaining between the tongue and the floor of the mouth. There may be a genetic predisposition to ankyloglossia. This congenital anomaly typically occurs in isolation CPS Ankyloglossia

Other benefits of breastfeeding

linked to a decrease in Sudden Infant Death Syndrome (SIDS) enhanced performance on neurocognitive testing decrease in the incidence of both breast and ovarian cancers, and a delay in the return of ovulation and greater postpartum weight loss CPS baby friendly

Dif betwn live birth, fetal death and infant death

live birth = expulsion/extraction (evidence of life) Fetal death - death before expulsion Infant death - death after expulsion Neo Notes

NO is generated in the _____ from ______ by the enzymatic activity of NO synthases NO diffuses into vascular muscle cells and activates the enzyme ______, leading to increased ________ production, pulmonary vasodilation and improved ventilation/perfusion matching

lung endothelium Larginine guanylate cyclase cyclic guanosine monophosphate CPS iNO

Most centres now γ-irradiate blood to deactivate ____ and prevent ______

lymphocytes graft-versus-host disease Irradiation = destroy WBC DNA Prevent graft vs. host disease Donor t lymphocytes attack recipient lymphoid tissue Usually recipient immune system destroys this If recipient immunocompromised OR the recipient shares HLA haplotypes with donor (direct relative donor), recipient cannot destroy it. If irradiate, should use in 24hrs because K+ leakage will increase after irradiation CPS Red blood cell transfusion in newborn infants

When a pregnant woman is at risk of giving birth between 22 and 25 weeks GA, the primary HCP should consult with a ______. Transfer to a tertiary perinatal centre is recommended.

maternal-fetal medicine specialist CPS Counselling and management for anticipated extremely preterm birth

Secondary surfactant deficiency or dysfunction occurs in other newborn respiratory disorders, including _______

meconium aspiration syndrome, pneumonia and pulmonary hemorrhage Intubated infants with meconium aspiration syndrome requiring more than 50% oxygen should receive exogenous surfactant therapy CPS surfactant

Potential complications of exchange transfusion for hyperbili

metabolic acidosis, electrolyte abnormalities, hypoglycemia, hypocalcemia, thrombocytopenia, volume overload, arrhythmias, NEC, infection, graft versus host disease, and death Neo Notes

1st trimester exposure to medications/drug =

miscarriage or congenital malformations Neo Notes

Surfactant replacement therapy, either as a rescue treatment or a prophylactic natural surfactant therapy, reduces ______ and several aspects of morbidity. These morbidities include Surfactant replacement increases the likelihood of surviving without bronchopulmonary dysplasia (BPD, also known as chronic lung disease of the preterm) largely by The increase in survival is achieved with no increase in _______

mortality deficits in oxygenation, the incidence of pulmonary air leaks (pneumothorax and pulmonary interstitial emphysema) and the duration of ventilatory support improving survival rather than the incidence of BPD Babies treated with surfactants have shorter hospital stays and lower costs of intensive care treatment adverse neurodevelopmental outcome CPS surfactant

There is both observational evidence and clinical consensus that sick, hypoglycemic infants, particularly those with _____, should be treated immediately with an intravenous infusion of glucose

neurological signs CPS Hypoglycemia

Breastmilk advantages in premature babies

nutritional advantages, protection against infection, decreased risk of NEC, lower risk of SIDS, possibly lower risk of childhood/adolescent obesity and improved neurodevelopmental outcome o Need to supplement with breast milk fortifier containing protein, calcium, and phosphorus o Iron stores in VLBW adequate until birthweight doubled, then start iron supplementation (2 mg/kg/24 hr) Neo Notes

Prognosis by wt at birth

o 1,501-2,500 g : 95% or greater chance of survival o Survival 15% at 23 wk, 56% at 24 wk, and 79% at 25 wk Neo Notes

RDS Symptoms usually reach a peak at

o 3 days, after which improvement is gradual ♣ Often heralded by spontaneous diuresis Neo Notes

longer term prognosis in VLBW

o 30-50% of VLBW children have poor school performance at 7 yr of age (repeating of grades, special classes, learning disorders, poor speech and language), despite a normal IQ. ♣ Risk factors: BW< 750 g, severe IVH, PVL, BPD, cerebral atrophy, posthemorrhagic hydrocephalus, IUGR, low SES ♣ Antenatal magnesium sulfate may have neuroprotective effects o Both premature and IUGR infants are at risk for significant metabolic conditions (obesity, type II diabetes) and cardiovascular disorders (ischemic heart disease, hypertension) as adults. Neo Notes

Screening guidelines for ROP

o AAP Screening guidelines: o Infants with a birth weight of <1,500 g o gestational age of ≤32 wk o selected infants with a birth weight between 1,500 and 2,000 g or gestational age of >32 wk with an unstable clinical course, o Timing depends on age, follow-up usually at 2 wk or less. Neo Notes

Antiseptic Skin and Cord Care:

o Careful removal of blood from the skin shortly after birth may reduce the risk of infection with blood-borne agents. o Once temperature stabilized, the entire skin and cord should be cleansed with warm water or a mild nonmedicated soap solution and rinsed with water to reduce the incidence of skin and periumbilical colonization with pathogenic bacteria and subsequent infectious complications. o To reduce colonization with Staphylococcus aureus and other pathogenic bacteria, the umbilical cord may be treated daily with a bactericidal or antimicrobial agents such as triple dye or bacitracin. Neo Notes

Classification of ROP by location

o Classification: o Location: ♣ Zone 1: inner zone, extends twice the disc-macular distance, or 30 degrees in all directions from the optic disc. ♣ Zone II: the middle zone, extends from the outer edge of zone I to the ora serrata nasally and to the anatomic equator temporally. Zone III: the outer zone, extends from the outer border of zone II to the ora serrata temporally. Neo Notes

Prevention/prediction of RDS

o Estimation of fetal head circumference and determination of lecithin concentration in amniotic fluid (lecithin: sphigomyelin ratio) An L-S ratio of 2.4 or more indicates fetal lung maturity and a relatively low risk of infant respiratory distress syndrome, and an L/S ratio of less than 1.5 is associated with a high risk of infant respiratory distress syndrome. o Betamethasone to women 48 hrs before delivery of fetuses between 24-34 weeks gestation significantly reduces incidence/mortality o Administration of 1st does of surfactant to symptomatic infants immediately after birth (prophylactic) or during first few hrs of life (early rescue) reduces airl leak and mortality, but does NOT alter the incidence of BPD Neo Notes

Factors Related to Premature Birth and Low Birthweight

o Factors Related to Premature Birth and Low Birthweight o low socioeconomic status: maternal undernutrition, anemia, illness; inadequate prenatal care; drug misuse; obstetric complications; and maternal history of reproductive inefficiency o single-parent families o teenage pregnancies o short interpregnancy interval o more than 4 prevoius children o maternal smoking Neo Notes

Hematologic dz in infants of DM mothers

o Hematologic: increased incidence of hyperbilirubinemia, polycythemia and RV thrombosis Other: o Congenital anomalies 3x greater risk (cardiac defects, NTDs, renal anomalies, etc) Neo Notes

Lung dz in infants of DM mothers

o Higher incidence of RDS (antagonistic effect of insulin on surfactant synthesis) o Asymmetric septal hypertrophy cardiomegaly heart failure (5-10%) Neo Notes

Most common cause of neonatal seizures

o Hypoxic-Ischemic Encephalopathy: most common cause, 50-60%, occurs within 12h of birth o Vascular Events: intracranial bleeds and ischemic strokes, account for 10-20% o Intracranial Infections: 5-10%, bacterial meningitis, TORCH, particularly HSV o Brain Malformations: 5-10% o Metabolic Disturbances: disturbances in glucose, calcium, magnesium, other electrolytes, amino acids, or organic acids and pyridoxine dependency. o Drug Withdrawal: narcotic analgesics, sedative-hypnotics; appear during the first 3 days of life. Neo Notes

Idiopathic apnea of prematurity

o Idiopathic apnea of prematurity: no predisposing disease o Obstructive apnea: absent airflow but persistent chest wall motion (pharyngeal instability, neck flexion, nasal occlusion) o Central apnea: decreased CNS stimuli to respiratory muscles, absent airflow and chest wall motion. In premature infants, immaturity of the brainstem respiratory centers causes attenuated response to carbon dioxide and a paradoxical response to hypoxia that results in apnea rather than hyperventilation. o Mixed : most common (50-75%), with obstructive apnea preceding (usually) or following central apnea. Short episodes of apnea are usually central, whereas prolonged ones are often mixed. neo notes

1. Process of neonatal adaptation to extrauterine life

o In fetus: placenta is low-resistance and lungs high, resulting in right to left shunt o Foramen ovale: blood shunted from right atrium to left o PDA: blood shunted from pulmonary artery to aorta o Alveolar fluid is cleared during labour, delivery, with first breath o Alveoli are expanded and surfactant is released o Cord clamping results in removal of low-pressure placental circulation o Simultaneously, pulmonary vascular resistance decreases o This decreases the right to left shunt through the PDA, resulting in increased blood flow to lungs o With increased oxygenation, PDA closes o With increased pulmonary blood flow and pulmonary venous return, left atrial pressure transiently higher than right, resulting in closure of foramen ovale Neo Notes

Pneumothorax in newborns

o Increased in infants with lung disease (mec aspiration, RDS) and in those who have had vigorous resuscitation or receiving ventilation, and in infants with urinary tract abnormalities o Association with pulmonary hypoplasia is common ♣ Occurs on first day of life, due to reduced alveolar surface area and poor compliance ♣ Decreased amniotic fluid volume (Potter syndrome, renal agenesis, renal dysplasia, chronic amniotic fluid leak), decreased fetal breathing movement (oligohydramnios, neuromuscular disease), pulmonary space-occupying lesions (diaphragmatic hernia, pleural effusion, chylothorax), and thoracic abnormalities (asphyxiating thoracic dystrophies) Neo Notes

adjuvant treatment due to isoimmune hemolytic disease, used when approaching exchange levels

o Intravenous Immunoglobulin: adjuvant treatment due to isoimmune hemolytic disease, used when approaching exchange levels ♣ 0.5-1.0 g/kg/dose; repeat in 12 hr reduces the need for exchange transfusion in ABO+Rh hemolysis, presumably by reducing hemolysis. Neo Notes

Management of biliary atresia

o Management; ♣ All patients suspected should undergo exploratory laparotomy and direct cholangiography to determine the presence and site of obstruction ♣ Kasai operation (hepatoportoenterostomy) much higher success rate if performed before 8 weeks of life ♣ Adequate nutrition: ♣ Malabsorption from dietary long-chain> medium chain triglycerides ♣ Vitamin A deficiency (night blindness, thick skin): 10,000-15,000 IU/day as Aquasol A ♣ Vitamin E deficiency (neuromuscular degeneration): 50-400 IU/day as oral α-tocopherol or TPGS ♣ Vitamin D deficiency (metabolic bone disease): 5,000-8,000 IU/day of D2 or 3-5 μg/kg/day of 25- hydroxycholecalciferol ♣ Vitamin K deficiency (hypoprothrombinemia): 2.5-5 mg EOD as water-soluble derivative of menadione ♣ Micronutrient deficiency: Calcium, phosphate, or zinc supplementation ♣ Retention of biliary constituents such as cholesterol (itch or xanthomas), ursodeoxycholic acid 15-20 mg/kg/day ♣ Progressive liver disease; portal hypertension (variceal bleeding, ascites, hypersplenism) Supportive (control bleeding; salt restriction; spironolactone) End-stage liver disease (liver failure): transplantation Neo Notes

IM med that acts as a competitive enzymatic inhibition of the rate limiting conversion of heme-protein to biliverdin

o Metalloporphyrins: competitive enzymatic inhibition of the rate limiting conversion of heme-protein to biliverdin ♣ single IM dose on the 1st day of life may reduce the need for phototherapy ♣ may be beneficial when jaundice is anticipated (ABO incompatibility, G6PD def) or when blood products are discouraged as with Jehovah's Witness patients Neo Notes

Poor prognostic factors in neonatal seizures

o Mortality 20% o Strong correlation with EEG findings and prognosis ♣ Poor prognostic factors: abnormal background, elctrographic sz >10min, multifocal periodic electrographic discharges, and spread of the electrographic seizures to the contralateral hemisphere Neo notes

1. Aspects of drug therapy unique to the newborn

o Neonates have variable absorption, distribution, metabolism and renal excretion of drugs depending on gestational age o Need to be careful with dose, intervals, monitoring drug levels Neo Notes

Exam/Clinical ROP

o On exam: undervascularization of the peripheral retina, abnormal branching, tortuosity, or straightening of the retinal vessels, retinal pigmentary changes, dragging of the retina (dragged disc), ectopia of the macula, retinal folds, or retinal breaks, retinal detachment o Clinically: leukocoria, cataracts, glaucoma, and signs of inflammation, painful blind eye or a degenerated phthisical eye, myopia Neo Notes

1st choice drug for neonatal sz's

o Phenobarbital : 1st choice, 20mg/kg load, then 3-6mg/kg/d BID maintenance o Phenytoin : if 40mg/kg phenobarb not effective, can give phenytoin 15-20mg/kg o Fosphenytoin is preferable o Lorazepam : 1st or 2nd line, 0.05mg/kg o Diazepam : districuted and cleared very quickly therefore risk of recurrence o Midazolam: can be given as continuous infusion 0.5mcg/kg/min up to 2mcg/kg/min Neo notes

Prevention of ROP

o Prevention: prevention of premature birth, lower oxygen saturation (85-92%) at age <34 wk and maintaining them at higher oxygen saturation (92-97%) at age >34 wk. Neo Notes

Biliary Atresia - etiology and dx

o Progressive obliterative cholangiopathy, usually postnatal onset o Diagnosis: ♣ Ultrasound: may detect associated anomalies (polysplenia and vascular malformations) ♣ hepatobiliary scintigraphy used to differentiate from nonobstructive causes of cholestasis (hepatic uptake of the agent is normal, but excretion into the intestine is absent) Gold standard = liver biopsy Neo Notes

Development of ROP

o ROP acute phase: cessation of vasculogenesis, abrupt termination of the vessels, marked by a line in the retina. o The line can then grow into a ridge o Cell division and differentiation might later resume, and vascularization of the retina can proceed. o Alternatively, there may be progression to an abnormal proliferation of vessels out of the plane of the retina, into the vitreous, and over the surface of the retina. o Cicatrization and traction on the retina can follow, leading to retinal detachment. Normally: At wk 16, retinal angiogenesis normally proceeds from the optic disc to the periphery, reaching the outer rim of the retina (ora serrata) nasally at about 36 wk and extending temporally by approximately 40 wk. Neo Notes

1. Premature baby with apneas, temperature instability, abdomen distended with bowel loops. Palpable, not tolerating feeds. Management / treatment x 4 lines:

o Referring to Necrotizing Enterocolitis, also consider sepsis 1) Stop feeds 2) Blood culture 3) Abdominal x-ray 4) Systemic Antibiotics (cover GP, GN, anaerobes) 5) Assist ventilation (apneas) 6) Intravascular volume replacement, cardiovascular support, correction of hematologic, metabolic, and electrolyte abnormalities Nelson's: NEC is life-threatening, generally occurs in premature neonates o onset usually in 2nd-3rd week of life but can be later; age of onset inversely related to GA o 1st signs may be nonspecific (lethargy, temp instability) or GI (abdo distension, gastric retention, bloody stools in 25%) o Diagnosis by AXR: pnematosis intestinalis, portal venous gas, pneumoperitoneum o Treatment: o supportive care, and prevention of further injury with cessation of feeding, nasogastric decompression, administration of IV fluids o surgery indicated if perforation; consider if failure of medical management, palpable mass, fixed bowel loop on radiographs o Prevention: o Less risk in breast-fed newborns o Gut stimulation protocol, followed by judicious volume advancement Necrotizing enterocolitis management: • NPO • NG tube to suction • IV fluids • CBC, blood cultures, lytes, BUN, Cr • Surgical consult • Broad spectrum antibiotics (amp, cefotax, flagyl) Intubate and ventilate npo / insert ng investigations -AXR, CBC, BCx Antibiotic coverage with Ampicillin, Gentamicin, Flagyl Surgical consultation to R/O necrotizing enterocolitis and need for surgical intervention (wait until you get your investigations)

1. Baby born at 41 wks. Meconium staining. Flat babe requiring resuscitation. Apgars 2 at 1 minute and 3 at 5 minutes and 6 at 7 minutes. a. What 5 things may you expect with this baby in the near future? b. What 2 tests at discharge, if normal, would suggest a good neurological outcome for this child?

o Seizures (due to HIE/cerebral edema, hypocalcemia, hypoglycemia, or infection) o Persistent pulmonary hypertension o Respiratory distress syndrome o Hematuria o Acute tubular necrosis o Heart failure/cardiogenic shock - - Head MRI - Normal aEEG a. What five things may you expect with this baby in the near future: • Apnea requiring intubation • Metabolic acidosis and electrolyte abnormalities • Possible hypotension requiring fluid bolus/inotropes • Seizures • Hypoglycemia • Poor feeding b. What 2 tests at discharge, if normal, would suggest a good neurological outcome for this child? • MRI • EEG What 5 things may you expect with this baby in the near future? Seizures Hypoglycemia Respiratory distress Poor feeding Hypotonia Apnea Electrolyte abnormalities What 2 tests at discharge, if normal, would suggest a good neurological outcome for this child? MRI EEG N p 719

how to differentiate jitters from seizure

o Seizures vs. Jitteriness: o jitteriness : rapid motor activities, such as a tremor or shake, that can be ended by flexion or holding the limb. Often induced by a stimulus o Seizures: don't end with tactile or motor suppression. Often involve eye deviation and autonomic changes. Neo Notes

Grades of IVH

o Severity of hemorrhage: • Grade I - bleeding isolated to the subependymal area • Grade II - bleeding within the ventricle but no ventricular dilatation • Grade III - intraventricular hemorrhage with ventricular dilatation • Grade IV - intraventricular and parenchymal hemorrhage • Ventriculomegaly is defined as mild (0.5-1 cm), moderate (1.0-1.5 cm), or severe (>1.5 cm). Neo Notes

Classification of ROP by severity

o Severity: ♣ Stage 1: demarcation line that separates vascularized from avascular retina, in plane of retina, flat and white. ♣ Stage 2: demarcation ridge; out of the plane of the retina. ♣ Stage 3 : ridge + extraretinal fibrovascular tissue ♣ Stage 4 : subtotal retinal detachment caused by traction from the proliferating tissue in the vitreous or on the retina. A. subtotal retinal detachment not involving the macula B. subtotal retinal detachment involving the macula. ♣ Stage 5: total retinal detachment. Neo Notes

Prevention of IVH

o Tenacious care in LBW infants: respiratory status, fluid and electrolyte, avoiding acidosis, hypocarbia, hypoxia, hypotension, wide fluctuations in neonatal blood pressure, and pneumothorax o A single course of antenatal corticosteroids at 24-34 wk gestation if at risk for preterm delivery: decrease the risk of death, grade III and IV IVH and PVL o Indomethacin given to VLBW infants decreases incidence of severe IVH but does not affect long-term outcome, and also can decrease cerebral blood flow transiently Neo Notes

Tx of ROP

o Treatment: for select cases, cryotherapy or laser photocoagulation of the avascular retina reduces the more-severe complications of progressive ROP. Neo Notes

Maintenance of Body Heat:

o at risk for heat loss and hypothermia: large body surface area, thin insulating layer in low birthweight and preterm infants o heat loss occurs by four mechanisms: (1) convection of heat energy to the cooler surrounding air, (2) conduction of heat to the colder materials touching the infant, (3) heat radiation from the infant to other nearby cooler objects, (4) evaporation from skin and lungs. o Heat production is augmented by increasing the metabolic rate and oxygen consumption in part by releasing norepinephrine, which results in nonshivering thermo-genesis through oxidation of fat, particularly brown fat. o Hypoglycemic or hypoxic infants cannot increase their oxygen consumption when exposed to a cold environment, and their central temperature decreases. o to reduce heat loss, it is desirable to ensure that infants are dried and either wrapped in blankets or placed under radiant warmers. o Skin-to-skin contact with the mother is the optimal method of maintaining temperature in the stable newborn. Neo Notes

o Prematurity + IUGR due to medical conditions that interfere with

o circulation and efficiency of the placenta o development or growth of the fetus o general health and nutrition of the Neo Notes

Other factors suggesting nonphysiologic cause of jaundice

o family history of hemolytic disease, pallor, hepatomegaly, splenomegaly, failure of phototherapy to lower bilirubin, vomiting, lethargy, poor feeding, excessive weight loss, apnea, bradycardia, abnormal vital signs, hypothermia, light-colored stools, dark urine, and signs of kernicterus. Neo Notes

Clinical Manifestations of PPHN

o ill in the delivery room or within the 1st 12 hr of life ♣ Initially signs of resp distress may be minimal ♣ Multiorgan involvement may be present. o Hypoxia is quite labile and often out of proportion to CXR • Diagnosis: o Hypoxia is universal and is unresponsive to 100% oxygen, but it may respond transiently to hyperoxic hyperventilation. o Pao2 gradient between a preductal (right radial artery) and a postductal (umbilical artery) site of blood sampling >20 mm Hg suggests RL shunting through the PDA, as does a saturation gradient >5% between a preductal and postductal site by pulse oximetry. o Tricuspid or mitral insufficiency may be noted on auscultation as a holosystolic murmur. The degree of tricuspid regurgitation can be used to estimate pulmonary artery pressure. Neo Notes

o Premature + appropriate weight for gestational age due to

o inability of the uterus to retain the fetus o interference with the course of the pregnancy o premature rupture of the amniotic membranes o premature separation of the placenta o multifetal gestation o undetermined stimulus to effective uterine contractions before term. o Overt or asymptomatic bacterial infection: Appropriate antibiotic therapy reduces the risk of fetal infection and may prolong gestation. Neo Notes

Respiratory Distress Syndrome (Hyaline Membrane Disease) Risk increases with

o maternal diabetes, multiple births, C/S, precipitous delivery, asphyxia, cold stress and hx of previously affected infants (highest in preterm male or white infants) o Risk reduced with PIH, heroin use, PROM, antenatal steroids Neo Notes

IVH pathogenesis

o occurs in the subependymal germinal matrix o due to immature blood vessels and poor tissue vascular support o Risk factors: prematurity, RDS, , hypoxic-ischemic or hypotensive injury, reperfusion injury of damaged vessels, ∆ cerebral blood flow, reduced vascular integrity, increased venous pressure, PTx, hypervolemia, and Htn. Neo Notes

Bronchopulmonary dysplasia

o oxygen requirement for 28 days postnatally Neo Notes

• Pulmonary interstitial emphysema

o progressive enlargement of blebs of air may that result in cystic dilatation and respiratory deterioration o If severe, precedes the development of BPD o Prevention - avoidance of high inspiratory or mean airway pressures o Treatment may include bronchoscopy in patients with evidence of mucous plugging, selective intubation and ventilation of the uninvolved bronchus, oxygen, general respiratory care, and HFV. Neo Notes

Discharge from the Hospital Criteria

o taking all nutrition by nipple, either bottle or breast (Some medically fragile infants may be discharged home while receiving gavage feedings after the parents have received appropriate training and education) o Growth of approximately 30 g/day. o Temperature stable in an open crib. o no recent episodes of apnea or bradycardia o no IV meds o Stable infants with BPD may be discharged on home O2 with oximetry and close follow-up o ROP screen in: ♣ all infants with birthweight <1,500 g ♣ birthweights between 1,500 and 2,000 g with an unstable clinical course requiring oxygen o Hearing test in all children o Check blood pressure if had indwelling vascular catheter o Check hemoglobin/hematocrit o Discharge if stable and weight 1,800-2,100 g o Home Care: o Parental education o At least one home care visit o Early developmental intervention improves cognitive outcomes up until preschool Neo Notes

other congenital defects associated with Hirschsprung's

other congenital defects, including Down, Smith-Lemli-Opitz, Waardenburg, cartilage-hair hypoplasia, and congenital hypoventilation syndromes and urogenital or cardiovascular abnormalities. Seen in association with microcephaly, MR, and abnormal facies; with autism; or with cleft palate, hydrocephalus, and micrognathia. Neo Notes

Treatment of symptomatic polycythemic newborns is _____. The Hct level at which to perform a partial exchange transfusion in an asymptomatic infant is unclear but should not be considered if the Hct is ____. Partial exchange will lower the Hct and viscosity and improve acute symptoms.

partial exchange transfusion with normal saline. ≤70-75% Neo Notes

• Infants with RDS who have persistent or recurrent oxygen and ventilatory requirements (defined as _____) within the first _____ should have repeated doses of surfactant and it may be given as early as ____ after the initial dose or, more commonly, _____ after the initial dose Administering more than _____ doses has not been shown to have a benefit

persistent or recurrent oxygen requirement of 30% or more 72 h of life 2 h 4 h to 6 h three CPS surfactant

Early-onset neonatal bacterial sepsis (EOS) - The CSF culture in early onset sepsis work up may be negative when the LP is performed after antibiotics have been started but ______ may be observed with meningitis. In term infants, a cerebrospinal fluid (CSF) white blood cell (WBC) count of _____ is considered abnormal What else should be cultured?

pleocytosis, low glucose and/or elevated protein concentration >20-25 cells/mm3 Cultures of urine, gastric aspirates and body surface have limited value in the evaluation for EOS and are not recommended for newly born infants CPS term infants at increased risk for early onset bacterial sepsis

The finding of _____ confirms the clinical suspicion of NEC and is diagnostic; _____ of patients have _____ when treatment is started. ______ is a sign of severe disease, and pneumoperitoneum indicates a perforation.

pneumatosis intestinalis (air in the bowel wall) 50-75% of patients have pneumatosis Portal venous gas Neo Notes

Indications for surgery for NEC include

pneumoperitoneum or positive abdominal paracentesis (stool or organism on Gram stain from peritoneal fluid). Failure of medical management, a single fixed bowel loop on roentgenograms, abdominal wall erythema, or a palpable mass are relative indications for exploratory laparotomy. Ideally, surgery should be performed after intestinal necrosis develops, but before perforation and peritonitis occurs. The role of peritoneal drainage in lieu of laparotomy in a patient with perforation secondary to NEC remains to be determined. Peritoneal drainage may be helpful for patients in extremis with peritonitis who are too unstable to undergo surgery. In many patients with isolated intestinal perforation treated by drainage, no further surgical procedure is needed; a small subgroup may require later surgery to repair an intestinal stricture or fistula. Neo Notes

The blood smear in hemolytic anemia of the NB typically shows ______ and the reticulocyte count is ______. _______ may develop in severe cases.

polychromasia and a marked increase in nucleated RBCs increased Thrombocytopenia Neo Notes

Phototherapy is contraindicated in the presence of

porphyria Neo Notes

The incidence of neonatal polycythemia is increased in

postmature (3%) vs term (1-2%) infants; in SGA (8%) vs LGA (3%) vs AGA (1%) infants; during the 1st day of life (peak, 2-3 hr); in the recipient infant of a twin-twin transfusion; after delayed clamping of the umbilical cord; in infants of diabetic mothers; in trisomy 13, 18, or 21; in adrenogenital syndrome; in neonatal Graves disease; in hypothyroidism; in infants of hypertensive mothers or those on propranolol; and in Beckwith-Wiedemann syndrome. Neo Notes

The most important neonatal factor predisposing to infection is

prematurity. Neo Notes

The main concerns involved with iNO toxicity include ______

production of NO2 and methemoglobin, decreased platelet aggregation, increased risk of bleeding and surfactant dysfunction Toxicities NO2 production Methemoglobin production Decreased platelet aggregation Increased bleeding Surfactant dysfunction **no toxicity at starting 20-40ppm doses NO2: NO2 toxic byproduct of iNO metabolism in high oxygen concentrations esp in circuit NO2 is cytotoxic Cause pulmonary injury at [>5ppm] At no time did the NO2 concentration become a problem at a dose of <80ppm NO2 production: One of the concerns with the use of iNO is the chemical reaction that converts NO to NO2 in high oxygen concentrations in the ventilator circuit and airways. NO2 is cytotoxic and can cause pulmonary injury at concentrations >5 ppm. In the iNO clinical trials, the peak level of NO2 was 0.8±1.2 ppm; no gas was discontinued due to toxicity. iNO at doses <80 ppm is not associated with significant NO2 levels. Methemoglobin: NO is absorbed into the blood where its binds to the ion of the heme protein, subsequently producing nitrosyl-hemoglobin, which is oxidized to methemoglobin with the release of nitrates. Methemoglobin levels should be measured frequently and kept <2.5%. With standard iNO doses, methemoglobinemia is unusual. In the Neonatal Inhaled Nitric Oxide Study Group (NiNOS) trial, the peak level of methemoglobin was 2.4%±1.85%. Premature infants are at a higher risk of methemoglobin toxicity due to reduced levels of methemoglobin reductase, but at iNO doses of <20 ppm, methemoglobin levels were not elevated CPS iNO

Indications for delivery in hemolytic dz of the NB include _____.

pulmonary maturity, fetal distress, complications of PUBS, or 35-37 wk of gestation Neo Notes

Inspissated bile syndrome

refers to the rare occurrence of persistent icterus in association with significant elevations in direct and indirect bilirubin in infants with hemolytic disease. The cause is unclear, but the jaundice clears spontaneously within a few weeks or months. Neo Notes

Contraindications to oral feeding

respiratory distress, hypoxia, circulatory insufficiency, excessive secretions, gagging, sepsis, central nervous system depression, severe immaturity, or signs of serious illness. Neo Notes

Chorioamnionitis

results from microbial invasion of amniotic fluid. RFs: ROM>18h, traumatic and premature delivery. Aspiration or ingestion of bacteria can lead to congenital pneumonia or systemic infection. Neo Notes

Conventional treatment of ROP is ________, directed toward the _____ part of the retina, with the goal of decreasing production of angiogenic growth factors. Ideally, treatment should be initiated for type 1 ROP within ____ of its detection How should ROP be treated? CPS Recommendation: Who should have retinal ablation therapy?

retinal ablation (The effectiveness of laser photocoagulation is well established) avascular 72 h tx: Retinal ablation: Cryotherapy and laser photocoagulation Purpose is to decrease angiogenic growth factor production in non-vascularized areas Retinal Ablation: Considered: Zone 1 any stage w/ plus disease Zone 1 stage 3 with or without plus disease Zone 2 stage 2 or 3 w/ plus disease For sure: Zone 1 or zone 2 w/ stage 3 and plus Do it within 72h of exam CPS ROP

Early-onset neonatal bacterial sepsis (EOS) is

sepsis occurring within the first seven days of life. CPS term infants at increased risk for early onset bacterial sepsis

After ______, dexamethasone has been shown to decrease the rate of _____ age with less impact on neurodevelopmental outcome.

seven days of life CLD at 36 weeks' postmenstrual No trials have examined whether the benefits of corticosteroids outweigh the adverse effects for infants at high risk of, or with, severe CLD. While routine dexamethasone therapy of all ventilated infants is not recommended, clinicians may consider a short course of low-dose dexamethasone for individual infants at high risk of or with severe CLD. There is no evidence that hydrocortisone is an effective or safe alternative to dexamethasone and little evidence to support routine use of inhaled corticosteroids for prevention or treatment. The impact of dexamethasone in both decreasing rates of CLD and increasing adverse neurodevelopmental outcome was much greater than that of hydrocortisone. In summary, the benefits of early corticosteroid therapy, in particular dexamethasone, did not appear to outweigh the adverse effects Because late dexamethasone (>7d) appeared to have both beneficial and harmful effects, it was suggested that its use be reserved for infants who could not be weaned from mechanical ventilation A systematic review designed to determine whether death as well as pulmonary and neurodevelopmental sequelae were modified by cumulative dexamethasone dosage concluded that higher doses were more effective in reducing the risk of mortality and BPD, but that lower dosage was not associated with a reduction in neurodevelopmental sequelae.[22] At present, there is insufficient evidence to demonstrate the safety of routine low-dose dexamethasone use. Hydrocortisone has been proposed as an alternative to dexamethasone because it is a less potent glucocorticoid and some have speculated that it may have fewer side effects. Furthermore, in addition to anti-inflammatory actions, hydrocortisone may mitigate against adrenal insufficiency experienced by some preterm infants and, thus, decrease the incidence of CLD. Overall, hydrocortisone therapy did not decrease mortality, the need for oxygen at 36 weeks' PMA, combined outcome of death or CLD, percent of survivors discharged home on oxygen, or the rate of failure to extubate. Follow-up data from five trials showed no significant difference in the rate of cerebral palsy or of combined mortality and cerebral palsy between hydrocortisone and a placebo. The seven trials used a variety of inhaled corticosteroids (beclomethasone, fluticasone, budesonide, flunisolide) for approximately two to four weeks. Compared to placebo, there were no differences in CLD at 36 weeks' PMA, death by 36 weeks PMA, death or CLD at 36 weeks, as well as no differences in side effects, including hyperglycemia, hypertension and infection. It is unclear whether the benefits of late dexamethasone therapy outweigh the adverse effects for infants who are at high risk of CLD or for those with prolonged ventilator-dependence. If clinicians choose, after parental agreement, to treat an infant who is ventilator-dependent, at risk of severe CLD or who has severe CLD, low-dose dexamethasone (initial dose 0.15 mg/kg/day to 0.2 mg/ kg/day) should be used in tapering doses over a short course (seven to 10 days). Inhaled corticosteroids may be considered as an alternative to dexamethasone, but the most effective dose and duration of therapy is not known. (Grade C recommendation) CPS Postnatal corticosteroids to treat/prevent BPD

What are the risks of exogenous surfactant therapy?

short-term risks of surfactant replacement therapy include bradycardia and hypoxemia during instillation as well as blockage of the endotracheal tube There may also be an increase in pulmonary hemorrhage following surfactant treatment; however, mortality ascribed to pulmonary hemorrhage is not increased and overall mortality is lower after surfactant therapy. the large increase in functional residual capacity due to the recruitment of lung volume but unless administered pressures are reduced, overdistension can occur. Hyperventilation with very low PCO2 can also sometimes accidentally occur. Thus, weaning of administered airway pressures and ventilator settings should be expected within a few minutes of the administration of natural surfactants, and the caregivers must be aware of the nature and speed of these changes. CPS surfactant

By _____, 50% of boys can retract their foreskins, although the process of separation may not be complete until _____: 95% of boys have retractile foreskin by 17 years of age

six years of age - by 6, 1/2 still stick puberty CPS circumcision

Injury to the spine/spinal cord is rare, but can be devastating, including vertebral #, spinal cord transaction, hemorrhage and edema. It is most often due to ____. The injury usually occurs at ____ with ____ presentations and ____ with ____ presentations.

strong traction during a difficult delivery C4 cephalic C8-T1 breech Neo Notes

It is recommended that _____ be treated with intravenous dextrose solution

symptomatic, hypoglycemic infants (and asymptomatic infants who have failed to respond to enteral supplementation) CPS Hypoglycemia

What are the physiological responses to intubation? Intubation/laryngoscopy causes

systemic and pulmonary hypertension, bradycardia, intracranial hypertension and hypoxia. CPS Intubation

In seizures, if the EEG at the time of discharge is not paroxysmal, then medications are usually ______

tapered. Neo Notes

An LP must be done when working up for early onset sepsis whenever _____

the blood culture is positive. CPS term infants at increased risk for early onset bacterial sepsis

iNO improves oxygenation and decreases _____ in infants _____ gestational age at birth. Its role in managing preterm infants ______ gestational age is _______. The recommended starting dose is _____ with gradual reduction of the dose following improvement in oxygenation.

the combined outcome of death or need for extracorporeal membrane oxygenation (iNO has no impact on adverse neurodevelopmental outcomes) ?35 weeks' <35 weeks' not yet established 20 ppm iNO is usually started in infants with an OI >20 to 25, or when the PaO2 remains less than 100 mmHg, despite optimal ventilation with 100% oxygen iNO has a short half-life (2 s to 6 s) and has been used in newborns at doses of 1 ppm to 80 ppm, which is then titrated to achieve the desired effect. Doses greater than 40 ppm have the potential to increase toxicity without additional benefits. Current evidence supports the use of iNO for infants ≥35 weeks' gestational age at birth with hypoxemic respiratory failure who fail to respond to appropriate respiratory management. CPS iNO

dx of hemolytic anemia of the newborn

the direct Coombs test is usually positive, and anemia is generally present. Neo Notes

Post-surfactant vent management

the large increase in functional residual capacity due to the recruitment of lung volume but unless administered pressures are reduced, overdistension can occur. Hyperventilation with very low PCO2 can also sometimes accidentally occur. Thus, weaning of administered airway pressures and ventilator settings should be expected within a few minutes of the administration of natural surfactants, and the caregivers must be aware of the nature and speed of these changes. many infants can be very rapidly weaned and extubated to nasal continuous positive airway pressure (CPAP) within 1 h of intubation and surfactant administration. To do this, the premedication used for intubation should only cause a brief duration of respiratory depression and staff must be trained and skilled in rapid ventilator weaning. Such weaning is often performed with few or no blood gases, relying instead on the infant's clinical condition and spontaneous respiratory effort and with consideration of the oxygen requirements as determined from pulse oximetry and sometimes with the use of transcutaneous CO2 measurements. There is currently no proof that a rapid wean and extubation approach improves long-term outcomes compared with the more traditional weaning approach. CPS surfactant

Erythroblastosis fetalis is caused by

the transplacental passage of maternal antibody (Mom type O has Anti a and Anti B Antibodies) active against paternal RBC antigens of the infant and is characterized by an increased rate of RBC destruction. Significant disease is associated primarily with the D antigen of the Rh group and with incompatibility of ABO factors. When small quantities of Rh+ fetal blood containing D antigen inherited from an Rh+ father enter the maternal circulation, antibody formation against D antigen may be induced in the unsensitized Rh- recipient mother. Once sensitization has taken place, considerably smaller doses of antigen can stimulate an increase in antibody titer. Initially, a rise in IgM antibody occurs, which is later replaced by IgG antibody; the latter readily crosses the placenta and causes hemolytic manifestations. Hemolytic disease rarely occurs during a first pregnancy because transfusion of Rh+ fetal blood into an Rh- mother occurs near the time of delivery, too late for the mother to become sensitized and transmit antibody to her infant before delivery. The overall incidence of isoimmunization of Rh- mothers at risk is low, with antibody to D detected in <10%; only about 5% ever have babies with hemolytic disease. When the mother and fetus are also incompatible with respect to group A or B, the mother is partially protected against sensitization by the rapid removal of Rh+ cells from her circulation by her preexisting anti-A or anti-B, which are IgM antibodies and do not cross the placenta. Once a mother has been sensitized, her infant is likely to have hemolytic disease. The severity of Rh illness worsens with successive pregnancies. Neo Notes

Infants who are at a significant risk for RDS should receive prophylactic natural surfactant therapy as soon as ______

they are stable within a few minutes after intubation However, giving the surfactant as soon as possible once stabilization has occurred seems to be important. The open study of infants at high risk of or with respiratory insufficiency - the role of surfactant (OSIRIS) [52] demonstrated that the combined incidence of death or BPD was reduced by about 11% when surfactant was given at a mean postnatal age of 2 h rather than 3 h (RR=0.89, 95% CI 0.79 to 1.00, evidence level 1b), showing that even fairly short delays in therapy worsen outcomes CPS surfactant

Prenatal US w/ CDH

ultrasound polyhydramnios, chest mass, mediastinal shift, gastric bubble or a liver in the thoracic cavity, and fetal hydrops Neo notes

The risk of CMV infection is reduced by ______

universal leukoreduction leuCCCoreduced vs W-irradiate = irriate those WBC's CPS Red blood cell transfusion in newborn infants

very premature infants - Does a low hemoglobin concentration threshold for blood transfusion safely reduce the need for transfusions?

using the lower threshold of Hb concentration when transfusing is recommended, always recognizing the need for individualized targets based on the patient's health status. For infants in the first and second week of life, minimum hemoglobin levels of 100 g/L and 85 g/L, respectively, are recommended. Infants requiring respiratory support may require transfusions at a higher threshold. In view of the concerns regarding the increased risk for cognitive delay, clinicians should avoid using thresholds lower than those tested in published RCTs CPS Minimizing blood loss and the need for transfusions in very premature infants

Dx of ABO incompatibility

weakly to moderately positive direct Coombs test result, and spherocytes in the blood smear, which may suggest the presence of hereditary spherocytosis. Hyperbilirubinemia is often the only other laboratory abnormality. Reticulocytes may be increased to 10-15%, with extensive polychromasia and increased numbers of nucleated RBCs. In 10-20% of affected infants, the unconjugated serum bilirubin level may reach 20 mg/dL or more unless phototherapy is administered. Neo notes

When can you use conventional ptx according to CPS?

when TSB [] 35-50micrmol/L below threshold CPS Hyperbili

Does folic acid reduce the risk of neural tube defects period?

yes CPS Folic Acid and Neural tube defects

Are late prems more susceptible to temperature instability? How should you prepare for a late prems thermal instability?

yes Poor thermoregulation Incr surface area to volume ratio Make sure home environment safe ~18deg C for lightly dressed infant sleeping in cot NO HEAVIER CLOTHING TO MAKE UP FOR COLD ENVIRONMENT! (INCR SIDS) CPS facilitating discharge of late preterms infants

What is the rate of FAS/FAE in industrialized countries? What are three reasons why FAS/FAE is so hard to identify?

~1-3/1000 (FAE may be more frequent) Hard to diagnose at birth Confused with other health problems Diagnosis made considering physical exam and past medical history CPS Preventing FAS

Why is supplementation not enough?

~50% of pregnancies in Canada are UNPLANNED! Can't have everyone taking it everyday! CPS Folic Acid and Neural tube defects

Drugs contraindicated in breast-feeding

· Immunosuppressants · Antineoplastic agents · Cyclophosphamide · Doxorubicin · Radiopharmaceuticals · Bromocriptine · Diethylstilbestrol · Iodides · Methimazole · Thiouracil · Chloramphenicol · Clozapine · Lithium · Amphetamines · Cocaine · Heroin · Phencyclidine (PCP) · Ergots · Gold salts neo notes

Prevention of Mec aspiration

• : rapid identification of fetal distress and prompt delivery o Routine intrapartum nasopharyngeal suctioning in pregnancies with meconium-stained amniotic fluid does not reduce the risk for MAS and, on rare occasions, may cause nasopharyngeal trauma or cause a cardiac arrhythmia Neo Notes

Canadian Pediatric Review 2018 Perinatal-Neonatal Medicine When to consider CPAP

• Babies with laboured breathing or persistent cyanosis • NOT for apnea in L&D • If ongoing positive pressure ventilation is required - PEEP of 5-7 cm of water should be used • Value of CPAP - Decreases rate & duration of ventilation - Decrease surfactant use

Canadian Pediatric Review 2018 Perinatal-Neonatal Medicine Respiratory disease in the newborn: clues

• By onset: - Early: TTN, RDS, severe malformations (CHD), pulmonary hypoplasia - Evolving: pneumothorax, RDS, MAS, PPHN, pneumonia, lobar emphysema - Later onset: TEF, CHD, pneumonia • By clinical clues - Prematurity, uncontrolled diabetes: RDS - Term, elective C-section: TTN - History oligohydramnios: pulmonary hypoplasia - History polyhydramnios or ++ secretions: TEF - Infection risk factors (GBS positive, chlamydia); 'well' then onset of resp symptoms: pneumonia - Acute, asymmetric features, systemic sx: pneumothorax - DDx of cardiac symptoms: PPHN

very premature infants - Is there evidence to support the use of in-line or other continuous monitoring techniques for reducing PRBC transfusions? Is there evidence to support limiting routine blood sampling?

• Caregivers should scrutinize and limit blood sampling to the minimum required for safe clinical care. Using marked tubes to indicate the minimum volume to collect, and clustering blood samples to reduce the number of skin breaks and painful procedures are recommended CPS Minimizing blood loss and the need for transfusions in very premature infants

Canadian Pediatric Review 2018 Perinatal-Neonatal Medicine RBC transfusions in newborns CPS 2015, 2016

• Delayed cord clamping reduces need for transfusion in preterms • Avoid excess bloodwork, use non-invasive monitoring, EPO not routine • Transfusion Risks: - Transfusion-transmitted infections (viral, bacterial, parasitical, prional), - Leukocytes (including immunomodulation, graft-versus-host disease; transfusion-related acute lung injury and alloimmunization), - Acute volume or electrolyte disturbances, and - Blood group incompatibilities (often mistransfusion errors) • Volumes: 20 ml/kg, at 5 ml/kg/h, - For hemorrhagic shock: push 20ml then 10m/kg/h (watch for hyperkalemia) • Iron (2-3 mg/kg/day) once on full feeds; treatment 4-6 mg/kg/d • Thresholds (TABLE 1 anemia of prematurity) Suggested hemoglobin levels and hematocrit thresholds for transfusing infants with anemia of prematurity Postnatal age Respiratory support* No respiratory support Week 1 115 (35) 100 (30) Week 2 100 (30) 85 (25) Week 3 and older 85 (25) 75 (23) Data presented as hemoglobin, g/L (hematocrit, %). *Respiratory support is defined as an inspired oxygen requirement in excess of 25% or the need for mechanical increase in airway pressure (Adapted from reference 6)

Breast milk jaundice

• Develops in 2 percent of breast-fed term infants after day 7 o Maximum bilirubin levels in 2-3rd week of life, may persists up to 10 wks • Presence of glucuronidase in some breast milk • If BF discontinued - bili falls, with resumption levels seldom return to high levels o Phototherapy may be of benefit Neo Notes

Early-onset sepsis/infections: acquired ______

• Early-onset infections: acquired before or during delivery o Coagulase-negative staph most frequent neonatal nosocomial pathogen • The most important neonatal factor predisposing to infection is prematurity. Neo Notes

Canadian Pediatric Review 2018 Perinatal-Neonatal Medicine Neonatal adaptation: Fluids, growth, nutrition

• Fluids - Newborn: TBW 75% at term (60% adult) - Increased insensible losses with: decreasing GA, radiant warmers, phototherapy - TFI: 60 [term] - 80 [preterm] cc/kg/day • Kidneys - Decreased GFR, decreased concentrating ability - 99% have urine output in 1st 24hrs - Normal urine output: 1-2cc/kg/h - Renal insufficiency caused by: hypoperfusion, urinary tract obstruction, congenital anomalies, vascular (thrombosis / stenosis) • Antenatal renal insufficiency: - oligohydramnios, Potter's, pulmonary hypoplasia • NPO infants: - Term: D10W Surgical: D10W+Na; - Preterm <1500g: Electrolyte free TPN + Ca • Term baby at end of 1st week: 100 kcal/kg/day - Formula / breast milk: 20 kcal /oz (30ml) • Breastfeeding - Benefits: prevent SIDS, enhance cognitive development, social, immune / allergy / infection (IgA, WBC, bifidus factor) - Contraindication: Few drugs (street, alcohol++, chemotherapy, immunosuppressants), HIV, localized HSV. Hep B, CMV ok. • Growth - WHO growth charts for Canada - Weight loss <10% 1st week, regain D10-14 - Weight gain: "1 oz/day except on Sunday" - HC: grows 0.5cm/week for 1st 2mos • Then ~1cm/month from 2-6mos • 96% term infants pass meconium by 24hrs - Obstructive problems: • Meconium plug, Hirschprung's, meconium ileus (CF), imperforate anus, small left colon (IDM)

o acute deterioration Day 2 or 3 with apnea, pallor, cyanosis; poor suck; abnormal eye signs; a high-pitched, shrill cry; muscular twitching, convulsions, or decreased muscle tone; metabolic acidosis; shock; and a decreased hematocrit

• IVH: o majority asymptomatic o rarely present at birth; 75% within 3 days of life. o Small amount will have late hemorrhage between days 14 and 30. Neo Notes

Risk estimation for anticipated mortality or long-term NDD: Extremely high likelihood of mortality or severe NDD* Suggested level of care: Palliative care Clinical examples that usually meet the risk estimation

• Infant born at 22 weeks GA, irrespective of additional risk factors*** • Infant born at 24 weeks GA, with an estimated weight of 350 g CPS Counselling and management for anticipated extremely preterm birth

Managing newborns exposed to C trachomatis:

• Infants born either vaginally or by Caesarian section to mothers with an untreated chlamydia infection should be closely monitored for symptoms (eg, conjunctivitis, pneumonitis) and treated if infection occurs. Routine cultures should not be performed on asymptomatic infants. • Prophylaxis of exposed newborns is not recommended because of the association of macrolides with pyloric stenosis, but may be considered when infant follow-up cannot be guaranteed CPS Preventing ophthalmia neonatorum

Late-onset sepsis/infections: acquired ______

• Late-onset infections: acquired after delivery in the nursery, NICU, or the community Neo Notes

Early signs of kernicterus

• Lethargy, poor feeding, and loss of the Moro reflex are common initial signs. o Overt neurologic signs have a grave prognosis: >75 percent of such infants die and 80 percent of affected survivors have choreoathetosis, involuntary muscle spasms, MR, deafness, and spastic quadriplegia Neo Notes

Management of CDH

• Management: o Human studies have shown no benefit for in utero repair o Rapid intubation, sedation and possibly paralysis ♣ Do not give prolonged IPPV ♣ Gentle ventilation with permissive hypercapnea ♣ Maintain oxygenation without inducing barotraumas o Surgical repair Neo notes

Canadian Pediatric Review 2018 Perinatal-Neonatal Medicine Risk of IUGR

• Maternal - Hypertensive, preeclampsia - Renal disease - Diabetes - Antiphospholipid syndrome - Severe nutrition deficiency - Smoking / substances - Maternal hypoxia (CHD, lung) • Fetal - Multiple gestation - Placental abnormalities - Infection (viral) - Congenital anomaly, chromosomes

Maternal and neonatal risk factors for early onset bacterial sepsis in term infants What are maternal risk factors for GBS? How often were they present? How many invasive GBS infants did it identify?

• Maternal intrapartum GBS colonization during the current pregnancy • GBS bacteruria at any time during the current pregnancy • A previous infant with invasive GBS disease • Prolonged rupture of membranes ≥18 h • Maternal fever (temperature ≥ 38oC)/Signs of chorioamnionitis ROM >18h Present in 22% of mothers = 0.2% infant dz prevented Catch only 50% of infants who developed invasive GBS disease CPS term infants at increased risk for early onset bacterial sepsis

Pathophysiology of meconium aspiration pneumonia

• Meconium inactivates surfactant and obstructs small airways • Clinical Manifestations: o Resp distress and cyanosis o Pneumothorax/pneumomediastinum o usually improves within 72 hr, but tachypnea may persist for days to weeks o If requiring ventilation high risk of mortality Neo Notes

• Most dramatic events in growth and dev't occur before birth

• Most dramatic events in growth and dev't occur before birth o Transformatin of fertilized egg embryo fetus o Elaboration of nervous system o Emergence of behaviour Neo Notes

very premature infants - Does the use of noninvasive monitoring in preterm infants reduce the need for transfusions?

• Noninvasive CO2 monitoring should be considered when caring for ventilated preterm infants. Exercise caution in infants with significant pulmonary disease because of potential for greater inaccuracy of note - In infants with significant pulmonary disease, the end-tidal CO2 partial pressure of CO2 may be quite discrepant due to large dead space volumes. Using noninvasive bilirubin monitoring devices before phototherapy is recommended CPS Minimizing blood loss and the need for transfusions in very premature infants

Breastfeeding jaundice

• Occurs in the first week of life in breast-fed infants o due to decreased milk intake with dehydration • Prevention: frequent breast-feeding (>10x/day), rooming-in, discouraging D5W or water supplementation and ongoing lactation support Neo Notes

Canadian Pediatric Review 2018 Perinatal-Neonatal Medicine Newborn preventive care

• Ophthalmic prophylaxis: - Erythromycin ophthalmic (not routinely effective!) • Vitamin K prophylaxis 0.5 (<1500g) 1mg (>1500g) IM - Oral alternative if parents refuse: • 2mg at 1st feed, repeat at 2-4 weeks and 6-8 weeks • Early hemorrhagic disease of the newborn - 1st 24 hours, due to maternal medication • Classic hemorrhagic disease of the newborn - 1:400, bleeding 1st wk of life - Vitamin K deficiency, Maternal coumadin may still be a risk factor • Late hemorrhagic disease - 1-7:100 000, bleeding 2nd-12th wk of life (3-8 weeks in CPS) - Exclusive breastfeeding, no Vit K (or 1 oral dose!!), fat malabsorption • Tx: Vitamin K, FFP

How does PVL present

• PVL o usually asymptomatic until later infancy: spastic motor deficits. Neo Notes

PVL pathogenesis

• PVL : o focal necrotic lesions in the periventricular white matter. o Causes CP/motor abnormalities because corticospinal tracts descend through the periventricular white matter Neo Notes

Neonatal ocular prophylaxis with erythromycin, the only agent currently available in Canada for this purpose, may no longer be useful and, therefore, should not be routinely recommended.

• Paediatricians and other physicians caring for newborns, along with midwives and other health care providers, should become familiar with local legal requirements concerning ocular prophylaxis. CPS Preventing ophthalmia neonatorum

Pathophysiology of PPHN

• Pathophysiology: o Persistence of the fetal circulatory pattern of RL (pulmonary systemic) through the PDA & PFO after birth is due to excessively high pulmonary vascular resistance. May be due to: ♣ maladaptive from an acute injury (no normal vasodilation in response to increased oxygen and other changes after birth) ♣ increased pulmonary artery medial muscle thickness and extension of smooth muscle layers into the usually nonmuscular, more peripheral pulmonary arterioles in response to chronic fetal hypoxia ♣ pulmonary hypoplasia (diaphragmatic hernia, Potter syndrome) ♣ obstruction as a result of polycythemia or TAPVR, or alveolar capillary dysplasia. Neo Notes

very premature infants - Is there evidence to support the use of point-of-care testing techniques for reducing PRBC transfusions?

• Point-of-care testing should be considered in preterm infants. This strategy requires support from the institution's biochemistry laboratory and compliance with accreditation standards CPS Minimizing blood loss and the need for transfusions in very premature infants

Prognosis in RDS

• Prognosis - less than 10 percent mortality, much better with increased gestation and BW >1500 g o Normal lung function in most infants in longterm (unless severe) Neo Notes

Prognosis of pulmonary hemorrhage in neonates

• Prognosis is very poor, death occurs in the first 48 hrs in 65 percent of infants wo come to autopsy Neo Notes

Prognosis of CDH

• Prognosis: overall survival is 67% o Poor prognosis: associated major anomaly, symptoms before 24 hr of age, severe pulmonary hypoplasia, herniation to the contralateral lung, and the need for ECMO. o Long-term: BPD, obstructive and restrictive lung disease, GERD(>50% with 25% requiring an antireflux procedure), intestinal obstruction (20%), delayed growth in the 1st 2 yr of life with "catch up" by the time they are 2 yr old, neurocognitive defects, pectus excavatum and scoliosis. Neo Notes

Dx of CDH

• RESP DISTRESS (immediately, or after honeymoon period of up to 48 hr) o CXR and NG tube is all that is usually required to confirm dx Neo notes

very premature infants - Does treatment with recombinant human erythropoietin safely reduce the need for PRBC transfusions?

• Routine use of erythropoietin (SC, IV or PO) is not recommended. Dosage and delivery method should be individualized if parents withhold consent to transfuse blood Overall, although erythropoietin use has been associated with a reduced number of transfusions, the number of donor exposures may not be reduced and concern about a higher risk for RoP remains. Therefore, erythropoietin use should be individualized for patients whose parents have refused other blood products. CPS Minimizing blood loss and the need for transfusions in very premature infants

Canadian Pediatric Review 2018 Perinatal-Neonatal Medicine Risk of preterm delivery

• SES status - <20 or >40 years - Very low SES, Low BMI • Past Gyne/OB - Pyelonephritis - Uterine / cervical anomalies - Multiple abortions - Preterm delivery • Lifestyle - >10 cigarettes/day - Heavy work • Pregnancy - Multiples

Etiology of TTN

• Secondary of slow absorption of fetal lung fluid decreased pulmonary compliance and tidal volume and increased dead space. Neo Notes

Delayed presentation in CDH

• Small number have delayed presentation o Vomiting due to intestinal obstruction o Mild respiratory symtoms o Well described as presenting after a documented GBS sepsis Neo notes

very premature infants - Does enteral iron supplementation reduce the need for blood transfusions?

• Supplemental iron does not appear to reduce the need for blood transfusion in preterm neonates. For treatment beyond two months of age, however, iron supplements may improve hematological values and help to avoid iron deficiency anemia. Supplementation with physiological doses (2 mg/kg/day to 3 mg/kg/day, or 4 mg/kg/day to 6 mg/kg/day in newborns who are iron deficient) should be considered However, administering supplemental iron in physiological doses (2 mg/kg/day to 3 mg/kg/day, or 4mg/kg/day to 6 mg/kg/day if an infant is believed to be iron deficient), once full oral feeds have been achieved, has been demonstrated to improve hemoglobin and ferritin levels after two months of treatment and to reduce the risk for iron-deficiency anemia (along with an associated risk for lower cognitive function) in the first year of life. Human milk fortifier and enriched preterm formulas provide at least 2 mg/kg/day of iron to infants who are not fluid restricted CPS Minimizing blood loss and the need for transfusions in very premature infants

Tx of TTN

• Treatment is supportive (no evidence of use of furosemide!) • Secondary of slow absorption of fetal lung fluid decreased pulmonary compliance and tidal volume and increased dead space. o severe cases result in PPHN Neo Notes

Tx of PPHN

• Treatment: o Correct any predisposing disease o Improve poor tissue oxygenation (supportive care) ♣ Often require sedation, use of paralytics is controversial o Nitric Oxide - relaxes vascular smooth muscle ♣ Starting dose = 20 ppm, wean slowly ♣ High doses methemoglobinemia ♣ An OI > 40 that is unresponsive to nitric oxide inhalation predict a high mortality rate and are indications for ECMO. OI = (MAP × Fio2 × 100) ÷ Postductal Pao2 • Prognosis: related to associated HIE and ability to reduce PVR Neo notes

Tx of pulmonary hemorrhage

• Treatment: blood replacement, PEEP, suctioning to clear the airway, intratracheal administration of epinephrine, and, in some cases, HFV o administration of exogenous surfactant after the bleeding has occurred can improve lung compliance since the presence of intra-alveolar blood and protein can inactive surfactant. Neo Notes

Treatment of Mec aspiration

• Treatment: depressed ETT suctioning, then supportive and standard care for resp distress o Surfactant if requiring ventilation (may decrease need for ECMO) o May be complicated by PPHN o May benefit from HFV, iNO or ECMO Neo Notes

Canadian Pediatric Review 2018 Perinatal-Neonatal Medicine Newborn screening

• Universal hearing screen (Oto-acoustic emissions) - Risks: prematurity, hyperbilirubinemia, infection (TORCH), HIE, genetic syndrome, ototoxic drugs, - Results: Pass, fail, refer (repeat in community - further testing) • Blood spot at >24 hours of age - Screen for: Endocrine (congenital hypothyroidism, adrenal hyperplasia); Heme (Sickle cell, beta-thalassemia); Metabolic (galactosemia, fatty acid, amino acid and organic acid disorders); Cystic fibrosis - Positive screen: • further testing needed to confirm diagnosis • Ex. Repeat TSH/T4 and reassess • Bilirubin at 24 hours (see later) & 48 hours (late preterm) • O2 saturation: screening for congenital heart disease For reference Pulse oximetry screening • Improves detection rates for critical congenital heart disease • Recommended for all term / latepreterm newborns • After 24 hours post-birth to lower false positives • Right hand and one foot - lower false negative - 99% specificity - 76% specificity • FAIL screening -> clinical evaluation - could include consultation with a paediatrician - If a cardiac diagnosis cannot be excluded, referral to a paediatric cardiologist for consultation and echocardiogram

Transient Tachypnea of the Newborn Characterized by:

• Usually follows normal pregnancy and delivery at term • Characterized by: o Early onset of tachypnea and resp distress o Minimal oxygen requirements (<40%) o Rapid recovery within 3 days o severe cases result in PPHN • Treatment is supportive (no evidence of use of furosemide!) Neo Notes

Physiologic Jaundice (icterus neonatorum) occurs o Search to determine cause should be made if:

• Visible on 2-3rd day, usually peaking between 2-4th day ♣ Appears in the 1st 24-36 hr of life ♣ Serum bilirubin is rising at a rate faster than 75 mmol/L/24 hr ♣ Serum bilirubin is > 200 mmol/L in full-term infants (especially in the absence of risk factors) or 170-200 mmol/L in preterm infants ♣ Persists after 10-14 days of life ♣ Direct biliruvin is > 30 mmol/L Neo Notes

w/ HIE The greatest risk of adverse outcome is seen in infants with

• fetal acidosis (pH <7.0), a 5-min Apgar score of 0-3, hypoxic-ischemic encephalopathy (altered tone, depressed level of consciousness, seizures), and other multiorgan system signs. Hypoxic-ischemia Encephalopathy - important cause of permanent damage to CNS tissue that may result in neonatal death or manifest later as cerbral palsy or developmental delay • 15-20% of infants with HIE die in the neonatal period • 25-30% have permanent neurodevelopmental abnormalities (CP, MR) Neo Notes

Large for Gestational age

• mortality increases > 4000 g o Predisposing factors: maternal diabetes and obesity o Higher incidence of : ♣ birth injuries (cervical and brachial plexus, phrenic nerve damage, clavical #, cephalohematomoas, subdurals, ecchymoses) ♣ congenital anomolies (CHD) ♣ hypoglycemia ♣ intellectual and developmental retardatio Neo Notes

• Grade 3 or 4 IVH occurs in (by wt)

• overall incidence has decreased with improved perinatal care • Grade 3 or 4 IVH occurs in o 66% of infants 500-750 g o 10% of infants <1000g o 5% of infants 1250-1500 g. o Overall incidence is 30% in infants <1500 g. • Incidence of PVL is 20% in infants <1000 g. Neo Notes

Non prematurity related IVH etiologies

• rarely, from a primary hemorrhagic disturbance or congenital vascular anomaly • In utero hemorrhage due to fetal alloimmune thrombocytopenia can cause severe cerebral hemorrhage or a porencephalic cyst after resolution of a fetal cortical hemorrhage. • Other causes: DIC, isoimmune thrombocytopenia, and vitamin K deficiency (especially if mom on phenobarbital or phenytoin) Although the main source and etiology of IVH in about half of the term neonates remained unknown, but mother risk factors such as preeclampsia, urogenital tract infections, chorioamnionitis and some neonatal risks including asphyxia, traumas, vitamin K deficiency, thrombocytopenia and sinovenous thrombotic events have a major role for appearing this event (9). The source of IVH in term and pre-term infants are also different. Although most IVH events in preterm neonates are originated from the fragility of capillaries in the germinal matrix, IVH mostly emanate from residual germinal matrix tissue, the choroid plexus, and the thalamus, in term neonates (10). IVH in both preterm and term neonates has significant consequences, particularly leading to adverse neurodevelopment and even death particularly occurred in early onset IVH (11). The early onset IVH is usually associated with some underlying factors including a lower gestation and birth weight, steroids therapy, antenatal and postnatal complications as well as the mode of delivery (12). Although optimal management and surgical interventions have led to the appropriate outcome in neonates with IVH, but because of severe neurological defects and hemodynamic instabilities, notable number of affected neonates died from this event. Neo notes

PATHOGENESIS of Perinatal asphyxia

↓ Uteroplacental perfusion during labor ± chronic fetal hypoxia-acidosis; Meconium aspiration syndrome Neo Notes

ANCS (Antenatal corticosteroids) should be offered to women at risk for extremely preterm birth at _____ GA when early intensive care is a management option.

≥22 weeks 154-181days CPS Counselling and management for anticipated extremely preterm birth

what drugs increase central respiratory drive by lowering the threshold of response to hypercarbia, enhancing contractility of the diaphragm and preventing diaphragmatic fatigue.

♣ Methylxanthines increase central respiratory drive by lowering the threshold of response to hypercarbia, enhancing contractility of the diaphragm and preventing diaphragmatic fatigue. Neo Notes

When to give surfactant for RDS

♣ Rescue treatment is initiated as soon as possible in the first 24 hrs of life • Repeat dosing every 6-12 hr for total of 2-4 doses Neo Notes

Complications of phototherapy

♣ diarrhea, erythematous macular rash, purpuric rash with transient porphyrinemia, overheating, dehydration, hypothermia ♣ Phototherapy is contraindicated in the presence of porphyria Bronze baby syndrome = dark, brown skin discoloration; occurs with significant elevation of direct bilirubin and cholestasis (may be due to photo-induced modification of prophyrins); may last for many months; phototherapy can continue if needed Neo Notes

endocrinopathy associated w/ cholestasis

♣ hypothyroidism, panhypopituitarism), also metabolic galactosemia, tyrosinemia ♣ Can be the initial manifestation of homozygous α1-antitrypsin deficiency or of cystic fibrosis Neo Notes

Assisted Mechanical Ventilation indications for RDS

♣ pH < 7.20 ♣ PCO2 60 mmHg or higher ♣ PO2 50 mmHg or less at oxygen concentrations of 70-100% ♣ Persistent apnea Neo Notes

2. Preterm 29 weeks with RDS FiO2 0.8, intubated, hypoglycemia; Baby develops respiratory distress and febrile. Is intubated and on antibiotics. What 5 things are you going to do to get the infant ready for transport, CBC done, sat 89%.

- Maintain normothermia - maintain normotension - Increase FiO2 - ? Administer surfactant - Start IVF @ 80mL/kg/day, follow glucose - CXR to confirm position of tube CPS statement: Transport of critically ill newborn - get consent from family, ensure copies of medical records and images, retain placenta for pathology, ensure documented information in chart, provide family with information on tertiary care centre

1. Mom with Parvo REPEAT a. fetal hydrops

- Parvovirus B19: erythema infectiosum/fifth disease - Primary target of B19 infection is erythroid cell line -> cell lysis, depletion of erythroid precursors, transient arrest of erythropoiesis - Some manifestations are direct result of virus (ex. Aplastic crisis), some are post-infectious phenomena secondary to immune response (ex. Exanthema, arthritis) - Can cross placenta -> fetal anemia, high-output cardiac failure, fetal hydrops

7. 35week baby ready for d/c after a very uncomplicated NICU stay (feeding and growing I think). Name five things this baby is at risk for being re-admitted for.

- hyperbilirubinemia, feeding problems, apnea/ALTE, suspected sepsis, respiratory problems, hypothermia (direct from CPS statement)

4. A baby with hypoglycemia needing GIR >10mg/kg/min a) Please list 3 types of hypoglycemia that would have a normal GIR requirement and 3 needed a higher than normal GIR: -Normal GIR requirement - High GIR requirement b) This baby is LGA and has a protruding tongue. What is the physiology of hypoglycemia in this baby. - Physiology:

-Normal GIR requirement: IUGR, SGA, inborn errors of metabolism (GH deficiency, cortisol deficiency) - High GIR requirement: hyperinsulinism/IDM, sepsis, asphyxia, Beckwith-Wiedemann b) This baby is LGA and has a protruding tongue. What is the physiology of hypoglycemia in this baby. - Beckwith-Wiedemann - Physiology: Transient hyperinsulinism secondary to beta-cell hypertrophy

76. A newborn presents in the first week of life with fatigue. Labs demonstrate Na 151, K 4.1, Cl 122, Glucose 4.5. What is the most likely cause of this 1. Poor breastfeeding 2. CAH 3. Munchausen by proxy 4. RTA

1. Poor breastfeeding

4. This is a baby with jaundice and significant conjugated hyperbilirubinemia. Two tables with one column where you had to write the differentials and then the corresponding definitive investigation?

Dubin-Johnson syndrome: urinary coproporphyrin excretion Rotor syndrome: urinary coproporphyrin excretion TPN cholestasis: none Biliary atresia: U/S, liver biopsy, HIDA scan Infectious hepatitis (CMV): Urine/saliva for CMV Alagille: Genetics (JAG1, NOTCH2) Alpha-1 antitripsin: Alpha-1 antitripsin Sepsis/UTI: BCx, UCx Hypothyroidism: TSH Galactosemia: RBC GALT Choledochal cyst: ultrasound PFIC (progressive familial intrahepatic cholestasis): genetics

1. Baby with difficult delivery, now present with hard, red purple nodules. a. What is the likely diagnosis? b. What is the best test to confirm the diagnosis?

What is the likely diagnosis? Subcutaneous fat necrosis - associated with hypoxia, therapeutic cooling, maternal hypertension What is the best test to confirm the diagnosis? Biopsy is the gold standard confirmatory test (but typically a clinical diagnosis)

33. You are assessing a 2 day old term baby for discharge home. He was born at 3.5kg and currently weights 3.22kg. He is vigorous and is feeding well. Breastfeeding is well established. The boy's serum total bilirubin at 48 hours of age is 220; he has no known risk factors for hyperbilirubinemia. At this time, you will recommend: a) Discharge home with follow-up in 24hrs for weight and total serum bilirubin b) Discharge home with follow-up in 72hrs for weight and total serum bilirubin c) Begin standard (single) phototherapy d) Begin intensive (double or triple) phototherapy

a) Discharge home with follow-up in 24hrs for weight and total serum bilirubin

34. A baby is born by spontaneous vaginal delivery with thick meconium. As part of the neonatal resuscitation, he undergoes endotracheal intubation with suctioning for meconium below the cords. He is admitted to the nursery due to oxygen requirements (FIO2 100%). Umbilical catheters are inserted and blood gases are performed. A radial arterial gas reveals pH 7.26, PaO2 90. The UAC gas reveals pH 7.25, PaO2 60. You diagnose: a) PPHN b) Cyanotic congenital heart disease c) Meconium aspiration syndrome d) Non-cyanotic congenital heart disease

a) PPHN - PPHN = R->L shunt - suspect in infants with cyanosis, RF's, unlikely to respond to 100% oxygen - diagnosis by shunting: >20mmHg PaO2 gradient between pre and post-ductal sampling, >5% pre-post ductal saturation difference

38. A term infant was the product of an uncomplicated pregnancy to a healthy mother with protective serologies, normal ultrasounds, and routine prenatal care. The delivery was marked by a slow 2nd stage due to cephalopelvic disproportion. A shoulder dystocia required multiple maneuvers to extract the baby. He was born vigorous, without need for resuscitation, but the obstetricians are concerned that he may have an Erb's palsy. Should that be the case, what are you likely to find on your physical exam? a) Presence of a grasping reflex in the affected limb b) Presence of a symmetric Moro reflex c) Presence of brachial deep tendon reflexes in the affected limb (lose biceps reflex!) d) Significant swelling of the affected limb

a) Presence of a grasping reflex in the affected limb

1. Optho assessment in NICU indicated in which of the following: a. 31 weeks and <1250g b. < 1500g c. less than 35 weeks

a. 31 weeks and <1250g CPS Statement: - screen any infants born <30+6 weeks, <1250g, or more mature with clinical course that puts them at higher risk screening all infants born ≤306/7 weeks' GA (regardless of birth weight) as well as infants having a birth weight ≤1250 g and more mature infants believed to be at high risk for ROP, has a very small likelihood of an unscreened baby having treatable ROP.

1. Newborn, Down's with vomiting post feeds. Abdo slightly distended. Passed meconium. Looks well. What test next? Double bubble for duodenal atresia a. AXR b. US c. UGI with small bowel follow through d. Barium enema

a. AXR - Nelson's: Clinical Features of Down Syndrome in Neonatal Period o CNS - hypotonia, developmental delay o Craniofacial - flat face, epicanthal folds, microcephaly... o Cardiovascular: endocardial cushing defects, VSDs, ASDs... o Musculoskeletal: joint hyperflexibility, wide gap between 1st and 2nd toes... o Gastrointestinal: duodenal atresia, TEF, Hirschsprung, imperforate anus o Miscellaneous: Cutis marmorata - Feed intolerance - investigate with AXR - Duodenal atresia - can have thin membrane that occludes lumen (most common), fibrous cord connecting 2 blind pouches, or gap spanning 2 nonconnecting ends of duodenum o Strong associations: prematurity, Down syndrome o Presents with bilious vomiting without abdominal distension, usually first day of life o Diagnosis suggested by 'double-bubble' on AXR (distended and gas-filled stomach, and proximal duodenum) o Often a prenatal diagnosis

54. (Repeat question): A 3 week old baby presents with poor feeding and poor weight gain. He is jaundiced and has hepatosplenomegaly. His bilirubin is 170 with conjugated 115. Which imaging would you do next? a. Abdominal ultrasound with dopplers b. HIDA scan c. CT abdomen d. MRI abdomen

a. Abdominal ultrasound with dopplers

1. Baby born to Mom with remote history of heroin use. Baby starts seizing on day of life one. Glucose was normal. Lytes all normal. What to do? a. Bolus morphine b. Give phenobarb c. Give Narcan d. Give glucose

a. Bolus morphine - Nelson's: Causes of neonatal seizures days 1-4: HIE (most common!), drug withdrawal, drug toxicity (lidocaine, penicillin), IVH, metabolic (hypocalcemia, hypoglycemia, hypomagnesemia, hypo/hypernatremia), sepsis, IeM, pyridoxine deficiency - Drug withdrawal seizures (barbiturates, benzodiazepines, heroin, methadone) can occur several weeks after drug cessation due to prolonged excretion by neonate - Treatment: o Underlying disorder - if high suspicion of NAS, bolus morphine o Pharmacotherapy ♣ Phenobarbitol is drug of first choice, then Pheny/Fospheny, then Benzos o *in my opinion, if underlying reason not clear, start by treating with pharmacotherapy as try to treat underlying disorder, but if confident, treat NAS.

64. 8 day old baby. Na 165, K 4.7. a. Breastfeeding failure b. CAH c. ? d. ?

a. Breastfeeding failure -11-beta hydroxylase deficiency - loss of mineralocorticoid precursors - hypernatremia, hypokalemia...

1. Newborn, thin MEC. No resus. At 12 hours of life -> respiratory distress and cystic lesion in RUL with slight midline shift. What is the most likely diagnosis: a. CCAM b. Pulmonary sequestration c. Meconium Aspiration d. Congenital Lobar Emphysema

a. CCAM - already reviewed; can present in neonatal period with respiratory distress. CXR - cystic lesion, shift away from lesion

1. Description of newborn twins with weight discrepancy. Larger twin had hematocrit of 70, smaller hematocrit of 40. What is smaller twin at risk for? (twin to twin transfusion) a. CHF b. Hypervolemia c. Hyperviscosity d. Cyanosis

a. CHF - TTTS is complication of monochorionic twin pregnancies; placental vascular anastomoses (usually artery from one twin delivers blood that drains into vein of other twin) - Recipient Twin: Large, polycythemia, hypervolemia, cardiac hypertrophy, large glomeruli - Donor Twin: Small, anemia, oligohydramnios, hypovolemia, thin arteriole walls, small glomeruli

1. Propranolol, SSRI and breastfeeding is it okay a. Continue reassure b. Stop SSRI only c. Stop Propraniolol only d. Stop Both

a. Continue reassure - CPS Statement: "Postpartum use of SSRIs is not a contraindication to breastfeeding" - Nelson's: Propranolol is in 'probably safe' category

1. Benefits from surfactant a. Decreased mortality from RDS b. Decreased BPD c. Increases IVH d. Increases risk of pneumothorax

a. Decreased mortality from RDS - CPS statement: Surfactant decreases mortality, but not rates of BPD; decreases risk of pneumothorax

39) Newborn with plethora and lethargy. Hct 0.72, Hgb 240, glc 3.2 on venous blood. What to do? a. Exchange transfusion b. IV D10W c. Septic W/U

a. Exchange transfusion -this infant is sxatic as described A N18 p 773 Plethora in the Newborn Infant ♣ Plethora = a ruddy, deep red-purple appearance associated with a high Hct, is often due to polycythemia, defined as a central Hct of 65% of higher - this exceeds the mean Hct found in 'normal' newborns by 2 standard deviations ♣ Causes: o High altitudes o Postmature o SGA o 1st day of life o After delayed clamping of the cord o IDMs o Trisomy 13, 18, 21 o Adrenogenital syndrome o Neonatal Graves and hypothyroidism o Hypertensive mothers o Beckwith Wiedemann syndrome ♣ Infants of diabetic mothers and hypertensive mothers or those with growth restriction may have been exposed to chronic fetal hypoxia, which stimulates erythropoietin prodxn and increases RBC prodxn ♣ Clinical manifestations: o Irritability and tremulousness o Lethargy and hypotonia o Tachypnea o Respiratory distress o Congestive heart failure o Cyanosis o Feeding disturbances o Hyperbilirubinemia o Hypoglycemia o Thrombocytopenia o Seizures or stroke o Pulmonary hypertension o NEC o Renal vein thrombosis o Renal failure ♣ Most are asxatic ♣ Rx - partial exchange transfusion (with normal saline) ♣ The Hct level at which to perform a partial exchange transfusion in an asymptomatic infant is unclear but should not be considered if the Hct is < 70-75% Partial exchange Transfusion ♣ Partial xchg Txns can be performed through an umbilical venous catheter, an umbilical arterial catheter, or a peripheral venous catheter. ♣ Aliquots equal to 5% of the estimated blood volume are withdrawn and historically have been replaced either with FFP, plamanate, 5% albumin, or normal saline. Normal saline posses the least risk. ♣ Formula: Blood vol to be xchg = Observed Hct - Desired Hct Observed Hct x blood vol (80ml/kg) PS p445

1. Mum blood type A+, DAT negative, Hb low, male child, Asian with jaundice requiring phototherapy, what is most likely diagnosis a. G6PD b. ABO incompatability c. RH incompatability

a. G6PD - X-linked, most commonly presents as neonatal jaundice, acute hemolysis NOT b. ABO incompatability (typically O moms) c. RH incompatability (Mom must be Rh-negative for this to occur)

52. (Repeat question): You attend delivery of a 33 week infant, who requires 15 minutes of resuscitation including PPV, CPR and 2 rounds of epinephrine. Apgars are 2, 4 and 4. Initial gas is 6.98. What disqualifies this patient from cooling? a. Gestational age b. Length of resuscitation c. APGAR at 10 min d. Gas

a. Gestational age

1. 33 weeker with HIE. Bad gas. Apgar at 10 minutes = 4. What is the contraindication to cooling? a. Gestational age b. Apgar at 10 minutes c. Gas

a. Gestational age CPS Statement: inclusion criteria (infants >33 weeks, HIE, <6 hours of age) Current evidence shows that the infants who benefit from hypothermia are term and late preterm infants ≥36 weeks' gestation with HIE who are ≤6 h of age and who meet both treatment criteria A and B: Criteria A Any two of the following: • Apgar score <5 at 10 min of age. • Continued need for ventilation and resuscitation at 10 min of age. • Metabolic acidosis with pH <7 or base deficit >16 mmol/L in cord or arterial blood gases measured within 1 h of birth. AND Criteria B • Moderate (Sarnat stage II) or severe (Sarnat stage III) encephalopathy demonstrated by the presence of seizures or at least one sign in at least three of the six categories shown in Table 1 [14]. All infants who are depressed at birth should be assessed to determine whether they fulfill criteria A. Infants who fulfill criteria A should then undergo a careful neurological examination to determine whether they fulfill criteria B. Infants who meet both criteria should be offered hypothermia. If possible, it is helpful to assess infants with an amplitude-integrated electroencephalogram for at least 20 min before 5.5 h of age to document abnormal tracings or seizures Asphyxia - Hypoxic Ischemic Encephalopathy • Causes: - disruption of umbilical flow (prolapse, compression), - failure of gas placental gas exchange (abruption), - compromised fetus not tolerating labour (IUGR, anemia), - 'failure of postnatal transition' • Pathophysiology: - Initial necrosis, cell death à2nd reperfusion injury • Diagnosis: - history of perinatal depression + - acidosis, low APGAR 0-3 @ 5 min, ventilation at 10 min - neuro sx: LOC, tone, reflexes, seizures - investigations: • MRI (diffusion restriction), CFM (bedside) / full EEG, EPs HIE management (CPS statement) Therapeutic hypothermia Indications: (>36 weeks) BOTH 1&2 1. Any 2 of: - APGAR < 5 at 10 min, - ventilation 10min, - acidosis pH <7 BD >16 (on cord or at 1h) 2. Signs of moderate to severe encephalopathy Interventions: • Temperature: 34C +/- 0.5 - Passive cooling (in the community) - Active cooling (tertiary centre; servo controlled, cold compresses) - Method: total body or selective head cooling • Timing: ASAP, within 1st 6 hours • Complications: - hypotension, bradycardia, ?coagulopathy, Fat necrosis Outcomes • Prognosis: - Severe: 80% morbidity - Moderate: 30-50% - Mild: usually no deficits • Benefits of cooling - Risk reduction 25% combined mortality & major NDD - NNT 11 to prevent 1 mortality - Risk reduction 20% NDD in survivors

61. What other test would do immediately in an 8-hour neonate with elevated 17 OHP a. Glucose b. Sodium c. Testosterone d. Karyotype

a. Glucose - >90% of CAH caused by 21-hydroxylase deficiency -> problems of synthesis of aldosterone and cortisol - salt-wasting form - both hormones are deficient - develop hyponatremia at 5-7 days of life; earlier develop hypoglycemia

1. Baby with persistent hypoglycemia, now on D10 at TFI 120. What is most likely cause? a. Hyperinsulinism b. GH def c. Cortisol def d. Fatty acid oxidation defect

a. Hyperinsulinism - Nelson's: "Hyperinsulinism is most common cause of persistent hypoglycemia in early infancy" o Insulin concentrations inappropriately high at times of hypoglycemia o May not have evidence of maternal diabetes o Can occur spontaneously, or associated with genetic mutations, Beckwith-Wiedemann, congenital disorders of glycosylation

1. Baby with thrombocytopenia. Mom's plts are normal. What do you give? a. Maternal PLA-1 neg plts b. IVIG c. Steroids

a. Maternal PLA-1 neg plts - Nelson's: Neonatal thrombocytopenia usually secondary to systemic illness or transfer of maternal antibodies directed against fetal platelets (viral/bacterial/protozoal infection, NAIT, maternal ITP) - NATP (Neonatal alloimmune thrombocytopenia purpura, aka NAIT) caused by development of maternal antibodies against antigens present on fetal platelets that are shared with father (like Rh disease) - well-appearing child, normal maternal platelet count, neonatal thrombocytopenia - up to 30% have intracranial hemorrhage - diagnosis made by checking for presence of maternal alloantibodies directed against father's platelets - treat with IVIG prenatally to mother; after birth can give washed maternal platelets (have maternal alloantigens - won't be hit by the antibodies)

74. Baby with large congenital (melanocytic?) nevus on face. What is she at risk for? a. Melanocytosis of the leptomeninges

a. Melanocytosis of the leptomeninges - increased ICP, hydrocephalus, seizures, intellectual disability, melanoma

89. [SAME PICTURE AS PREV YEARS - CEPHALOHEMATOMA] You note a baby in the nursery with the shown lesion. What is the appropriate treatment? a. No treatment necessary - it will self-resolve in 2-12 weeks b. Monitor carefully for head growth and signs of shock in the first few days (subgaleal haemorrhage) c. (Sorry don't remember) d. (Sorry don't remember)

a. No treatment necessary - it will self-resolve in 2-12 weeks

80. Mom who is IVDU. Mom is Hep C PCR positive and has Hep C antibody. Baby is 6 months old, Hep C antibody negative and has normal liver enzymes. What do you do next? a. Nothing b. Hep C PCR c. Refer for liver biopsy d. Repeat antibody in six months

a. Nothing CPS Statement: Infant at any age with no Hep C antibodies = either never infected or cleared the antibody, not going to have issues

1. Neonate term, drying them off and stimulating, HR 40 and apnea what is next step a. PPV b. Intubate c. start compression d. give epi

a. PPV

91. 4 day old neonate who is doing well and appears jaundiced. You perform a serum bilirubin showing an unconjugated bilirubin of 187 and conjugated bilirubin of 13. What is your next step? (No bilirubin charts provided) a. Reassure, no treatment at this point b. Send to nearest hospital for phototherapy c. Check liver enzymes d. Stop breast feeding for 24 hours

a. Reassure, no treatment at this point

1. Picture of ECG of newborn. Looks like 3rd degree block. What is Mom's diagnosis? a. SLE b. Maternal Graves c. Myasthenia Gravis d. ?

a. SLE - Most common cause of complete heart block in Newborn = maternal SLE

1. Child born to Mom on methadone (NAI?) a. Sneezing b. Constipation c. Lethargy d. Hypotonia

a. Sneezing - Heroine/methadone addiction: o LBW infants (less in methadone than heroine) o Symptoms of withdrawal usually within first 48 hours o *tremors, hyperirritability, hypertonicity, diarrhea, high-pitched cry, poor feeding, sneezing, hiccups... o treatment based on presence of signs

1. Child with difficult delivery, shoulder dystocia, forceps, now has increased work of breathing...CXR shows poor movement of L hemidiaphragm and child L arm pronated what is the prognosis a. Spont recovery in a few weeks b. persistence of brachial c. persistence of thoracic d. will need surgical exploration

a. Spont recovery in a few weeks Nelson's: - Brachial plexus injury with involvement of the phrenic nerve (ipsilateral injury), diagnosis by US/fluoroscopy - Typically see full recovery by 1-3 months, rarely surgical plication of diaphragm may be indicated

90. A baby is in the NICU. Very hypotonic, on gavage feeds. Noted to have puffy hands and feet. Birth weight was 4.4 kg. What does this baby most likely have? a. Turner syndrome b. SMA c. Myotonic dystrophy d. Prader Willi Syndrome

a. Turner syndrome

3. Baby born at home. No healthcare contact. Comes in with lethargy and bruising. Elevated PT/PTT. a. What is likely cause? b. How much vitamin K should be given at birth? c. What investigation would you do immediately? d. If mom refuses: Oral Vitamin K can be given with first feed;

a. What is likely cause? Hemorrhagic Disease of the Newborn b. How much vitamin K should be given at birth? 1mg IM if >1500g, 0.5mg if <1500f c. What investigation would you do immediately? CT Head d. If mom refuses: Oral Vitamin K can be given with first feed; repeat at 2-4 weeks, 6-8 weeks Classic disease occurs at 2-7 days, GI/intracranial bleeding

83. 1 month old with sx of colic, thriving otherwise and well. Parents ask for advice. a. continue breastfeeding b. simethicone c. supplement with cow's milk formula d. supplement with soy formula

a. continue breastfeeding

79. You are about to start prostaglandin on an infant with suspected cyanotic heart disease. What are the side effects you need to monitor for? a. hypoventilation, b. hypertension c. Tachycardia d. Irritability

a. hypoventilation,

37. A term baby was born following an uncomplicated pregnancy to a healthy mother. The delivery was complicated by decelerations. At birth, the baby was stunned and required resuscitation with IPPV, but recovered. At 2 hours of life, the neonate is irritable, has a flexed posture, increased deep tendon reflexes and a brisk Moro. What is his Sarnat score? a) 0 b) 1 c) 2 d) 3

b) 1

42. A baby has had a difficult birth. Current, he is irritable, with increased reflexes. Heart rate is increased. His pupils are mydriatic. There are no seizures. His SARNAT score is: a) 0 b) 1 c) 2 d) 3

b) 1

51. A term baby has been distressed during labour, with a worrisome fetal heart tracing. She is born limp and apneic, with absent heart rate. After how long of absent heart rate with adequate resucitation can you consider discontinue your efforts? a) 5 minutes b) 10 minutes c) 20 minutes d) 30 minutes.

b) 10 minutes

36. A woman in labour has a history of maculopapular rash in the context of penicillin use during the birth of her first child. She is in active labour and tested positive for group B strep at 36weeks of gestation. The antibiotic of choice for this labouring mum is: a) Amoxicillin b) Cefazolin c) Clindamycin d) Penicillin

b) Cefazolin (Penicillin intolerant, not anaphylaxis)

40. A baby is referred to your clinic for a "funny head shape". The resident working with you diagnoses a positional plagiocephaly. What would be a worrisome sign of physical examination? a) The ipsilateral ear is anteriorly displaced b) The ipsilateral ear is posteriorly displaced c) The baby has a tendency to look more towards the affected side d) The baby has a symmetric Moro

b) The ipsilateral ear is posteriorly displaced - Positional plagiocephaly: unilateral flattening of occiput, ipsilateral anterior displacement of ear - Craniosynostosis: ipsilateral frontal bossing, posterior displacement of ipsilateral ear

39. An infant born at 28weeks of gestation is currently 2 months old (chronological age) and well. He is in your level 2 nursery on supplemental oxygen via low flow nasal cannulae and learning how to feed at the breast and bottle. When will you prescribe his first series of immunizations? a) At 2 months corrected gestational age b) Today c) Once he is off oxygen d) On the day of discharge home

b) Today CPS: Infants should be given at chronological age as long as in stable condition

45. A newborn with the following lesion had hypoglycemia. What is this child at risk for? a) Neuroblastoma b) Wilm's tumour + hepatoblastoma c) Leukemia d) Duodenal atresia

b) Wilm's tumour + hepatoblastoma - Omphalocele associated with trisomy syndromes, cardiac defects, Beckwith-Wiedemann, Bladder extrophy - This is Beckwith-Wiedemann (large, macroglossia) - at risk of WIlm's and adrenal-cortical carcinoma, hepatoblastoma

48. A strictly vegan mother plans to exclusively breastfeed her baby. Which supplementation does the baby MOST need: a) iron b) calcium c) B12 d) ?

b) calcium

47. A baby born at 38 weeks has a bilirubin of 274 at 30 hours of life. DAT is negative and blood smear is negative for hemolysis. The baby is well. You are provided with all 3 nomograms. What do you do next? (bili charts provided) a) repeat bili in 24-48 hours b) intensive phototherapy and recheck in 6 hours c) IV fluids and prepare for Exchange transfusion d) IV fluids and IVIG

b) intensive phototherapy and recheck in 6 hours

46. A 4500g newborn baby is found 8 hours later to be lethargic, tachypenic. With poor feeding. The Hb is 240, hct 0.72, WBC 22, platelets 340. The gas shows a pH of 7.4 CO2 40 pO2 80 Bicarb 22. The birth was otherwise uneventful. What is your next step in management? a) intubate and ventilate b) partial exchange transfusion c) broad spectrum antibiotics

b) partial exchange transfusion - severe polycythemia (Hct >0.65) but asymptomatic and otherwise totally well - hydrate - severe polycythemia with being unwell, elevated WBC, etc - partial exchange transfusion

77. Baby born with a difficult delivery, required resuscitation. On examination was irritable, with dilated pupils, increased tone. What is the Sarnat score? a. 0 b. 1 c. 2 d. 3

b. 1

68. Baby born at 42 weeks after placental abruption, asystolic since birth and ongoing appropriate resus. After how long can you stop resuscitating? a. 15 minutes b. 10 minutes c. 20 minutes d. 30 minutes

b. 10 minutes

1. Baby born to mom on 40mg methadone daily during pregnancy. What is minimum time you have to watch baby a. 2 days b. 5 days c. 10 days

b. 5 days - AAP for neonatal drug withdrawal: Observe for 5-7 days - Onset with heroine ~24 hours, methadone 48-72 hours

67. Mother has recurrent HSV. There were no active lesion at delivery. For how long after delivery is the infant at risk for PERINATAL transmission? a. 2 weeks b. 6 weeks c. 16 weeks d. 36 weeks

b. 6 weeks CPS Statement: typically presents within 4 weeks, can occur up to 6 weeks

1. Mom is GBS negative but clinical chorio during labour. Mom did not get abx. Baby looks well at birth. What to do? a. observe b. CBC and observe c. CBC, cultures and observe d. FSWU and Abx

b. CBC and observe - CPS Statement: Risk of sepsis with chorioamnionitis is low. CDC and AAP recommend cultures and antibiotics for these patients, but should use individualized approach depending on severity of RFs and maternal antibiotic therapy; close observation for 24 hours is reasonable approach; "CBC 'may be helpful' - I chose CBC and observe as Mom did not get any ABx Table 1 Maternal and neonatal risk factors for early onset bacterial sepsis in term infants • Maternal intrapartum GBS colonization during the current pregnancy • GBS bacteruria at any time during the current pregnancy • A previous infant with invasive GBS disease • Prolonged rupture of membranes ≥18 h • Maternal fever (temperature ≥ 38oC) GBS colonization and intrapartum antibiotic prophylaxis In the absence of IAP, approximately 1% to 2% of infants born to mothers colonized with GBS develop EOGBS sepsis. Current guidelines recommend screening pregnant women for GBS colonization at 35 weeks' to 37 weeks' GA and providing IAP for those who screen positive as well as for those with GBS bacteruria or a previous GBS-infected infant.[1][14] If GBS status is unknown, IAP should be offered if any other risk factors (Table 1) are present. Adequate IAP consists of at least one dose given at least 4 h before birth of: • IV penicillin G (initial dose 5 million units) or ampicillin (initial dose 2 grams) OR • IV cefazolin (initial dose 2 grams) if the mother is allergic to penicillin but at low risk for anaphylaxis Penicillin-allergic women with a high risk of anaphylaxis should be treated with IV clindamycin when the GBS isolate is sensitive to clindamycin and erythromycin OR with IV vancomycin when the isolate is resistant to clindamycin or susceptibilities are unknown. Because the efficacy of the latter two regimes has not been confirmed in clinical trials, they should be considered inadequate IAP when managing the neonate

1. Term neonate at 1 hour of life still sat 88%, good response to O2, C-Section, Resp rate 80, CXR shown fluid in fissure, small R pleural effusion, grunting, normal heart, normal exam what to do next: a. O2 supportive b. CPAP c. lasix d. intubate

b. CPAP - Nelson's: TTN - early tachypnea, occasionally cyanosis relieved with minimal oxygen supplementation, chest sounds clear, CXR shows prominent vascular markings, fluid in fissures, rarely small pleural effusions (often a diagnosis of exclusion)

1. Photo of bili charts 48 hrs old term, bili 210, DAT neg what to do next, feeding well breastfeeding a. rpt bili in 24 hours and weight b. F/U peds in 72 hours c. Conventional Phototherapy d. Intensive phototherapy

b. F/U peds in 72 hours - CPS Statement o Measure TSB/TcB in all infants between 24-72 hours of life, and plot of predictive nomogram ****If term and DAT-negative, anything other than high-risk zone, can proceed with routine care o If 210 at 48 hours, High-Intermediate risk zone, continue with routine care *but should have clinical assessment 24-48 hours later

66. 4.2 kg infant born to a mother with gestational diabetes. Glucose at 2 hours of age is 2.1. What is your management? a. Continue to measure glucose every 3-4 hours for the next 36 hours. b. Feed the infant. Recheck glucose in 1 hour. c. Feed the infant. Recheck glucose before next feed in 3-4 hours. d. Continue to monitor glucose every 3-4 hours for the next 12 hours.

b. Feed the infant. Recheck glucose in 1 hour.

84. You are assessing a baby being transferred to NICU on day of life 3. The baby has features of tachypnea with nasal flaring, poor feeding, fussiness, and myoclonic jerks. Blood sugar was checked on admission and was normal. Given the most likely diagnosis, what is the most appropriate treatment for this baby? a. Diazepam b. Morphine c. Pyridoxine d. Ampicillin and Gentamicin

b. Morphine (Neonatal Abstinence Syndrome) d. Ampicillin and Gentamicin

24h old newborn with the picture shown above. What to do/tell parents? a. Head imaging to see extent b. No further intervention and it will resolve spontaneously in 2-12 weeks c. Provide analgesia and it will resolve spontaneously in the next few days d. Monitor head circumference and Hgb x 24 hours as can become hemorrhagic shock

b. No further intervention and it will resolve spontaneously in 2-12 weeks Cephalohematoma: subperiosteal haemorrhage, limited to surface of one cranial bone - most resorb within 2 weeks-3 months, some calcify for years - no therapy needed; if extensive, at risk of hyperbili and may need phototx Caput - bruising, swelling, crosses suture lines Subgaleal hemorhage - fluctuating mass, crossing suture lines; can have significant haemorrhage; usually resolve after 2-3 weeks

69. Baby with G-tube that looks like this: (This was the exact photo on the exam) a. Reassure b. Silver nitrate cautery c. Topic ABx d. Fungal abx cream

b. Silver nitrate cautery AAP document on tube feeding - granuloma tissue, manage with silver nitrate cautery

85. A 3-month-old baby presents to hospital after a few days of poor feeding. In the emergency room, he is pale and not responsive. Heart rate is 260 beats per minute. Peripheral pulses are still palpable with a capillary refill time of 3 second. The liver is 5 cm below the costal margin. What is the most likely diagnosis? a. Cardiac failure due to myocarditis b. Supraventricular tachycardia c. Lactic acidosis d. Metabolic disorder

b. Supraventricular tachycardia

53. (Repeat question): Neonate with E. coli bacteremia, midline abdominal mass on physical examination. Increased serum creatinine. What investigation would most likely confirm the diagnosis? a. Renal ultrasound b. VCUG c. CT abdomen d. Urine culture

b. VCUG - palpable bladder in context of posterior urethral valves; most common cause of severe obstructive uropathy; distended bladder. NOt b. CT abdomen - renal abscess c. Urine culture

41. Baby APGARs. At 1 minute he is limp and breathing irregularly, with heart rate 80 and he is blue. He grimaces with nasal suctioning. At 5 minutes, his HR is 140, with acrocyanosis. He does not grimace with nasal suctioning. His respirations is irregular. His tone improved to mild flexion. What is his APGARs? a) 2,4 b) 3,4 c) 3,5 d) 4,5

c) 3,5 1 minute: Appearance 0, Pulse 1, Grimace 1, Activity 0, Respirations 1 = 3 5 minute: Appearance 1, Pulse 2, Grimace 0, Activity 1, Respirations 1= 5

70. A 29+2 week GA infant in the NICU. When should ROP screening start? a) 31 weeks b) 32 weeks c) 33 weeks d) 34 weeks

c) 33 weeks CPS Statement: ROP takes longest to develop in immature infants; time of exam based on post-menstrual age (not chronological) - first exam at 31 weeks for infants born GA 22-27 weeks; then add one PMA week for each GA week (ex. GA 29, exam at 33 weeks) Screening: - Who: o <31wks (30+6 or lower) o <1250g o <2001 g if: ♣ Hypotension needing inotropes ♣ Respiratory disease severe and unstable ♣ Prolonged ventilation and oxygen - When: o </=27wks @31wks o >27wks at 4wks chronological age - By whom: o Trained ophthalmologist skilled in ROP - What to use for pain: o Topical anesthetics, soother, sucrose, swaddle - When do follow up exams: o In 1-3wks depending on severity - When do you stop doing exams: o CGA 45wks and no "pre-threshold disease" or worse o Full revascularization o Zone 3 vascularization w/o zone ½ ROP o Regression of ROP Treatment: - Cryotherapy or laser photocoagulation - Decreases angiogenic growth factor in non vascularized areas - When do you treat? o Within 72h of the examination o Consider for ♣ Zone 1 = plus disease OR stage 3 ♣ Zone 2 = plus disease AND stage 2/3 o Treat Threshold disease - 5 consecutive clock hours/ 8 total clock hours of stage 3 in zone 1 or 2 with plus disease - Long term follow up? o All at risk for visual acuity issues regardless of treatment o Need long term ophtho followup Prevention: - Prevent premature birth - <34wks Lower oxygen saturations (85-92%) but this is associated with higher mortality! - >36wks 92-97%

57. You are called to a twin delivery. Twin A is 2800g with Hct 0.70 and Twin B is 2100g and Hct 0.40 What are you most likely to see in Twin A: a) Hypoglycemia b) RDS c) CHF d) ??

c) CHF

59. A 7 day old term Asian baby presents with jaundice. He looks well. Mom's blood type AB+, baby B+. Hgb 104, bilirubin 207, retics 8%. Most likely etiology: a) Sepsis b) ABO incompatibility c) G6PD d) Thalassemia

c) G6PD

44. A woman presents in labour at 36 weeks. She is GBS positive. She had a maculopapular rash fro receiving penicillin for a previous pregnancy. Which antibiotics do you use for her? a) clindamycin b) erythromycin c) cefazolin d) vancomycin

c) cefazolin

58. A baby is diagnosed with Erb's palsy. What are you likely to see: a) symmetric Moro b) intact biceps reflex c) intact grasp d) intact wrist extension

c) intact grasp ERB the waiter grasps the tray erb the waiter grasps the tray = grasp intact Klumpke = fist in a clump C5-C6 - Erb-Duchenne; inability to abduct shoulder, externally rotate arm, and supinate forearm -> adduction and internal rotation of arm, pronation of forearm the arm hangs by the side and is rotated medially; the forearm is extended and pronated. The arm cannot be raised from the side; all power of flexion of the elbow is lost, as is also supination of the forearm".[7] The resulting biceps damage is the main cause of this classic physical position commonly called "waiter's tip". paralysis of the deltoid, biceps and brachioradialis, clinically apparent as internal rotation at the shoulder, extension and pronation of the forearm resulting in a "waiter's tip" position. Klumpke C7-C8 - Klumpke paralysis (when with T1) - paralyzed hand and ipsilateral ptosis = hand in klump = fist alphabetical = erb -> klumpke (c5-6 -> c7 - T1) Klumpke-Dejerine Type Injury (Lower Plexus Root) Lower plexus root injuries (C8-T1) show more sensory (ulnar side of the hand) and vasomotor involvement, with paralysis of the flexors and extensors of the forearm and intrinsic muscles of the hand. If the 1st thoracic root is involved, Horner syndrome and cervical sympathetic damage is likely. what aspect in erbs/klumpkes, if present predicts worse outcome horners, not grasp.

56. Mother uses marijuana for chronic pain. Breastfeeding. What do you tell her about the risks to the baby: a) continue breastfeeding. Benefits of breastfeeding outweigh risk of marijuana b) marijuana is contraindicated due to risks on the developing brain c) recent studies on the legal use of marajuana found no risk c) Counsel to stop using marijuana. Risks to baby unknown. d) Call CAS

c) recent studies on the legal use of marajuana found no risk - Mother risk statement

81. Asian newborn with sacral dimple. Which is the most concerning feature? a. Mongolian spot (not a concerning cutaneous sign) b. 3 mm diameter c. Above gluteal folds d. More than 2 cm from the anus

c. Above gluteal folds

65. Newborn with sacral dimple. Which would make you do further investigation? a. Slate grey nevus (mongolian spot) over dimple b. 2 cm from the anus c. Above the gluteal fold d. 3 mm in diameter

c. Above the gluteal fold Nelson's: imaging indicated if deep, >5mm, >25mm from anal verge, associated with mass/vascular lesion/hairy patch

73. 35+6 week infant born to a mother who is GBS unknown. ROM x 12 hours. Infant is well. What do you do? a. CBC, Cx and Empiric abx x 36h. b. CBC and observe if WBC <5 c. CBC and observe if WBC >5

c. CBC and observe if WBC >5

1. Baby almost ready for discharge. Has tracheostomy tube. Has acute desat, increased WOB. Nurse suctioned no effect. What to do? a. endotracheal intubation b. CXR c. Change the trach d. Gas

c. Change the trach

1. Mom was adequately treated for syphilis during pregnancy (had greater than four-fold drop in titers). How do you manage baby? (CPS guideline) a. Do Syphillis serology in baby b. Monitor for symptoms only c. Do serologies and monitor for symptoms

c. Do serologies and monitor for symptoms ***0, 3, 6, 18 months, monitor for symptoms monthly until 3 months of age - CPS statement: o Treponemal tests remains reactive for life, so look for RPR titre decline (expected decline with adequate therapy = four-fold drop at 12 months, 16-fold drop at 24 months) o Mother treated for Syphilis during pregnancy, more than 4 weeks before delivery, with adequate fall in RPR titre - baseline and monthly assessment for symptoms for first three months, syphilis serological tests at 0, 3, 6, and 18 months; BUT no further testing or treatment

86. You are seeing a 2-day-old neonate who appears quite pale. There is no evidence of bruising or petechiae. You are working up the cause of her pallor and decide to send a Kleinhuer-Betke test. What sample is needed and what are you looking for? a. Neonate's blood, looking for maternal cells b. Neonate's blood, looking for blood type and reaction with mother's blood c. Maternal blood, looking for neonatal cells d. Maternal blood, looking for blood type and reaction with fetal blood

c. Maternal blood, looking for neonatal cells Nelson's: Assess for transplacental haemorrhage, not usually sufficient to cause clinically apparent anemia at birth

1. Neonate with low platelets, low hemoglobin, normal white count, mum's CBC was normal. Neonate given platelets with response (30 increased to 40) what to do next? a. recheck platelets in 8 hours b. IVIG c. PLA 1 negative platelets d. Transfuse regular platelets

c. PLA 1 negative platelets - NAIT, treat with maternally washed platelets - Nelson's: Neonatal thrombocytopenia usually secondary to systemic illness or transfer of maternal antibodies directed against fetal platelets (viral/bacterial/protozoal infection, NAIT, maternal ITP) - NATP (Neonatal alloimmune thrombocytopenia purpura, aka NAIT) caused by development of maternal antibodies against antigens present on fetal platelets that are shared with father (like Rh disease) - well-appearing child, normal maternal platelet count, neonatal thrombocytopenia - up to 30% have intracranial hemorrhage - diagnosis made by checking for presence of maternal alloantibodies directed against father's platelets - treat with IVIG prenatally to mother; after birth can give washed maternal platelets (have maternal alloantigens - won't be hit by the antibodies)

1. Counselling parents of 23 weeker. a. Because they are teens medical team decides b. Against the law to resuscitate in Canada c. Parents feelings need to be taken into account d. You do not have to offer resuscitation.

c. Parents feelings need to be taken into account

72. (Repeat) Most common cause of hypertension in a newborn? a. Hydronephrosis b. Coarctation of aorta c. Renovascular d. CAH

c. Renovascular

1. Mom on carbamazepine, what do you get in baby? a. NTD b. Cardiac defects c. Spina bifida

c. Spina bifida (vs NTD because Spina Bifida is more specific?) THINK carbamazeSPINE - Carbamazepine: spina bifida, possible neurodevelopmental delay - Cardiac defects: alcohol, amphetamines, MMF, Vit D (supravalacular AS), Lithium (Ebstein's)

1. Resuscitating 36 weeker. After drying and stimulating not breathing and HR 48 what do you do next? a. Chest compressions b. Start ventilation with 100% 02 (only ever use FiO2 21 or 30%) c. Start ventilating with room air d. Start ventilation with 100% o2 and chest compressions.

c. Start ventilating with room air

62. Baby with suspected trisomy 21 with a petechial rash, high WBC, anemia, and thrombocytopenia. On exam, has hepatosplenomegaly. What is the most likely reason for his presentation? a. Sepsis b. CMV infection c. Transient myeloproliferative disorder

c. Transient myeloproliferative disorder Nelson's: Transient myelioproliferative disorder characterized by high leukocyte counts, blast cells, anemia, thrombocytopenia, hepatosplenomegaly; resolves within first 3 months of life - 20-30% will develop leukemia by age 3

35. A baby born at 28 weeks gestation is now at 39 weeks corrected gestational age. He received caffeine for several weeks in the context of apnea of prematurity, and has been off it since 34 weeks CGA. He continues to have apneic events. Which of the following is the most likely etiology for these spells? a) Periventricular leucomalacia b) Seizures c) Uncoordinated feeding d) Apnea of prematurity

d) Apnea of prematurity (usually resolves by 36 weeks) CPS Statement: most prems free of apnea by 36 weeks; very preterm infants show more variability - may persist to 44 weeks not a) Periventricular leucomalacia (asymptomatic until neurologic sequelae of white matter damage occur later in infancy)

32. After having breastfed for several weeks and wanting to get back to an easier routine, the mother of an ex-33 week premature baby girl decides to implement formula feeding. She purchases powdered formulawhich she prepares at home. Bacterial contamination is most likely with which of the following pathogens? a) Listeria monocytogenes b) Escherichia coli c) Staphylococcus aureus d) Enterobacter sakazakii

d) Enterobacter sakazakii (found in formula; cause infection in preterm infants)

49. A baby presents at 36 hours with this rash. What is it? a) erythema toxicum neonatorum b) HSV infection c) Miliaria d) Pustular melanosis

d) Pustular melanosis Not a) erythema toxicum neonatorum - benign, self-limited, occurs in 50% of full-term infants, small papules on surrounding erythematous base, peak incidence on second day of life. b) HSV infection - usually presents on day 5-11 of life; skin, eye and mouth disease c) Miliaria (heat rash) - clear vesicles suddenly erupting in sweaty places d) Pustular melanosis - lesions usually present at birth but can be within first couple days of life; hyperpigmented macules, fine scaling, pustules

43. A baby is born with meconium stained amniotic fluid. He is apneic and limp. You bring the baby to the radiant warmer. What do you do? a) tracheal suction b) oral suction c) dry d) stimulate ??

d) stimulate ??

78. Baby depressed at 1 minute - no tone, blue, irregular resps, grimace with suction, HR 80, then at 5min, some flexion, acrocyanosis, HR 140, no grimace with suction, irregular breathing. APGAR? a. 2, 4 b. 2, 5 c. 3, 4 d. 3, 5

d. 3, 5

1. 3 days of age. Feeding, vomiting & lethargy. Glucose of 3. Ph 7.25. Ammonia normal. Full septic w/u - normal CBC and normal LP. Normal Apgars at birth. Examines normally except for slightly lethargic. What is most likely diagnosis: a. Inborn Error of Metabolism b. GBS Sepsis- should see some abnormality on CBC c. HIE d. IVH

d. IVH - usually spontaneous in premature babies, or can occur with trauma or asphyxia; rarely from primary hemorrhagic disturbance or congenital vascular anomaly; usually asymptomatic or have acute deterioration (hypotension, apnea...) - Nelson's: IeM o infant usually appears normal at birth (differentiates from birth trauma, intrauterine insults, chromosomal abnormalities) o often presents like sepsis in the newborn (non-specific; poor feeding, vomiting, lethargy, convulsion, coma) o Approach to IeM: obtain ammonia, pH and CO2; ♣ High ammonia, normal pH/CO2 - urea cycle defect ♣ High anion gap with high or normal ammonia, acidosis - organic acidemia ♣ Normal ammonia, normal anion gap - aminoacidopathy or galactosemia

1. 3 week old baby, not moving R arm. Not opening R eye very well, ptosis and miosis. What to do to work this up? a. Thoracic MRI b. Visual evoked Potentials c. Urine Catecholamines d. Observe

d. Observe - Nelson's: 'Klumpke Paralysis' - rare form of brachial palsy, in which injury to 7th and 8th cranial nerves and 1st thoracic nerve produces paralyzed hand, and ipsilateral ptosis and miosis (Horner syndrome) - Erb palsy = shoulder adducted, internally rotated, forearm extended, sparing of hand and fingers - Must differentiate from cerebral injury, fracture/dislocation; MRI demonstrates nerve root rupture or avulsion; but not useful in neonatal period - Choose to observe

88. A baby is born to a mother known to have illicit drug use. He failed his hearing screen and has thrombocytopenia. His urine is positive for CMV on PCR. What is your next step in treatment? a. No medical treatment b. IV ganciclovir for 2 weeks c. PO valganciclovir for 4 weeks d. PO valganciclovir for six months

d. PO valganciclovir for six months - CPS Statement - If asymptomatic and only failed hearing screen, controversial

87. You are seeing a newborn baby born with normal APGARs with no resuscitation necessary. He has a large, swollen vesicle on the back of his right at hand just adjacent to his wrist. It is 8mm by 5mm in diameter. It is swollen and full of fluid. Baby is otherwise well. How do you treat this? a. Treat with IV ampicillin and Gentamicin b. Treat with IV acyclovir c. Surgical consult d. Reassure and watch

d. Reassure and watch - "sucking blister" - Nelson's: solitary or scattered bullae on upper limbs of infants at birth, radial aspects of forearm, thumb, index finger, presumed secondary to intense sucking in utero

1. 2 month old cholestatic jaundice, frontal bossing, murmur, butterfly vertebrae. What is the most likely ocular finding a. aniridia b. cataracts c. chorioretinitis d. posterior embryotoxon

d. posterior embryotoxon - Nelson's: Alagille Syndrome - most common syndrome with intrahepatic bile duct paucity -> cholestasis - CP: facial characteristics (broad forehead, long straight nose, underdeveloped mandible), ocular abnormalities (posterior embryotoxon, shallow anterior chamber), CV abnormalities (esp. PPS; ToF, PA, VSD), vertebral defects, tubulointerstitial nephropathy - Posterior embryotoxon = prominent, anterior displaced 'Schwalbe line' (anatomic line demarcating outer limit of corneal endothelium layer)


Related study sets

Ch. 31: The Nurse in the Schools

View Set

Difference between reducing valve and regulator, operation of flow restrictors, Bourdon gauges, and Thorpe tubes, effects of downstream pressure on meters, laboratory value that increases during hypoxia, clinical signs of mild/moderate and severe hypoxia,

View Set

Chapter 12: Marketing Channels: Delivering Customer Value

View Set

Chapter 18: Marketing in a Global Economy

View Set

POL California Life: Contract Law

View Set

Ch 18 Intraoperative Nursing Management

View Set

AP Gov. Chapter 2 Quiz Questions

View Set

Sleep, Internal Regulation (Exam 3)

View Set

Political science 2302 Test 2 lamar university

View Set

Chapter 68: Management of Patients With Neurologic Trauma

View Set

Flannery O'Connor "A Good Man is Hard to Find"

View Set